Saltar al contenido principal
LibreTexts Español

2.1: Propiedades básicas

  • Page ID
    111659
  • \( \newcommand{\vecs}[1]{\overset { \scriptstyle \rightharpoonup} {\mathbf{#1}} } \) \( \newcommand{\vecd}[1]{\overset{-\!-\!\rightharpoonup}{\vphantom{a}\smash {#1}}} \)\(\newcommand{\id}{\mathrm{id}}\) \( \newcommand{\Span}{\mathrm{span}}\) \( \newcommand{\kernel}{\mathrm{null}\,}\) \( \newcommand{\range}{\mathrm{range}\,}\) \( \newcommand{\RealPart}{\mathrm{Re}}\) \( \newcommand{\ImaginaryPart}{\mathrm{Im}}\) \( \newcommand{\Argument}{\mathrm{Arg}}\) \( \newcommand{\norm}[1]{\| #1 \|}\) \( \newcommand{\inner}[2]{\langle #1, #2 \rangle}\) \( \newcommand{\Span}{\mathrm{span}}\) \(\newcommand{\id}{\mathrm{id}}\) \( \newcommand{\Span}{\mathrm{span}}\) \( \newcommand{\kernel}{\mathrm{null}\,}\) \( \newcommand{\range}{\mathrm{range}\,}\) \( \newcommand{\RealPart}{\mathrm{Re}}\) \( \newcommand{\ImaginaryPart}{\mathrm{Im}}\) \( \newcommand{\Argument}{\mathrm{Arg}}\) \( \newcommand{\norm}[1]{\| #1 \|}\) \( \newcommand{\inner}[2]{\langle #1, #2 \rangle}\) \( \newcommand{\Span}{\mathrm{span}}\)\(\newcommand{\AA}{\unicode[.8,0]{x212B}}\)

    Introducción

    Acerca de este libro

    Este libro es un curso de un semestre en análisis básico. Comenzó su vida como mis notas de conferencia para enseñar Matemáticas 444 en la Universidad de Illinois en Urbana-Champaign (UIUC) en el semestre de otoño de 2009. Posteriormente agregué el capítulo del espacio métrico para enseñar Matemáticas 521 en la Universidad de Wisconsin—Madison (UW). Un requisito previo para este curso es un curso básico de prueba, utilizando por ejemplo,, o.

    Debería ser posible utilizar el libro tanto para un curso básico para estudiantes que no necesariamente deseen ir a la escuela de posgrado (como UIUC 444), sino también como un curso de un semestre más avanzado que abarque también temas como espacios métricos (como UW 521). Aquí están mis sugerencias sobre qué cubrir en un curso semestral. Para un curso más lento como UIUC 444:

    §0.3, §1.1—§1.4, §2.1—§2.5, §3.1—§3.4, §4.1—§4.2, §5.1—§5.3, §6.1—§6.3

    Para un curso más riguroso que cubre espacios métricos que se ejecuta bastante más rápido (como UW 521):

    §0.3, §1.1—§1.4, §2.1—§2.5, §3.1—§3.4, §4.1—§4.2, §5.1—§5.3, §6.1—§6.2, §7.1—§7.6

    También debería ser posible correr un curso más rápido sin espacios métricos que cubran todas las secciones de los capítulos 0 al 6. El número aproximado de conferencias dictadas en las notas de sección hasta el capítulo 6 son una estimación muy aproximada y fueron diseñadas para el curso más lento. Los primeros capítulos del libro pueden ser utilizados en un curso introductorio de pruebas como se hace por ejemplo en Iowa State University Math 201, donde este libro se utiliza en conjunto con el libro de prueba de Hammack.

    El libro que normalmente se utiliza para la clase en la UIUC es Bartle y Sherbert, Introducción al Análisis Real tercera edición. La estructura del inicio del libro sigue un poco el programa de estudios estándar de UIUC Math 444 y por lo tanto tiene algunas similitudes con. Una diferencia importante es que definimos la integral de Riemann usando sumas Darboux y particiones no etiquetadas. El enfoque Darboux es mucho más apropiado para un curso de este nivel.

    Nuestro enfoque nos permite encajar un curso como UIUC 444 dentro de un semestre y aún así dedicar algún tiempo extra al intercambio de límites y terminar con el teorema de Picard sobre la existencia y singularidad de soluciones de ecuaciones diferenciales ordinarias. Este teorema es un maravilloso ejemplo que utiliza muchos resultados comprobados en el libro. Para los estudiantes más avanzados, el material puede cubrirse más rápido para que lleguemos a espacios métricos y probemos el teorema de Picard usando el teorema de punto fijo como es habitual.

    Existen otros libros excelentes. Mi favorito son los excelentes Principios de Análisis Matemático de Rudin o como comúnmente y amorosamente se le llama bebé Rudin (para distinguirlo de su otro gran libro de texto de análisis). Tomé mucha inspiración e ideas de Rudin. No obstante, Rudin es un poco más avanzado y ambicioso que este curso actual. Para aquellos que deseen continuar con las matemáticas, Rudin es una buena inversión. Una alternativa económica y algo más simple a Rudin es la Introducción al Análisis de Rosenlicht. También está la Introducción al Análisis Real descargable gratuitamente de William Trench.

    Una nota sobre el estilo de algunas de las pruebas: Muchas pruebas tradicionalmente hechas por contradicción, prefiero hacerlo por una prueba directa o por contrapositiva. Si bien el libro sí incluye pruebas por contradicción, solo lo hago cuando la declaración contrapositiva me pareció demasiado incómoda, o cuando la contradicción sigue bastante rápido. En mi opinión, la contradicción es más probable que meta en problemas a los estudiantes principiantes, ya que estamos hablando de objetos que no existen.

    Trato de evitar el formalismo innecesario donde es inútil. Además, las pruebas y el lenguaje se vuelven un poco menos formales a medida que avanzamos en el libro, ya que cada vez se dejan más detalles para evitar el desorden.

    Como regla general, uso\(:=\) en lugar de\(=\) definir un objeto en lugar de simplemente mostrar igualdad. Utilizo este símbolo de manera bastante más generosa de lo habitual para el énfasis. Lo uso incluso cuando el contexto es “local”, es decir, simplemente puedo definir una función\(f(x) := x^2\) para un solo ejercicio o ejemplo.

    Por último, quiero agradecer a Jana Maříková, Glen Pugh, Paul Vojta, Frank Beatrous, Sönmez Şahutoğlu, Jim Brandt, Kenji Kozai y Arthur Busch, por enseñar con el libro y darme muchos comentarios útiles. Frank Beatrous escribió las extensiones de versión de la Universidad de Pittsburgh, que sirvieron de inspiración para muchas de las adiciones recientes. También quiero agradecer a Dan Stoneham, Jeremy Sutter, Eliya Gwetta, Daniel Pimentel-Alarcón, Steve Hoerning, Yi Zhang, Nicole Caviris, Kristopher Lee, Baoyue Bi, Hannah Lund, Trevor Mannella, Mitchel Meyer, Gregory Beauregard, Chase Meadors, Andreas Giannopoulos, un lector anónimo, y en general todos los alumnos en mis clases por sugerencias y búsqueda de errores y errores tipográficos.

    Acerca del análisis

    El análisis es la rama de las matemáticas que se ocupa de las desigualdades y los límites. El presente curso aborda los conceptos más básicos en análisis. El objetivo del curso es familiarizar al lector con pruebas rigurosas en el análisis y también establecer una base firme para el cálculo de una variable.

    El cálculo te ha preparado, el estudiante, para usar las matemáticas sin decirte por qué lo que aprendiste es verdad. Para usar, o enseñar, las matemáticas de manera efectiva, no se puede simplemente saber qué es verdad, hay que saber por qué es verdad. Este curso te muestra por qué el cálculo es cierto. Está aquí para darle una buena comprensión del concepto de límite, el derivado, y lo integral.

    Usemos una analogía. Un mecánico de autos que haya aprendido a cambiar el aceite, arreglar los faros rotos y cargar la batería, solo podrá hacer esas tareas simples. No podrá trabajar de manera independiente para diagnosticar y solucionar problemas. Un maestro de secundaria que no entienda la definición de la integral de Riemann o el derivado puede no ser capaz de responder adecuadamente a todas las preguntas de los estudiantes. Hasta el día de hoy recuerdo varias declaraciones sin sentido que escuché de mi maestro de cálculo en la secundaria, quien simplemente no entendía el concepto del límite, aunque podía “hacer” todos los problemas en el cálculo.

    Comenzamos con una discusión sobre el sistema de números reales, lo más importante es su propiedad de integridad, que es la base de todo lo que viene después. Luego discutimos la forma más simple de un límite, el límite de una secuencia. Posteriormente, estudiamos las funciones de una variable, la continuidad y la derivada. A continuación, definimos la integral de Riemann y demostramos el teorema fundamental del cálculo. Se discuten secuencias de funciones y el intercambio de límites. Finalmente, damos una introducción a los espacios métricos.

    Demos la diferencia más importante entre el análisis y el álgebra. En álgebra, probamos las igualdades directamente; demostramos que un objeto, quizás un número, es igual a otro objeto. En análisis, solemos demostrar desigualdades. Para ilustrar el punto, considere la siguiente declaración.

    \(x\)Déjese ser un número real. Si\(0 \leq x < \epsilon\) es cierto para todos los números reales\(\epsilon > 0\), entonces\(x = 0\).

    Esta afirmación es la idea general de lo que hacemos en análisis. Si queremos demostrar eso\(x = 0\), lo demostramos\(0 \leq x < \epsilon\) para todos positivos\(\epsilon\).

    El término análisis real es un poco erróneo. Prefiero usar simplemente análisis. El otro tipo de análisis, el análisis complejo, realmente se construye sobre el material presente, en lugar de ser distinto. Además, un curso más avanzado sobre análisis real hablaría a menudo de números complejos. Sospecho que la nomenclatura es un bagaje histórico.

    Sigamos con el espectáculo...

    Teoría básica de conjuntos

    Nota: 1—3 conferencias (algún material puede omitirse o cubrirse ligeramente)

    Antes de empezar a hablar de análisis necesitamos arreglar algún lenguaje. El análisis moderno 1 utiliza el lenguaje de los conjuntos, y por lo tanto ahí es donde empezamos. Hablamos de conjuntos de una manera bastante informal, utilizando la llamada “teoría de conjuntos ingenua”. No te preocupes, eso es lo que usa la mayoría de los matemáticos, y es difícil meterse en problemas.

    Suponemos que el lector ha visto la teoría básica de conjuntos y ha tenido un curso de redacción básica de pruebas. Esta sección debe ser pensada como un repaso.

    Sets

    Un conjunto es una colección de objetos llamados elementos o miembros. Un conjunto sin objetos se llama conjunto vacío y se denota por\(\emptyset\) (o a veces por\(\{ \}\)).

    Piense en un set como un club con cierta membresía. Por ejemplo, los alumnos que juegan al ajedrez son miembros del club de ajedrez. No obstante, no lleves demasiado lejos la analogía. Un conjunto solo lo definen los miembros que forman el conjunto; dos conjuntos que tienen los mismos miembros son el mismo conjunto.

    La mayoría de las veces consideraremos conjuntos de números. Por ejemplo, el conjunto\[S := \{ 0, 1, 2 \}\] es el conjunto que contiene los tres elementos 0, 1 y 2. Escribimos\[1 \in S\] para denotar que el número 1 pertenece al conjunto\(S\). Es decir, 1 es miembro de\(S\). De igual manera escribimos\[7 \notin S\] para denotar que el número 7 no está en\(S\). Es decir, 7 no es miembro de\(S\). Los elementos de todos los conjuntos bajo consideración provienen de algún conjunto que llamamos el universo. Por simplicidad, a menudo consideramos que el universo es el conjunto que contiene solo los elementos que nos interesan. El universo se entiende generalmente desde el contexto y no se menciona explícitamente. En este curso, nuestro universo será la mayoría de las veces el conjunto de números reales.

    Si bien los elementos de un conjunto suelen ser números, otros objetos, como otros conjuntos, pueden ser elementos de un conjunto. Un conjunto también puede contener algunos de los mismos elementos que otro conjunto. Por ejemplo,\[T := \{ 0, 2 \}\] contiene los números 0 y 2. En este caso todos los elementos de\(T\) también pertenecen a\(S\). Escribimos\(T \subset S\). De manera más formal hacemos la siguiente definición.

    1. Un conjunto\(A\) es un subconjunto de un conjunto\(B\) si\(x \in A\) implica\(x \in B\), y escribimos\(A \subset B\). Es decir, todos los miembros de también\(A\) son miembros de\(B\).
    2. Dos conjuntos\(A\) y\(B\) son iguales si\(A \subset B\) y\(B \subset A\). Escribimos\(A = B\). Es decir,\(A\) y\(B\) contienen exactamente los mismos elementos. Si no es cierto eso\(A\) y\(B\) son iguales, entonces escribimos\(A \not= B\).
    3. Un conjunto\(A\) es un subconjunto apropiado de\(B\) if\(A \subset B\) y\(A \not= B\). Escribimos\(A \subsetneq B\).

    Por ejemplo, para\(S\) y\(T\) definido anteriormente\(T \subset S\), pero\(T \not= S\). Entonces\(T\) es un subconjunto apropiado de\(S\). Si\(A = B\), entonces\(A\) y\(B\) son simplemente dos nombres para el mismo conjunto exacto. Mencionemos la notación de construcción de conjuntos,\[\{ x \in A : P(x) \} .\] Esta notación se refiere a un subconjunto del conjunto\(A\) que contiene todos los elementos de\(A\) que satisfacen la propiedad\(P(x)\). La notación a veces se abrevia, no\(A\) se menciona cuando se entiende desde el contexto. Además, a veces\(x \in A\) se sustituye por una fórmula para facilitar la lectura de la notación.

    Los siguientes son conjuntos que incluyen las notaciones estándar.

    1. El conjunto de números naturales,\({\mathbb{N}}:= \{ 1, 2, 3, \ldots \}\).
    2. El conjunto de enteros,\({\mathbb{Z}}:= \{ 0, -1, 1, -2, 2, \ldots \}\).
    3. El conjunto de números racionales,\({\mathbb{Q}}:= \{ \frac{m}{n} : m, n \in {\mathbb{Z}} \text{ and } n \not= 0 \}\).
    4. El conjunto de números naturales pares,\(\{ 2m : m \in {\mathbb{N}}\}\).
    5. El conjunto de números reales,\({\mathbb{R}}\).

    Tenga en cuenta que\({\mathbb{N}}\subset {\mathbb{Z}}\subset {\mathbb{Q}}\subset {\mathbb{R}}\).

    Hay muchas operaciones que queremos hacer con conjuntos.

    1. Una unión de dos conjuntos\(A\) y\(B\) se define como\[A \cup B := \{ x : x \in A \text{ or } x \in B \} .\]
    2. Una intersección de dos conjuntos\(A\) y\(B\) se define como\[A \cap B := \{ x : x \in A \text{ and } x \in B \} .\]
    3. Un complemento de\(B\) relativo a\(A\) (o establecer diferencia teórica de\(A\) y\(B\)) se define como\[A \setminus B := \{ x : x \in A \text{ and } x \notin B \} .\]
    4. Decimos complemento de\(B\) y escribimos\(B^c\) en lugar de\(A \setminus B\) si el conjunto\(A\) es o bien el universo entero o es el conjunto obvio que contiene\(B\), y se entiende desde el contexto.
    5. Decimos conjuntos\(A\) y\(B\) somos disjuntos si\(A \cap B = \emptyset\).

    La notación\(B^c\) puede ser un poco vaga en este punto. Si el conjunto\(B\) es un subconjunto de los números reales\({\mathbb{R}}\), entonces\(B^c\) significa\({\mathbb{R}}\setminus B\). Si\(B\) es naturalmente un subconjunto de los números naturales, entonces\(B^c\) es\({\mathbb{N}}\setminus B\). Si alguna vez surgiera ambigüedad, usaremos la notación de diferencia establecida\(A \setminus B\).

    Ilustramos las operaciones en los diagramas de Venn en. Establezcamos ahora uno de los teoremas más básicos sobre conjuntos y lógica.

    \(A, B, C\)Dejen ser conjuntos. Entonces\[\begin{aligned} {(B \cup C)}^c &= B^c \cap C^c , \\ {(B \cap C)}^c &= B^c \cup C^c ,\end{aligned}\] o, de manera más general,\[\begin{aligned} A \setminus (B \cup C) &= (A \setminus B) \cap (A \setminus C) , \\ A \setminus (B \cap C) &= (A \setminus B) \cup (A \setminus C) .\end{aligned}\]

    El primer enunciado es probado por el segundo enunciado si asumimos que el conjunto\(A\) es nuestro “universo”.

    Demostrémoslo\(A \setminus (B \cup C) = (A \setminus B) \cap (A \setminus C)\). Recuerda la definición de igualdad de conjuntos. Primero, debemos demostrar que si\(x \in A \setminus (B \cup C)\), entonces\(x \in (A \setminus B) \cap (A \setminus C)\). Segundo, también debemos demostrar que si\(x \in (A \setminus B) \cap (A \setminus C)\), entonces\(x \in A \setminus (B \cup C)\).

    Entonces, supongamos\(x \in A \setminus (B \cup C)\). Entonces\(x\) está adentro\(A\), pero no en\(B\) ni\(C\). De ahí\(x\) está adentro\(A\) y no en\(B\), es decir,\(x \in A \setminus B\). De igual manera\(x \in A \setminus C\). Así\(x \in (A \setminus B) \cap (A \setminus C)\).

    Por otro lado supongamos\(x \in (A \setminus B) \cap (A \setminus C)\). En particular\(x \in (A \setminus B)\), así\(x \in A\) y\(x \notin B\). También como\(x \in (A \setminus C)\), entonces\(x \notin C\). De ahí\(x \in A \setminus (B \cup C)\).

    La prueba de la otra igualdad se deja como ejercicio.

    También necesitaremos intersectar o unir varios conjuntos a la vez. Si solo hay finitamente muchos, entonces simplemente aplicamos la operación de unión o intersección varias veces. Sin embargo, supongamos que tenemos una colección infinita de conjuntos (un conjunto de conjuntos)\(\{ A_1, A_2, A_3, \ldots \}\). Definimos\[\begin{aligned} & \bigcup_{n=1}^\infty A_n := \{ x : x \in A_n \text{ for some $n \in {\mathbb{N}}$} \} , \\ & \bigcap_{n=1}^\infty A_n := \{ x : x \in A_n \text{ for all $n \in {\mathbb{N}}$} \} .\end{aligned}\]

    También podemos tener conjuntos indexados por dos enteros. Por ejemplo, podemos tener el conjunto de conjuntos\(\{ A_{1,1}, A_{1,2}, A_{2,1}, A_{1,3}, A_{2,2}, A_{3,1}, \ldots \}\). Entonces escribimos\[\bigcup_{n=1}^\infty \bigcup_{m=1}^\infty A_{n,m} = \bigcup_{n=1}^\infty \left( \bigcup_{m=1}^\infty A_{n,m} \right) .\] Y de manera similar con las intersecciones.

    No es difícil ver que podemos tomar los sindicatos en cualquier orden. Sin embargo, generalmente no se permite cambiar el orden de los sindicatos e intersecciones sin pruebas. Por ejemplo:\[\bigcup_{n=1}^\infty \bigcap_{m=1}^\infty \{ k \in {\mathbb{N}}: mk < n \} = \bigcup_{n=1}^\infty \emptyset = \emptyset .\] Sin embargo,\[\bigcap_{m=1}^\infty \bigcup_{n=1}^\infty \{ k \in {\mathbb{N}}: mk < n \} = \bigcap_{m=1}^\infty {\mathbb{N}} = {\mathbb{N}}.\]

    En ocasiones, el conjunto de índices no son los números naturales. En este caso necesitamos una notación más general. Supongamos que\(I\) es algún conjunto y para cada uno\(\iota \in I\), tenemos un conjunto\(A_\iota\). Luego definimos\[\bigcup_{\iota \in I} A_\iota := \{ x : x \in A_\iota \text{ for some $\iota \in I$} \} \qquad \bigcap_{\iota \in I} A_\iota := \{ x : x \in A_\iota \text{ for all $\iota \in I$} \} .\]

    Inducción

    Cuando una declaración incluye un número natural arbitrario, un método común de prueba es el principio de inducción. Comenzamos con el conjunto de números naturales\({\mathbb{N}}= \{ 1,2,3,\ldots \}\), y les damos su orden natural, es decir,\(1 < 2 < 3 < 4 < \cdots\). Al\(S \subset {\mathbb{N}}\) tener un elemento mínimo, queremos decir que existe un\(x \in S\), tal que para cada\(y \in S\), tenemos\(x \leq y\).

    Los números naturales\({\mathbb{N}}\) ordenados de manera natural poseen la llamada propiedad bien ordenada. Tomamos esta propiedad como axioma; simplemente asumimos que es verdad.

    Cada subconjunto no vacío de\({\mathbb{N}}\) tiene un elemento mínimo (más pequeño).

    El principio de inducción es el siguiente teorema, el cual equivale a la propiedad bien ordenada de los números naturales.

    [induction:thm] Let\(P(n)\) Ser una sentencia dependiendo de un número natural\(n\). Supongamos que

    1. (declaración de base)\(P(1)\) es cierto,
    2. (paso de inducción) si\(P(n)\) es verdadero, entonces\(P(n+1)\) es verdadero.

    Entonces\(P(n)\) es cierto para todos\(n \in {\mathbb{N}}\).

    Supongamos que\(S\) es el conjunto de números naturales\(m\) para los cuales no\(P(m)\) es cierto. Supongamos que no\(S\) está vacío. Entonces\(S\) tiene un elemento mínimo por el bien ordenando la propiedad. Llamemos\(m\) al menos elemento de\(S\). Sabemos\(1 \notin S\) por suposición. Por lo tanto\(m > 1\) y\(m-1\) es un número natural también. Ya que\(m\) fue el elemento menos de\(S\), sabemos que eso\(P(m-1)\) es cierto. Pero por el paso de inducción vemos que eso\(P(m-1+1) = P(m)\) es cierto, contradiciendo la afirmación de que\(m \in S\). Por lo tanto\(S\) está vacío y\(P(n)\) es cierto para todos\(n \in {\mathbb{N}}\).

    A veces es conveniente comenzar en un número diferente al 1, pero todo lo que cambia es el etiquetado. La suposición que\(P(n)\) es verdadera en “si\(P(n)\) es verdadera, entonces\(P(n+1)\) es verdadera” suele llamarse hipótesis de inducción.

    Demostremos que para todos\(n \in {\mathbb{N}}\),\[2^{n-1} \leq n! .\] Dejemos que\(P(n)\) sea la afirmación que\(2^{n-1} \leq n!\) sea cierta. Al enchufar\(n=1\), vemos que eso\(P(1)\) es cierto.

    Supongamos que\(P(n)\) es verdad. Es decir, supongamos que\(2^{n-1} \leq n!\) sostiene. Multiplicar ambos lados por 2 para obtener\[2^n \leq 2(n!) .\] Como\(2 \leq (n+1)\) cuando\(n \in {\mathbb{N}}\), tenemos\(2(n!) \leq (n+1)(n!) = (n+1)!\). Eso es,\[2^n \leq 2(n!) \leq (n+1)!,\] y por lo tanto\(P(n+1)\) es cierto. Por el principio de inducción, vemos que eso\(P(n)\) es cierto para todos\(n\), y por lo tanto\(2^{n-1} \leq n!\) es cierto para todos\(n \in {\mathbb{N}}\).

    Afirmamos que para todos\(c \not= 1\),\[1 + c + c^2 + \cdots + c^n = \frac{1-c^{n+1}}{1-c} .\]

    Prueba: Es fácil verificar que la ecuación se mantiene con\(n=1\). Supongamos que es cierto para\(n\). Entonces\[\begin{split} 1 + c + c^2 + \cdots + c^n + c^{n+1} & = ( 1 + c + c^2 + \cdots + c^n ) + c^{n+1} \\ & = \frac{1-c^{n+1}}{1-c} + c^{n+1} \\ & = \frac{1-c^{n+1} + (1-c)c^{n+1}}{1-c} \\ & = \frac{1-c^{n+2}}{1-c} . \end{split}\]

    Existe un principio equivalente llamado inducción fuerte. La prueba de que la inducción fuerte es equivalente a la inducción se deja como ejercicio.

    Dejar\(P(n)\) ser una declaración dependiendo de un número natural\(n\). Supongamos que

    1. (declaración de base)\(P(1)\) es cierto,
    2. (paso de inducción) si\(P(k)\) es verdadero para todos\(k = 1,2,\ldots,n\), entonces\(P(n+1)\) es cierto.

    Entonces\(P(n)\) es cierto para todos\(n \in {\mathbb{N}}\).

    Funciones

    Informalmente, una función teórica de conjunto que\(f\) toma un conjunto\(A\) a un conjunto\(B\) es un mapeo que a cada uno\(x \in A\) asigna un único\(y \in B\). Escribimos\(f \colon A \to B\). Por ejemplo, definimos una función\(S = \{ 0, 1, 2 \}\) que\(f \colon S \to T\) toma a\(T = \{ 0, 2 \}\) asignando\(f(0) := 2\),\(f(1) := 2\), y\(f(2) := 0\). Es decir, una función\(f \colon A \to B\) es una caja negra, en la que pegamos un elemento de\(A\) y la función escupe un elemento de\(B\). A veces\(f\) se llama mapeo y decimos\(f\) mapas\(A\) a\(B\).

    A menudo, las funciones se definen por algún tipo de fórmula, sin embargo, realmente deberías pensar en una función como una tabla de valores muy grande. El problema sutil aquí es que una sola función puede tener varias fórmulas diferentes, todas dando la misma función. Además, para muchas funciones, no existe una fórmula que exprese sus valores.

    Para definir rigurosamente una función primero definamos el producto cartesiano.

    Dejar\(A\) y\(B\) ser conjuntos. El producto cartesiano es el conjunto de tuplas definido como\[A \times B := \{ (x,y) : x \in A, y \in B \} .\]

    Por ejemplo, el conjunto\([0,1] \times [0,1]\) es un conjunto en el plano delimitado por un cuadrado con vértices\((0,0)\),\((0,1)\),\((1,0)\), y\((1,1)\). Cuando\(A\) y\(B\) son el mismo conjunto a veces usamos un superíndice 2 para denotar tal producto. Por ejemplo\([0,1]^2 = [0,1] \times [0,1]\), o\({\mathbb{R}}^2 = {\mathbb{R}}\times {\mathbb{R}}\) (el plano cartesiano).

    Una función\(f \colon A \to B\) es un subconjunto\(f\) de\(A \times B\) tal que para cada uno\(x \in A\), hay una única\((x,y) \in f\). Entonces escribimos\(f(x) = y\). A veces el conjunto\(f\) se llama la gráfica de la función en lugar de la función en sí misma.

    El conjunto\(A\) se llama el dominio de\(f\) (y a veces confusamente denotado\(D(f)\)). El conjunto\[R(f) := \{ y \in B : \text{there exists an $x$ such that %$(x,y) \in f$ $f(x)=y$ } \}\] se llama el rango de\(f\).

    Tenga en cuenta que posiblemente\(R(f)\) puede ser un subconjunto adecuado de\(B\), mientras que el dominio de siempre\(f\) es igual a\(A\). Suele suponer que el dominio de no\(f\) está vacío.

    Desde el cálculo, usted está más familiarizado con las funciones que llevan números reales a números reales. Sin embargo, también viste algunos otros tipos de funciones. Por ejemplo, la derivada es una función que mapea el conjunto de funciones diferenciables con el conjunto de todas las funciones. Otro ejemplo es la transformación de Laplace, que también lleva funciones a funciones. Otro ejemplo más es la función que toma una función continua\(g\) definida en el intervalo\([0,1]\) y devuelve el número\(\int_0^1 g(x) dx\).

    Dejar\(f \colon A \to B\) ser una función, y\(C \subset A\). Definir la imagen (o imagen directa) de\(C\) como\[f(C) := \{ f(x) \in B : x \in C \} .\] Let\(D \subset B\). Definir la imagen inversa como\[f^{-1}(D) := \{ x \in A : f(x) \in D \} .\]

    Definir la función\(f \colon {\mathbb{R}}\to {\mathbb{R}}\) por\(f(x) := \sin(\pi x)\). Entonces\(f([0,\nicefrac{1}{2}]) = [0,1]\),\(f^{-1}(\{0\}) = {\mathbb{Z}}\), etc...

    [st:propinv] Vamos\(f \colon A \to B\). Dejar\(C, D\) ser subconjuntos de\(B\). Entonces\[\begin{aligned} & f^{-1}( C \cup D) = f^{-1} (C) \cup f^{-1} (D) , \\ & f^{-1}( C \cap D) = f^{-1} (C) \cap f^{-1} (D) , \\ & f^{-1}( C^c) = {\left( f^{-1} (C) \right)}^c .\end{aligned}\]

    Leer la última línea como\(f^{-1}( B \setminus C) = A \setminus f^{-1} (C)\).

    Empecemos por el sindicato. Supongamos\(x \in f^{-1}( C \cup D)\). Eso significa\(x\) mapas a\(C\) o\(D\). Por lo tanto\(f^{-1}( C \cup D) \subset f^{-1} (C) \cup f^{-1} (D)\). Por el contrario si\(x \in f^{-1}(C)\), entonces\(x \in f^{-1}(C \cup D)\). De manera similar para\(x \in f^{-1}(D)\). De ahí\(f^{-1}( C \cup D) \supset f^{-1} (C) \cup f^{-1} (D)\), y tenemos igualdad.

    El resto de la prueba se deja como ejercicio.

    La proposición no se sostiene para las imágenes directas. Sí tenemos el siguiente resultado más débil.

    [st:propfor] Vamos\(f \colon A \to B\). Dejar\(C, D\) ser subconjuntos de\(A\). Entonces\[\begin{aligned} & f( C \cup D) = f (C) \cup f (D) , \\ & f( C \cap D) \subset f (C) \cap f (D) .\end{aligned}\]

    La prueba se deja como ejercicio.

    Dejar\(f \colon A \to B\) ser una función. \(f\)Se dice que la función es inyectiva o uno-a-uno si\(f(x_1) = f(x_2)\) implica\(x_1 = x_2\). En otras palabras, para todo\(y \in B\) el conjunto\(f^{-1}(\{y\})\) está vacío o consiste en un solo elemento. Llamamos a\(f\) tal inyección.

    \(f\)Se dice que la función es suryectiva o sobre si\(f(A) = B\). Llamamos a\(f\) tal sobrejección.

    Se dice\(f\) que una función que es a la vez una inyección y una sobreyección es biyectiva, y decimos que\(f\) es una biyección.

    Cuando\(f \colon A \to B\) es una bijección, entonces siempre\(f^{-1}(\{y\})\) es un elemento único de\(A\), y podemos considerar\(f^{-1}\) como una función\(f^{-1} \colon B \to A\). En este caso, llamamos a\(f^{-1}\) la función inversa de\(f\). Por ejemplo, para la bijección\(f \colon {\mathbb{R}}\to {\mathbb{R}}\) definida por\(f(x) := x^3\) tenemos\(f^{-1}(x) = \sqrt[3]{x}\).

    Una última pieza de notación para las funciones que necesitamos es la composición de funciones.

    Vamos\(f \colon A \to B\),\(g \colon B \to C\). La función\(g \circ f \colon A \to C\) se define como\[(g \circ f)(x) := g\bigl(f(x)\bigr) .\]

    Cardinalidad

    Un tema sutil en la teoría de conjuntos y uno que genera una considerable confusión entre los estudiantes es el de la cardinalidad, o “tamaño” de los conjuntos. El concepto de cardinalidad es importante en las matemáticas modernas en general y en el análisis en particular. En esta sección, veremos el primer teorema realmente inesperado.

    Dejar\(A\) y\(B\) ser conjuntos. Decimos\(A\) y\(B\) tenemos la misma cardinalidad cuando existe una biyección\(f \colon A \to B\). Denotamos por\(\left\lvert {A} \right\rvert\) la clase de equivalencia de todos los conjuntos con la misma cardinalidad que\(A\) y simplemente llamamos\(\left\lvert {A} \right\rvert\) la cardinalidad de\(A\).

    Tenga en cuenta que\(A\) tiene la misma cardinalidad que el conjunto vacío si y solo si\(A\) es el conjunto vacío. Entonces escribimos\(\left\lvert {A} \right\rvert := 0\).

    Supongamos que\(A\) tiene la misma cardinalidad que\(\{ 1,2,3,\ldots,n \}\) para algunos\(n \in {\mathbb{N}}\). Entonces escribimos\(\left\lvert {A} \right\rvert := n\), y decimos que\(A\) es finito. Cuando\(A\) es el conjunto vacío, también llamamos\(A\) finito.

    Decimos que\(A\) es infinito o “de infinita cardinalidad” si no\(A\) es finito.

    Que la notación\(\left\lvert {A} \right\rvert = n\) esté justificada lo dejamos como ejercicio. Es decir, para cada conjunto finito no vacío\(A\), existe un número natural único\(n\) tal que existe una bijección de\(A\) a\(\{ 1,2,3,\ldots,n \}\).

    Podemos ordenar juegos por tamaño.

    [def:comparecards] Escribimos\[\left\lvert {A} \right\rvert \leq \left\lvert {B} \right\rvert\] si existe una inyección de\(A\) a\(B\). Escribimos\(\left\lvert {A} \right\rvert = \left\lvert {B} \right\rvert\) si\(A\) y\(B\) tenemos la misma cardinalidad. Escribimos\(\left\lvert {A} \right\rvert < \left\lvert {B} \right\rvert\) si\(\left\lvert {A} \right\rvert \leq \left\lvert {B} \right\rvert\), pero\(A\) y\(B\) no tenemos la misma cardinalidad.

    Declaramos sin pruebas que\(\left\lvert {A} \right\rvert = \left\lvert {B} \right\rvert\) tienen la misma cardinalidad si y sólo si\(\left\lvert {A} \right\rvert \leq \left\lvert {B} \right\rvert\) y\(\left\lvert {B} \right\rvert \leq \left\lvert {A} \right\rvert\). Este es el llamado teorema de Cantor-Bernstein-Schroeder. Además, si\(A\) y\(B\) son cualesquiera dos conjuntos, siempre podemos escribir\(\left\lvert {A} \right\rvert \leq \left\lvert {B} \right\rvert\) o\(\left\lvert {B} \right\rvert \leq \left\lvert {A} \right\rvert\). Los temas que rodean esta última declaración son muy sutiles. Como no requerimos ninguna de estas dos declaraciones, omitimos pruebas.

    Los casos verdaderamente interesantes de cardinalidad son conjuntos infinitos. Comenzamos con la siguiente definición.

    Si\(\left\lvert {A} \right\rvert = \left\lvert

    ParseError: invalid DekiScript (click for details)
    Callstack:
        at (Matematicas/Analisis/Introducción_al_Análisis_Real_(Lebl)/02:_Números_reales/2.01:_Propiedades_básicas), /content/body/div[1]/div[3]/div[4]/p[11]/span[1]/span, line 1, column 1
    
    \right\rvert\), entonces\(A\) se dice que es contablemente infinito. Si\(A\) es finito o contablemente infinito, entonces decimos que\(A\) es contable. Si no\(A\) es contable, entonces\(A\) se dice que es incontable.

    La cardinalidad de\({\mathbb{N}}\) suele denotarse como\(\aleph_0\) (leer como aleph-nada) 2.

    El conjunto de números parejos naturales tiene la misma cardinalidad que\({\mathbb{N}}\). Prueba: Dado un número parejo natural, escríbalo como\(2n\) para algunos\(n \in {\mathbb{N}}\). Después crear una bijección tomando\(2n\) a\(n\).

    De hecho, mencionemos sin pruebas la siguiente caracterización de conjuntos infinitos: Un conjunto es infinito si y solo si está en correspondencia uno a uno con un subconjunto propio de sí mismo.

    \({\mathbb{N}}\times {\mathbb{N}}\)es un conjunto contablemente infinito. Prueba: Arreglar los elementos de\({\mathbb{N}}\times {\mathbb{N}}\) lo siguiente\((1,1)\),\((1,2)\),\((2,1)\),\((1,3)\),\((2,2)\),\((3,1)\),... Es decir, siempre anote primero todos los elementos cuyas dos entradas sumen a\(k\), luego anote todos los elementos cuyas entradas sumen a\(k+1\) y así sucesivamente. Después defina una bijección con\({\mathbb{N}}\) dejando que 1 vaya a\((1,1)\), 2 vaya a\((1,2)\) y así sucesivamente.

    El conjunto de números racionales es contable. Prueba: (informal) Seguir el mismo procedimiento que en el ejemplo anterior, escrito\(\nicefrac{1}{1}\),\(\nicefrac{1}{2}\),\(\nicefrac{2}{1}\), etc... No obstante, omita cualquier fracción (como\(\nicefrac{2}{2}\)) que ya haya aparecido.

    Para completar mencionamos la siguiente declaración. Si\(A \subset B\) y\(B\) es contable, entonces\(A\) es contable. Del mismo modo si\(A\) es incontable, entonces\(B\) es incontable. Como no vamos a necesitar esta afirmación en la secuela, y como la prueba requiere el teorema de Cantor-Bernstein-Schroeder mencionado anteriormente, no lo vamos a dar aquí.

    Damos el primer resultado verdaderamente llamativo. Primero, necesitamos una notación para el conjunto de todos los subconjuntos de un conjunto.

    Si\(A\) es un conjunto, definimos el conjunto de potencia de\(A\), denotado por\({\mathcal{P}}(A)\), para que sea el conjunto de todos los subconjuntos de\(A\).

    Por ejemplo, si\(A := \{ 1,2\}\), entonces\({\mathcal{P}}(A) = \{ \emptyset, \{ 1 \}, \{ 2 \}, \{ 1, 2 \} \}\). Para un conjunto finito\(A\) de cardinalidad\(n\), la cardinalidad de\({\mathcal{P}}(A)\) es\(2^n\). Este hecho se deja como ejercicio. De ahí que para conjuntos finitos la cardinalidad de\({\mathcal{P}}(A)\) es estrictamente mayor que la cardinalidad de\(A\). Lo que es un hecho inesperado y llamativo es que esta afirmación sigue siendo cierta para conjuntos infinitos.

    \(\left\lvert {A} \right\rvert < \left\lvert {{\mathcal{P}}(A)} \right\rvert\). En particular, no existe una sobrejección de\(A\) en adelante\({\mathcal{P}}(A)\).

    Existe una inyección\(f \colon A \to {\mathcal{P}}(A)\). Para cualquier\(x \in A\), defina\(f(x) := \{ x \}\). Por lo tanto\(\left\lvert {A} \right\rvert \leq \left\lvert {{\mathcal{P}}(A)} \right\rvert\).

    Para terminar la prueba, debemos demostrar que ninguna función\(f \colon A \to {\mathcal{P}}(A)\) es una suryección. Supongamos que\(f \colon A \to {\mathcal{P}}(A)\) es una función. Entonces para\(x \in A\),\(f(x)\) es un subconjunto de\(A\). Definir el conjunto\[B := \{ x \in A : x \notin f(x) \} .\] Afirmamos que no\(B\) está en el rango de\(f\) y por lo tanto no\(f\) es una suryección. Supongamos que existe\(x_0\) tal eso\(f(x_0) = B\). \(x_0 \in B\)O bien\(x_0 \notin B\). Si\(x_0 \in B\), entonces\(x_0 \notin f(x_0) = B\), que es una contradicción. Si\(x_0 \notin B\), entonces\(x_0 \in f(x_0) = B\), que vuelve a ser una contradicción. Por lo tanto, tal\(x_0\) no existe. Por lo tanto, no\(B\) está en el rango de\(f\), y no\(f\) es una suryección. Al igual\(f\) que una función arbitraria, no existe ninguna sobrejección.

    Una consecuencia particular de este teorema es que sí existen conjuntos incontables, como\({\mathcal{P}}({\mathbb{N}})\) deben ser incontables. Un hecho relacionado es que el conjunto de números reales (que estudiamos en el siguiente capítulo) es incontable. La existencia de conjuntos incontables puede parecer poco intuitiva, y el teorema causó bastante polémica en el momento en que se anunció. El teorema no sólo dice que existen conjuntos incontables, sino que de hecho existen progresivamente conjuntos infinitos cada vez más grandes\({\mathbb{N}}\),\({\mathcal{P}}({\mathbb{N}})\),\({\mathcal{P}}({\mathcal{P}}({\mathbb{N}}))\),\({\mathcal{P}}({\mathcal{P}}({\mathcal{P}}({\mathbb{N}})))\), etc...

    Ejercicios

    Espectáculo\(A \setminus (B \cap C) = (A \setminus B) \cup (A \setminus C)\).

    Demostrar que el principio de inducción fuerte es equivalente a la inducción estándar.

    Terminar la prueba de.

    1. Demostrar.
    2. Encuentra un ejemplo para el cual la igualdad de conjuntos en\(f( C \cap D) \subset f (C) \cap f (D)\) falla. Es decir, encontrar un\(f\),\(A\),\(B\),\(C\), y\(D\) tal que\(f( C \cap D)\) sea un subconjunto adecuado de\(f(C) \cap f(D)\).

    Demostrar que si\(A\) es finito, entonces existe un número único\(n\) tal que existe una biyección entre\(A\) y\(\{ 1, 2, 3, \ldots, n \}\). Es decir, la notación\(\left\lvert {A} \right\rvert := n\) está justificada. Pista: Demuestre que si\(n > m\), entonces no hay inyección de\(\{ 1, 2, 3, \ldots, n \}\) a\(\{ 1, 2, 3, \ldots, m \}\).

    Demostrar

    1. \(A \cap (B \cup C) = (A \cap B) \cup (A \cap C)\)
    2. \(A \cup (B \cap C) = (A \cup B) \cap (A \cup C)\)

    Dejar\(A \Delta B\) denotar la diferencia simétrica, es decir, el conjunto de todos los elementos que pertenecen a cualquiera\(A\) o\(B\), pero no a ambos\(A\) y\(B\).

    1. Dibuja un diagrama de Venn para\(A \Delta B\).
    2. Espectáculo\(A \Delta B = (A \setminus B) \cup (B \setminus A)\).
    3. Espectáculo\(A \Delta B = (A \cup B) \setminus ( A \cap B)\).

    Para cada uno\(n \in {\mathbb{N}}\), vamos\(A_n := \{ (n+1)k : k \in {\mathbb{N}}\}\).

    1. Encuentra\(A_1 \cap A_2\).
    2. Encuentra\(\bigcup_{n=1}^\infty A_n\).
    3. Encuentra\(\bigcap_{n=1}^\infty A_n\).

    Determine\({\mathcal{P}}(S)\) (el conjunto de potencia) para cada uno de los siguientes:

    1. \(S = \emptyset\),
    2. \(S = \{1\}\),
    3. \(S = \{1,2\}\),
    4. \(S = \{1,2,3,4\}\).

    Dejar\(f \colon A \to B\) y\(g \colon B \to C\) ser funciones.

    1. Demostrar que si\(g \circ f\) es inyectivo, entonces\(f\) es inyectivo.
    2. Demostrar que si\(g \circ f\) es suryectiva, entonces\(g\) es suryectiva.
    3. Encuentra un ejemplo explícito donde\(g \circ f\) es biyectiva, pero ni\(f\) ni\(g\) son biyectivas.

    Demostrarlo\(n < 2^n\) por inducción.

    Demostrar que para un conjunto finito\(A\) de cardinalidad\(n\), la cardinalidad de\({\mathcal{P}}(A)\) es\(2^n\).

    Demostrar\(\frac{1}{1\cdot 2} + \frac{1}{2\cdot 3} + \cdots + \frac{1}{n(n+1)} = \frac{n}{n+1}\) para todos\(n \in {\mathbb{N}}\).

    Demostrar\(1^3 + 2^3 + \cdots + n^3 = {\left( \frac{n(n+1)}{2} \right)}^2\) para todos\(n \in {\mathbb{N}}\).

    Demostrar que\(n^3 + 5n\) es divisible por\(6\) para todos\(n \in {\mathbb{N}}\).

    Encuentra el más pequeño\(n \in {\mathbb{N}}\) tal que\(2{(n+5)}^2 < n^3\) y llámalo\(n_0\). \(2{(n+5)}^2 < n^3\)Demuéstralo para todos\(n \geq n_0\).

    Encuentra todas\(n \in {\mathbb{N}}\) esas cosas que\(n^2 < 2^n\).

    Terminar la prueba de que el es equivalente a la propiedad bien ordenando de\({\mathbb{N}}\). Es decir, probar la propiedad bien ordenando por\({\mathbb{N}}\) usar el principio de inducción.

    Dé un ejemplo de una colección contable de conjuntos finitos\(A_1, A_2, \ldots\), cuya unión no es un conjunto finito.

    Dar un ejemplo de una colección contable de conjuntos infinitos\(A_1, A_2, \ldots\), con\(A_j \cap A_k\) ser infinito para todos\(j\) y\(k\), tal que\(\bigcap_{j=1}^\infty A_j\) es no vacío y finito.

    Números reales

    Propiedades básicas

    Nota: 1.5 conferencias

    El principal objeto con el que trabajamos en el análisis es el conjunto de números reales. Como este conjunto es tan fundamental, muchas veces se dedica mucho tiempo a construir formalmente el conjunto de números reales. Sin embargo, tomamos un enfoque más fácil aquí y solo asumimos que existe un conjunto con las propiedades correctas. Tenemos que comenzar con las definiciones de esas propiedades.

    Un conjunto ordenado es un conjunto\(S\), junto con una relación\(<\) tal que

    1. Para cualquiera\(x, y \in S\), exactamente uno de\(x < y\),\(x=y\), o\(y < x\) sostiene.
    2. Si\(x < y\) y\(y < z\), entonces\(x < z\).

    Escribimos\(x \leq y\) si\(x < y\) o\(x=y\). Definimos\(>\) y\(\geq\) de la manera obvia.

    Por ejemplo, el conjunto de números racionales\({\mathbb{Q}}\) es un conjunto ordenado dejando\(x < y\) si y solo si\(y-x\) es un número racional positivo, es decir si\(y-x = \nicefrac{p}{q}\) donde\(p,q \in {\mathbb{N}}\). Del mismo modo,\({\mathbb{N}}\) y también\({\mathbb{Z}}\) se ordenan los conjuntos.

    Hay otros juegos ordenados además de juegos de números. Por ejemplo, el conjunto de países se puede ordenar por masa terrestre, así por ejemplo India\(>\) Lichtenstein. Cada vez que ordenas un conjunto de alguna manera, estás haciendo un conjunto ordenado. Un conjunto ordenado típico que has usado desde la primaria es el diccionario. Es el conjunto ordenado de palabras donde el orden es el llamado ordenamiento lexicográfico. Tales conjuntos ordenados aparecen a menudo, por ejemplo, en la informática. En esta clase nos interesará mayormente en conjunto ordenado de números sin embargo.

    Vamos\(E \subset S\), donde\(S\) es un conjunto ordenado.

    1. Si existe\(b \in S\) tal que\(x \leq b\) para todos\(x \in E\), entonces decimos que\(E\) está delimitado arriba y\(b\) es un límite superior de\(E\).
    2. Si existe\(b \in S\) tal que\(x \geq b\) para todos\(x \in E\), entonces decimos que\(E\) está delimitado por debajo y\(b\) es un límite inferior de\(E\).
    3. Si existe un límite superior\(b_0\) de\(E\) tal que siempre que\(b\) sea algún límite superior para\(E\) nosotros tenemos\(b_0 \leq b\), entonces\(b_0\) se llama el límite inferior superior o el supremo de\(E\). Escribimos\[\sup\, E := b_0 .\]
    4. De igual manera, si existe un límite inferior\(b_0\) de\(E\) tal que siempre que\(b\) sea algún límite inferior para\(E\) nosotros tenemos\(b_0 \geq b\), entonces\(b_0\) se llama el mayor límite inferior o el infimum de\(E\). Escribimos\[\inf\, E := b_0 .\]

    Cuando un conjunto\(E\) está acotado tanto arriba como acotado abajo, decimos simplemente que\(E\) está acotado.

    Un supremum o infimum para\(E\) (aunque existan) no necesita estar en\(E\). Por ejemplo, el conjunto\(E := \{ x \in {\mathbb{Q}}: x < 1 \}\) tiene un límite superior mínimo de 1, pero 1 no está en el conjunto en\(E\) sí. Por otro lado, si tomamos\(G := \{ x \in {\mathbb{Q}}: x \leq 1 \}\), entonces el límite inferior superior de\(G\) es claramente también 1, y en este caso\(1 \in G\). Por otro lado, el conjunto no\(P := \{ x \in {\mathbb{Q}}: x \geq 0 \}\) tiene límite superior (¿por qué?) y por lo tanto no puede tener menos límite superior. Por otro lado 0 es el mayor límite inferior de\(P\).

    [defn:lub] Un conjunto ordenado\(S\) tiene la propiedad de límite mínimo superior si cada subconjunto no vacío\(E \subset S\) que está delimitado por encima tiene un límite mínimo superior, que\(\sup\, E\) existe en\(S\).

    La propiedad con límite superior a veces se llama propiedad de integridad o propiedad de integridad Dedekind.

    El conjunto\({\mathbb{Q}}\) de números racionales no tiene la propiedad de límite superior mínimo. El subconjunto\(\{ x \in {\mathbb{Q}}: x^2 < 2 \}\) no tiene un supremo en\({\mathbb{Q}}\). El supremo obvio no\(\sqrt{2}\) es racional. Supongamos\(x \in {\mathbb{Q}}\) tal que\(x^2 = 2\). Escribe\(x=\nicefrac{m}{n}\) en los términos más bajos. Entonces\({(\nicefrac{m}{n})}^2 = 2\) o\(m^2 = 2n^2\). De ahí\(m^2\) que sea divisible por 2 y así\(m\) es divisible por 2. Escribe\(m = 2k\) y así\({(2k)}^2 = 2n^2\). Dividir por 2 y anotar eso\(2k^2 = n^2\), y por lo tanto\(n\) es divisible por 2. Pero eso es una contradicción como lo\(\nicefrac{m}{n}\) fue en términos más bajos.

    Eso\({\mathbb{Q}}\) no tiene la propiedad menos límite superior es una de las razones más importantes por las que trabajamos\({\mathbb{R}}\) en el análisis. El conjunto\({\mathbb{Q}}\) está bien para los algebraistas. Pero los analistas requieren la propiedad con menos límites superiores para hacer cualquier trabajo. También requerimos que nuestros números reales tengan muchas propiedades algebraicas. En particular, requerimos que sean un campo.

    Un conjunto\(F\) se denomina campo si tiene dos operaciones definidas en él, suma\(x+y\) y multiplicación\(xy\), y si satisface los siguientes axiomas.

    1. Si\(x \in F\) y\(y \in F\), entonces\(x+y \in F\).
    2. (conmutatividad de adición)\(x+y = y+x\) para todos\(x,y \in F\).
    3. (asociatividad de adición)\((x+y)+z = x+(y+z)\) para todos\(x,y,z \in F\).
    4. Existe un elemento\(0 \in F\) tal que\(0+x = x\) para todos\(x \in F\).
    5. Por cada elemento\(x\in F\) existe un elemento\(-x \in F\) tal que\(x + (-x) = 0\).
    1. Si\(x \in F\) y\(y \in F\), entonces\(xy \in F\).
    2. (conmutatividad de multiplicación)\(xy = yx\) para todos\(x,y \in F\).
    3. (asociatividad de multiplicación)\((xy)z = x(yz)\) para todos\(x,y,z \in F\).
    4. Existe un elemento\(1 \in F\) (y\(1 \not= 0\)) tal que\(1x = x\) para todos\(x \in F\).
    5. Por cada\(x\in F\) tal que\(x \not= 0\) existe un elemento\(\nicefrac{1}{x} \in F\) tal que\(x(\nicefrac{1}{x}) = 1\).
    6. (derecho distributivo)\(x(y+z) = xy+xz\) para todos\(x,y,z \in F\).

    El conjunto\({\mathbb{Q}}\) de números racionales es un campo. Por otro lado no\({\mathbb{Z}}\) es un campo, ya que no contiene inversos multiplicativos. Por ejemplo, no hay\(x \in {\mathbb{Z}}\) tal que\(2x = 1\), por lo que (M5) no esté satisfecho. Se puede comprobar que (M5) es la única propiedad que falla 3.

    Asumiremos los hechos básicos sobre campos que se prueban fácilmente a partir de los axiomas. Por ejemplo,\(0x = 0\) se prueba fácilmente al señalar que\(xx = (0+x)x = 0x+xx\), usando (A4), (D) y (M2). Luego usando (A5) en\(xx\), junto con (A2), (A3) y (A4), obtenemos\(0 = 0x\).

    Se dice que un campo\(F\) es un campo ordenado si también\(F\) es un conjunto ordenado tal que:

    1. [defn:ordfield:i] Para\(x,y,z \in F\),\(x < y\) implica\(x+z < y+z\).
    2. [defn:ordfield:ii] Para\(x,y \in F\),\(x > 0\) e\(y > 0\) implica\(xy > 0\).

    Si\(x > 0\), decimos que\(x\) es positivo. Si\(x < 0\), decimos que\(x\) es negativo. También decimos que no\(x\) es negativo si\(x \geq 0\), y no\(x\) es positivo si\(x \leq 0\).

    Por ejemplo, se puede verificar que los números racionales\({\mathbb{Q}}\) con el orden estándar sean un campo ordenado.

    [prop:bordfield] Dejar\(F\) ser un campo ordenado y\(x,y,z \in F\). Entonces:

    1. [prop:bordfield:i] Si\(x > 0\), entonces\(-x < 0\) (y viceversa).
    2. [prop:bordfield:ii] Si\(x > 0\) y\(y < z\), entonces\(xy < xz\).
    3. [prop:bordfield:iii] Si\(x < 0\) y\(y < z\), entonces\(xy > xz\).
    4. [prop:bordfield:iv] Si\(x \not= 0\), entonces\(x^2 > 0\).
    5. [prop:bordfield:v] Si\(0 < x < y\), entonces\(0 < \nicefrac{1}{y} < \nicefrac{1}{x}\).

    Tenga en cuenta que [prop:bordfield:iv] implica en particular que\(1 > 0\).

    Probemos [prop:bordfield:i]. La desigualdad\(x > 0\) implica por ítem [defn:ordfield:i] de definición de campo ordenado que\(x + (-x) > 0 + (-x)\). Ahora aplicar las propiedades algebraicas de los campos a obtener\(0 > -x\). A lo “viceversa” le sigue un cálculo similar.

    Para [prop:bordfield:ii], primer aviso que\(y < z\) implica\(0 < z - y\) aplicar ítem [defn:ordfield:i] de la definición de campos ordenados. Ahora aplica ítem [defn:ordfield:ii] de la definición de campos ordenados a obtener\(0 < x(z-y)\). Por propiedades algebraicas obtenemos\(0 < xz - xy\), y nuevamente aplicando item [defn:ordfield:i] de la definición que obtenemos\(xy < xz\).

    Parte [prop:bordfield:iii] se deja como ejercicio.

    Para probar parte [prop:bordfield:iv] primero supongamos\(x > 0\). Entonces por ítem [defn:ordfield:ii] de la definición de campos ordenados obtenemos eso\(x^2 > 0\) (use\(y=x\)). Si\(x < 0\), usamos parte [prop:bordfield:iii] de esta proposición. Enchufe\(y=x\) y\(z=0\).

    Por último para probar parte [prop:bordfield:v], observe que\(\nicefrac{1}{x}\) no puede ser igual a cero (¿por qué?). Supongamos\(\nicefrac{1}{x} < 0\), entonces\(\nicefrac{-1}{x} > 0\) por [prop:bordfield:i]. Luego aplica part [prop:bordfield:ii] (as\(x > 0\)) para obtener\(x(\nicefrac{-1}{x}) > 0x\) o\(-1 > 0\), lo que contradice\(1 > 0\) usando part [prop:bordfield:i] nuevamente. De ahí\(\nicefrac{1}{x} > 0\). De igual manera\(\nicefrac{1}{y} > 0\). Así\((\nicefrac{1}{x})(\nicefrac{1}{y}) > 0\) por definición de campo ordenado y por parte [prop:bordfield:ii]\[(\nicefrac{1}{x})(\nicefrac{1}{y})x < (\nicefrac{1}{x})(\nicefrac{1}{y})y .\] Por propiedades algebraicas obtenemos\(\nicefrac{1}{y} < \nicefrac{1}{x}\).

    Producto de dos números positivos (elementos de un campo ordenado) es positivo. Sin embargo, no es cierto que si el producto es positivo, entonces cada uno de los dos factores debe ser positivo.

    Vamos\(x,y \in F\) donde\(F\) es un campo ordenado. Supongamos\(xy > 0\). Entonces ambos\(x\) y\(y\) son positivos, o ambos son negativos.

    Claramente ambas conclusiones pueden suceder. Si cualquiera\(x\) y\(y\) son cero, entonces\(xy\) es cero y por lo tanto no es positivo. De ahí que supongamos que\(x\) y\(y\) son distintos de cero, y simplemente necesitamos demostrarlo si tienen signos opuestos, entonces\(xy < 0\). Sin pérdida de generalidad supongamos\(x > 0\) y\(y < 0\). Multiplicar\(y < 0\) por\(x\) para obtener\(xy < 0x = 0\). El resultado sigue por contrapositivo.

    El lector también puede conocer sobre los números complejos, generalmente denotados por\({\mathbb{C}}\). Es decir,\({\mathbb{C}}\) es el conjunto de números de la forma\(x + iy\), donde\(x\) y\(y\) son números reales, y\(i\) es el número imaginario, un número tal que\(i^2 = -1\). El lector puede recordar por álgebra que también\({\mathbb{C}}\) es un campo, sin embargo, no es un campo ordenado. Si bien uno puede hacer\({\mathbb{C}}\) en un conjunto ordenado de alguna manera, se puede probar que no es posible poner un pedido\({\mathbb{C}}\) que lo convierta en un campo ordenado.

    Ejercicios

    Demostrar parte [prop:bordfield:iii] de.

    [exercise:finitesethasminmax] Dejar\(S\) ser un conjunto ordenado. Dejar\(A \subset S\) ser un subconjunto finito no vacío. Entonces\(A\) se acota. Además,\(\inf\, A\) existe y está en\(A\) y\(\sup\, A\) existe y está en\(A\). Pista: Uso.

    Vamos\(x, y \in F\), donde\(F\) es un campo ordenado. Supongamos\(0 < x < y\). \(x^2 < y^2\)Demuéstralo.

    Deja\(S\) ser un conjunto ordenado. Dejar\(B \subset S\) estar acotado (arriba y abajo). Dejar\(A \subset B\) ser un subconjunto no vacío. Supongamos que todos los\(\inf\)'s y\(\sup\)'s existen. Demostrar que\[\inf\, B \leq \inf\, A \leq \sup\, A \leq \sup\, B .\]

    Deja\(S\) ser un conjunto ordenado. Dejar\(A \subset S\) y suponer\(b\) es un límite superior para\(A\). Supongamos\(b \in A\). \(b = \sup\, A\)Demuéstralo.

    Deja\(S\) ser un conjunto ordenado. Dejar\(A \subset S\) ser un subconjunto no vacío que está delimitado arriba. Supongamos\(\sup\, A\) que existe y\(\sup\, A \notin A\). Mostrar que\(A\) contiene un subconjunto infinitamente contable. En particular,\(A\) es infinito.

    Encuentre un orden (no estándar) del conjunto de números naturales de\({\mathbb{N}}\) tal manera que exista un subconjunto propio no vacío\(A \subsetneq {\mathbb{N}}\) y tal que\(\sup\, A\) exista en\({\mathbb{N}}\), pero\(\sup\, A \notin A\).

    \(F = \{ 0, 1, 2 \}\)Dejar. a) Demostrar que existe exactamente una manera de definir suma y multiplicación para que\(F\) sea un campo si\(0\) y\(1\) tienen su significado habitual de (A4) y (M4). b) Mostrar que\(F\) no puede ser un campo ordenado.

    [exercise:dominatingb] Dejar\(S\) ser un conjunto ordenado y\(A\) es un subconjunto no vacío tal que\(\sup \, A\) existe. Supongamos que hay\(B \subset A\) tal que siempre que\(x \in A\) haya\(y \in B\) tal que\(x \leq y\). Demostrar que\(\sup \, B\) existe y\(\sup \, B = \sup \, A\).

    \(D\)Sea el conjunto ordenado de todas las palabras posibles (no solo palabras en inglés, todas las cadenas de letras de longitud arbitraria) usando el alfabeto latino usando solo letras minúsculas. El orden es el orden lexicográfico como en un diccionario (por ejemplo,\(<\) puerta de\(<\) perro aaa). \(A\)Sea el subconjunto de\(D\) contener las palabras cuya primera letra es 'a' (por ejemplo\(\in A\), a, abcd\(\in A\)). Demuestre que\(A\) tiene un supremo y encuentra lo que es.

    El conjunto de números reales

    Nota: 2 conferencias, los números reales extendidos son opcionales

    El conjunto de números reales

    Finalmente llegamos al sistema de números reales. Para simplificar las cosas, en lugar de construir el conjunto de números reales a partir de los números racionales, simplemente declaramos su existencia como un teorema sin pruebas. Observe que\({\mathbb{Q}}\) es un campo ordenado.

    Existe un campo único 4 ordenado\({\mathbb{R}}\) con el tal que\({\mathbb{Q}}\subset {\mathbb{R}}\).

    Tenga en cuenta que también\({\mathbb{N}}\subset {\mathbb{Q}}\). Eso lo vimos\(1 > 0\). Por (ejercicio) podemos demostrarlo\(n > 0\) para todos\(n \in {\mathbb{N}}\). De igual manera podemos verificar fácilmente todas las afirmaciones que conocemos sobre los números racionales y su ordenamiento natural.

    Demostremos uno de los resultados más básicos pero útiles sobre los números reales. La siguiente proposición es esencialmente cómo un analista demuestra que un número es cero.

    Si\(x \in {\mathbb{R}}\) es tal que\(x \geq 0\) y\(x \leq \epsilon\) para todos\(\epsilon \in {\mathbb{R}}\) dónde\(\epsilon > 0\), entonces\(x = 0\).

    Si\(x > 0\), entonces\(0 < \nicefrac{x}{2} < x\) (¿por qué?). Tomar\(\epsilon = \nicefrac{x}{2}\) obtiene una contradicción. Por lo tanto\(x=0\).

    Un hecho sencillo relacionado es que cada vez que tenemos dos números reales\(a < b\), entonces hay otro número real\(c\) tal que\(a < c < b\). Solo toma por ejemplo\(c = \frac{a+b}{2}\) (¿por qué?). De hecho, hay infinitamente muchos números reales entre\(a\) y\(b\).

    La propiedad más útil de\({\mathbb{R}}\) para los analistas no es sólo que es un campo ordenado, sino que tiene el. Esencialmente queremos\({\mathbb{Q}}\), pero también queremos tomar suprema (e infima) a voluntad y no. Entonces lo que hacemos es arrojar números suficientes para obtener\({\mathbb{R}}\).

    Ya mencionamos que\({\mathbb{R}}\) deben contener elementos que no están en\({\mathbb{Q}}\) debido a la. Vimos que no hay raíz cuadrada racional de dos. El conjunto\(\{ x \in {\mathbb{Q}}: x^2 < 2 \}\) implica la existencia del número real\(\sqrt{2}\), aunque este hecho requiere un poco de trabajo.

    [ejemplo:sqrt2] Reclamación: Existe un número real positivo único\(r\) tal que\(r^2 = 2\). Denotamos\(r\) por\(\sqrt{2}\).

    Toma el set\(A := \{ x \in {\mathbb{R}}: x^2 < 2 \}\). Primero si\(x^2 < 2\), entonces\(x < 2\). Para ver este hecho, tenga en cuenta que\(x \geq 2\) implica\(x^2 \geq 4\) (use, no mencionaremos explícitamente su uso a partir de ahora), de ahí cualquier número\(x\) tal que no\(x \geq 2\) esté en\(A\). Así\(A\) queda delimitado arriba. Por otro lado,\(1 \in A\), así\(A\) es no vacío.

    Definamos\(r := \sup\, A\). Lo demostraremos\(r^2 = 2\) al mostrar eso\(r^2 \geq 2\) y\(r^2 \leq 2\). Esta es la forma en que los analistas muestran igualdad, al mostrar dos desigualdades. Eso ya lo sabemos\(r \geq 1 > 0\).

    A continuación, puede parecer que estamos sacando ciertas expresiones de un sombrero. Al escribir una prueba como esta, por supuesto, se nos ocurrirían las expresiones solo después de jugar con lo que deseamos probar. El orden en que escribimos la prueba no es necesariamente el orden en que se nos ocurre la prueba.

    Primero demostremos eso\(r^2 \geq 2\). Toma un número positivo\(s\) tal que\(s^2 < 2\). Deseamos encontrar una\(h > 0\) tal que\({(s+h)}^2 < 2\). Como\(2-s^2 > 0\), tenemos\(\frac{2-s^2}{2s+1} > 0\). Elegimos un\(h \in {\mathbb{R}}\) tal que\(0 < h < \frac{2-s^2}{2s+1}\). Además, asumimos\(h < 1\).

    \[\begin{aligned} {(s+h)}^2 - s^2 & = h(2s + h) \\ & < h(2s+1) & & \quad \,\bigl(\text{since } h < 1\bigr) \\ & < 2-s^2 & & \quad \left(\text{since } h < \frac{2-s^2}{2s+1} \right) . \end{aligned}\]

    Por lo tanto,\({(s+h)}^2 < 2\). De ahí\(s+h \in A\), pero como\(h > 0\) lo hemos hecho\(s+h > s\). Entonces\(s < r = \sup\, A\). Al igual\(s\) que un número positivo arbitrario tal que\(s^2 < 2\), de ello se deduce\(r^2 \geq 2\).

    Ahora toma un número positivo\(s\) tal que\(s^2 > 2\). Deseamos encontrar una\(h > 0\) tal que\({(s-h)}^2 > 2\). Como\(s^2-2 > 0\) lo hemos hecho\(\frac{s^2-2}{2s} > 0\). Elegimos un\(h \in {\mathbb{R}}\) tal que\(0 < h < \frac{s^2-2}{2s}\) y\(h < s\). \[\begin{split} s^2 - {(s-h)}^2 & = 2sh - h^2 \\ & < 2sh \\ & < s^2-2 \qquad \left( \text{since } h < \frac{s^2-2}{2s} \right) . \end{split}\]Al restar\(s^2\) de ambos lados y multiplicar por\(-1\), nos encontramos\({(s-h)}^2 > 2\). Por lo tanto\(s-h \notin A\).

    Además, si\(x \geq s-h\), entonces\(x^2 \geq {(s-h)}^2 > 2\) (como\(x > 0\) y\(s-h > 0\)) y así\(x \notin A\). Así\(s-h\) es un límite superior para\(A\). Sin embargo\(s-h < s\),, o en otras palabras\(s > r = \sup\, A\). Así\(r^2 \leq 2\).

    Juntos,\(r^2 \geq 2\) e\(r^2 \leq 2\) implican\(r^2 = 2\). La parte de existencia está terminada. Todavía necesitamos manejar la singularidad. Supongamos\(s \in {\mathbb{R}}\) tal que\(s^2 = 2\) y\(s > 0\). Por lo tanto\(s^2 = r^2\). Sin embargo, si\(0 < s < r\), entonces\(s^2 < r^2\). De igual manera\(0 < r < s\) implica\(r^2 < s^2\). De ahí\(s = r\).

    El número\(\sqrt{2} \notin {\mathbb{Q}}\). El conjunto\({\mathbb{R}}\setminus {\mathbb{Q}}\) se llama el conjunto de números irracionales. Acabamos de ver que no\({\mathbb{R}}\setminus {\mathbb{Q}}\) está vacío. No sólo no está vacío, veremos más adelante que es realmente muy grande.

    Utilizando la misma técnica anterior, podemos demostrar que\(x^{1/n}\) existe un número real positivo para todos\(n\in {\mathbb{N}}\) y para todos\(x > 0\). Es decir, para cada uno\(x > 0\), existe un número real positivo único\(r\) tal que\(r^n = x\). La prueba se deja como ejercicio.

    Propiedad de Arquímedes

    Como hemos visto, hay muchos números reales en cualquier intervalo. Pero también hay infinitamente muchos números racionales en cualquier intervalo. El siguiente es uno de los hechos fundamentales sobre los números reales. Las dos partes del siguiente teorema son en realidad equivalentes, aunque no lo parezca a primera vista.

    [thm:arco]

    1. [thm:arch:i] (propiedad de Arquímedes) Si\(x, y \in {\mathbb{R}}\) y\(x > 0\), entonces existe\(n \in {\mathbb{N}}\) tal que\[nx > y .\]
    2. [thm:arch:ii] (\({\mathbb{Q}}\)es denso en\({\mathbb{R}}\)) Si\(x, y \in {\mathbb{R}}\) y\(x < y\), entonces existe\(r \in {\mathbb{Q}}\) tal que\(x < r < y\).

    Demostremos [thm:arch:i]. Dividimos por\(x\) y luego [thm:arch:i] dice que para cualquier número real\(t:= \nicefrac{y}{x}\), podemos encontrar un número natural\(n\) tal que\(n > t\). En otras palabras, [thm:arch:i] dice que no\({\mathbb{N}}\subset {\mathbb{R}}\) está delimitado arriba. Supongamos por contradicción que\({\mathbb{N}}\) está delimitada arriba. Vamos\(b := \sup {\mathbb{N}}\). El número\(b-1\) no puede ser posiblemente un límite superior\({\mathbb{N}}\) ya que es estrictamente menor que\(b\) (el supremo). Así existe\(m \in {\mathbb{N}}\) tal que\(m > b-1\). Agregamos uno para obtener\(m+1 > b\), lo que contradice\(b\) ser un límite superior.

    Abordemos [thm:arch:ii]. Primero asuma\(x \geq 0\). Tenga en cuenta que\(y-x > 0\). Por [thm:arch:i], existe\(n \in {\mathbb{N}}\) tal que\[n(y-x) > 1.\] también por [thm:arch:i] el conjunto no\(A := \{ k \in {\mathbb{N}}: k > nx \}\) está vacío. Por el bien ordenar la propiedad de\({\mathbb{N}}\),\(A\) tiene un elemento mínimo\(m\). Como\(m \in A\), entonces\(m > nx\). Nos dividimos por\(n\) para conseguir\(x < \nicefrac{m}{n}\). Como\(m\) es el elemento menor de\(A\),\(m-1 \notin A\). Si\(m > 1\), entonces\(m-1 \in {\mathbb{N}}\), pero\(m-1 \notin A\) y así\(m-1 \leq nx\). Si\(m=1\), entonces\(m-1 = 0\), y\(m-1 \leq nx\) todavía se mantiene como\(x \geq 0\). Es decir,\[m-1 \leq nx \qquad \text{or} \qquad m \leq nx+1 . % < m .\] por otra parte de\(n(y-x) > 1\) obtenemos\(ny > 1+nx\). De ahí\(ny > 1+nx \geq m\), y por lo tanto\(y > \nicefrac{m}{n}\). Armando todo lo que obtenemos\(x < \nicefrac{m}{n} < y\). Así que vamos\(r = \nicefrac{m}{n}\).

    Ahora asuma\(x < 0\). Si\(y > 0\), entonces solo tomamos\(r=0\). Si\(y \leq 0\), entonces\(0 \leq -y < -x\), y nos encontramos con un racional\(q\) tal que\(-y < q < -x\). Entonces toma\(r = -q\).

    Declaremos y probemos un corolario sencillo pero útil de la.

    \(\inf \{ \nicefrac{1}{n} : n \in {\mathbb{N}}\} = 0\).

    Vamos\(A := \{ \nicefrac{1}{n} : n \in {\mathbb{N}}\}\). Obviamente no\(A\) está vacío. Además,\(\nicefrac{1}{n} > 0\) y así 0 es un límite inferior, y\(b := \inf\, A\) existe. Como 0 es un límite inferior, entonces\(b \geq 0\). Ahora toma una arbitraria\(a > 0\). Por el existe\(n\) tal que\(na > 1\), o en otras palabras\(a > \nicefrac{1}{n} \in A\). Por lo tanto\(a\) no puede ser un límite inferior para\(A\). De ahí\(b=0\).

    Usando supremum e infimum

    Queremos asegurarnos de que suprema e infima sean compatibles con las operaciones algebraicas. Para un conjunto\(A \subset {\mathbb{R}}\) y un número\(x \in {\mathbb{R}}\) definir\[\begin{aligned} x + A & := \{ x+y \in {\mathbb{R}}: y \in A \} , \\ xA & := \{ xy \in {\mathbb{R}}: y \in A \} .\end{aligned}\]

    [prop:supinfalg] Dejar\(A \subset {\mathbb{R}}\) ser no vacío.

    1. Si\(x \in {\mathbb{R}}\) y\(A\) está delimitado arriba, entonces\(\sup (x+A) = x + \sup\, A\).
    2. Si\(x \in {\mathbb{R}}\) y\(A\) está delimitado por debajo, entonces\(\inf (x+A) = x + \inf\, A\).
    3. Si\(x > 0\) y\(A\) está delimitado arriba, entonces\(\sup (xA) = x ( \sup\, A )\).
    4. Si\(x > 0\) y\(A\) está delimitado por debajo, entonces\(\inf (xA) = x ( \inf\, A )\).
    5. Si\(x < 0\) y\(A\) está delimitado por debajo, entonces\(\sup (xA) = x ( \inf\, A )\).
    6. Si\(x < 0\) y\(A\) está delimitado arriba, entonces\(\inf (xA) = x ( \sup\, A )\).

    Tenga en cuenta que multiplicar un conjunto por un número negativo cambia supremum por un infimum y viceversa. También, como la proposición implica que supremum (resp. infimum) de\(x+A\) o\(xA\) existe, también implica que\(x+A\) o no\(xA\) está vacío y delimitado desde arriba (resp. desde abajo).

    Sólo probemos la primera declaración. El resto se dejan como ejercicios.

    Supongamos que\(b\) es un límite superior para\(A\). Es decir,\(y \leq b\) para todos\(y \in A\). Entonces\(x+y \leq x+b\) para todos\(y \in A\), y así\(x+b\) es un límite superior para\(x+A\). En particular, si\(b = \sup\, A\), entonces\[\sup (x+A) \leq x+b = x+ \sup\, A .\]

    La otra dirección es similar. Si\(b\) es un límite superior para\(x+A\), entonces\(x+y \leq b\) para todos\(y \in A\) y así\(y \leq b-x\) para todos\(y \in A\). Así\(b-x\) es un límite superior para\(A\). Si\(b = \sup (x+A)\), entonces\[\sup\, A \leq b-x = \sup (x+A) -x .\] Y el resultado sigue.

    A veces necesitamos aplicar supremum o infimum dos veces. Aquí hay un ejemplo.

    [infsupineq:prop] Let\(A, B \subset {\mathbb{R}}\) be nonempty establece tal manera que\(x \leq y\) siempre\(x \in A\) y\(y \in B\). Entonces\(A\) se delimita arriba,\(B\) se limita por debajo, y\(\sup\, A \leq \inf\, B\).

    Cualquiera\(x \in A\) es un límite inferior para\(B\). Por lo tanto\(x \leq \inf\, B\) para todos\(x \in A\), así\(\inf\, B\) es un límite superior para\(A\). De ahí,\(\sup\, A \leq \inf\, B\).

    Debemos tener cuidado con las desigualdades estrictas y tomar suprema e infima. Obsérvese que\(x < y\) siempre\(x \in A\) y\(y \in B\) todavía solo implica\(\sup\, A \leq \inf\, B\), y no una estricta desigualdad. Este es un importante punto sutil que surge a menudo. Por ejemplo, tomar\(A := \{ 0 \}\) y tomar\(B := \{ \nicefrac{1}{n} : n \in {\mathbb{N}}\}\). Entonces\(0 < \nicefrac{1}{n}\) para todos\(n \in {\mathbb{N}}\). Sin embargo,\(\sup\, A = 0\) y\(\inf\, B = 0\).

    La prueba del siguiente hecho elemental de uso frecuente se deja al lector. Una declaración similar sostiene para infima.

    [prop:existsxepsfromsup] Si\(S \subset {\mathbb{R}}\) es un conjunto no vacío, acotado desde arriba, entonces para cada\(\epsilon > 0\) existe\(x \in S\) tal que\((\sup\, S) - \epsilon < x \leq \sup\, S\).

    Para que el uso de suprema e infima sea aún más fácil, es posible que queramos escribir\(\sup\, A\) y\(\inf\, A\) sin preocuparnos por\(A\) estar acotados y no vacíos. Hacemos las siguientes definiciones naturales.

    \(A \subset {\mathbb{R}}\)Déjese ser un conjunto.

    1. Si\(A\) está vacío, entonces\(\sup\, A := -\infty\).
    2. Si no\(A\) está delimitado arriba, entonces\(\sup\, A := \infty\).
    3. Si\(A\) está vacío, entonces\(\inf\, A := \infty\).
    4. Si no\(A\) está acotado por debajo, entonces\(\inf\, A := -\infty\).

    Por conveniencia,\(\infty\) y a veces\(-\infty\) son tratados como si fueran números, salvo que no permitimos la aritmética arbitraria con ellos. Nos\({\mathbb{R}}^* := {\mathbb{R}}\cup \{ -\infty , \infty\}\) convertimos en un conjunto ordenado dejando\[-\infty < \infty \quad \text{and} \quad -\infty < x \quad \text{and} \quad x < \infty \quad \text{for all $x \in {\mathbb{R}}$}.\] El conjunto\({\mathbb{R}}^*\) se llama el conjunto de números reales extendidos. Es posible definir alguna aritmética en\({\mathbb{R}}^*\). La mayoría de las operaciones se extienden de manera obvia, pero debemos irnos\(\infty-\infty\),\(0 \cdot (\pm\infty)\), e\(\frac{\pm\infty}{\pm\infty}\) indefinidos. Nos abstenemos de utilizar esta aritmética, nos lleva a errores fáciles ya\({\mathbb{R}}^*\) que no es un campo. Ahora podemos tomar suprema e infima sin miedo al vacío o a la falta de límites. En este libro sobre todo evitamos usar\({\mathbb{R}}^*\) fuera de los ejercicios, y dejamos tales generalizaciones al lector interesado.

    Máximas y mínimas

    Por sabemos que un conjunto finito de números siempre tiene un supremum o un infimum que está contenido en el propio conjunto. En este caso no solemos utilizar las palabras supremum o infimum.

    Cuando un conjunto\(A\) de números reales está delimitado arriba, tal que\(\sup\, A \in A\), entonces podemos usar la palabra máximo y la notación\(\max A\) para denotar lo supremo. De manera similar para infimum; cuando un conjunto\(A\) está acotado por debajo y\(\inf\, A \in A\), entonces podemos usar la palabra mínimo y la notación\(\min\, A\). Por ejemplo,\[\begin{aligned} & \max \{ 1,2.4,\pi,100 \} = 100 , \\ & \min \{ 1,2.4,\pi,100 \} = 1 .\end{aligned}\] mientras se escribe\(\sup\) y\(\inf\) puede ser técnicamente correcto en esta situación,\(\max\) y\(\min\) generalmente se utilizan para enfatizar que el supremo o infimum está en el propio conjunto.

    Ejercicios

    Demostrar que si\(t > 0\) (\(t \in {\mathbb{R}}\)), entonces existe\(n \in {\mathbb{N}}\) tal que\(\dfrac{1}{n^2} < t\).

    Demostrar que si\(t \geq 0\) (\(t \in {\mathbb{R}}\)), entonces existe\(n \in {\mathbb{N}}\) tal que\(n-1 \leq t < n\).

    Terminar la prueba de.

    Vamos\(x, y \in {\mathbb{R}}\). Supongamos\(x^2 + y^2 = 0\). Demostrar eso\(x = 0\) y\(y = 0\).

    Demostrar que\(\sqrt{3}\) es irracional.

    Vamos\(n \in {\mathbb{N}}\). Demostrar que o bien\(\sqrt{n}\) es un entero o es irracional.

    Demostrar la desigualdad media aritmética-geométrica. Es decir, para dos números reales positivos\(x,y\) tenemos\[\sqrt{xy} \leq \frac{x+y}{2} .\] Además, la igualdad ocurre si y sólo si\(x=y\).

    Demostrar que para dos números reales cualesquiera\(x\) y\(y\) tal que\(x < y\), existe un número irracional\(s\) tal que\(x < s < y\). Pista: Aplicar la densidad de\({\mathbb{Q}}\) a\(\dfrac{x}{\sqrt{2}}\) y\(\dfrac{y}{\sqrt{2}}\).

    [ejercicio:supofsum] Dejar\(A\) y\(B\) ser dos conjuntos delimitados no vacíos de números reales. Vamos\(C := \{ a+b : a \in A, b \in B \}\). Demostrar que\(C\) es un conjunto acotado y que\[\sup\,C = \sup\,A + \sup\,B \qquad \text{and} \qquad \inf\,C = \inf\,A + \inf\,B .\]

    Dejar\(A\) y\(B\) ser dos conjuntos delimitados no vacíos de números reales no negativos. Definir el conjunto\(C := \{ ab : a \in A, b \in B \}\). Demostrar que\(C\) es un conjunto acotado y que\[\sup\,C = (\sup\,A )( \sup\,B) \qquad \text{and} \qquad \inf\,C = (\inf\,A )( \inf\,B).\]

    Dado\(x > 0\) y\(n \in {\mathbb{N}}\), muestran que existe un número real positivo único\(r\) tal que\(x = r^n\). Por lo general\(r\) se denota por\(x^{1/n}\).

    Demostrar.

    [ejercicio:bernoulliineq] Demostrar la llamada desigualdad de Bernoulli 5: Si\(1+x > 0\) entonces por todo\(n \in {\mathbb{N}}\) lo que tenemos\((1+x)^n \geq 1+nx\).

    Valor absoluto

    Nota: 0.5—1 conferencia

    Un concepto que encontraremos una y otra vez es el concepto de valor absoluto. Se quiere pensar en el valor absoluto como el “tamaño” de un número real. Demos una definición formal. \[\left\lvert {x} \right\rvert := \begin{cases} x & \text{ if $x \geq 0$}, \\ -x & \text{ if $x < 0$} . \end{cases}\]

    Demos las principales características del valor absoluto como propuesta.

    [prop:absbas]

    1. [prop:absbas:i]\(\left\lvert {x} \right\rvert \geq 0\), y\(\left\lvert {x} \right\rvert=0\) si y solo si\(x = 0\).
    2. [prop:absbas:ii]\(\left\lvert {-x} \right\rvert = \left\lvert {x} \right\rvert\) para todos\(x \in {\mathbb{R}}\).
    3. [prop:absbas:iii]\(\left\lvert {xy} \right\rvert = \left\lvert {x} \right\rvert\left\lvert {y} \right\rvert\) para todos\(x,y \in {\mathbb{R}}\).
    4. [prop:absbas:iv]\(\left\lvert {x} \right\rvert^2 = x^2\) para todos\(x \in {\mathbb{R}}\).
    5. [prop:absbas:v]\(\left\lvert {x} \right\rvert \leq y\) si y solo si\(-y \leq x \leq y\).
    6. [prop:absbas:vi]\(-\left\lvert {x} \right\rvert \leq x \leq \left\lvert {x} \right\rvert\) para todos\(x \in {\mathbb{R}}\).

    [prop:absbas:i]: Esta afirmación no es difícil de ver a partir de la definición.

    [prop:absbas:ii]: Supongamos\(x > 0\), entonces\(\left\lvert {-x} \right\rvert = -(-x) = x = \left\lvert {x} \right\rvert\). Del mismo modo cuando\(x < 0\), o\(x = 0\).

    [prop:absbas:iii]: Si\(x\) o\(y\) es cero, entonces el resultado es obvio. Cuando\(x\) y ambos\(y\) son positivos, entonces\(\left\lvert {x} \right\rvert\left\lvert {y} \right\rvert = xy\). \(xy\)también es positivo y por lo tanto\(xy = \left\lvert {xy} \right\rvert\). Si\(x\) y\(y\) son ambos negativos entonces\(xy\) sigue siendo positivo y\(xy = \left\lvert {xy} \right\rvert\), y\(\left\lvert {x} \right\rvert\left\lvert {y} \right\rvert = (-x)(-y) = xy\). Siguiente asumir\(x > 0\) y\(y < 0\). Entonces\(\left\lvert {x} \right\rvert\left\lvert {y} \right\rvert = x(-y) = -(xy)\). Ahora\(xy\) es negativo y por lo tanto\(\left\lvert {xy} \right\rvert = -(xy)\). Del mismo modo si\(x < 0\) y\(y > 0\).

    [prop:absbas:iv]: Obvio si\(x \geq 0\). Si\(x < 0\), entonces\(\left\lvert {x} \right\rvert^2 = {(-x)}^2 = x^2\).

    [prop:absbas:v]: Supongamos\(\left\lvert {x} \right\rvert \leq y\). Si\(x \geq 0\), entonces\(x \leq y\). Obviamente\(y \geq 0\) y por lo tanto\(-y \leq 0 \leq x\) así\(-y \leq x \leq y\) se sostiene. Si\(x < 0\), entonces\(\left\lvert {x} \right\rvert \leq y\) significa\(-x \leq y\). Negando ambos lados obtenemos\(x \geq -y\). De nuevo\(y \geq 0\) y así\(y \geq 0 > x\). De ahí,\(-y \leq x \leq y\).

    Por otro lado, supongamos que\(-y \leq x \leq y\) es cierto. Si\(x \geq 0\), entonces\(x \leq y\) es equivalente a\(\left\lvert {x} \right\rvert \leq y\). Si\(x < 0\), entonces\(-y \leq x\) implica\((-x) \leq y\), que es equivalente a\(\left\lvert {x} \right\rvert \leq y\).

    [prop:absbas:vi]: Aplicar [prop:absbas:v] con\(y = \left\lvert {x} \right\rvert\).

    Una propiedad utilizada con la frecuencia suficiente para darle un nombre es la llamada desigualdad triangular.

    \(\left\lvert {x+y} \right\rvert \leq \left\lvert {x} \right\rvert+\left\lvert {y} \right\rvert\)para todos\(x, y \in {\mathbb{R}}\).

    De nosotros tenemos\(- \left\lvert {x} \right\rvert \leq x \leq \left\lvert {x} \right\rvert\) y\(- \left\lvert {y} \right\rvert \leq y \leq \left\lvert {y} \right\rvert\). Añadimos estas dos desigualdades para obtener\[- (\left\lvert {x} \right\rvert+\left\lvert {y} \right\rvert) \leq x+y \leq \left\lvert {x} \right\rvert+ \left\lvert {y} \right\rvert .\] De nuevo por lo que tenemos\(\left\lvert {x+y} \right\rvert \leq \left\lvert {x} \right\rvert+\left\lvert {y} \right\rvert\).

    Hay otras versiones a menudo aplicadas de la desigualdad triangular.

    Vamos\(x,y \in {\mathbb{R}}\)

    1. (desigualdad del triángulo inverso)\(\bigl\lvert (\left\lvert {x} \right\rvert-\left\lvert {y} \right\rvert) \bigr\rvert \leq \left\lvert {x-y} \right\rvert\).
    2. \(\left\lvert {x-y} \right\rvert \leq \left\lvert {x} \right\rvert+\left\lvert {y} \right\rvert\).

    Vamos a enchufar\(x=a-b\) y\(y=b\) entrar en la desigualdad triángulo estándar para obtener\[\left\lvert {a} \right\rvert = \left\lvert {a-b+b} \right\rvert \leq \left\lvert {a-b} \right\rvert + \left\lvert {b} \right\rvert ,\] o\(\left\lvert {a} \right\rvert-\left\lvert {b} \right\rvert \leq \left\lvert {a-b} \right\rvert\). Cambiando los roles de\(a\) y\(b\) obtenemos o\(\left\lvert {b} \right\rvert-\left\lvert {a} \right\rvert \leq \left\lvert {b-a} \right\rvert = \left\lvert {a-b} \right\rvert\). Ahora aplicando de nuevo obtenemos la desigualdad del triángulo inverso.

    La segunda versión de la desigualdad triangular se obtiene a partir de la estándar simplemente reemplazando\(y\) con\(-y\) y señalando nuevamente eso\(\left\lvert {-y} \right\rvert = \left\lvert {y} \right\rvert\).

    Vamos\(x_1, x_2, \ldots, x_n \in {\mathbb{R}}\). Entonces\[\left\lvert {x_1 + x_2 + \cdots + x_n} \right\rvert \leq \left\lvert {x_1} \right\rvert + \left\lvert {x_2} \right\rvert + \cdots + \left\lvert {x_n} \right\rvert .\]

    Procedemos por. La conclusión se sostiene trivialmente para\(n=1\), y para\(n=2\) ello es la desigualdad estándar del triángulo. Supongamos que el corolario se sostiene para\(n\). Toma\(n+1\) números\(x_1,x_2,\ldots,x_{n+1}\) y primero usa la desigualdad triangular estándar, luego la hipótesis de inducción\[\begin{split} \lvert {x_1 + x_2 + \cdots + x_n + x_{n+1}} \rvert & \leq \lvert {x_1 + x_2 + \cdots + x_n} \rvert + \lvert {x_{n+1}} \rvert \\ & \leq \lvert {x_1} \rvert + \lvert {x_2} \rvert + \cdots + \lvert {x_n} \rvert + \lvert {x_{n+1}} \rvert . \qedhere \end{split}\]

    Veamos un ejemplo del uso de la desigualdad triangular.

    Encuentra un número\(M\) tal que\(\lvert {x^2-9x+1} \rvert \leq M\) para todos\(-1 \leq x \leq 5\).

    Usando la desigualdad del triángulo, escribe\[\lvert {x^2-9x+1} \rvert \leq \lvert {x^2} \rvert+\lvert {9x} \rvert+\lvert {1} \rvert = \lvert {x} \rvert^2+9\lvert {x} \rvert+1 .\] Es obvio que\(\lvert {x} \rvert^2+9\lvert {x} \rvert+1\) es más grande cuando\(\left\lvert {x} \right\rvert\) es más grande. En el intervalo proporcionado,\(\left\lvert {x} \right\rvert\) es mayor cuando\(x=5\) y así\(\left\lvert {x} \right\rvert=5\). Una posibilidad para\(M\) es\[M = 5^2+9(5)+1 = 71 .\] Hay, por supuesto, otros\(M\) que funcionan. El límite del 71 es mucho más alto de lo necesario, pero no pedimos lo mejor posible\(M\), solo uno que funcione.

    El último ejemplo nos lleva al concepto de funciones acotadas.

    Supongamos que\(f \colon D \to {\mathbb{R}}\) es una función. Decimos que\(f\) está acotado si existe un número\(M\) tal que\(\left\lvert {f(x)} \right\rvert \leq M\) para todos\(x \in D\).

    En el ejemplo que probamos\(x^2-9x+1\) es acotado cuando se considera como una función on\(D = \{ x : -1 \leq x \leq 5 \}\). Por otro lado, si consideramos el mismo polinomio como una función en toda la línea real\({\mathbb{R}}\), entonces no está acotada.

    Para una función\(f \colon D \to {\mathbb{R}}\)\[\begin{aligned} & \sup_{x \in D} f(x) := \sup\, f(D) , \\ & \inf_{x \in D} f(x) := \inf\, f(D) .\end{aligned}\] escribimos También a veces reemplazamos el “\(x \in D\)” con una expresión. Por ejemplo si, como antes\(f(x) = x^2-9x+1\), para\(-1 \leq x \leq 5\), un poco de cálculo muestra\[\sup_{x \in D} f(x) = \sup_{-1 \leq x \leq 5} ( x^2 -9x+1 ) = 11, \qquad \inf_{x \in D} f(x) = \inf_{-1 \leq x \leq 5} ( x^2 -9x+1 ) = \nicefrac{-77}{4} .\]

    [prop:funcsupinf] Si\(f \colon D \to {\mathbb{R}}\) y\(g \colon D \to {\mathbb{R}}\) (\(D\)nonempty) están delimitadas 6 funciones y\[f(x) \leq g(x) \qquad \text{for all $x \in D$},\] luego\[\label{prop:funcsupinf:eq} \sup_{x \in D} f(x) \leq \sup_{x \in D} g(x) \qquad \text{and} \qquad \inf_{x \in D} f(x) \leq \inf_{x \in D} g(x) .\]

    Debes tener cuidado con las variables. El\(x\) en el lado izquierdo de la desigualdad en [prop:funcsupinf:eq] es diferente de la\(x\) de la derecha. Realmente deberías pensar en la primera desigualdad como\[\sup_{x \in D} f(x) \leq \sup_{y \in D} g(y) .\] Demostremos esta desigualdad. Si\(b\) es un límite superior para\(g(D)\), entonces\(f(x) \leq g(x) \leq b\) para todos\(x \in D\), y por lo tanto\(b\) es un límite superior para\(f(D)\). Tomando el límite inferior superior obtenemos eso para todos\(x \in D\)\[f(x) \leq \sup_{y \in D} g(y) .\] Por lo tanto\(\sup_{y \in D} g(y)\) es un límite superior para\(f(D)\) y por lo tanto mayor que o igual al límite inferior superior de\(f(D)\). \[\sup_{x \in D} f(x) \leq \sup_{y \in D} g(y) .\]La segunda desigualdad (la afirmación sobre el inf) se deja como ejercicio.

    Un error común es concluir\[\label{rn:av:ltnottrue} \sup_{x \in D} f(x) \leq \inf_{y \in D} g(y) .\] La desigualdad [rn:av:ltnottrue] no es cierta dada la hipótesis de la afirmación anterior. Para esta desigualdad más fuerte necesitamos la hipótesis más fuerte\[f(x) \leq g(y) \qquad \text{for all $x \in D$ and $y \in D$.}\] La prueba así como un contraejemplo se deja como ejercicio.

    Ejercicios

    Demuéstralo\(\left\lvert {x-y} \right\rvert < \epsilon\) si y solo si\(x-\epsilon < y < x+\epsilon\).

    Demostrar que

    1. \(\max \{x,y\} = \frac{x+y+\left\lvert {x-y} \right\rvert}{2}\)
    2. \(\min \{x,y\} = \frac{x+y-\left\lvert {x-y} \right\rvert}{2}\)

    Encuentra un número\(M\) tal que\(\lvert {x^3-x^2+8x} \rvert \leq M\) para todos\(-2 \leq x \leq 10\).

    Terminar la prueba de. Es decir, probar que dado cualquier conjunto\(D\), y dos funciones acotadas\(f \colon D \to {\mathbb{R}}\) y\(g \colon D \to {\mathbb{R}}\) tal que\(f(x) \leq g(x)\) para todos\(x \in D\), entonces\[\inf_{x\in D} f(x) \leq \inf_{x\in D} g(x) .\]

    Let\(f \colon D \to {\mathbb{R}}\) and\(g \colon D \to {\mathbb{R}}\) be funciones (\(D\)no vacías).

    1. Supongamos\(f(x) \leq g(y)\) para todos\(x \in D\) y\(y \in D\). Demostrar que\[\sup_{x\in D} f(x) \leq \inf_{x\in D} g(x) .\]
    2. Encontrar un específico\(D\),\(f\), y\(g\), tal que\(f(x) \leq g(x)\) para todos\(x \in D\), pero\[\sup_{x\in D} f(x) > \inf_{x\in D} g(x) .\]

    Demostrar sin el supuesto de que las funciones están acotadas. Pista: Necesitas usar los números reales extendidos.

    [exercise:sumofsup] Dejar\(D\) ser un conjunto no vacío. Supongamos\(f \colon D \to {\mathbb{R}}\) y\(g \colon D \to {\mathbb{R}}\) son funciones acotadas. a) Mostrar\[\sup_{x\in D} \bigl(f(x) + g(x) \bigr) \leq \sup_{x\in D} f(x) + \sup_{x\in D} g(x) \qquad \text{and} \qquad \inf_{x\in D} \bigl(f(x) + g(x) \bigr) \geq \inf_{x\in D} f(x) + \inf_{x\in D} g(x) .\] b) Encuentra ejemplos donde obtenemos desigualdades estrictas.

    Intervalos y el tamaño de R

    Nota: 0.5—1 conferencia (prueba de incontabilidad de\({\mathbb{R}}\) puede ser opcional)

    Seguramente viste la notación para intervalos antes, pero vamos a dar aquí una definición formal. Para\(a, b \in {\mathbb{R}}\) tal que\(a < b\) definimos\[\begin{aligned} & [a,b] := \{ x \in {\mathbb{R}}: a \leq x \leq b \}, \\ & (a,b) := \{ x \in {\mathbb{R}}: a < x < b \}, \\ & (a,b] := \{ x \in {\mathbb{R}}: a < x \leq b \}, \\ & [a,b) := \{ x \in {\mathbb{R}}: a \leq x < b \} .\end{aligned}\] El intervalo\([a,b]\) se llama intervalo cerrado y\((a,b)\) se llama intervalo abierto. Los intervalos de la forma\((a,b]\) y\([a,b)\) se denominan intervalos semiabiertos.

    Los intervalos anteriores fueron todos intervalos acotados, ya que ambos\(a\) y\(b\) fueron números reales. Definimos intervalos no acotados,\[\begin{aligned} & [a,\infty) := \{ x \in {\mathbb{R}}: a \leq x \}, \\ & (a,\infty) := \{ x \in {\mathbb{R}}: a < x \}, \\ & (-\infty,b] := \{ x \in {\mathbb{R}}: x \leq b \}, \\ & (-\infty,b) := \{ x \in {\mathbb{R}}: x < b \} .\end{aligned}\] Para completar definimos\((-\infty,\infty) := {\mathbb{R}}\).

    En definitiva, un intervalo es un conjunto\(I \subset {\mathbb{R}}\) con al menos 2 elementos, de tal manera que si\(a < b < c\) y\(a,c \in I\), entonces\(b \in I\). Ver.

    Ya hemos visto que cualquier intervalo abierto\((a,b)\) (donde por\(a < b\) supuesto) debe estar no vacío. Por ejemplo, contiene el número\(\frac{a+b}{2}\). Un hecho inesperado es que desde una perspectiva teórica de conjuntos, todos los intervalos tienen el mismo “tamaño”, es decir, todos tienen la misma cardinalidad. Por ejemplo, el mapa\(f(x) := 2x\) toma el intervalo\([0,1]\) bijectivamente al intervalo\([0,2]\).

    Quizás lo más interesante, la función\(f(x) := \tan(x)\) es un mapa biyective de\((-\nicefrac{\pi}{2},\nicefrac{\pi}{2})\) a\({\mathbb{R}}\). De ahí que el intervalo acotado\((-\nicefrac{\pi}{2},\nicefrac{\pi}{2})\) tenga la misma cardinalidad que\({\mathbb{R}}\). No es del todo sencillo construir un mapa biyective\([0,1]\) a partir de decir\((0,1)\), pero es posible.

    Y no te preocupes, sí existe una manera de medir el “tamaño” de subconjuntos de números reales que “ve” la diferencia entre\([0,1]\) y\([0,2]\). No obstante, su adecuada definición requiere mucha más maquinaria de la que tenemos ahora mismo.

    Digamos más sobre la cardinalidad de los intervalos y por lo tanto sobre la cardinalidad de\({\mathbb{R}}\). Hemos visto que existen números irracionales, es decir, no\({\mathbb{R}}\setminus {\mathbb{Q}}\) está vacío. La pregunta es: ¿Cuántos números irracionales hay? Resulta que hay números mucho más irracionales que números racionales. Hemos visto que\({\mathbb{Q}}\) es contable, y vamos a demostrar que\({\mathbb{R}}\) es incontable. De hecho, la cardinalidad de\({\mathbb{R}}\) es la misma que la cardinalidad de\({\mathcal{P}}({\mathbb{N}})\), aunque aquí no vamos a probar esta afirmación.

    \({\mathbb{R}}\)es incontable.

    Damos una versión modificada de la prueba original de Cantor de 1874 ya que esta prueba requiere la menor configuración. Normalmente esta prueba se afirma como prueba de contradicción, pero una prueba por contrapositiva es más fácil de entender.

    \(X \subset {\mathbb{R}}\)Sea un subconjunto contablemente infinito tal que para dos números reales cualesquiera\(a < b\), haya un\(x \in X\) tal que\(a < x < b\). Eran\({\mathbb{R}}\) contables, entonces podríamos tomar\(X = {\mathbb{R}}\). Si demostramos que\(X\) es necesariamente un subconjunto propio, entonces\(X\) no puede ser igual\({\mathbb{R}}\), y\({\mathbb{R}}\) debe ser incontable.

    Como\(X\) es contablemente infinito, hay una bijección de\({\mathbb{N}}\) a\(X\). En consecuencia, escribimos\(X\) como una secuencia de números reales\(x_1, x_2, x_3,\ldots\), tal que cada número en\(X\) viene dado por\(x_j\) para algunos\(j \in {\mathbb{N}}\).

    Construyamos inductivamente dos secuencias de números reales\(a_1,a_2,a_3,\ldots\) y\(b_1,b_2,b_3,\ldots\). Dejar\(a_1 := x_1\) y\(b_1 := x_1+1\). Tenga en cuenta que\(a_1 < b_1\) y\(x_1 \notin (a_1,b_1)\). Porque\(k > 1\), supongamos\(a_{k-1}\) y se\(b_{k-1}\) ha definido. Supongamos también que\((a_{k-1},b_{k-1})\) no contiene ninguna\(x_j\) para ninguna\(j=1,\ldots,k-1\).

    1. Definir\(a_k := x_j\), dónde\(j\) está el más pequeño\(j \in {\mathbb{N}}\) tal que\(x_j \in (a_{k-1},b_{k-1})\). Tal\(x_j\) existe por nuestra suposición sobre\(X\).
    2. A continuación, definir\(b_k := x_j\) dónde\(j\) está el más pequeño\(j \in {\mathbb{N}}\) tal que\(x_j \in (a_{k},b_{k-1})\).

    Observe que\(a_k < b_k\) y\(a_{k-1} < a_k < b_k < b_{k-1}\). También aviso que\((a_{k},b_{k})\) no contiene\(x_k\) y por lo tanto no contiene ninguno\(x_j\) para\(j=1,\ldots,k\).

    Reclamación:\(a_j < b_k\) para todos\(j\) y\(k\) en\({\mathbb{N}}\). Supongamos primero\(j < k\). Entonces\(a_j < a_{j+1} < \cdots < a_{k-1} < a_k < b_k\). De manera similar para\(j > k\). El reclamo sigue.

    Dejar\(A = \{ a_j : j \in {\mathbb{N}}\}\) y\(B = \{ b_j : j \in {\mathbb{N}}\}\). Por y el reclamo anterior tenemos\[\sup\, A \leq \inf\, B .\] Definir\(y := \sup\, A\). El número\(y\) no puede ser miembro de\(A\). Si\(y = a_j\) para algunos\(j\), entonces\(y < a_{j+1}\), lo cual es imposible. De igual manera\(y\) no puede ser miembro de\(B\). Por lo tanto,\(a_j < y\) para todos\(j\in {\mathbb{N}}\) y\(y < b_j\) para todos\(j\in {\mathbb{N}}\). En otras palabras\(y \in (a_j,b_j)\) para todos\(j\in {\mathbb{N}}\).

    Por último debemos demostrarlo\(y \notin X\). Si lo hacemos, entonces habremos construido un número real no en\(X\) mostrar que\(X\) debió haber sido un subconjunto propio. Toma cualquiera\(x_k \in X\). Por la construcción anterior\(x_k \notin (a_k,b_k)\), así\(x_k \not= y\) como\(y \in (a_k,b_k)\).

    Por lo tanto, la secuencia\(x_1,x_2,\ldots\) no puede contener todos los elementos de\({\mathbb{R}}\) y por lo tanto\({\mathbb{R}}\) es incontable.

    Ejercicios

    Para\(a < b\), construir una bijección explícita de\((a,b]\) a\((0,1]\).

    Supongamos que\(f \colon [0,1] \to (0,1)\) es una biyección. Usando\(f\), construir una bijección de\([-1,1]\) a\({\mathbb{R}}\).

    [exercise:intervaldef] Supongamos que\(I \subset {\mathbb{R}}\) es un subconjunto con al menos 2 elementos tales que si\(a < b < c\) y\(a, c \in I\), entonces es uno de los nueve tipos de intervalos dados explícitamente en esta sección. Además, probar que los intervalos dados en esta sección satisfacen todos esta propiedad.

    Construir una biyección explícita de\((0,1]\) a\((0,1)\). Pista: Un enfoque es el siguiente: Primero mapear\((\nicefrac{1}{2},1]\) a\((0,\nicefrac{1}{2}]\), luego mapear\((\nicefrac{1}{4},\nicefrac{1}{2}]\) a\((\nicefrac{1}{2},\nicefrac{3}{4}]\), etc... Anote el mapa explícitamente, es decir, anote un algoritmo que te diga exactamente qué número va a dónde. Entonces demuestre que el mapa es una biyección.

    Construir una biyección explícita de\([0,1]\) a\((0,1)\).

    a) Mostrar que cada intervalo cerrado\([a,b]\) es la intersección de muchos intervalos abiertos. b) Mostrar que cada intervalo abierto\((a,b)\) es una unión contable de intervalos cerrados. c) Mostrar que una intersección de una familia posiblemente infinita de intervalos cerrados está vacía, un solo punto o una cerrada intervalo.

    Supongamos que\(S\) es un conjunto de intervalos abiertos disjuntos en\({\mathbb{R}}\). Es decir, si\((a,b) \in S\) y\((c,d) \in S\), entonces cualquiera\((a,b) = (c,d)\) o\((a,b) \cap (c,d) = \emptyset\). Probar\(S\) es un conjunto contable.

    Demostrar que la cardinalidad de\([0,1]\) es la misma que la cardinalidad de\((0,1)\) demostrando eso\(\left\lvert {[0,1]} \right\rvert \leq \left\lvert {(0,1)} \right\rvert\) y\(\left\lvert {(0,1)} \right\rvert \leq \left\lvert {[0,1]} \right\rvert\). Ver. Tenga en cuenta que esto requiere el teorema de Cantor-Bernstein-Schroeder que afirmamos sin pruebas. También tenga en cuenta que esta prueba no le da una biyección explícita.

    Un número\(x\) es algebraico si\(x\) es una raíz de un polinomio con coeficientes enteros, en otras palabras,\(a_n x^n + a_{n-1} x^{n-1} + \ldots + a_1 x + a_0 = 0\) donde todos\(a_n \in {\mathbb{Z}}\). a) Demostrar que solo hay contablemente muchos números algebraicos. b) Demostrar que existen números no algebraicos (seguir los pasos de Cantor, utilizar la inccountabilidad de\({\mathbb{R}}\)). Pista: Siéntase libre de usar el hecho de que un polinomio de grado\(n\) tiene como máximo raíces\(n\) reales.

    Representación decimal de los reales

    Nota: 1 conferencia (opcional)

    A menudo pensamos en los números reales como su representación decimal. Para un entero positivo\(n\), encontramos los dígitos\(d_K,d_{K-1},\ldots,d_2,d_1,d_0\) para algunos\(K\), donde cada uno\(d_j\) es un número entero entre\(0\) y\(9\), luego\[n = d_K {10}^K + d_{K-1} {10}^{K-1} + \cdots + d_2 {10}^2 + d_1 10 + d_0 .\] Suponemos a menudo\(d_K \not= 0\). Para representar\(n\) escribimos la secuencia de dígitos:\(n = d_K d_{K-1} \cdots d_2 d_1 d_0\). Por un dígito (decimal), nos referimos a un entero entre\(0\) y\(9\).

    Del mismo modo representamos algunos números racionales. Es decir, para ciertos números\(x\), podemos encontrar entero negativo\(-M\), un entero positivo\(K\), y dígitos\(d_K,d_{K-1},\ldots,d_1,d_0,d_{-1},\ldots,d_{-M}\), tal que\[x = d_K {10}^K + d_{K-1} {10}^{K-1} + \cdots + d_2 {10}^2 + d_1 10 + d_0 + d_{-1} {10}^{-1} + d_{-2} {10}^{-2} + \cdots + d_{-M} {10}^{-M} .\] Nosotros escribimos\(x = d_K d_{K-1} \cdots d_1 d_0 \, . \, d_{-1} d_{-2} \cdots d_{-M}\).

    No todos los números reales tienen tal representación, ni siquiera el número racional simple\(\nicefrac{1}{3}\) no. El número irracional\(\sqrt{2}\) tampoco tiene tal representación. Para obtener una representación para todos los números reales debemos permitir infinitamente muchos dígitos.

    Consideremos a partir de ahora solo números reales en el intervalo\((0,1]\). Si encontramos una representación para estos, simplemente les agregamos enteros para obtener una representación para todos los números reales. Supongamos que tomamos una secuencia infinita de dígitos decimales: Es\[0.d_1d_2d_3\ldots.\] decir, tenemos un dígito\(d_j\) para cada\(j \in {\mathbb{N}}\). Hemos renumerado los dígitos para evitar los signos negativos. Decimos que esta secuencia de dígitos representa un número real\(x\) si\[x = \sup_{n \in {\mathbb{N}}} \left( \frac{d_1}{10} + \frac{d_2}{{10}^2} + \frac{d_3}{{10}^3} + \cdots + \frac{d_n}{{10}^n} \right) .\] llamamos\[D_n := \frac{d_1}{10} + \frac{d_2}{{10}^2} + \frac{d_3}{{10}^3} + \cdots + \frac{d_n}{{10}^n}\] al truncamiento de\(x\) a dígitos\(n\) decimales.

    [prop:decimalprop]

    1. Cada secuencia infinita de dígitos\(0.d_1d_2d_3\ldots\) representa un número real único\(x \in [0,1]\).
    2. Por cada\(x \in (0,1]\) existe una secuencia infinita de dígitos\(0.d_1d_2d_3\ldots\) que representa\(x\). Existe una representación única tal que\[D_n < x \leq D_n+\frac{1}{{10}^n} \qquad \text{for all $n \in {\mathbb{N}}$}.\]

    Empecemos con el primer ítem. Supongamos que hay una secuencia infinita de dígitos\(0.d_1d_2d_3\ldots\). Utilizamos la fórmula de suma geométrica para escribir\[\begin{split} D_n = \frac{d_1}{10} + \frac{d_2}

    ParseError: EOF expected (click for details)
    Callstack:
        at (Matematicas/Analisis/Introducción_al_Análisis_Real_(Lebl)/02:_Números_reales/2.01:_Propiedades_básicas), /content/body/div[7]/div/p[7]/span[2]/span, line 1, column 3
    
    {1-\nicefrac{1}{10}} \right) = 1-{(\nicefrac{1}{10})}^{n} < 1 . \end{split}\] En particular,\(D_n < 1\) para todos\(n\). Como\(D_n \geq 0\) es obvio, obtenemos\[0 \leq \sup_{n\in {\mathbb{N}}} \, D_n \leq 1 ,\] y por lo tanto\(0.d_1d_2d_3\ldots\) representa un número único\(x \in [0,1]\).

    Pasamos al segundo ítem. Tomar\(x \in (0,1]\). Primero abordemos la existencia. Por convención definimos\(D_0 := 0\), luego automáticamente obtenemos\(D_0 < x \leq D_0 + {10}^{-0}\). Supongamos para inducción que definimos todos los dígitos\(d_1,d_2,\ldots,d_n\), y eso\(D_n < x \leq D_n + {10}^{-n}\). Tenemos que definir\(d_{n+1}\).

    Por el de los números reales encontramos un entero\(j\) tal que\(x-D_n \leq j {10}^{-(n+1)}\). Tomamos lo menos así\(j\) y obtenemos\[\label{eq:theDnjineq} (j-1){10}^{-(n+1)} < x-D_n \leq j {10}^{-(n+1)} .\] Let\(d_{n+1} := j-1\). Como\(D_n < x\), entonces\(d_{n+1} = j-1 \geq 0\). Por otro lado ya que\(x-D_n \leq {10}^{-n}\) tenemos eso\(j\) es como máximo 10, y por lo tanto\(d_{n+1} \leq 9\). Entonces\(d_{n+1}\) es un dígito decimal. Ya que\(D_{n+1} = D_n + d_{n+1} {10}^{-(n+1)}\) sumamos\(D_n\) a la desigualdad [EQ:TheDNJineQ] anterior:\[D_{n+1} = D_n + (j-1){10}^{-(n+1)} < x \leq %D_n + j {10}^{-(n+1)} = D_n + (j-1) {10}^{-(n+1)} + {10}^{-(n+1)} = D_{n+1} + {10}^{-(n+1)} .\] Y así se\(D_{n+1} < x \leq D_{n+1} + {10}^{-(n+1)}\) sostiene. Hemos definido inductivamente una secuencia infinita de dígitos\(0.d_1d_2d_3\ldots\). En\(D_n < x\) cuanto a todos\(n\), entonces\(\sup \{ D_n : n \in {\mathbb{N}}\} \leq x\). Como\(x-{10}^{-n} \leq D_n\), entonces\(x-{10}^{-n} \leq \sup \{ D_m : m \in {\mathbb{N}}\}\) para todos\(n\). Las dos desigualdades juntas implican\(\sup \{ D_n : n \in {\mathbb{N}}\} = x\).

    Lo que queda por mostrar es la singularidad. Supongamos que\(0.e_1e_2e_3\ldots\) es otra representación de\(x\). Dejar\(E_n\) ser el truncamiento\(n\) -dígito de\(0.e_1e_2e_3\ldots\), y supongamos\(E_n < x \leq E_n + {10}^{-n}\) para todos\(n \in {\mathbb{N}}\). Supongamos para algunos\(K \in {\mathbb{N}}\),\(e_n = d_n\) para todos\(n < K\), así\(D_{K-1} = E_{K-1}\). Entonces\[E_K = D_{K-1} + e_K{10}^{-K} < x \leq E_K + {10}^{-K} = D_{K-1} + e_K{10}^{-K} + {10}^{-K} .\] restando\(D_{K-1}\) y multiplicando por obtenemos\[e_K < (x - D_{K-1}){10}^K \leq e_K + 1 .\] Del mismo modo\({10}^{K}\) obtenemos De\[d_K < (x - D_{K-1}){10}^K \leq d_K + 1 .\] ahí, ambos\(e_K\) y\(d_K\) son el entero más grande\(j\) tal que\(j < (x - D_{K-1}){10}^K\), y por lo tanto\(e_K = d_K\). Es decir, la representación es única.

    La representación no es única si no requerimos la condición extra en la proposición. Por ejemplo, para el número\(\nicefrac{1}{2}\) el método en la prueba obtiene la representación\[0.49999\ldots .\] Sin embargo, también tenemos la representación\(0.5000\ldots\). El requisito clave que hace que la representación sea única es\(D_n < x\) para todos\(n\). La desigualdad\(x \leq D_n + {10}^{-n}\) es cierta para cada representación por el cómputo en el inicio de la prueba.

    Los únicos números que tienen representaciones no únicas son los que terminan ya sea en una secuencia infinita de\(0\) s o\(9\) s, porque la única representación para la cual\(D_n = x\) es aquella donde todos los dígitos pasados\(n\) th uno son cero. En este caso hay exactamente dos representaciones de\(x\) (ver los ejercicios).

    Demos otra prueba de la incontabilidad de los reales usando representaciones decimales. Esta es la segunda prueba de Cantor, y probablemente sea más conocida. Si bien esta prueba puede parecer más corta, es porque ya hemos hecho la parte dura anterior y nos queda un truco hábil para demostrar que\({\mathbb{R}}\) es incontable. Este truco se llama diagonalización Cantor y encuentra uso en otras pruebas también.

    El conjunto\((0,1]\) es incontable.

    Dejar\(X := \{ x_1,x_2,x_3,\ldots \}\) ser cualquier subconjunto contable de números reales en\((0,1]\). Vamos a construir un número real no en\(X\). \[x_n = 0.d_1^nd_2^nd_3^n\ldots\]Sea la representación única de la proposición,\(d_j^n\) es decir, el dígito\(j\) th del número\(n\) th. Deja\(e_n := 1\) si\(d_n^n \not= 1\), y deja\(e_n := 2\) si\(d_n^n = 1\). Dejar\(E_n\) ser el truncamiento\(n\) -dígito de\(y = 0.e_1e_2e_3\ldots\). Porque todos los dígitos son distintos de cero lo conseguimos\(E_n < E_{n+1} \leq y\). Por lo tanto\[E_n < y \leq E_n + {10}^{-n}\] para todos\(n\), y la representación es la única para\(y\) desde la proposición. Pero para cada\(n\), el dígito\(n\) th de\(y\) es diferente del dígito\(n\) th de\(x_n\), entonces\(y \not= x_n\). Por lo tanto\(y \notin X\), y como\(X\) era un subconjunto contable arbitrario,\((0,1]\) debe ser incontable.

    Usando dígitos decimales también podemos encontrar muchos números que no son racionales. La siguiente proposición es cierta para cada número racional, pero la damos sólo\(x \in (0,1]\) por simplicidad.

    [prop:rationaldecimal] Si\(x \in (0,1]\) es un número racional y\(x = 0.d_1d_2d_3\ldots\), entonces los dígitos decimales eventualmente comienzan a repetirse. Es decir, hay enteros positivos\(N\) y\(P\), tal que para todos\(n \geq N\),\(d_n = d_{n+P}\).

    Dejar\(x = \nicefrac{p}{q}\) para enteros positivos\(p\) y\(q\). Supongamos que\(x\) es un número con una representación única, ya que de lo contrario hemos visto anteriormente que ambas representaciones se están repitiendo.

    Para calcular el primer dígito tomamos\(10 p\) y dividimos por\(q\). El cociente es el primer dígito\(d_1\) y el resto\(r\) es algún número entero entre 0 y\(q-1\). Es decir,\(d_1\) es el entero más grande tal que\(d_1 q \leq 10p\) y luego\(r = 10p - d_1q\).

    El siguiente dígito se calcula dividiendo\(10 r\) por\(q\), y así sucesivamente. Notamos que en cada paso hay como máximo restos\(q\) posibles y de ahí que en algún momento el proceso deba comenzar a repetirse. De hecho vemos que\(P\) es a lo sumo\(q\).

    Lo contrario de la proposición también es cierto y se deja como ejercicio.

    El número\[x = 0.101001000100001000001\ldots,\] es irracional. Es decir, los dígitos son\(n\) ceros, luego un uno, luego\(n+1\) ceros, luego uno, y así sucesivamente y así sucesivamente. El hecho de que\(x\) sea irracional se desprende de la proposición; los dígitos nunca empiezan a repetirse. Por cada\(P\), si vamos lo suficientemente lejos, nos encontramos con un 1 que es seguido por al menos\(P+1\) ceros.

    Ejercicios

    ¿Cuál es la representación decimal de\(1\) garantizada por? Asegúrese de demostrar que sí satisface la condición.

    Demostrar lo contrario de, es decir, si los dígitos en la representación decimal de finalmente\(x\) se repiten, entonces\(x\) debe ser racional.

    Mostrar que los números reales\(x \in (0,1)\) con representación decimal no única son exactamente los números racionales que se pueden escribir como\(\frac{m}{10^n}\) para algunos enteros\(m\) y\(n\). En este caso muestran que existen exactamente dos representaciones de\(x\).

    Dejar\(b \geq 2\) ser un entero. Definir una representación de un número real\([0,1]\) en términos de base en\(b\) lugar de base 10 y probar para base\(b\).

    Utilizando el ejercicio anterior con\(b=2\) (binario), mostrar que la cardinalidad de\({\mathbb{R}}\) es la misma que la cardinalidad de\({\mathcal{P}}({\mathbb{N}})\), obteniendo otra prueba (aunque relacionada) que\({\mathbb{R}}\) es incontable. Consejo: Construir dos inyecciones, una de\([0,1]\) a\({\mathcal{P}}({\mathbb{N}})\) y otra de\({\mathcal{P}}({\mathbb{N}})\) a\([0,1]\). Pista 2: Dado un conjunto\(A \subset {\mathbb{N}}\), deje que el dígito binario\(n\) th de\(x\) sea 1 si\(n\in A\).

    Construir una biyección entre\([0,1]\) y\([0,1] \times [0,1]\). Pista: considere los dígitos pares e impares, y tenga cuidado con la singularidad de la representación.

    Secuencias y series

    Secuencias y límites

    Nota: 2.5 conferencias

    El análisis consiste esencialmente en tomar límites. El tipo más básico de un límite es un límite de una secuencia de números reales. Ya hemos visto secuencias utilizadas de manera informal. Demos la definición formal.

    Una secuencia (de números reales) es una función\(x \colon {\mathbb{N}}\to {\mathbb{R}}\). En lugar de\(x(n)\) que usualmente denotamos el elemento\(n\) th en la secuencia por\(x_n\). Usamos la notación\(\{ x_n \}\), o más precisamente\[\{ x_n \}_{n=1}^\infty,\] para denotar una secuencia.

    Una secuencia\(\{ x_n \}\) está delimitada si existe\(B \in {\mathbb{R}}\) tal que\[\left\lvert {x_n} \right\rvert \leq B \qquad \text{for all $n \in {\mathbb{N}}$.}\] En otras palabras, la secuencia\(\{x_n\}\) se limita siempre que el conjunto\(\{ x_n : n \in {\mathbb{N}}\}\) esté acotado.

    Cuando necesitamos dar una secuencia concreta a menudo damos cada término como una fórmula en términos de\(n\). Por ejemplo\(\{ \nicefrac{1}{n} \}_{n=1}^\infty\), o simplemente\(\{ \nicefrac{1}{n} \}\), significa la secuencia\(1, \nicefrac{1}{2}, \nicefrac{1}{3}, \nicefrac{1}{4}, \nicefrac{1}{5}, \ldots\). La secuencia\(\{ \nicefrac{1}{n} \}\) es una secuencia acotada (\(B=1\)bastará). Por otro lado la secuencia\(\{ n \}\) representa\(1,2,3,4,\ldots\), y esta secuencia no está acotada (¿por qué?).

    Mientras que la notación para una secuencia es similar 7 a la de un conjunto, las nociones son distintas. Por ejemplo, la secuencia\(\{ {(-1)}^n \}\) es la secuencia\(-1,1,-1,1,-1,1,\ldots\), mientras que el conjunto de valores, el rango de la secuencia, es solo el conjunto\(\{ -1, 1 \}\). Podemos escribir este conjunto como\(\{ {(-1)}^n : n \in {\mathbb{N}}\}\). Cuando puede surgir ambigüedad, utilizamos las palabras secuencia o conjunto para distinguir los dos conceptos.

    Otro ejemplo de una secuencia es la llamada secuencia constante. Esa es una secuencia que\(\{ c \} = c,c,c,c,\ldots\) consiste en una sola constante que se\(c \in {\mathbb{R}}\) repite indefinidamente.

    Ahora llegamos a la idea de un límite de una secuencia. Veremos en que la notación a continuación está bien definida. Es decir, si existe un límite, entonces es único. Entonces tiene sentido hablar del límite de una secuencia.

    \(\{ x_n \}\)Se dice que una secuencia converge a un número\(x \in {\mathbb{R}}\), si por cada\(\epsilon > 0\), existe un\(M \in {\mathbb{N}}\) tal que\(\left\lvert {x_n - x} \right\rvert < \epsilon\) para todos\(n \geq M\). Se dice que el número\(x\) es el límite de\(\{ x_n \}\). Escribimos\[\lim_{n\to \infty} x_n := x .\]

    Se dice que una secuencia que converge es convergente. De lo contrario, se dice que la secuencia es divergente.

    Es bueno saber intuitivamente lo que significa un límite. Significa que eventualmente cada número en la secuencia está cerca del número\(x\). Más precisamente, podemos acercarnos arbitrariamente al límite, siempre que vayamos lo suficientemente lejos en la secuencia. No significa que lleguemos alguna vez al límite. Es posible, y bastante común, que no haya\(x_n\) en la secuencia que iguale al límite\(x\). Ilustramos el concepto en. En la figura primero pensamos en la secuencia como una gráfica, ya que es una función de\({\mathbb{N}}\). En segundo lugar también lo trazamos como una secuencia de puntos etiquetados en la línea real.

    Cuando escribimos\(\lim\, x_n = x\) para algún número real\(x\), estamos diciendo dos cosas. Primero, eso\(\{ x_n \}\) es convergente, y segundo que el límite es\(x\).

    La definición anterior es una de las definiciones más importantes en el análisis, y es necesario entenderla perfectamente. El punto clave en la definición es que dado cualquiera\(\epsilon > 0\), podemos encontrar un\(M\). El\(M\) puede depender\(\epsilon\), así que solo elegimos una\(M\) vez que conocemos\(\epsilon\). Ilustremos este concepto con algunos ejemplos.

    La secuencia constante\(1,1,1,1,\ldots\) es convergente y el límite es 1. Para cada\(\epsilon > 0\), elegimos\(M = 1\).

    Reclamación: La secuencia\(\{ \nicefrac{1}{n} \}\) es convergente y\[\lim_{n\to \infty} \frac{1}{n} = 0 .\] Prueba: Dada una\(\epsilon > 0\), encontramos una\(M \in {\mathbb{N}}\) tal que\(0 < \nicefrac{1}{M} < \epsilon\) (en el trabajo). Entonces por todos\(n \geq M\) tenemos eso\[\left\lvert {x_n - 0} \right\rvert = \left\lvert {\frac{1}{n}} \right\rvert = \frac{1}{n} \leq \frac{1}{M} < \epsilon .\]

    La secuencia\(\{ {(-1)}^n \}\) es divergente. Prueba: Si hubiera un límite\(x\), entonces para\(\epsilon = \frac{1}{2}\) nosotros esperamos una\(M\) que satisfaga la definición. Supongamos que tal\(M\) existe, entonces para un par\(n \geq M\) calculamos\[\nicefrac{1}{2} > \left\lvert {x_n - x} \right\rvert = \left\lvert {1 - x} \right\rvert \qquad \text{and} \qquad \nicefrac{1}{2} > \left\lvert {x_{n+1} - x} \right\rvert = \left\lvert {-1 - x} \right\rvert .\] Pero\[2 = \left\lvert {1 - x - (-1 -x)} \right\rvert \leq \left\lvert {1 - x} \right\rvert + \left\lvert {-1 -x} \right\rvert < \nicefrac{1}{2} + \nicefrac{1}{2} = 1 ,\] y eso es una contradicción.

    [prop:limisunique] Una secuencia convergente tiene un límite único.

    La prueba de esta proposición exhibe una técnica útil en el análisis. Muchas pruebas siguen el mismo esquema general. Queremos mostrar que cierta cantidad es cero. Escribimos la cantidad usando la desigualdad del triángulo como dos cantidades, y estimamos cada una por números arbitrariamente pequeños.

    Supongamos que la secuencia\(\{ x_n \}\) tiene el límite\(x\) y el límite\(y\). Toma una arbitraria\(\epsilon > 0\). De la definición encontrar un\(M_1\) tal que para todos\(n \geq M_1\),\(\left\lvert {x_n-x} \right\rvert < \nicefrac{\epsilon}{2}\). De igual manera encontramos un\(M_2\) tal que por todo\(n \geq M_2\) lo que tenemos\(\left\lvert {x_n-y} \right\rvert < \nicefrac{\epsilon}{2}\). Tomar\(M := \max \{M_1, M_2\}\). Para\(n \geq M\) (para que ambos\(n \geq M_1\) y\(n \geq M_2\)) tengamos\[\begin{split} \left\lvert {y-x} \right\rvert & = \left\lvert {x_n-x - (x_n -y)} \right\rvert \\ & \leq \left\lvert {x_n-x} \right\rvert + \left\lvert {x_n -y} \right\rvert \\ & < \frac{\epsilon}{2} + \frac{\epsilon}{2} = \epsilon . \end{split}\] Como\(\left\lvert {y-x} \right\rvert < \epsilon\) para todos\(\epsilon > 0\), entonces\(\left\lvert {y-x} \right\rvert = 0\) y\(y=x\). De ahí que el límite (si existe) sea único.

    Una secuencia convergente\(\{ x_n \}\) está delimitada.

    Supongamos que\(\{ x_n \}\) converge a\(x\). Así existe\(M \in {\mathbb{N}}\) tal que por todo\(n \geq M\) lo que tenemos\(\left\lvert {x_n - x} \right\rvert < 1\). Vamos\(B_1 := \left\lvert {x} \right\rvert+1\) y notemos que para\(n \geq M\) nosotros tenemos\[\begin{split} \left\lvert {x_n} \right\rvert & = \left\lvert {x_n - x + x} \right\rvert \\ & \leq \left\lvert {x_n - x} \right\rvert + \left\lvert {x} \right\rvert \\ & < 1 + \left\lvert {x} \right\rvert = B_1 . \end{split}\] El conjunto\(\{ \left\lvert {x_1} \right\rvert, \left\lvert {x_2} \right\rvert, \ldots, \left\lvert {x_{M-1}} \right\rvert \}\) es un conjunto finito y por lo tanto\[B_2 := \max \{ \left\lvert {x_1} \right\rvert, \left\lvert {x_2} \right\rvert, \ldots, \left\lvert {x_{M-1}} \right\rvert \} .\] let Let\(B := \max \{ B_1, B_2 \}\). Entonces por todo\(n \in {\mathbb{N}}\) lo que tenemos\[\left\lvert {x_n} \right\rvert \leq B. \qedhere\]

    La secuencia\(\{ {(-1)}^n \}\) muestra que lo contrario no se sostiene. Una secuencia acotada no es necesariamente convergente.

    Vamos a mostrar la secuencia\(\left\{ \frac{n^2+1}{n^2+n} \right\}\) converge y\[\lim_{n\to\infty} \frac{n^2+1}{n^2+n} = 1 .\]

    Dado\(\epsilon > 0\), encuentra\(M \in {\mathbb{N}}\) tal que\(\frac{1}{M+1} < \epsilon\). Entonces para cualquiera\(n \geq M\) tenemos\[\begin{split} %\abs{\frac{n^2+1}{n^2+n} - 1} & = %\abs{\frac{n^2+1 - (n^2+n)}{n^2+n}} \\ \left\lvert {\frac{n^2+1}{n^2+n} - 1} \right\rvert = \left\lvert {\frac{n^2+1 - (n^2+n)}{n^2+n}} \right\rvert & = \left\lvert {\frac{1 - n}{n^2+n}} \right\rvert \\ & = \frac{n-1}{n^2+n} \\ & \leq \frac{n}{n^2+n} = \frac{1}{n+1} \\ & \leq \frac{1}{M+1} < \epsilon . \end{split}\] Por lo tanto,\(\lim \frac{n^2+1}{n^2+n} = 1\).

    Secuencias monótona

    El tipo más simple de una secuencia es una secuencia monótona. Comprobar que una secuencia monótona converge es tan fácil como verificar que esté delimitada. También es fácil encontrar el límite para una secuencia monótona convergente, siempre que podamos encontrar el supremo o infimum de un conjunto contable de números.

    Una secuencia\(\{ x_n \}\) es monótona aumentando si es\(x_n \leq x_{n+1}\) para todos\(n \in {\mathbb{N}}\). Una secuencia\(\{ x_n \}\) es monótona decreciente si es\(x_n \geq x_{n+1}\) para todos\(n \in {\mathbb{N}}\). Si una secuencia es monótona creciente o monótona decreciente, simplemente podemos decir que la secuencia es monótona. Algunos autores también utilizan la palabra monotónico.

    Por ejemplo,\(\{ \nicefrac{1}{n} \}\) es monótona decreciente, la secuencia constante\(\{ 1 \}\) es tanto monótona creciente como monótona decreciente, y no\(\{ {(-1)}^n \}\) es mon Los primeros términos de una secuencia monótona creciente muestra se muestran en.

    [thm:monotoneconv] Una secuencia monótona\(\{ x_n \}\) se limita si y sólo si es convergente.

    Además, si\(\{ x_n \}\) es monótona creciente y acotada, entonces\[\lim_{n\to \infty} x_n = \sup \{ x_n : n \in {\mathbb{N}}\} .\] Si\(\{ x_n \}\) es monótona decreciente y acotada, entonces\[\lim_{n\to \infty} x_n = \inf \{ x_n : n \in {\mathbb{N}}\} .\]

    Supongamos que la secuencia es monótona creciente. Supongamos que la secuencia está acotada, entonces existe\(B\) tal que\(x_n \leq B\) para todos\(n\), es decir, el conjunto\(\{ x_n : n \in {\mathbb{N}}\}\) está acotado desde arriba. \[x := \sup \{ x_n : n \in {\mathbb{N}}\} .\]Dejemos\(\epsilon > 0\) ser arbitrarios. Como\(x\) es el supremo, entonces debe haber por lo menos uno\(M \in {\mathbb{N}}\) tal que\(x_{M} > x-\epsilon\) (porque\(x\) es el supremo). Como\(\{ x_n \}\) es monótono aumentando, entonces es fácil ver (por) eso\(x_n \geq x_{M}\) para todos\(n \geq M\). De ahí\[\left\lvert {x_n-x} \right\rvert = x-x_n \leq x-x_{M} < \epsilon .\] Por lo tanto la secuencia converge a\(x\). Ya sabemos que se delimita una secuencia convergente, lo que completa la otra dirección de la implicación.

    La prueba de secuencias decrecientes monótonas se deja como ejercicio.

    Toma la secuencia\(\{ \frac{1}{\sqrt{n}} \}\).

    Primero\(\frac{1}{\sqrt{n}} > 0\) para todo\(n \in {\mathbb{N}}\), y de ahí la secuencia está delimitada desde abajo. Demostremos que es monótona decreciente. Empezamos con\(\sqrt{n+1} \geq \sqrt{n}\) (¿por qué es eso cierto?). De esta desigualdad obtenemos\[\frac{1}{\sqrt{n+1}} \leq \frac{1}{\sqrt{n}} .\] Así que la secuencia es monótona decreciente y acotada desde abajo (de ahí acotada). Aplicamos el teorema para señalar que la secuencia es convergente y de hecho ya\[\lim_{n\to \infty} \frac{1}{\sqrt{n}} = \inf \left\{ \frac{1}{\sqrt{n}} : n \in {\mathbb{N}}\right\} .\] sabemos que el infimum es mayor o igual a 0, ya que 0 es un límite inferior. Toma un número\(b \geq 0\) tal que\(b \leq \frac{1}{\sqrt{n}}\) para todos\(n\). Cuadramos ambos lados para obtener\[b^2 \leq \frac{1}{n} \qquad \text{for all $n \in {\mathbb{N}}$}.\] Hemos visto antes que esto implica que\(b^2 \leq 0\) (una consecuencia de la). Como también lo hemos hecho\(b^2 \geq 0\), entonces\(b^2 = 0\) y así\(b = 0\). De ahí\(b=0\) es el mayor límite inferior, y\(\lim \frac{1}{\sqrt{n}} = 0\).

    Una palabra de precaución: Debemos demostrar que una secuencia monótona está acotada para poder usarla. Por ejemplo, la secuencia\(\{ 1 + \nicefrac{1}{2} + \cdots + \nicefrac{1}{n} \}\) es una secuencia monótona creciente que crece muy lentamente. Veremos, una vez lleguemos a la serie, que esta secuencia no tiene límite superior y por lo tanto no converge. No es en absoluto obvio que esta secuencia no tenga límite superior.

    Un ejemplo común de dónde surgen las secuencias monótonas es la siguiente proposición. La prueba se deja como ejercicio.

    [prop:supinfseq] Dejar\(S \subset {\mathbb{R}}\) ser un conjunto delimitado no vacío. Luego existen secuencias\(\{ x_n \}\) monótonas y\(\{ y_n \}\) tal que\(x_n, y_n \in S\) y\[\sup\,S = \lim_{n\to \infty} x_n \qquad \text{and} \qquad \inf\,S = \lim_{n\to\infty} y_n .\]

    Cola de una secuencia

    Para una secuencia\(\{ x_n \}\), la \(K\)-cola (donde\(K \in {\mathbb{N}}\)) o simplemente la cola de la secuencia es la secuencia que comienza en\(K+1\), generalmente escrita como\[\{ x_{n+K} \}_{n=1}^\infty \qquad \text{or} \qquad \{ x_n \}_{n=K+1}^\infty .\]

    El resultado principal sobre la cola de una secuencia es la siguiente proposición.

    Dejar\(\{ x_n \}_{n=1}^\infty\) ser una secuencia. Entonces las siguientes declaraciones son equivalentes:

    1. [prop:ktail:i] La secuencia\(\{ x_n \}_{n=1}^\infty\) converge.
    2. [prop:ktail:ii] La\(K\) -cola\(\{ x_{n+K} \}_{n=1}^\infty\) converge para todos\(K \in {\mathbb{N}}\).
    3. [prop:ktail:iii] La\(K\) -cola\(\{ x_{n+K} \}_{n=1}^\infty\) converge para algunos\(K \in {\mathbb{N}}\).

    Además, si existe alguno (y por lo tanto todos) de los límites, entonces para cualquier\(K \in {\mathbb{N}}\)\[\lim_{n\to \infty} x_n = \lim_{n \to \infty} x_{n+K} .\]

    Es claro que [prop:ktail:ii] implica [prop:ktail:iii]. Por lo tanto mostraremos primero que [prop:ktail:i] implica [prop:ktail:ii], y luego mostraremos que [prop:ktail:iii] implica [prop:ktail:i]. En el proceso también mostraremos que los límites son iguales.

    Empecemos con [prop:ktail:i] implica [prop:ktail:ii]. Supongamos que\(\{x_n \}\) converge a algunos\(x \in {\mathbb{R}}\). Que\(K \in {\mathbb{N}}\) sea arbitrario. Definir\(y_n := x_{n+K}\), deseamos mostrar que\(\{ y_n \}\) converge a\(x\). Es decir, dada una\(\epsilon > 0\), existe\(M \in {\mathbb{N}}\) tal que\(\left\lvert {x-x_n} \right\rvert < \epsilon\) para todos\(n \geq M\). Tenga en cuenta que\(n \geq M\) implica\(n+K \geq M\). Por lo tanto, es cierto que para todos\(n \geq M\) tenemos eso\[\left\lvert {x-y_n} \right\rvert = \left\lvert {x-x_{n+K}} \right\rvert < \epsilon .\] Por lo tanto\(\{ y_n \}\) converge a\(x\).

    Pasemos a [prop:ktail:iii] implica [prop:ktail:i]. \(K \in {\mathbb{N}}\)Déjese dar, definir\(y_n := x_{n+K}\), y supongamos que eso\(\{ y_n \}\) converge\(x \in {\mathbb{R}}\). Es decir, dada una\(\epsilon > 0\), existe\(M' \in {\mathbb{N}}\) tal que\(\left\lvert {x-y_n} \right\rvert < \epsilon\) para todos\(n \geq M'\). Vamos\(M := M'+K\). Entonces\(n \geq M\) implica\(n-K \geq M'\). Así, siempre que\(n \geq M\) tengamos\[\left\lvert {x-x_n} \right\rvert = \left\lvert {x-y_{n-K}} \right\rvert < \epsilon.\] Por lo tanto\(\{ x_n \}\) converge a\(x\).

    Esencialmente, al límite no le importa cómo comienza la secuencia, solo le importa la cola de la secuencia. Es decir, el inicio de la secuencia puede ser arbitrario.

    Por ejemplo, la secuencia definida por\(x_n := \frac{n}{n^2+16}\) es decreciente si empezamos en\(n=4\) (está aumentando antes). Es decir,\(\{ x_n \} = \nicefrac{1}{17}, \nicefrac{1}{10}, \nicefrac{3}{25}, \nicefrac{1}{8}, \nicefrac{5}{41}, \nicefrac{3}{26}, \nicefrac{7}{65}, \nicefrac{1}{10}, \nicefrac{9}{97}, \nicefrac{5}{58},\ldots\), y\[\nicefrac{1}{17} < \nicefrac{1}{10} < \nicefrac{3}{25} < \nicefrac{1}{8} > \nicefrac{5}{41} > \nicefrac{3}{26} > \nicefrac{7}{65} > \nicefrac{1}{10} > \nicefrac{9}{97} > \nicefrac{5}{58} > \ldots .\] Eso es si tiramos a la basura los 3 primeros términos y miramos la cola 3 está disminuyendo. La prueba se deja como ejercicio. Dado que la cola 3 es monótona y está delimitada por debajo de cero, es convergente, y por lo tanto la secuencia es convergente.

    Subsecuencias

    Un concepto muy útil relacionado con las secuencias es el de una subsecuencia. Una subsecuencia de\(\{ x_n \}\) es una secuencia que contiene solo algunos de los números de\(\{ x_n \}\) en el mismo orden.

    Dejar\(\{ x_n \}\) ser una secuencia. Dejar\(\{ n_i \}\) ser una secuencia estrictamente creciente de números naturales (es decir\(n_1 < n_2 < n_3 < \cdots\)). La secuencia\[\{ x_{n_i} \}_{i=1}^\infty\] se llama una subsecuencia de\(\{ x_n \}\).

    Por ejemplo, tomemos la secuencia\(\{ \nicefrac{1}{n} \}\). La secuencia\(\{ \nicefrac{1}{3n} \}\) es una subsecuencia. Para ver cómo encajan estas dos secuencias en la definición, toma\(n_i := 3i\). Los números en la subsecuencia deben provenir de la secuencia original, por lo que no\(1,0,\nicefrac{1}{3},0, \nicefrac{1}{5},\ldots\) es una subsecuencia de\(\{ \nicefrac{1}{n} \}\). Del mismo modo se debe conservar el orden, por lo que la secuencia no\(1,\nicefrac{1}{3},\nicefrac{1}{2},\nicefrac{1}{5},\ldots\) es una subsecuencia de\(\{ \nicefrac{1}{n} \}\).

    Una cola de una secuencia es un tipo especial de una subsecuencia. Para una subsecuencia arbitraria, tenemos la siguiente proposición sobre la convergencia.

    [prop:seqtosubseq] Si\(\{ x_n \}\) es una secuencia convergente, entonces cualquier subsecuencia también\(\{ x_{n_i} \}\) es convergente y\[\lim_{n\to \infty} x_n = \lim_{i\to \infty} x_{n_i} .\]

    Supongamos\(\lim_{n\to \infty} x_n = x\). Eso significa que por cada\(\epsilon > 0\) tenemos un\(M \in {\mathbb{N}}\) tal que para todos No es\(n \geq M\)\[\left\lvert {x_n - x} \right\rvert < \epsilon .\] difícil de probar (¡hazlo!) por eso\(n_i \geq i\). De ahí\(i \geq M\) implica\(n_i \geq M\). Así, por todo\(i \geq M\) lo que tenemos\[\left\lvert {x_{n_i} - x} \right\rvert < \epsilon ,\] y estamos hechos.

    La existencia de una subsecuencia convergente no implica convergencia de la secuencia misma. Toma la secuencia\(0,1,0,1,0,1,\ldots\). Es decir,\(x_n = 0\) si\(n\) es impar, y\(x_n = 1\) si\(n\) es par. La secuencia\(\{ x_n \}\) es divergente, sin embargo, la subsecuencia\(\{ x_{2n} \}\) converge a 1 y la subsecuencia\(\{ x_{2n+1} \}\) converge a 0. Comparar.

    Ejercicios

    En los siguientes ejercicios, siéntete libre de usar lo que sabes del cálculo para encontrar el límite, si existe. Pero debes probar que encontraste el límite correcto, o probar que la serie es divergente.

    ¿Está\(\{ 3n \}\) delimitada la secuencia? Demostrar o desacreditar.

    ¿La secuencia es\(\{ n \}\) convergente? Si es así, ¿cuál es el límite?

    ¿La secuencia es\(\left\{ \dfrac{{(-1)}^n}{2n} \right\}\) convergente? Si es así, ¿cuál es el límite?

    ¿La secuencia es\(\{ 2^{-n} \}\) convergente? Si es así, ¿cuál es el límite?

    ¿La secuencia es\(\left\{ \dfrac{n}{n+1} \right\}\) convergente? Si es así, ¿cuál es el límite?

    ¿La secuencia es\(\left\{ \dfrac{n}{n^2+1} \right\}\) convergente? Si es así, ¿cuál es el límite?

    [ejercicio:absconv] Dejar\(\{ x_n \}\) ser una secuencia.

    1. Mostrar que\(\lim\, x_n = 0\) (es decir, el límite existe y es cero) si y solo si\(\lim \left\lvert {x_n} \right\rvert = 0\).
    2. Encuentra un ejemplo tal que\(\{ \left\lvert {x_n} \right\rvert \}\) converja y\(\{ x_n \}\) diverja.

    ¿La secuencia es\(\left\{ \dfrac{2^n}{n!} \right\}\) convergente? Si es así, ¿cuál es el límite?

    Demuestre que la secuencia\(\left\{ \dfrac{1}{\sqrt[3]{n}} \right\}\) es monótona, acotada y utilízala para encontrar el límite.

    Demuestre que la secuencia\(\left\{ \dfrac{n+1}{n} \right\}\) es monótona, acotada y utilízala para encontrar el límite.

    Terminar la prueba de secuencias decrecientes monótona.

    Demostrar.

    Dejar\(\{ x_n \}\) ser una secuencia monótona convergente. Supongamos que existe\(k \in {\mathbb{N}}\) tal que\[\lim_{n\to \infty} x_n = x_k .\] Mostrar eso\(x_n = x_k\) para todos\(n \geq k\).

    Encuentra una subsecuencia convergente de la secuencia\(\{ {(-1)}^n \}\).

    Let\(\{x_n\}\) Ser una secuencia definida por\[x_n := \begin{cases} n & \text{if $n$ is odd} , \\ \nicefrac{1}{n} & \text{if $n$ is even} . \end{cases}\]

    1. ¿Está delimitada la secuencia? (probar o desacreditar)
    2. ¿Existe una subsecuencia convergente? Si es así, encuéntralo.

    Dejar\(\{ x_n \}\) ser una secuencia. Supongamos que hay dos subsecuencias convergentes\(\{ x_{n_i} \}\) y\(\{ x_{m_i} \}\). Supongamos\[\lim_{i\to\infty} x_{n_i} = a \qquad \text{and} \qquad \lim_{i\to\infty} x_{m_i} = b,\] dónde\(a \not= b\). Demostrar que no\(\{ x_n \}\) es convergente, sin usar.

    Encuentra una secuencia\(\{ x_n \}\) tal que para cualquiera\(y \in {\mathbb{R}}\), existe una subsecuencia\(\{ x_{n_i} \}\) convergente a\(y\).

    Dejar\(\{ x_n \}\) ser una secuencia y\(x \in {\mathbb{R}}\). Supongamos que para cualquiera\(\epsilon > 0\), hay\(M\) tal que para todos\(n \geq M\),\(\left\lvert {x_n-x} \right\rvert \leq \epsilon\). \(\lim\, x_n = x\)Demuéstralo.

    Que\(\{ x_n \}\) sea una secuencia y\(x \in {\mathbb{R}}\) tal que exista\(k \in {\mathbb{N}}\) tal que para todos\(n \geq k\),\(x_n = x\). Demostrar que\(\{ x_n \}\) converge a\(x\).

    Dejar\(\{ x_n \}\) ser una secuencia y definir una secuencia\(\{ y_n \}\) por\(y_{2k} := x_{k^2}\) y\(y_{2k-1} = x_k\) para todos\(k \in {\mathbb{N}}\). Demostrar que\(\{ x_n \}\) converge si y sólo si\(\{ y_n \}\) converge. Además, demostrar que si convergen, entonces\(\lim\, x_n = \lim\, y_n\).

    Mostrar que la cola 3 de la secuencia definida por\(x_n := \frac{n}{n^2+16}\) es monótona decreciente. Pista: Supongamos\(n \geq m \geq 4\) y consideremos el numerador de la expresión\(x_n-x_m\).

    Supongamos que\(\{ x_n \}\) es una secuencia tal que las subsecuencias\(\{ x_{2n} \}\)\(\{ x_{2n-1} \}\),, y\(\{ x_{3n} \}\) todas convergen. Demostrar que\(\{ x_n \}\) es convergente.

    Datos sobre los límites de las secuencias

    Nota: 2—2.5 conferencias, secuencias definidas recursivamente se pueden omitir de forma segura

    En esta sección repasamos algunos resultados básicos sobre los límites de las secuencias. Comenzamos por observar cómo las secuencias interactúan con las desigualdades.

    Límites y desigualdades

    Un lema básico sobre límites y desigualdades es el llamado lema squeeze. Permite mostrar convergencia de secuencias en casos difíciles si encontramos otras dos secuencias convergentes más simples que “aprietan” la secuencia original.

    [squeeze:lemma] Dejar\(\{ a_n \}\),\(\{ b_n \}\), y\(\{ x_n \}\) ser secuencias tales que\[a_n \leq x_n \leq b_n \quad \text{ for all $n \in {\mathbb{N}}$} .\] Supongamos\(\{ a_n \}\) y\(\{ b_n \}\) convergen y\[\lim_{n\to \infty} a_n = \lim_{n\to \infty} b_n .\] Entonces\(\{ x_n \}\) converge y\[\lim_{n\to \infty} x_n = \lim_{n\to \infty} a_n = \lim_{n\to \infty} b_n .\]

    La idea intuitiva de la prueba se ilustra en. Si\(x\) es el límite de\(a_n\) y\(b_n\), entonces si ambos están dentro\(\nicefrac{\epsilon}{3}\) de\(x\), entonces la distancia entre\(a_n\) y\(b_n\) es como mucho\(\nicefrac{2\epsilon}{3}\). Como\(x_n\) es entre\(a_n\) y\(b_n\) es a lo sumo\(\nicefrac{2\epsilon}{3}\) de\(a_n\). Dado que\(a_n\) está a lo sumo\(\nicefrac{\epsilon}{3}\) lejos de\(x\), entonces\(x_n\) debe estar a lo sumo\(\epsilon\) lejos de\(x\). Sigamos rigurosamente esta intuición.

    Vamos\(x := \lim\, a_n = \lim\, b_n\). Dejemos\(\epsilon > 0\) que se den.

    Encontrar un\(M_1\) tal que por todos\(n \geq M_1\) tenemos eso\(\left\lvert {a_n-x} \right\rvert < \nicefrac{\epsilon}{3}\), y un\(M_2\) tal que para todos\(n \geq M_2\) tenemos\(\left\lvert {b_n-x} \right\rvert < \nicefrac{\epsilon}{3}\). Set\(M := \max \{M_1, M_2 \}\). Supongamos\(n \geq M\). Calculamos\[\begin{split} \left\lvert {x_n - a_n} \right\rvert = x_n-a_n & \leq b_n-a_n \\ & = \left\lvert {b_n - x + x - a_n} \right\rvert \\ & \leq \left\lvert {b_n - x} \right\rvert + \left\lvert {x - a_n} \right\rvert \\ & < \frac{\epsilon}{3} + \frac{\epsilon}{3} = \frac{2\epsilon}{3} . \end{split}\] Armados con esta información estimamos\[\begin{split} \left\lvert {x_n - x} \right\rvert &= \left\lvert {x_n - x + a_n - a_n} \right\rvert \\ &\leq \left\lvert {x_n - a_n} \right\rvert + \left\lvert {a_n - x} \right\rvert \\ & < \frac{2\epsilon}{3} + \frac{\epsilon}{3} = \epsilon . \end{split}\] Y ya terminamos.

    Una aplicación de la es calcular límites de secuencias usando límites que ya se conocen. Por ejemplo, supongamos que tenemos la secuencia\(\{ \frac{1}{n\sqrt{n}} \}\). Ya que\(\sqrt{n} \geq 1\) para todos\(n \in {\mathbb{N}}\), tenemos\[0 \leq \frac{1}{n\sqrt{n}} \leq \frac{1}{n}\] para todos\(n \in {\mathbb{N}}\). Ya lo sabemos\(\lim \nicefrac{1}{n} = 0\). Por lo tanto, usando la secuencia constante\(\{ 0 \}\) y la secuencia\(\{ \nicefrac{1}{n} \}\) en el lema squeeze, concluimos\[\lim_{n\to\infty} \frac{1}{n\sqrt{n}} = 0 .\]

    Los límites también preservan las desigualdades.

    [limandineq:lemma] Dejar\(\{ x_n \}\) y\(\{ y_n \}\) ser secuencias convergentes y\[x_n \leq y_n ,\] para todos\(n \in {\mathbb{N}}\). Entonces\[\lim_{n\to\infty} x_n \leq \lim_{n\to\infty} y_n .\]

    Dejar\(x := \lim\, x_n\) y\(y := \lim\, y_n\). Dejemos\(\epsilon > 0\) que se den. Encuentra un\(M_1\) tal que por todo\(n \geq M_1\) lo que tenemos\(\left\lvert {x_n-x} \right\rvert < \nicefrac{\epsilon}{2}\). Encuentra un\(M_2\) tal que por todo\(n \geq M_2\) lo que tenemos\(\left\lvert {y_n-y} \right\rvert < \nicefrac{\epsilon}{2}\). En particular, para algunos\(n \geq \max\{ M_1, M_2 \}\) tenemos\(x-x_n < \nicefrac{\epsilon}{2}\) y\(y_n-y < \nicefrac{\epsilon}{2}\). Añadimos estas desigualdades para obtener\[y_n-x_n+x-y < \epsilon, \qquad \text{or} \qquad y_n-x_n < y-x+ \epsilon .\] Ya que\(x_n \leq y_n\) tenemos\(0 \leq y_n-x_n\) y por lo tanto\(0 < y-x+ \epsilon\). En otras palabras\[x-y < \epsilon .\] Porque\(\epsilon > 0\) fue arbitrario obtenemos\(x-y \leq 0\), ya que hemos visto que un número no negativo menor que cualquier positivo\(\epsilon\) es cero. Por lo tanto\(x \leq y\).

    Se demuestra un corolario fácil usando secuencias constantes en. La prueba se deja como ejercicio.

    [limandineq:cor]

    1. Que\(\{ x_n \}\) sea una secuencia convergente tal que\(x_n \geq 0\), entonces\[\lim_{n\to\infty} x_n \geq 0.\]
    2. Dejemos\(a,b \in {\mathbb{R}}\) y dejemos\(\{ x_n \}\) ser una secuencia convergente tal que\[a \leq x_n \leq b ,\] para todos\(n \in {\mathbb{N}}\). Entonces\[a \leq \lim_{n\to\infty} x_n \leq b.\]

    En y no podemos simplemente sustituir todas las desigualdades no estrictas por desigualdades estrictas. Por ejemplo, vamos\(x_n := \nicefrac{-1}{n}\) y\(y_n := \nicefrac{1}{n}\). Entonces\(x_n < y_n\),\(x_n < 0\), y\(y_n > 0\) para todos\(n\). Sin embargo, estas desigualdades no son preservadas por la operación límite como lo hemos hecho nosotros\(\lim\, x_n = \lim\, y_n = 0\). La moraleja de este ejemplo es que las desigualdades estrictas pueden convertirse en desigualdades no estrictas cuando se aplican límites; si sabemos\(x_n < y_n\) para todos\(n\), solo podemos concluir\[\lim_{n \to \infty} x_n \leq \lim_{n \to \infty} y_n .\] Este tema es una fuente común de errores.

    Continuidad de las operaciones algebraicas

    Los límites interactúan muy bien con las operaciones algebraicas.

    [prop:contalg] Dejar\(\{ x_n \}\) y\(\{ y_n \}\) ser secuencias convergentes.

    1. [prop:contalg:i] La secuencia\(\{ z_n \}\), donde\(z_n := x_n + y_n\), converge y\[\lim_{n \to \infty} (x_n + y_n) = \lim_{n \to \infty} z_n = \lim_{n \to \infty} x_n + \lim_{n \to \infty} y_n .\]
    2. [prop:contalg:ii] La secuencia\(\{ z_n \}\), donde\(z_n := x_n - y_n\), converge y\[\lim_{n \to \infty} (x_n - y_n) = \lim_{n \to \infty} z_n = \lim_{n \to \infty} x_n - \lim_{n \to \infty} y_n .\]
    3. [prop:contalg:iii] La secuencia\(\{ z_n \}\), donde\(z_n := x_n y_n\), converge y\[\lim_{n \to \infty} (x_n y_n) = \lim_{n \to \infty} z_n = \left( \lim_{n \to \infty} x_n \right) \left( \lim_{n \to \infty} y_n \right) .\]
    4. [prop:contalg:iv] Si\(\lim\, y_n \not= 0\) y\(y_n \not= 0\) para todos\(n \in {\mathbb{N}}\), entonces la secuencia\(\{ z_n \}\), donde\(z_n := \dfrac{x_n}{y_n}\), converge y\[\lim_{n \to \infty} \frac{x_n}{y_n} = \lim_{n \to \infty} z_n = %\frac{\lim_{n \to \infty} x_n}{\lim_{n \to \infty} y_n} . \frac{\lim\, x_n}{\lim\, y_n} .\]

    Empecemos con [prop:contalg:i]. Supongamos\(\{ x_n \}\) y\(\{ y_n \}\) son secuencias convergentes y escribir\(z_n := x_n + y_n\). Vamos\(x := \lim\, x_n\),\(y := \lim\, y_n\), y\(z := x+y\).

    Dejemos\(\epsilon > 0\) que se den. Encuentra un\(M_1\) tal que por todo\(n \geq M_1\) lo que tenemos\(\left\lvert {x_n - x} \right\rvert < \nicefrac{\epsilon}{2}\). Encuentra un\(M_2\) tal que por todo\(n \geq M_2\) lo que tenemos\(\left\lvert {y_n - y} \right\rvert < \nicefrac{\epsilon}{2}\). Tomar\(M := \max \{ M_1, M_2 \}\). Por todo\[\begin{split} \left\lvert {z_n - z} \right\rvert &= \left\lvert {(x_n+y_n) - (x+y)} \right\rvert = \left\lvert {x_n-x + y_n-y} \right\rvert \\ & \leq \left\lvert {x_n-x} \right\rvert + \left\lvert {y_n-y} \right\rvert \\ & < \frac{\epsilon}{2} + \frac{\epsilon}{2} = \epsilon. \end{split}\] lo que\(n \geq M\) tenemos Por lo tanto [prop:contalg:i] está probado. La prueba de [prop:contalg:ii] es casi idéntica y se deja como ejercicio.

    Abordemos [prop:contalg:iii]. Supongamos de nuevo eso\(\{ x_n \}\) y\(\{ y_n \}\) son secuencias convergentes y escribir\(z_n := x_n y_n\). Vamos\(x := \lim\, x_n\),\(y := \lim\, y_n\), y\(z := xy\).

    Dejemos\(\epsilon > 0\) que se den. Como\(\{ x_n \}\) es convergente, está acotada. Por lo tanto, encuentra un\(B >0\) tal que\(\left\lvert {x_n} \right\rvert \leq B\) para todos\(n \in {\mathbb{N}}\). Encuentra un\(M_1\) tal que por todo\(n \geq M_1\) lo que tenemos\(\left\lvert {x_n - x} \right\rvert < \frac{\epsilon}{2(\left\lvert {y} \right\rvert+1)}\). Encuentra un\(M_2\) tal que por todo\(n \geq M_2\) lo que tenemos\(\left\lvert {y_n - y} \right\rvert < \frac{\epsilon}{2B}\). Tomar\(M := \max \{ M_1, M_2 \}\). Por todo\(n \geq M\) lo que tenemos\[\begin{split} \left\lvert {z_n - z} \right\rvert &= \left\lvert {(x_ny_n) - (xy)} \right\rvert \\ & = \left\lvert {x_ny_n - (x+x_n-x_n)y} \right\rvert \\ & = \left\lvert {x_n(y_n -y) + (x_n - x)y} \right\rvert \\ & \leq \left\lvert {x_n(y_n -y)} \right\rvert + \left\lvert {(x_n - x)y} \right\rvert \\ & = \left\lvert {x_n} \right\rvert\left\lvert {y_n -y} \right\rvert + \left\lvert {x_n - x} \right\rvert\left\lvert {y} \right\rvert \\ & \leq B\left\lvert {y_n -y} \right\rvert + \left\lvert {x_n - x} \right\rvert\left\lvert {y} \right\rvert \\ & < B\frac{\epsilon}{2B} + \frac{\epsilon}{2(\left\lvert {y} \right\rvert+1)}\left\lvert {y} \right\rvert \\ & < \frac{\epsilon}{2} + \frac{\epsilon}{2} = \epsilon . \end{split}\]

    Finalmente abordemos [prop:contalg:iv]. En lugar de probar [prop:contalg:iv] directamente, probamos la siguiente afirmación más simple:

    Reclamación: Si\(\{ y_n \}\) es una secuencia convergente tal que\(\lim\, y_n \not= 0\) y\(y_n \not= 0\) para todos\(n \in {\mathbb{N}}\), entonces\[\lim_{n\to\infty} \frac{1}{y_n} = \frac{1}{\lim\, y_n} .\]

    Una vez probado el reclamo, tomamos la secuencia\(\{ \nicefrac{1}{y_n} \}\), la multiplicamos por la secuencia\(\{ x_n \}\) y aplicamos ítem [prop:contalg:iii].

    Comprobante de reclamo: Dejar\(\epsilon > 0\) ser entregado. Vamos\(y := \lim\, y_n\). Encuentra un\(M\) tal que por todos\(n \geq M\) tenemos\[\left\lvert {y_n - y} \right\rvert < \min \left\{ \left\lvert {y} \right\rvert^2\frac{\epsilon}{2}, \, \frac{\left\lvert {y} \right\rvert}{2} \right\} .\] Aviso que podemos hacer esta afirmación ya que el lado derecho es positivo porque\(\left\lvert {y} \right\rvert \not= 0\). Por lo tanto por todo lo que\(n \geq M\) tenemos\(\left\lvert {y - y_n} \right\rvert < \frac{\left\lvert {y} \right\rvert}{2}\), y así\[\left\lvert {y} \right\rvert = \left\lvert {y - y_n + y_n } \right\rvert \leq \left\lvert {y - y_n} \right\rvert + \left\lvert { y_n } \right\rvert < \frac{\left\lvert {y} \right\rvert}{2} + \left\lvert {y_n} \right\rvert.\] restando\(\nicefrac{\left\lvert {y} \right\rvert}{2}\) de ambos lados obtenemos\(\nicefrac{\left\lvert {y} \right\rvert}{2} < \left\lvert {y_n} \right\rvert\), o en otras palabras,\[\frac{1}{\left\lvert {y_n} \right\rvert} < \frac{2}{\left\lvert {y} \right\rvert} .\] Ahora terminamos la prueba del reclamo:\[\begin{split} \left\lvert {\frac{1}{y_n} - \frac{1}{y}} \right\rvert &= \left\lvert {\frac{y - y_n}{y y_n}} \right\rvert \\ & = \frac{\left\lvert {y - y_n} \right\rvert}{\left\lvert {y} \right\rvert \left\lvert {y_n} \right\rvert} \\ & < \frac{\left\lvert {y - y_n} \right\rvert}{\left\lvert {y} \right\rvert} \, \frac{2}{\left\lvert {y} \right\rvert} \\ & < \frac{\left\lvert {y} \right\rvert^2 \frac{\epsilon}{2}}{\left\lvert {y} \right\rvert} \, \frac{2}{\left\lvert {y} \right\rvert} = \epsilon . \end{split}\] Y ya hemos terminado.

    Al enchufar secuencias constantes, obtenemos varios corolarios fáciles. Si\(c \in {\mathbb{R}}\) y\(\{ x_n \}\) es una secuencia convergente, entonces por ejemplo\[\lim_{n \to \infty} c x_n = c \left( \lim_{n \to \infty} x_n \right) \qquad \text{and} \qquad \lim_{n \to \infty} (c + x_n) = c + \lim_{n \to \infty} x_n .\] Similarmente con resta y división constantes.

    Como podemos tomar límites más allá de la multiplicación podemos mostrar (ejercer) eso\(\lim\, x_n^k = {(\lim\, x_n)}^k\) para todos\(k \in {\mathbb{N}}\). Es decir, podemos tomar límites poderes pasados. Veamos si podemos hacer lo mismo con raíces.

    Que\(\{ x_n \}\) sea una secuencia convergente tal que\(x_n \geq 0\). Entonces\[\lim_{n\to\infty} \sqrt{x_n} = \sqrt{ \lim_{n\to\infty} x_n } .\]

    Por supuesto para incluso hacer esta afirmación, necesitamos aplicar para demostrar eso\(\lim\, x_n \geq 0\), para que podamos tomar la raíz cuadrada sin preocupaciones.

    Dejar\(\{ x_n \}\) ser una secuencia convergente y dejar\(x := \lim\, x_n\).

    Primero supongamos\(x=0\). Dejemos\(\epsilon > 0\) que se den. Entonces hay\(M\) tal que por todo\(n \geq M\) lo que tenemos\(x_n = \left\lvert {x_n} \right\rvert < \epsilon^2\), o en otras palabras\(\sqrt{x_n} < \epsilon\). De ahí\[\left\lvert {\sqrt{x_n} - \sqrt{x}} \right\rvert = \sqrt{x_n} < \epsilon.\]

    Ahora supongamos\(x > 0\) (y por lo tanto\(\sqrt{x} > 0\)). \[\begin{split} \left\lvert {\sqrt{x_n}-\sqrt{x}} \right\rvert &= \left\lvert {\frac{x_n-x}{\sqrt{x_n}+\sqrt{x}}} \right\rvert \\ &= \frac{1}{\sqrt{x_n}+\sqrt{x}} \left\lvert {x_n-x} \right\rvert \\ & \leq \frac{1}{\sqrt{x}} \left\lvert {x_n-x} \right\rvert . \end{split}\]Dejamos el resto de la prueba al lector.

    Una prueba similar funciona para la raíz\(k\) th. Es decir, también obtenemos\(\lim\, x_n^{1/k} = {( \lim\, x_n )}^{1/k}\). Dejamos esto al lector como un ejercicio desafiante.

    También podemos querer tomar el límite más allá del signo de valor absoluto. Lo contrario de esta proposición no es cierto, véase la parte b).

    Si\(\{ x_n \}\) es una secuencia convergente, entonces\(\{ \left\lvert {x_n} \right\rvert \}\) es convergente y\[\lim_{n\to\infty} \left\lvert {x_n} \right\rvert = \left\lvert {\lim_{n\to\infty} x_n} \right\rvert .\]

    Simplemente notamos la desigualdad del triángulo inverso\[\big\lvert \left\lvert {x_n} \right\rvert - \left\lvert {x} \right\rvert \big\rvert \leq \left\lvert {x_n-x} \right\rvert .\] De ahí que si se\(\left\lvert {x_n -x} \right\rvert\) puede hacer arbitrariamente pequeña, así puede\(\big\lvert \left\lvert {x_n} \right\rvert - \left\lvert {x} \right\rvert \big\rvert\). Los detalles se dejan al lector.

    Veamos un ejemplo juntando las proposiciones anteriores. Ya que sabemos eso\(\lim \nicefrac{1}{n} = 0\), entonces es\[\lim_{n\to \infty} \left\lvert {\sqrt{1 + \nicefrac{1}{n}} - \nicefrac{100}{n^2}} \right\rvert = \left\lvert {\sqrt{1 + (\lim \nicefrac{1}{n})} - 100 (\lim \nicefrac{1}{n})(\lim \nicefrac{1}{n})} \right\rvert = 1.\] decir, el límite en el lado izquierdo existe porque existe el lado derecho. Realmente deberías leer la igualdad anterior de derecha a izquierda.

    Secuencias definidas recursivamente

    Ahora que sabemos que podemos intercambiar límites y operaciones algebraicas, podemos calcular los límites de muchas secuencias. Una de esas clases son secuencias definidas recursivamente, es decir, secuencias donde el siguiente número en la secuencia computado usando una fórmula a partir de un número fijo de elementos precedentes en la secuencia.

    \(\{ x_n \}\)Déjese definir por\(x_1 := 2\) y Primero\[x_{n+1} := x_n - \frac{x_n^2-2}{2x_n} .\] debemos averiguar si esta secuencia está bien definida; debemos demostrar que nunca dividimos por cero. Entonces debemos averiguar si la secuencia converge. Sólo entonces podremos intentar encontrar el límite.

    Primero probemos que\(x_n\) existe y\(x_n > 0\) para todos\(n\) (así la secuencia está bien definida y delimitada a continuación). Demostremos esto por. Eso lo sabemos\(x_1 = 2 > 0\). Para el paso de inducción, supongamos\(x_n > 0\). Entonces\[x_{n+1} = x_n - \frac{x_n^2-2}{2x_n} = \frac{2x_n^2 - x_n^2+2}{2x_n} = \frac{x_n^2+2}{2x_n} .\] Si\(x_n > 0\), entonces\(x_n^2+2 > 0\) y por lo tanto\(x_{n+1} > 0\).

    A continuación mostremos que la secuencia es monótona decreciente. Si lo demostramos\(x_n^2-2 \geq 0\) para todos\(n\), entonces\(x_{n+1} \leq x_n\) para todos\(n\). Obviamente\(x_1^2-2 = 4-2 = 2 > 0\). Para una arbitraria\(n\) tenemos\[x_{n+1}^2-2 = {\left( \frac{x_n^2+2}{2x_n} \right)}^2 - 2 = \frac{x_n^4+4x_n^2+4 - 8x_n^2}{4x_n^2} = \frac{x_n^4-4x_n^2+4}{4x_n^2} = \frac{{\left( x_n^2-2 \right)}^2}{4x_n^2} .\] Ya que cualquier número al cuadrado no es negativo, tenemos eso\(x_{n+1}^2-2 \geq 0\) para todos\(n\). Por lo tanto,\(\{ x_n \}\) es monótona decreciente y acotada (\(x_n > 0\)para todos\(n\)), y el límite existe. Queda por encontrar el límite.

    Escribamos\[2x_nx_{n+1} = x_n^2+2 .\] Ya que\(\{ x_{n+1} \}\) es la 1-cola de\(\{ x_n \}\), converge al mismo límite. Definamos\(x := \lim\, x_n\). Tomamos el límite de ambas partes para obtener\[2x^2 = x^2+2 ,\] o\(x^2 = 2\). En\(x_n > 0\) cuanto a todo\(n\) lo que obtenemos\(x \geq 0\), y por lo tanto\(x = \sqrt{2}\).

    Es posible que hayas visto la secuencia anterior antes. Es el método 8 de Newton para encontrar la raíz cuadrada de 2. Este método surge muy a menudo en la práctica y converge muy rápidamente. Observe que hemos utilizado el hecho de que\(x_1^2 -2 >0\), aunque no era estrictamente necesario mostrar convergencia al considerar una cola de la secuencia. De hecho la secuencia converge siempre y cuando\(x_1 \not= 0\), aunque con un negativo\(x_1\) llegaríamos a\(x=-\sqrt{2}\). Al sustituir el 2 en el numerador obtenemos la raíz cuadrada de cualquier número positivo. Estas declaraciones se dejan como ejercicio.

    Debe, sin embargo, tener cuidado. Antes de tomar algún límite, debes asegurarte de que la secuencia converja. Veamos un ejemplo.

    Supongamos\(x_1 := 1\) y\(x_{n+1} := x_n^2+x_n\). Si asumimos ciegamente que el límite existe (llamarlo\(x\)), entonces obtendríamos la ecuación\(x = x^2+x\), de la cual podríamos concluir\(x=0\). No obstante, no es difícil demostrar que\(\{ x_n \}\) no tiene límites y por lo tanto no converge.

    Lo que hay que notar en este ejemplo es que el método sigue funcionando, pero depende del valor inicial\(x_1\). Si establecemos\(x_1 := 0\), entonces la secuencia converge y el límite realmente es 0. Toda una rama de las matemáticas, llamada dinámica, trata precisamente de estos temas.

    Algunas pruebas de convergencia

    No siempre es necesario volver a la definición de convergencia para demostrar que una secuencia es convergente. Primero damos una prueba de convergencia simple. La idea principal es que\(\{ x_n \}\) converge a\(x\) si y sólo si\(\{ \left\lvert { x_n - x } \right\rvert \}\) converge a cero.

    [convzero:prop] Dejar\(\{ x_n \}\) ser una secuencia. Supongamos que hay una\(x \in {\mathbb{R}}\) y una secuencia convergente\(\{ a_n \}\) tal que\[\lim_{n\to\infty} a_n = 0\] y\[\left\lvert {x_n - x} \right\rvert \leq a_n\] para todos\(n\). Entonces\(\{ x_n \}\) converge y\(\lim\, x_n = x\).

    Dejemos\(\epsilon > 0\) que se den. Tenga en cuenta que\(a_n \geq 0\) para todos\(n\). Encuentra un\(M \in {\mathbb{N}}\) tal que por todo\(n \geq M\) lo que tenemos\(a_n = \left\lvert {a_n - 0} \right\rvert < \epsilon\). Entonces, por todo\(n \geq M\) lo que tenemos\[\left\lvert {x_n - x} \right\rvert \leq a_n < \epsilon . \qedhere\]

    Como muestra la proposición, estudiar cuando una secuencia tiene un límite es lo mismo que estudiar cuando otra secuencia va a cero. En general puede ser difícil decidir si una secuencia converge, pero para ciertas secuencias existen pruebas fáciles de aplicar que nos indican si la secuencia converge o no. Veamos una de esas pruebas. Primero calculemos el límite de una secuencia muy específica.

    Vamos\(c > 0\).

    1. Si\(c < 1\), entonces\[\lim_{n\to\infty} c^n = 0.\]
    2. Si\(c > 1\), entonces\(\{ c^n \}\) está sin límites.

    Primero supongamos\(c > 1\). Escribimos\(c = 1+r\) para algunos\(r > 0\). Por (o usando el teorema binomial si lo conoces) tenemos la desigualdad de Bernoulli (ver también):\[c^n = {(1+r)}^n \geq 1+nr .\] Por el de los números reales, la secuencia\(\{ 1+nr \}\) está sin límites (para cualquier número\(B\), encontramos\(n \in {\mathbb{N}}\) tal que\(nr \geq B-1\)). Por\(c^n\) lo tanto, no tiene límites.

    Ahora vamos\(c < 1\). Escribir\(c = \frac{1}{1+r}\), dónde\(r > 0\). Entonces\[c^n = \frac{1}{{(1+r)}^n} \leq \frac{1}{1+nr} \leq \frac{1}{r} \frac{1}{n} .\] As\(\{ \frac{1}{n} \}\) converge a cero, también lo hace\(\{ \frac{1}{r} \frac{1}{n} \}\). De ahí,\(\{ c^n \}\) converge a cero.

    Si nos fijamos en la proposición anterior, observamos que la relación entre el término\((n+1)\) th y el\(n\) th term es\(c\). Generalizamos este resultado simple a una clase más grande de secuencias. El siguiente lema volverá a aparecer una vez que lleguemos a series.

    [seq:ratiotest]\(\{ x_n \}\) Sea una secuencia tal que\(x_n \not= 0\) para todos\(n\) y tal que\[L := \lim_{n\to\infty} \frac{\left\lvert {x_{n+1}} \right\rvert}{\left\lvert {x_n} \right\rvert}\] exista el límite.

    1. Si\(L < 1\), entonces\(\{ x_n \}\) converge y\(\lim\, x_n = 0\).
    2. Si\(L > 1\), entonces\(\{ x_n \}\) está sin límites (de ahí diverge).

    Si\(L\) existe, pero\(L=1\), el lema no dice nada. No podemos llegar a ninguna conclusión basada únicamente en esa información. Por ejemplo, la secuencia\(\{ \nicefrac{1}{n} \}\) converge a cero, pero\(L=1\). La secuencia constante\(\{ 1 \}\) converge a 1, no a cero, y también\(L=1\). La secuencia\(\{ {(-1)}^n \}\) no converge en absoluto, y\(L=1\). Finalmente la secuencia\(\{ \ln n \}\) está sin límites, una vez más\(L=1\).

    Supongamos\(L < 1\). Como\(\frac{\left\lvert {x_{n+1}} \right\rvert}{\left\lvert {x_n} \right\rvert} \geq 0\), tenemos eso\(L \geq 0\). Escoge\(r\) tal que\(L < r < 1\). Deseamos comparar la secuencia con la secuencia\(r^n\). La idea es que si bien la secuencia no va a ser menor que\(L\) eventualmente, eventualmente será menor que\(r\), que sigue siendo menor que 1. La idea intuitiva de la prueba se ilustra en.

    Como\(r-L > 0\), existe\(M \in {\mathbb{N}}\) tal que por todo lo que\(n \geq M\) tenemos\[\left\lvert {\frac{\left\lvert {x_{n+1}} \right\rvert}{\left\lvert {x_n} \right\rvert} - L} \right\rvert < r-L .\] Por lo tanto,\[\frac{\left\lvert {x_{n+1}} \right\rvert}{\left\lvert {x_n} \right\rvert} < r .\] Para\(n > M\) (eso es para\(n \geq M+1\)) escribimos\[\left\lvert {x_n} \right\rvert = \left\lvert {x_M} \right\rvert \frac{\left\lvert {x_{M+1}} \right\rvert}{\left\lvert {x_{M}} \right\rvert} \frac{\left\lvert {x_{M+2}} \right\rvert}{\left\lvert {x_{M+1}} \right\rvert} \cdots \frac{\left\lvert {x_{n}} \right\rvert}{\left\lvert {x_{n-1}} \right\rvert} < \left\lvert {x_M} \right\rvert r r \cdots r = \left\lvert {x_M} \right\rvert r^{n-M} = (\left\lvert {x_M} \right\rvert r^{-M}) r^n .\] La secuencia\(\{ r^n \}\) converge a cero y de ahí\(\left\lvert {x_M} \right\rvert r^{-M} r^n\) converge a cero. Por, la\(M\) -cola de\(\{x_n\}\) converge a cero y por lo tanto\(\{x_n\}\) converge a cero.

    Ahora supongamos\(L > 1\). Escoge\(r\) tal que\(1 < r < L\). Como\(L-r > 0\), existe\(M \in {\mathbb{N}}\) tal que por todo lo que\(n \geq M\) tenemos\[\left\lvert {\frac{\left\lvert {x_{n+1}} \right\rvert}{\left\lvert {x_n} \right\rvert} - L} \right\rvert < L-r .\] Por lo tanto,\[\frac{\left\lvert {x_{n+1}} \right\rvert}{\left\lvert {x_n} \right\rvert} > r .\] De nuevo para\(n > M\) escribimos\[\left\lvert {x_n} \right\rvert = \left\lvert {x_M} \right\rvert \frac{\left\lvert {x_{M+1}} \right\rvert}{\left\lvert {x_{M}} \right\rvert} \frac{\left\lvert {x_{M+2}} \right\rvert}{\left\lvert {x_{M+1}} \right\rvert} \cdots \frac{\left\lvert {x_{n}} \right\rvert}{\left\lvert {x_{n-1}} \right\rvert} > \left\lvert {x_M} \right\rvert r r \cdots r = \left\lvert {x_M} \right\rvert r^{n-M} = (\left\lvert {x_M} \right\rvert r^{-M}) r^n .\] La secuencia\(\{ r^n \}\) es unbounded (since\(r > 1\)), y por lo tanto\(\{x_n\}\) no puede ser acotada (si\(\left\lvert {x_n} \right\rvert \leq B\) para todos\(n\), entonces \(r^n < \frac{B}{\left\lvert {x_M} \right\rvert} r^{M}\)para todos\(n\), lo cual es imposible). En consecuencia,\(\{ x_n \}\) no puede converger.

    Una simple aplicación del lema anterior es demostrar que\[\lim_{n\to\infty} \frac{2^n}{n!} = 0 .\]

    Prueba: Encontramos que\[\frac{2^{n+1} / (n+1)!}{2^n/n!} = \frac{2^{n+1}}{2^n}\frac{n!}{(n+1)!} = \frac{2}{n+1} .\] No es difícil ver que\(\{ \frac{2}{n+1} \}\) converja a cero. La conclusión sigue por el lema.

    Ejercicios

    Demostrar. Pista: Utilice secuencias constantes y.

    Demostrar parte [prop:contalg:ii] de.

    Demostrar que si\(\{ x_n \}\) es una secuencia convergente\(k \in {\mathbb{N}}\),, entonces\[\lim_{n\to\infty} x_n^k = {\left( \lim_{n\to\infty} x_n \right)}^k .\] Pista: Usar.

    Supongamos\(x_1 := \frac{1}{2}\) y\(x_{n+1} := x_n^2\). Demostrar que\(\{ x_n \}\) converge y encuentra\(\lim\, x_n\). Pista: ¡No se puede dividir por cero!

    Vamos\(x_n := \frac{n-\cos(n)}{n}\). Usa el para mostrar que\(\{ x_n \}\) converge y encuentra el límite.

    Dejar\(x_n := \frac{1}{n^2}\) y\(y_n := \frac{1}{n}\). Definir\(z_n := \frac{x_n}{y_n}\) y\(w_n := \frac{y_n}{x_n}\). ¿Hacer\(\{ z_n \}\) y\(\{ w_n \}\) converger? ¿Cuáles son los límites? ¿Se puede aplicar? ¿Por qué o por qué no?

    Verdadero o falso, probar o encontrar un contraejemplo. Si\(\{ x_n \}\) es una secuencia tal que\(\{ x_n^2 \}\) converge, entonces\(\{ x_n \}\) converge.

    Demostrar que\[\lim_{n\to\infty} \frac{n^2}{2^n} = 0 .\]

    Supongamos que\(\{ x_n \}\) es una secuencia y supongamos para algunos\(x \in {\mathbb{R}}\), el límite\[L := \lim_{n \to \infty} \frac{\left\lvert {x_{n+1}-x} \right\rvert}{\left\lvert {x_n-x} \right\rvert}\] existe y\(L < 1\). Espectáculo que\(\{ x_n \}\) converge a\(x\).

    Que\(\{ x_n \}\) sea una secuencia convergente tal que\(x_n \geq 0\) y\(k \in {\mathbb{N}}\). Entonces\[\lim_{n\to\infty} x_n^{1/k} = {\left( \lim_{n\to\infty} x_n \right)}^{1/k} .\] Pista: Encuentra una expresión\(q\) tal que\(\frac{x_n^{1/k}-x^{1/k}}{x_n-x} = \frac{1}{q}\).

    Vamos\(r > 0\). Mostrar que a partir de cualquiera\(x_1 \not= 0\), la secuencia definida por\[x_{n+1} := x_n - \frac{x_n^2-r}{2x_n}\] converge a\(\sqrt{r}\) si\(x_1 > 0\) y\(-\sqrt{r}\) si\(x_1 < 0\).

    a) Supongamos que\(\{ a_n \}\) es una secuencia acotada y\(\{ b_n \}\) es una secuencia que converge a 0. Espectáculo que\(\{ a_n b_n \}\) converge a 0.
    b) Encontrar un ejemplo donde no\(\{ a_n \}\) esté acotado,\(\{ b_n \}\) converja a 0, y no\(\{ a_n b_n \}\) sea convergente.
    c) Encontrar un ejemplo donde\(\{ a_n \}\) esté acotado,\(\{ b_n \}\) converja con algunos\(x \not= 0\), y no\(\{ a_n b_n \}\) sea convergente.

    Límite superior, límite inferior y Bolzano-Weierstrass

    Nota: 1—2 conferencias, prueba alternativa de BW opcional

    En esta sección se estudian las secuencias delimitadas y sus subsecuencias. En particular definimos el llamado límite superior y límite inferior de una secuencia acotada y hablamos de límites de subsecuencias. Además, probamos el teorema de Bolzano-Weierstrass 9, que es una herramienta indispensable en el análisis.

    Hemos visto que cada secuencia convergente está acotada, aunque existen muchas secuencias divergentes acotadas. Por ejemplo, la secuencia\(\{ {(-1)}^n \}\) está acotada, pero es divergente. Sin embargo, no todo se pierde y todavía podemos calcular ciertos límites con una secuencia divergente acotada.

    Límites superior e inferior

    Hay formas de crear secuencias monótonas a partir de cualquier secuencia, y de esta manera obtenemos el llamado límite superior y límite inferior. Estos límites siempre existen para secuencias delimitadas.

    Si una secuencia\(\{ x_n \}\) está delimitada, entonces el conjunto\(\{ x_k : k \in {\mathbb{N}}\}\) está delimitado. Entonces por cada\(n\) conjunto también\(\{ x_k : k \geq n \}\) está acotado (ya que es un subconjunto).

    [liminflimsup:def] Let\(\{ x_n \}\) Ser una secuencia acotada. Dejar\(a_n := \sup \{ x_k : k \geq n \}\) y\(b_n := \inf \{ x_k : k \geq n \}\). Definir\[\begin{aligned} \limsup_{n \to \infty} \, x_n & := \lim_{n \to \infty} a_n , \\ \liminf_{n \to \infty} \, x_n & := \lim_{n \to \infty} b_n .\end{aligned}\]

    Para una secuencia acotada, liminf y limsup siempre existen (ver abajo). Es posible definir liminf y limsup para secuencias no delimitadas si permitimos\(\infty\) y\(-\infty\). No es difícil generalizar los siguientes resultados para incluir secuencias no delimitadas, sin embargo, primero restringimos nuestra atención a las delimitadas.

    Dejar\(\{ x_n \}\) ser una secuencia acotada. Dejar\(a_n\) y\(b_n\) ser como en la definición anterior.

    1. La secuencia\(\{ a_n \}\) es limitada monótona decreciente y\(\{ b_n \}\) está acotada monótona creciente. En particular,\(\liminf x_n\) y\(\limsup x_n\) existir.
    2. \(\displaystyle \limsup_{n \to \infty} \, x_n = \inf \{ a_n : n \in {\mathbb{N}}\}\)y\(\displaystyle \liminf_{n \to \infty} \, x_n = \sup \{ b_n : n \in {\mathbb{N}}\}\).
    3. \(\displaystyle \liminf_{n \to \infty} \, x_n \leq \limsup_{n \to \infty} \, x_n\).

    Veamos por qué\(\{ a_n \}\) es una secuencia decreciente. Al igual\(a_n\) que el límite inferior superior para\(\{ x_k : k \geq n \}\), también es un límite superior para el subconjunto\(\{ x_k : k \geq (n+1) \}\). Por lo tanto\(a_{n+1}\), el límite inferior superior para\(\{ x_k : k \geq (n+1) \}\), tiene que ser menor o igual a\(a_n\), es decir,\(a_n \geq a_{n+1}\). Del mismo modo (un ejercicio),\(b_n\) es una secuencia creciente. Se deja como ejercicio para demostrar que si\(x_n\) está acotado, entonces\(a_n\) y\(b_n\) debe ser acotado.

    El segundo ítem de la proposición sigue como las secuencias\(\{ a_n \}\) y\(\{ b_n \}\) son monótonas.

    Para el tercer ítem, observamos que\(b_n \leq a_n\), como el\(\inf\) de un conjunto es menor o igual a su\(\sup\). Eso lo sabemos\(\{ a_n \}\) y\(\{ b_n \}\) convergen al limsup y al liminf (respectivamente). Aplicamos para obtener\[\lim_{n\to \infty} b_n \leq \lim_{n\to \infty} a_n. \qedhere\]

    \(\{ x_n \}\)Seamos definidos por\[x_n := \begin{cases} \frac{n+1}{n} & \text{ if $n$ is odd,} \\ 0 & \text{ if $n$ is even.} \end{cases}\] Vamos a computar el\(\liminf\) y\(\limsup\) de esta secuencia. Primero el límite inferior:\[\liminf_{n\to\infty} \, x_n = \lim_{n\to\infty} \left( \inf \{ x_k : k \geq n \} \right) = \lim_{n\to\infty} 0 = 0 .\] Para el límite superior escribimos No es\[\limsup_{n\to\infty} \, x_n = \lim_{n\to\infty} \left( \sup \{ x_k : k \geq n \} \right) .\] difícil ver que\[\sup \{ x_k : k \geq n \} = \begin{cases} \frac{n+1}{n} & \text{ if $n$ is odd,} \\ \frac{n+2}{n+1} & \text{ if $n$ is even.} \end{cases}\] Dejamos al lector demostrar que el límite es 1. Es decir,\[\limsup_{n\to\infty} \, x_n = 1 .\] Nótese que la secuencia no\(\{ x_n \}\) es una secuencia convergente.

    Nos asociamos con\(\limsup\) y\(\liminf\) ciertas subsecuencias.

    [subseqlimsupinf:thm] Si\(\{ x_n \}\) es una secuencia acotada, entonces existe una subsecuencia\(\{ x_{n_k} \}\) tal que\[\lim_{k\to \infty} x_{n_k} = \limsup_{n \to \infty} \, x_n .\] Del mismo modo, existe una subsecuencia (quizás diferente)\(\{ x_{m_k} \}\) tal que\[\lim_{k\to \infty} x_{m_k} = \liminf_{n \to \infty} \, x_n .\]

    Definir\(a_n := \sup \{ x_k : k \geq n \}\). Escribir\(x := \limsup \, x_n = \lim\, a_n\). Defina la subsecuencia de la siguiente manera. Escoja\(n_1 := 1\) y trabaje inductivamente. Supongamos que hemos definido la subsecuencia hasta\(n_k\) para algunos\(k\). Ahora escoge algunos de\(m > n_k\) tal manera que\[a_{(n_k+1)} - x_m < \frac{1}{k+1} .\] podamos hacer esto como\(a_{(n_k+1)}\) es un supremo del conjunto\(\{ x_n : n \geq n_k + 1 \}\) y de ahí que haya elementos de la secuencia arbitrariamente cercanos (o incluso posiblemente iguales) al supremo. Set\(n_{k+1} := m\). Se define la subsecuencia\(\{ x_{n_k} \}\). A continuación tenemos que demostrar que converge y tiene el límite adecuado.

    Tenga en cuenta que\(a_{(n_{k-1}+1)} \geq a_{n_k}\) (¿por qué?) y eso\(a_{n_{k}} \geq x_{n_k}\). Por lo tanto, por cada\(k > 1\) que tenemos\[\begin{split} \left\lvert {a_{n_k} - x_{n_k}} \right\rvert & = a_{n_k} - x_{n_k} \\ & \leq a_{(n_{k-1}+1)} - x_{n_k} \\ & < \frac{1}{k} . \end{split}\]

    Demostremos que\(\{ x_{n_k} \}\) converge a\(x\). Tenga en cuenta que la subsecuencia no necesita ser monótona. Dejemos\(\epsilon > 0\) que se den. A medida que\(\{ a_n \}\) converge a\(x\), entonces la subsecuencia\(\{ a_{n_k} \}\) converge a\(x\). Así existe\(M_1 \in {\mathbb{N}}\) tal que por todos\(k \geq M_1\) tenemos\[\left\lvert {a_{n_k} - x} \right\rvert < \frac{\epsilon}{2} .\] Encuentra un\(M_2 \in {\mathbb{N}}\) tal que\[\frac{1}{M_2} \leq \frac{\epsilon}{2}.\] Tome\(M := \max \{M_1 , M_2 \}\) y calme. Por todo\(k \geq M\) lo que tenemos\[\begin{split} \left\lvert {x- x_{n_k}} \right\rvert & = \left\lvert {a_{n_k} - x_{n_k} + x - a_{n_k}} \right\rvert \\ & \leq \left\lvert {a_{n_k} - x_{n_k}} \right\rvert + \left\lvert {x - a_{n_k}} \right\rvert \\ & < \frac{1}{k} + \frac{\epsilon}{2} \\ & \leq \frac{1}{M_2} + \frac{\epsilon}{2} \leq \frac{\epsilon}{2} + \frac{\epsilon}{2} = \epsilon . \end{split}\]

    Dejamos la declaración para\(\liminf\) como ejercicio.

    Usando límite inferior y límite superior

    La ventaja de\(\liminf\) y\(\limsup\) es que siempre podemos escribirlos para cualquier secuencia (acotada). Si de alguna manera pudiéramos calcularlos, también podríamos calcular el límite de la secuencia si existe, o mostrar que la secuencia diverge. Trabajar con\(\liminf\) y\(\limsup\) es un poco como trabajar con límites, aunque hay diferencias sutiles.

    [liminfsupconv:thm] Let\(\{ x_n \}\) Ser una secuencia acotada. Entonces\(\{ x_n \}\) converge si y solo si\[\liminf_{n\to \infty} \, x_n = \limsup_{n\to \infty} \, x_n.\] Además, si\(\{ x_n \}\) converge, entonces\[\lim_{n\to \infty} x_n = \liminf_{n\to \infty} \, x_n = \limsup_{n\to \infty} \, x_n.\]

    Definir\(a_n\) y\(b_n\) como en. Tenga en cuenta que\[b_n \leq x_n \leq a_n .\] Si\(\liminf \, x_n = \limsup \, x_n\), entonces sabemos eso\(\{ a_n \}\) y\(\{ b_n \}\) tenemos límites y que estos dos límites son los mismos. Por el lema squeeze (),\(\{ x_n \}\) converge y\[\lim_{n\to \infty} b_n = \lim_{n\to \infty} x_n = \lim_{n\to \infty} a_n .\]

    Ahora supongamos que\(\{ x_n \}\) converge a\(x\). Sabemos por eso existe una subsecuencia\(\{ x_{n_k} \}\) que converge a\(\limsup \, x_n\). A medida que\(\{ x_n \}\) converge a\(x\), cada subsecuencia converge hacia\(x\) y por lo tanto\(\limsup \, x_n = \lim\, x_{n_k} = x\). De igual manera\(\liminf \, x_n = x\).

    Límite superior y límite inferior se comportan muy bien con subsecuencias.

    [prop:subseqslimsupinf] Supongamos que\(\{ x_n \}\) es una secuencia acotada y\(\{ x_{n_k} \}\) es una subsecuencia. Entonces\[\liminf_{n\to\infty} \, x_n \leq \liminf_{k\to\infty} \, x_{n_k} \leq \limsup_{k\to\infty} \, x_{n_k} \leq \limsup_{n\to\infty} \, x_n .\]

    Ya se ha demostrado la desigualdad media. Demostraremos la tercera desigualdad, y dejaremos la primera desigualdad como ejercicio.

    Eso queremos demostrarlo\(\limsup \, x_{n_k} \leq \limsup \, x_n\). Definir\(a_j := \sup \{ x_k : k \geq j \}\) como de costumbre. Definir también\(c_j := \sup \{ x_{n_k} : k \geq j \}\). No es cierto que\(c_j\) sea necesariamente una subsecuencia de\(a_j\). No obstante, en\(n_k \geq k\) cuanto a todos\(k\), tenemos eso\(\{ x_{n_k} : k \geq j \} \subset \{ x_k : k \geq j \}\). Un supremo de un subconjunto es menor o igual al supremo del conjunto y por lo tanto\[c_j \leq a_j .\] aplicamos para concluir\[\lim_{j\to\infty} c_j \leq \lim_{j\to\infty} a_j ,\] cual es la conclusión deseada.

    Límite superior y límite inferior son los límites subsecuenciales más grandes y más pequeños. Si la subsecuencia en la proposición anterior es convergente, entonces tenemos eso\(\liminf \, x_{n_k} = \lim\, x_{n_k} = \limsup \, x_{n_k}\). Por lo tanto,\[\liminf_{n\to\infty} \, x_n \leq \lim_{k\to\infty} x_{n_k} \leq \limsup_{n\to\infty} \, x_n .\]

    De manera similar obtenemos la siguiente prueba útil para la convergencia de una secuencia acotada. Dejamos la prueba como ejercicio.

    [seqconvsubseqconv:thm] Una secuencia acotada\(\{ x_n \}\) es convergente y converge a\(x\) si y solo si cada subsecuencia convergente\(\{ x_{n_k} \}\) converge a\(x\).

    Teorema de Bolzano-Weierstrass

    Si bien no es cierto que una secuencia acotada sea convergente, el teorema de Bolzano-Weierstrass nos dice que al menos podemos encontrar una subsecuencia convergente. La versión de Bolzano-Weierstrass que presentamos en esta sección es la Bolzano-Weierstrass para secuencias.

    [thm:bwseq] Supongamos que una secuencia\(\{ x_n \}\) de números reales está delimitada. Entonces existe una subsecuencia convergente\(\{ x_{n_i} \}\).

    Utilizamos. Dice que existe una subsecuencia cuyo límite es\(\limsup \, x_n\).

    El lector podría quejarse ahora mismo de que es estrictamente más fuerte que el teorema de Bolzano-Weierstrass como se presentó anteriormente. Eso es cierto. No obstante, sólo se aplica a la línea real, pero Bolzano-Weierstrass se aplica en contextos más generales (es decir, in\({\mathbb{R}}^n\)) con más o menos exactamente la misma afirmación.

    Como el teorema es tan importante para el análisis, presentamos una prueba explícita. La siguiente prueba generaliza más fácilmente a diferentes contextos.

    Como la secuencia está acotada, entonces existen dos números\(a_1 < b_1\) tales que\(a_1 \leq x_n \leq b_1\) para todos\(n \in {\mathbb{N}}\).

    Vamos a definir una subsecuencia\(\{ x_{n_i} \}\) y dos secuencias\(\{ a_i \}\) y\(\{ b_i \}\), tal que\(\{ a_i \}\) es monótona creciente,\(\{ b_i \}\) es monótona decreciente,\(a_i \leq x_{n_i} \leq b_i\) y tal que\(\lim\, a_i = \lim\, b_i\). Eso\(x_{n_i}\) converge sigue por el.

    Definimos las secuencias inductivamente. Siempre tendremos eso\(a_i < b_i\), y eso\(x_n \in [a_i,b_i]\) para infinitamente muchos\(n \in {\mathbb{N}}\). Ya hemos definido\(a_1\) y\(b_1\). Tomamos\(n_1 := 1\), es decir\(x_{n_1} = x_1\).

    Ahora supongamos que hasta algunos\(k \in {\mathbb{N}}\) hemos definido la subsecuencia\(x_{n_1}, x_{n_2}, \ldots, x_{n_k}\), y las secuencias\(a_1,a_2,\ldots,a_k\) y\(b_1,b_2,\ldots,b_k\). Vamos\(y := \frac{a_k+b_k}{2}\). Claramente\(a_k < y < b_k\). Si existen infinitamente muchos\(j \in {\mathbb{N}}\) tales que\(x_j \in [a_k,y]\), entonces establecer\(a_{k+1} := a_k\),\(b_{k+1} := y\), y recoger\(n_{k+1} > n_{k}\) tal que\(x_{n_{k+1}} \in [a_k,y]\). Si no hay infinitamente muchos\(j\) tales que\(x_j \in [a_k,y]\), entonces debe ser cierto que hay infinitamente muchos\(j \in {\mathbb{N}}\) tales que\(x_j \in [y,b_k]\). En este caso escoja\(a_{k+1} := y\),\(b_{k+1} := b_k\), y escoja\(n_{k+1} > n_{k}\) tal que\(x_{n_{k+1}} \in [y,b_k]\).

    Ahora tenemos las secuencias definidas. Lo que queda por probar es eso\(\lim\, a_i = \lim\, b_i\). Obviamente los límites existen ya que las secuencias son monótonas. De la construcción, es obvio que\(b_i - a_i\) se corta por la mitad en cada paso. Por lo tanto\(b_{i+1} - a_{i+1} = \frac{b_i-a_i}{2}\). Por, obtenemos que\[b_i - a_i = \frac{b_1-a_1}{2^{i-1}} .\]

    Vamos\(x := \lim\, a_i\). Como\(\{ a_i \}\) es monótona tenemos eso\[x = \sup \{ a_i : i \in {\mathbb{N}}\}\] Ahora vamos\(y := \lim\, b_i = \inf \{ b_i : i \in {\mathbb{N}}\}\). Obviamente\(y \leq x\) en\(a_i < b_i\) cuanto a todos\(i\). Como las secuencias son monótona, entonces para cualquiera\(i\) tenemos (¿por qué?) \[y-x \leq b_i-a_i = \frac{b_1-a_1}{2^{i-1}} .\]Como\(\frac{b_1-a_1}{2^{i-1}}\) es arbitrariamente pequeño y\(y-x \geq 0\), tenemos eso\(y-x = 0\). Terminamos por el.

    Otra prueba más del teorema de Bolzano-Weierstrass es mostrar la siguiente afirmación, que se deja como un ejercicio desafiante. Reclamación: Cada secuencia tiene una subsecuencia monótona.

    Límites infinitos

    Si permitimos\(\liminf\) y tomamos\(\limsup\) los valores\(\infty\) y\(-\infty\), podemos aplicar\(\liminf\) y\(\limsup\) a todas las secuencias, no solo las delimitadas. Para cualquier secuencia, escribimos\[\limsup \, x_n := \inf \{ a_n : n \in {\mathbb{N}}\}, \qquad \text{and} \qquad \liminf \, x_n := \sup \{ b_n : n \in {\mathbb{N}}\},\] dónde\(a_n := \sup \{ x_k : k \geq n \}\) y\(b_n := \inf \{ x_k : k \geq n \}\) como antes.

    También a menudo definimos límites infinitos para ciertas secuencias divergentes.

    Decimos\(\{ x_n \}\) diverge al infinito 10 si por cada\(M \in {\mathbb{R}}\), existe\(N \in {\mathbb{N}}\) tal que por todo\(n \geq N\) lo que tenemos\(x_n > M\). En este caso escribimos\(\lim \, x_n := \infty\). De igual manera si por cada\(M \in {\mathbb{R}}\) existe un\(N \in {\mathbb{N}}\) tal que por todo\(n \geq N\) lo que tenemos\(x_n < M\), decimos\(\{ x_n \}\) diverge a menos infinito y escribimos\(\lim \, x_n := -\infty\).

    Esta definición se comporta como se esperaba con\(\limsup\) y\(\liminf\), ver ejercicios [ejercicio:infseqlimex] y [ejercicio:infseqlimlims].

    Si\(x_n := 0\) para impar\(n\) y\(x_n := n\) para par\(n\) entonces\[\lim_{n\to \infty} n = \infty, \qquad \lim_{n\to \infty} x_n \quad \text{does not exist}, \qquad \limsup_{n\to \infty} x_n = \infty.\]

    Ejercicios

    Supongamos que\(\{ x_n \}\) es una secuencia acotada. Definir\(a_n\) y\(b_n\) como en. \(\{ b_n \}\)Demuéstralo\(\{ a_n \}\) y están acotados.

    Supongamos que\(\{ x_n \}\) es una secuencia acotada. Definir\(b_n\) como en. Demostrar que\(\{ b_n \}\) es una secuencia creciente.

    Terminar la prueba de. Es decir, supongamos que\(\{ x_n \}\) es una secuencia acotada y\(\{ x_{n_k} \}\) es una subsecuencia. Demostrar\(\displaystyle \liminf_{n\to\infty}\, x_n \leq \liminf_{k\to\infty}\, x_{n_k}\).

    Demostrar.

    1. Vamos\(x_n := \dfrac{{(-1)}^n}{n}\), encuentra\(\limsup \, x_n\) y\(\liminf \, x_n\).
    2. Vamos\(x_n := \dfrac{(n-1){(-1)}^n}{n}\), encuentra\(\limsup \, x_n\) y\(\liminf \, x_n\).

    Dejar\(\{ x_n \}\) y\(\{ y_n \}\) ser secuencias acotadas de tal manera que\(x_n \leq y_n\) para todos\(n\). Entonces demuéstralo\[\limsup_{n\to\infty} \, x_n \leq \limsup_{n\to\infty} \, y_n\] y\[\liminf_{n\to\infty} \, x_n \leq \liminf_{n\to\infty} \, y_n .\]

    Dejar\(\{ x_n \}\) y\(\{ y_n \}\) ser secuencias acotadas.

    1. Espectáculo que\(\{ x_n + y_n \}\) está acotado.
    2. Mostrar esa\[(\liminf_{n\to \infty}\, x_n) + (\liminf_{n\to \infty}\, y_n) \leq \liminf_{n\to \infty}\, (x_n+y_n) .\] pista: Encuentra una subsecuencia\(\{ x_{n_i}+y_{n_i} \}\) de\(\{ x_n + y_n \}\) que converja. Entonces encuentra una subsecuencia\(\{ x_{n_{m_i}} \}\) de\(\{ x_{n_i} \}\) que converja. Entonces aplica lo que sabes de límites.
    3. Encuentra un explícito\(\{ x_n \}\) y\(\{ y_n \}\) tal que\[(\liminf_{n\to \infty}\, x_n) + (\liminf_{n\to \infty}\, y_n) < \liminf_{n\to \infty}\, (x_n+y_n) .\] Pista: Busca ejemplos que no tengan límite.

    Dejar\(\{ x_n \}\) y\(\{ y_n \}\) ser secuencias delimitadas (del ejercicio anterior sabemos que\(\{ x_n + y_n \}\) está acotada).

    1. Demuestre esa\[(\limsup_{n\to \infty}\, x_n) + (\limsup_{n\to \infty}\, y_n) \geq \limsup_{n\to \infty}\, (x_n+y_n) .\] Pista: Ver ejercicio anterior.
    2. Encuentra un explícito\(\{ x_n \}\) y\(\{ y_n \}\) tal que\[(\limsup_{n\to \infty}\, x_n) + (\limsup_{n\to \infty}\, y_n) > \limsup_{n\to \infty}\, (x_n+y_n) .\] Pista: Ver ejercicio anterior.

    Si\(S \subset {\mathbb{R}}\) es un conjunto, entonces\(x \in {\mathbb{R}}\) es un punto de clúster si por cada\(\epsilon > 0\), el conjunto no\((x-\epsilon,x+\epsilon) \cap S \setminus \{ x \}\) está vacío. Es decir, si hay puntos de\(S\) arbitrariamente cercanos a\(x\). Por ejemplo,\(S := \{ \nicefrac{1}{n} : n \in {\mathbb{N}}\}\) tiene un punto de clúster único (solo uno)\(0\), pero\(0 \notin S\). Demostrar la siguiente versión del teorema de Bolzano-Weierstrass:

    Teorema. \(S \subset {\mathbb{R}}\)Sea un conjunto infinito acotado, entonces existe al menos un punto de clúster de\(S\).

    Pista: Si\(S\) es infinito, entonces\(S\) contiene un subconjunto infinitamente contable. Es decir, hay una secuencia\(\{ x_n \}\) de números distintos en\(S\).

    a) Demostrar que cualquier secuencia contiene una subsecuencia monótona. Pista: Llamar a\(n \in {\mathbb{N}}\) un pico si es\(a_m \leq a_n\) para todos\(m \geq n\). Hay dos posibilidades: o la secuencia tiene como máximo finitamente muchos picos, o tiene infinitamente muchos picos.
    b) Concluir el teorema de Bolzano-Weierstrass.

    Demostremos una versión más fuerte de. Supongamos que\(\{ x_n \}\) es una secuencia tal que cada subsecuencia\(\{ x_{n_i} \}\) tiene una subsecuencia\(\{ x_{n_{m_i}} \}\) que converge\(x\) a. a) Primer espectáculo que\(\{ x_n \}\) se rebote. b) Ahora mostrar que\(\{ x_n \}\) converge a\(x\).

    Dejar\(\{x_n\}\) ser una secuencia acotada.
    a) Demostrar que existe\(s\) tal que para cualquiera\(r > s\) existe\(M \in {\mathbb{N}}\) tal que por todo\(n \geq M\) lo que tenemos\(x_n < r\).
    b) Si\(s\) es un número como en a), entonces probar\(\limsup \, x_n \leq s\).
    c) Demostrar que si\(S\) es el conjunto de todos\(s\) como en a), entonces\(\limsup \, x_n = \inf \, S\).

    [ejercicio:infseqlimex] Supongamos que\(\{ x_n \}\) es tal que\(\liminf \, x_n = -\infty\),\(\limsup \, x_n = \infty\). a) Mostrar que no\(\{ x_n \}\) es convergente, y también que\(\lim \, x_n = \infty\) ni ni\(\lim \, x_n = -\infty\) es cierto. b) Encuentra un ejemplo de tal secuencia.

    [ejercicio:infseqlimlims] Dada una secuencia\(\{ x_n \}\). a) Demostrar que\(\lim \, x_n = \infty\) si y solo\(\liminf \, x_n = \infty\) si. b) Luego mostrar que\(\lim \, x_n = - \infty\) si y solo si\(\limsup \, x_n = -\infty\). c) Si\(\{ x_n \}\) es monótona creciente, mostrar que o bien\(\lim \, x_n\) existe y es finito o\(\lim \, x_n = \infty\).

    Secuencias de Cauchy

    Nota: 0.5—1 conferencia

    Muchas veces queremos describir un cierto número por una secuencia que converja a él. En este caso, es imposible utilizar el número mismo en la prueba de que la secuencia converge. Sería bueno que pudiéramos comprobar la convergencia sin conocer el límite.

    Una secuencia\(\{ x_n \}\) es una secuencia Cauchy 11 si por cada\(\epsilon > 0\) existe una\(M \in {\mathbb{N}}\) tal que para todos\(n \geq M\) y para todos\(k \geq M\) tenemos\[\left\lvert {x_n - x_k} \right\rvert < \epsilon .\]

    Intuitivamente lo que significa es que los términos de la secuencia eventualmente se acercan arbitrariamente entre sí. Esperaríamos que tal secuencia fuera convergente. Resulta que eso es cierto porque\({\mathbb{R}}\) tiene el. Primero, veamos algunos ejemplos.

    La secuencia\(\{ \nicefrac{1}{n} \}\) es una secuencia de Cauchy.

    Prueba: Dado\(\epsilon > 0\), encuentra\(M\) tal que\(M > \nicefrac{2}{\epsilon}\). Entonces para\(n,k \geq M\) que tengamos eso\(\nicefrac{1}{n} < \nicefrac{\epsilon}{2}\) y\(\nicefrac{1}{k} < \nicefrac{\epsilon}{2}\). Por lo tanto para\(n, k \geq M\) nosotros tenemos\[\left\lvert {\frac{1}{n} - \frac{1}{k}} \right\rvert \leq \left\lvert {\frac{1}{n}} \right\rvert + \left\lvert {\frac{1}{k}} \right\rvert < \frac{\epsilon}{2} + \frac{\epsilon}{2} = \epsilon.\]

    La secuencia\(\{ \frac{n+1}{n} \}\) es una secuencia de Cauchy.

    Prueba: Dado\(\epsilon > 0\), encuentra\(M\) tal que\(M > \nicefrac{2}{\epsilon}\). Entonces para\(n,k \geq M\) que tengamos eso\(\nicefrac{1}{n} < \nicefrac{\epsilon}{2}\) y\(\nicefrac{1}{k} < \nicefrac{\epsilon}{2}\). Por lo tanto para\(n, k \geq M\) nosotros tenemos\[\begin{split} \left\lvert {\frac{n+1}{n} - \frac{k+1}{k}} \right\rvert & = \left\lvert {\frac{k(n+1)-n(k+1)}{nk}} \right\rvert \\ & = \left\lvert {\frac{kn+k-nk-n}{nk}} \right\rvert \\ & = \left\lvert {\frac{k-n}{nk}} \right\rvert \\ & \leq \left\lvert {\frac{k}{nk}} \right\rvert + \left\lvert {\frac{-n}{nk}} \right\rvert \\ & = \frac{1}{n} + \frac{1}{k} < \frac{\epsilon}{2} + \frac{\epsilon}{2} = \epsilon . \end{split}\]

    Una secuencia de Cauchy está delimitada.

    Supongamos que\(\{ x_n \}\) es Cauchy. Escoge\(M\) tal que por todo\(n,k \geq M\) lo que tenemos\(\left\lvert {x_n-x_k} \right\rvert < 1\). En particular, tenemos eso para todos\(n \geq M\)\[\left\lvert {x_n - x_M} \right\rvert < 1 .\] O por el triángulo inverso la desigualdad,\(\left\lvert {x_n} \right\rvert - \left\lvert {x_M} \right\rvert \leq \left\lvert {x_n - x_M} \right\rvert < 1\). De ahí\(n \geq M\) porque tenemos\[\left\lvert {x_n} \right\rvert < 1 + \left\lvert {x_M} \right\rvert.\] Let\[B := \max \{ \left\lvert {x_1} \right\rvert, \left\lvert {x_2} \right\rvert, \ldots, \left\lvert {x_{M-1}} \right\rvert, 1+ \left\lvert {x_M} \right\rvert \} .\] Entonces\(\left\lvert {x_n} \right\rvert \leq B\) para todos\(n \in {\mathbb{N}}\).

    Una secuencia de números reales es Cauchy si y sólo si converge.

    \(\epsilon > 0\)Déjese dar y supongamos que\(\{ x_n \}\) converge a\(x\). Entonces existe\(M\) tal que para\(n \geq M\) nosotros tenemos\[\left\lvert {x_n - x} \right\rvert < \frac{\epsilon}{2} .\] Por lo tanto para\(n \geq M\) y\(k \geq M\) tenemos\[\left\lvert {x_n - x_k} \right\rvert = \left\lvert {x_n - x + x - x_k} \right\rvert \leq \left\lvert {x_n-x} \right\rvert + \left\lvert {x-x_k} \right\rvert < \frac{\epsilon}{2} + \frac{\epsilon}{2} = \epsilon .\]

    Bien, esa dirección fue fácil. Ahora supongamos que\(\{ x_n \}\) es Cauchy. Nosotros hemos demostrado que\(\{ x_n \}\) está acotado. Si lo demostramos\[\liminf_{n\to \infty} \, x_n = \limsup_{n\to\infty} \, x_n ,\] entonces\(\{ x_n \}\) debe ser convergente por. Suponiendo que liminf y limsup existen es donde usamos el.

    Definir\(a := \limsup \, x_n\) y\(b := \liminf \, x_n\). Por, existen subsecuencias\(\{ x_{n_i} \}\) y\(\{ x_{m_i} \}\), tal que\[\lim_{i\to\infty} x_{n_i} = a \qquad \text{and} \qquad \lim_{i\to\infty} x_{m_i} = b.\] dado un\(\epsilon > 0\), existe un\(M_1\) tal que por todo\(i \geq M_1\) lo que tenemos\(\left\lvert {x_{n_i} - a} \right\rvert < \nicefrac{\epsilon}{3}\) y un\(M_2\) tal que para todo\(i \geq M_2\) lo que tenemos\(\left\lvert {x_{m_i} - b} \right\rvert < \nicefrac{\epsilon}{3}\). También existe\(M_3\) tal que por todo\(n,k \geq M_3\) lo que tenemos\(\left\lvert {x_n-x_k} \right\rvert < \nicefrac{\epsilon}{3}\). Vamos\(M := \max \{ M_1, M_2, M_3 \}\). Tenga en cuenta que si\(i \geq M\), entonces\(n_i \geq M\) y\(m_i \geq M\). De ahí\[\begin{split} \left\lvert {a-b} \right\rvert & = \left\lvert {a-x_{n_i}+x_{n_i} -x_{m_i}+x_{m_i} -b} \right\rvert \\ & \leq \left\lvert {a-x_{n_i}} \right\rvert + \left\lvert {x_{n_i} -x_{m_i}} \right\rvert + \left\lvert {x_{m_i} -b} \right\rvert \\ & < \frac{\epsilon}{3} + \frac{\epsilon}{3} + \frac{\epsilon}{3} = \epsilon . \end{split}\] En\(\left\lvert {a-b} \right\rvert < \epsilon\) cuanto a todos\(\epsilon > 0\), entonces\(a=b\) y la secuencia converge.

    El enunciado de esta proposición se utiliza a veces para definir la propiedad de integridad de los números reales. Decimos que un conjunto es Cauchy-complete (o a veces simplemente completo) si cada secuencia de Cauchy converge. Arriba probamos que como\({\mathbb{R}}\) tiene el, entonces\({\mathbb{R}}\) es Cauchy-completo. Podemos “completar”\({\mathbb{Q}}\) “lanzando” los puntos suficientes para hacer converger todas las secuencias de Cauchy (omitimos los detalles). El campo resultante tiene la propiedad de límite mínimo superior. La ventaja de usar secuencias de Cauchy para definir la integridad es que esta idea se generaliza a configuraciones más abstractas.

    Cabe señalar que el criterio de Cauchy es más fuerte que solo\(\left\lvert {x_{n+1}-x_n} \right\rvert\) (o\(\left\lvert {x_{n+j}-x_n} \right\rvert\) para un fijo\(j\)) ir a cero como\(n\) va al infinito. De hecho, cuando lleguemos a las sumas parciales de la serie armónica (ver en la siguiente sección), tendremos una secuencia tal que\(x_{n+1}-x_n = \nicefrac{1}{n}\), sin embargo,\(\{ x_n \}\) es divergente. De hecho, para esa secuencia es cierto que\(\lim_{n\to\infty} \left\lvert {x_{n+j}-x_n} \right\rvert = 0\) para cualquier\(j \in {\mathbb{N}}\) (conferir). El punto clave en la definición de Cauchy es ese\(n\) y\(k\) varían independientemente y pueden estar arbitrariamente alejados.

    Ejercicios

    Demostrar que\(\{ \frac{n^2-1}{n^2} \}\) es Cauchy usando directamente la definición de secuencias de Cauchy.

    \(\{ x_n \}\)Sea una secuencia tal que exista\(0 < C < 1\) tal que\[\left\lvert {x_{n+1} - x_n} \right\rvert \leq C \left\lvert {x_{n}-x_{n-1}} \right\rvert .\] Probar que\(\{ x_n \}\) es Cauchy. Pista: Puedes usar libremente la fórmula (para\(C \not= 1\))\[1+ C+ C^2 + \cdots + C^n = \frac{1-C^{n+1}}{1-C}.\]

    Supongamos que\(F\) es un campo ordenado que contiene los números racionales\({\mathbb{Q}}\), tal que\({\mathbb{Q}}\) es denso, es decir: cuando\(x,y \in F\) son tales que\(x < y\), entonces existe\(q \in {\mathbb{Q}}\) tal que\(x < q < y\). Digamos que una secuencia\(\{ x_n \}_{n=1}^\infty\) de números racionales es Cauchy si se le da alguno\(\epsilon \in {\mathbb{Q}}\) con\(\epsilon > 0\), existe\(M\) tal que por todo\(n,k \geq M\) lo que tenemos\(\left\lvert {x_n-x_k} \right\rvert < \epsilon\). Supongamos que cualquier secuencia Cauchy de números racionales tiene un límite en\(F\). Demostrar que\(F\) tiene el.

    Dejar\(\{ x_n \}\) y\(\{ y_n \}\) ser secuencias tales que\(\lim\, y_n =0\). Supongamos que para todos\(k \in {\mathbb{N}}\) y para todos\(m \geq k\) tenemos\[\left\lvert {x_m-x_k} \right\rvert \leq y_k .\] Show que\(\{ x_n \}\) es Cauchy.

    Supongamos que una secuencia de Cauchy\(\{ x_n \}\) es tal que para cada\(M \in {\mathbb{N}}\), existe una\(k \geq M\) y una\(n \geq M\) tal que\(x_k < 0\) y\(x_n > 0\). Usando simplemente la definición de una secuencia de Cauchy y de una secuencia convergente, mostrar que la secuencia converge a\(0\).

    Supongamos\(\left\lvert {x_n-x_k} \right\rvert \leq \nicefrac{n}{k^2}\) para todos\(n\) y\(k\). Demostrar que\(\{ x_n \}\) es Cauchy.

    Supongamos que\(\{ x_n \}\) es una secuencia de Cauchy tal que para infinitamente muchos\(n\),\(x_n = c\). Utilizando sólo la definición de secuencia de Cauchy lo demuestran\(\lim\, x_n = c\).

    Verdadero/Falso probar o encontrar un contraejemplo: Si\(\{ x_n \}\) es una secuencia de Cauchy entonces existe\(M\) tal que por todo\(n \geq M\) lo que tenemos\(\left\lvert {x_{n+1}-x_n} \right\rvert \leq \left\lvert {x_{n}-x_{n-1}} \right\rvert\).

    Serie

    Nota: 2 conferencias

    Un objeto fundamental en matemáticas es el de una serie. De hecho, cuando se estaban desarrollando los fundamentos del análisis, la motivación era comprender las series. Comprender las series es muy importante en aplicaciones de análisis. Por ejemplo, resolver ecuaciones diferenciales a menudo incluye series, y las ecuaciones diferenciales son la base para comprender casi toda la ciencia moderna.

    Definición

    Dada una secuencia\(\{ x_n \}\), escribimos el objeto formal\[\sum_{n=1}^\infty x_n \qquad \text{or sometimes just} \qquad \sum x_n\] y lo llamamos serie. Una serie converge, si la secuencia\(\{ s_k \}\) definida por\[s_k := \sum_{n=1}^k x_n = x_1 + x_2 + \cdots + x_k ,\] converge. Los números\(s_k\) se llaman sumas parciales. Si\(x := \lim\, s_k\), escribimos\[\sum_{n=1}^\infty x_n = x .\] En este caso, hacemos trampa un poco y tratamos\(\sum_{n=1}^\infty x_n\) como un número.

    Por otro lado, si la secuencia\(\{ s_k \}\) diverge, decimos que la serie es divergente. En este caso,\(\sum x_n\) es simplemente un objeto formal y no un número.

    Es decir, para una serie convergente que tenemos\[\sum_{n=1}^\infty x_n = \lim_{k\to\infty} \sum_{n=1}^k x_n .\] Debemos tener cuidado de usar solo esta igualdad si realmente existe el límite a la derecha. Es decir, el lado derecho no tiene sentido (el límite no existe) si la serie no converge.

    Antes de ir más allá, comentemos que a veces es conveniente iniciar la serie en un índice diferente al 1. Es decir, por ejemplo podemos escribir\[\sum_{n=0}^\infty r^n = \sum_{n=1}^\infty r^{n-1} .\] El lado izquierdo es más conveniente para escribir. La idea es la misma que la notación para la cola de una secuencia.

    Es común escribir la serie\(\sum x_n\) como\[x_1 + x_2 + x_3 + \cdots\] con el entendimiento de que los puntos suspensivos indican una serie y no una simple suma. No utilizamos esta notación ya que a menudo conduce a errores en las pruebas.

    La serie\[\sum_{n=1}^\infty \frac{1}{2^n}\] converge y el límite es 1. Es decir,\[\sum_{n=1}^\infty \frac{1}{2^n} = \lim_{k\to\infty} \sum_{n=1}^k \frac{1}{2^n} = 1 .\]

    Prueba: Primero probamos la siguiente igualdad\[\left( \sum_{n=1}^k \frac{1}{2^n} \right) + \frac{1}{2^k} = 1 .\] La igualdad es fácil de ver cuándo\(k=1\). \(k\)Sigue el comprobante para general, que dejamos al lector. \(s_k\)Sea la suma parcial. Escribimos\[\left\lvert { 1 - s_k } \right\rvert = \left\lvert { 1 - \sum_{n=1}^k \frac{1}{2^n} } \right\rvert = \left\lvert {\frac{1}{2^k}} \right\rvert = \frac{1}{2^k} .\] La secuencia\(\{ \frac{1}{2^k} \}\) y por lo tanto\(\{ \left\lvert {1-s_k} \right\rvert \}\) converge a cero. Entonces,\(\{ s_k \}\) converge a 1.

    Para\(-1 < r < 1\), la serie geométrica\[\sum_{n=0}^\infty r^n\] converge. De hecho,\(\sum_{n=0}^\infty r^n = \frac{1}{1-r}\). La prueba se deja como ejercicio al lector. El comprobante consiste en mostrar\[\sum_{n=0}^{k-1} r^n = \frac{1-r^k}{1-r} ,\] y luego tomar el límite como\(k\) va a\(\infty\).

    Un hecho que a menudo usamos es el siguiente análogo de mirar la cola de una secuencia.

    Dejar\(\sum x_n\) ser una serie. Vamos\(M \in {\mathbb{N}}\). Entonces\[\sum_{n=1}^\infty x_n \quad \text{converges if and only if} \quad \sum_{n=M}^\infty x_n \quad \text{converges.}\]

    Nos fijamos en las sumas parciales de las dos series (para\(k \geq M\))\[\sum_{n=1}^{k} x_n = \left( \sum_{n=1}^{M-1} x_n \right) + \sum_{n=M}^{k} x_n .\] Tenga en cuenta que\(\sum_{n=1}^{M-1} x_n\) es un número fijo. Ahora usa para terminar la prueba.

    Serie Cauchy

    Se dice que una serie\(\sum x_n\) es Cauchy o una serie de Cauchy, si la secuencia de sumas parciales\(\{ s_n \}\) es una secuencia de Cauchy.

    Una secuencia de números reales converge si y sólo si es Cauchy. Por lo tanto una serie es convergente si y sólo si es Cauchy.

    La serie\(\sum x_n\) es Cauchy si por cada\(\epsilon > 0\), existe una\(M \in {\mathbb{N}}\), tal que para cada\(n \geq M\) y\(k \geq M\) tenemos\[\left\lvert { \left( \sum_{j=1}^k x_j \right) - \left( \sum_{j=1}^n x_j \right) } \right\rvert < \epsilon .\] Sin pérdida de generalidad asumimos\(n < k\). Entonces escribimos\[\left\lvert { \left( \sum_{j=1}^k x_j \right) - \left( \sum_{j=1}^n x_j \right) } \right\rvert = \left\lvert { \sum_{j={n+1}}^k x_j } \right\rvert < \epsilon .\] Hemos demostrado la siguiente proposición simple.

    [prop:cachyser] La serie\(\sum x_n\) es Cauchy si por cada\(\epsilon > 0\), existe\(M \in {\mathbb{N}}\) tal que para todos\(n \geq M\) y cada uno que\(k > n\) tenemos\[\left\lvert { \sum_{j={n+1}}^k x_j } \right\rvert < \epsilon .\]

    Propiedades básicas

    \(\sum x_n\)Déjese ser una serie convergente. Entonces la secuencia\(\{ x_n \}\) es convergente y\[\lim_{n\to\infty} x_n = 0.\]

    Dejemos\(\epsilon > 0\) que se den. Como\(\sum x_n\) es convergente, es Cauchy. Así encontramos un\(M\) tal que por cada que\(n \geq M\) tenemos\[\epsilon > \left\lvert { \sum_{j={n+1}}^{n+1} x_j } \right\rvert = \left\lvert { x_{n+1} } \right\rvert .\] De ahí por cada\(n \geq M+1\) que tenemos\(\left\lvert {x_{n}} \right\rvert < \epsilon\).

    Entonces, si una serie converge, los términos de la serie van a cero. La implicación, sin embargo, va sólo en una dirección. Demos un ejemplo.

    [ejemplo:armónicasseries] La serie\(\sum \frac{1}{n}\) diverge (a pesar de que\(\lim \frac{1}{n} = 0\)). Se trata de la famosa serie armónica 12.

    Prueba: Demostraremos que la secuencia de sumas parciales no tiene límites, y por lo tanto no puede converger. Escribir las sumas parciales\(s_n\) para\(n = 2^k\) como:\[\begin{aligned} s_1 & = 1 , \\ s_2 & = \left( 1 \right) + \left( \frac{1}{2} \right) , \\ s_4 & = \left( 1 \right) + \left( \frac{1}{2} \right) + \left( \frac{1}{3} + \frac{1}{4} \right) , \\ s_8 & = \left( 1 \right) + \left( \frac{1}{2} \right) + \left( \frac{1}{3} + \frac{1}{4} \right) + \left( \frac{1}{5} + \frac{1}{6} + \frac{1}{7} + \frac{1}{8} \right) , \\ & ~~ \vdots \\ s_{2^k} & = 1 + \sum_{j=1}^k \left( \sum_{m=2^{j-1}+1}^{2^j} \frac{1}{m} \right) .\end{aligned}\] Tomamos nota de que\(\nicefrac{1}{3} + \nicefrac{1}{4} \geq \nicefrac{1}{4} + \nicefrac{1}{4} = \nicefrac{1}{2}\) y\(\nicefrac{1}{5} + \nicefrac{1}{6} + \nicefrac{1}{7} + \nicefrac{1}{8} \geq \nicefrac{1}{8} + \nicefrac{1}{8} + \nicefrac{1}{8} + \nicefrac{1}{8} = \nicefrac{1}{2}\). De manera más general\[\sum_{m=2^{k-1}+1}^{2^k} \frac{1}{m} \geq \sum_{m=2^{k-1}+1}^{2^k} \frac{1}{2^k} = (2^{k-1}) \frac{1}{2^k} = \frac{1}{2} .\] Por lo tanto\[s_{2^k} = 1 + \sum_{j=1}^k \left( \sum_{m=2^{k-1}+1}^{2^k} \frac{1}{m} \right) \geq 1 + \sum_{j=1}^k \frac{1}{2} = 1 + \frac{k}{2} .\] Como no\(\{ \frac{k}{2} \}\) está delimitado por el, eso significa que no\(\{ s_{2^k} \}\) está acotado, y por lo tanto no\(\{ s_n \}\) está acotado. De ahí\(\{ s_n \}\) diverge, y consecuentemente\(\sum \frac{1}{n}\) diverge.

    Las series convergentes son lineales. Es decir, podemos multiplicarlos por constantes y sumarlos y estas operaciones se hacen término por término.

    Dejar\(\alpha \in {\mathbb{R}}\)\(\sum x_n\) y\(\sum y_n\) ser series convergentes. Entonces

    1. \(\sum \alpha x_n\)es una serie convergente y\[\sum_{n=1}^\infty \alpha x_n = \alpha \sum_{n=1}^\infty x_n .\]
    2. \(\sum ( x_n + y_n )\)es una serie convergente y\[\sum_{n=1}^\infty ( x_n + y_n ) = \left( \sum_{n=1}^\infty x_n \right) + \left( \sum_{n=1}^\infty y_n \right) .\]

    Para el primer ítem, simplemente escribimos la\(k\) th suma parcial\[\sum_{n=1}^k \alpha x_n = \alpha \left( \sum_{n=1}^k x_n \right) .\] Miramos al lado derecho y notamos que el múltiplo constante de una secuencia convergente es convergente. De ahí que simplemente tomemos el límite de ambas partes para obtener el resultado.

    Para el segundo ítem también nos fijamos en la\(k\) ésima suma parcial\[\sum_{n=1}^k ( x_n + y_n ) = \left( \sum_{n=1}^k x_n \right) + \left( \sum_{n=1}^k y_n \right) .\] Nos fijamos en el lado derecho y observamos que la suma de secuencias convergentes es convergente. De ahí que simplemente tomemos el límite de ambas partes para obtener la proposición.

    Tenga en cuenta que multiplicar series no es tan simple como agregar, consulte la siguiente sección. No es cierto, claro, que podamos multiplicar término por término, ya que esa estrategia no funciona ni siquiera para sumas finitas. Por ejemplo,\((a+b)(c+d) \not= ac+bd\).

    Convergencia absoluta

    Dado que las secuencias monótonas son más fáciles de trabajar que las secuencias arbitrarias, generalmente es más fácil trabajar con series\(\sum x_n\) donde\(x_n \geq 0\) para todos\(n\). Entonces la secuencia de sumas parciales es monótona aumentando y converge si se limita desde arriba. Formalicemos esta declaración como una proposición.

    Si\(x_n \geq 0\) para todos\(n\), entonces\(\sum x_n\) converge si y sólo si la secuencia de sumas parciales está delimitada desde arriba.

    Como el límite de una secuencia monótona creciente es el supremo, tienen la desigualdad\[\sum_{n=1}^k x_n \leq \sum_{n=1}^\infty x_n .\]

    El siguiente criterio a menudo da una manera conveniente de probar la convergencia de una serie.

    Una serie\(\sum x_n\) converge absolutamente si la serie\(\sum \left\lvert {x_n} \right\rvert\) converge. Si una serie converge, pero no converge absolutamente, decimos que es condicionalmente convergente.

    Si la serie\(\sum x_n\) converge absolutamente, entonces converge.

    Una serie es convergente si y sólo si es Cauchy. De ahí supongamos que\(\sum \left\lvert {x_n} \right\rvert\) es Cauchy. Es decir, para cada\(\epsilon > 0\), existe\(M\) tal que para todos\(k \geq M\) y\(n > k\) tenemos\[\sum_{j=k+1}^n \left\lvert {x_j} \right\rvert = \left\lvert { \sum_{j=k+1}^n \left\lvert {x_j} \right\rvert } \right\rvert < \epsilon .\] Aplicamos el triángulo de desigualdad para una suma finita para obtener de\[\left\lvert { \sum_{j=k+1}^n x_j } \right\rvert \leq \sum_{j=k+1}^n \left\lvert {x_j} \right\rvert < \epsilon .\] ahí\(\sum x_n\) es Cauchy y por lo tanto converge.

    Por supuesto, si\(\sum x_n\) converge absolutamente, los límites de\(\sum x_n\) y\(\sum \left\lvert {x_n} \right\rvert\) son diferentes. Computar uno no nos ayuda a calcular el otro.

    Las series absolutamente convergentes tienen muchas propiedades maravillosas. Por ejemplo, las series absolutamente convergentes se pueden reorganizar arbitrariamente, o podemos multiplicar tales series juntas fácilmente. Las series condicionalmente convergentes por otro lado no suelen comportarse como cabría esperar. Ver la siguiente sección.

    Salimos como ejercicio para mostrar que\[\sum_{n=1}^\infty \frac{{(-1)}^n}{n}\] converge, aunque el lector debe terminar esta sección antes de intentarlo. Por otro lado probamos\[\sum_{n=1}^\infty \frac{1}{n}\] divergencias. Por lo tanto,\(\sum \frac{{(-1)}^n}{n}\) es una subsecuencia condicionalmente convergente.

    Prueba de comparación y la serie p

    Hemos señalado anteriormente que para que una serie converja los términos no sólo tienen que ir a cero, sino que tienen que ir a cero “lo suficientemente rápido”. Si conocemos la convergencia de una serie determinada podemos usar la siguiente prueba de comparación para ver si los términos de otra serie van a cero “lo suficientemente rápido”.

    Dejar\(\sum x_n\) y\(\sum y_n\) ser series tales que\(0 \leq x_n \leq y_n\) para todos\(n \in {\mathbb{N}}\).

    1. Si\(\sum y_n\) converge, entonces también lo hace\(\sum x_n\).
    2. Si\(\sum x_n\) diverge, entonces también lo hace\(\sum y_n\).

    Dado que los términos de la serie son todos no negativos, las secuencias de sumas parciales son monótonas crecientes. Ya que\(x_n \leq y_n\) para todos\(n\), las sumas parciales satisfacen para todos\(k\)\[\label{comptest:eq} \sum_{n=1}^k x_n \leq \sum_{n=1}^k y_n .\] Si la serie\(\sum y_n\) converge las sumas parciales para la serie están acotadas. Por lo tanto, el lado derecho de [comptest:eq] está acotado para todos\(k\). De ahí que también\(\sum x_n\) se acoplan las sumas parciales para. Dado que las sumas parciales son una secuencia monótona creciente, son convergentes. El primer ítem queda así probado.

    Por otro lado si\(\sum x_n\) diverge, la secuencia de sumas parciales debe estar sin límites ya que es monótona creciente. Es decir, las sumas parciales para\(\sum x_n\) son eventualmente mayores que cualquier número real. Armando esto con [comptest:eq] vemos que para cualquiera\(B \in {\mathbb{R}}\), hay\(k\) tal que de\[B \leq \sum_{n=1}^k x_n \leq \sum_{n=1}^k y_n .\] ahí las sumas parciales para también\(\sum y_n\) están ilimitadas, y\(\sum y_n\) también divergen.

    Una serie útil para usar con la prueba de comparación es la\(p\) serie -.

    Para\(p \in {\mathbb{R}}\), la serie\[\sum_{n=1}^\infty \frac{1}{n^p}\] converge si y solo si\(p > 1\).

    Primero supongamos\(p \leq 1\). Como\(n \geq 1\), tenemos\(\frac{1}{n^p} \geq \frac{1}{n}\). Dado que\(\sum \frac{1}{n}\) diverge, vemos que el\(\sum \frac{1}{n^p}\) debe divergir para todos\(p \leq 1\) por la prueba de comparación.

    Ahora supongamos\(p > 1\). Procedemos de una manera similar a la que hicimos en el caso de la serie armónica, pero en lugar de mostrar que la secuencia de sumas parciales no tiene límites demostramos que está acotada. Dado que los términos de la serie son positivos, la secuencia de sumas parciales es monótona creciente y convergerá si demostramos que está delimitada arriba. Dejar\(s_n\) denotar la\(n\) ésima suma parcial. \[\begin{aligned} s_1 & = 1 , \\ s_3 & = \left( 1 \right) + \left( \frac{1}{2^p} + \frac{1}{3^p} \right) , \\ s_7 & = \left( 1 \right) + \left( \frac{1}{2^p} + \frac{1}{3^p} \right) + \left( \frac{1}{4^p} + \frac{1}{5^p} + \frac{1}{6^p} + \frac{1}{7^p} \right) , \\ & ~~ \vdots \\ s_{2^k - 1} &= 1 + \sum_{j=1}^{k-1} \left( \sum_{m=2^j}^{2^{j+1}-1} \frac{1}{m^p} \right) .\end{aligned}\]En lugar de estimar desde abajo, estimamos desde arriba. En particular, como\(p\) es positivo, entonces\(2^p < 3^p\), y por lo tanto\(\frac{1}{2^p} + \frac{1}{3^p} < \frac{1}{2^p} + \frac{1}{2^p}\). De igual manera\(\frac{1}{4^p} + \frac{1}{5^p} + \frac{1}{6^p} + \frac{1}{7^p} < \frac{1}{4^p} + \frac{1}{4^p} + \frac{1}{4^p} + \frac{1}{4^p}\). Por lo tanto\[\begin{split} s_{2^k-1} & = 1+ \sum_{j=1}^k \left( \sum_{m=2^{j}}^{2^{j+1}-1} \frac{1}{m^p} \right) \\ & < 1+ \sum_{j=1}^k \left( \sum_{m=2^{j}}^{2^{j+1}-1} \frac{1}{{(2^j)}^p} \right) \\ & = 1+ \sum_{j=1}^k \left( \frac{2^j}{{(2^j)}^p} \right) \\ & = 1+ \sum_{j=1}^k {\left( \frac{1}{2^{p-1}} \right)}^j . \end{split}\] As\(p > 1\), entonces\(\frac{1}{2^{p-1}} < 1\). Entonces al usar el resultado de, observamos que\[\sum_{j=1}^\infty {\left( \frac{1}{2^{p-1}} \right)}^j\] converge. Por lo tanto\[s_{2^k-1} < 1+ \sum_{j=1}^k {\left( \frac{1}{2^{p-1}} \right)}^j \leq 1+ \sum_{j=1}^\infty {\left( \frac{1}{2^{p-1}} \right)}^j .\] Como\(\{ s_n \}\) es una secuencia monótona, entonces todo\(s_n \leq s_{2^k-1}\) para todos\(n \leq 2^k-1\). Así para todos\(n\),\[s_n < 1+ \sum_{j=1}^\infty {\left( \frac{1}{2^{p-1}} \right)}^j .\] La secuencia de sumas parciales es acotada y por lo tanto converge.

    Tenga en cuenta que ni la prueba\(p\) -series ni la prueba de comparación nos dicen a qué converge la suma. Sólo nos dicen que existe un límite de las sumas parciales. Por ejemplo, si bien sabemos que\(\sum \nicefrac{1}{n^2}\) converge es mucho más difícil encontrar 13 que sea el límite\(\nicefrac{\pi^2}{6}\). Si tratamos\(\sum \nicefrac{1}{n^p}\) como una función de\(p\), obtenemos la llamada\(\zeta\) función de Riemann. Comprender el comportamiento de esta función contiene uno de los problemas no resueltos más famosos de las matemáticas en la actualidad y tiene aplicaciones en áreas aparentemente no relacionadas como la criptografía moderna.

    La serie\(\sum \frac{1}{n^2+1}\) converge.

    Prueba: Primero tenga en cuenta que\(\frac{1}{n^2+1} < \frac{1}{n^2}\) para todos\(n \in {\mathbb{N}}\). Tenga en cuenta que\(\sum \frac{1}{n^2}\) converge por la prueba\(p\) -series. Por lo tanto, por la prueba de comparación,\(\sum \frac{1}{n^2+1}\) converge.

    Prueba de relación

    Déjese\(\sum x_n\) ser una serie tal que\[L := \lim_{n\to\infty} \frac{\left\lvert {x_{n+1}} \right\rvert}{\left\lvert {x_n} \right\rvert}\] existe. Entonces

    1. Si\(L < 1\), entonces\(\sum x_n\) converge absolutamente.
    2. Si\(L > 1\), entonces\(\sum x_n\) diverge.

    De notamos que si\(L > 1\), entonces\(x_n\) diverge. Ya que es una condición necesaria para la convergencia de series que los términos vayan a cero, sabemos que\(\sum x_n\) debe divergir.

    Así supongamos\(L < 1\). Vamos a argumentar que\(\sum \left\lvert {x_n} \right\rvert\) debe converger. La prueba es similar a la de. Por supuesto\(L \geq 0\). Escoge\(r\) tal que\(L < r < 1\). Como\(r-L > 0\), existe\(M \in {\mathbb{N}}\) tal que para todos\(n \geq M\)\[\left\lvert {\frac{\left\lvert {x_{n+1}} \right\rvert}{\left\lvert {x_n} \right\rvert} - L} \right\rvert < r-L .\] Por lo tanto,\[\frac{\left\lvert {x_{n+1}} \right\rvert}{\left\lvert {x_n} \right\rvert} < r .\] For\(n > M\) (es decir para\(n \geq M+1\)) escribir\[\left\lvert {x_n} \right\rvert = \left\lvert {x_M} \right\rvert \frac{\left\lvert {x_{M+1}} \right\rvert}{\left\lvert {x_{M}} \right\rvert} \frac{\left\lvert {x_{M+2}} \right\rvert}{\left\lvert {x_{M+1}} \right\rvert} \cdots \frac{\left\lvert {x_{n}} \right\rvert}{\left\lvert {x_{n-1}} \right\rvert} < \left\lvert {x_M} \right\rvert r r \cdots r = \left\lvert {x_M} \right\rvert r^{n-M} = (\left\lvert {x_M} \right\rvert r^{-M}) r^n .\] Para\(k > M\) escribimos la suma parcial\[\begin{split} \sum_{n=1}^k \left\lvert {x_n} \right\rvert & = \left(\sum_{n=1}^{M} \left\lvert {x_n} \right\rvert \right) + \left(\sum_{n=M+1}^{k} \left\lvert {x_n} \right\rvert \right) \\ & \leq \left(\sum_{n=1}^{M} \left\lvert {x_n} \right\rvert \right) + \left(\sum_{n=M+1}^{k} (\left\lvert {x_M} \right\rvert r^{-M}) r^n \right) \\ & \leq \left(\sum_{n=1}^{M} \left\lvert {x_n} \right\rvert \right) + (\left\lvert {x_M} \right\rvert r^{-M}) \left( \sum_{n=M+1}^{k} r^n \right) . \end{split}\] como A medida que\(\sum_{n=0}^{\infty} r^n\) converge\(0 < r < 1\) la serie geométrica, así\(\sum_{n=M+1}^{\infty} r^n\) converge también (¿por qué?). Tomamos el límite como\(k\) va al infinito en el lado derecho arriba para obtener\[\begin{split} \sum_{n=1}^k \left\lvert {x_n} \right\rvert & \leq \left(\sum_{n=1}^{M} \left\lvert {x_n} \right\rvert \right) + (\left\lvert {x_M} \right\rvert r^{-M}) \left( \sum_{n=M+1}^{k} r^n \right) \\ & \leq \left(\sum_{n=1}^{M} \left\lvert {x_n} \right\rvert \right) + (\left\lvert {x_M} \right\rvert r^{-M}) \left( \sum_{n=M+1}^{\infty} r^n \right) . \end{split}\] El lado derecho es un número del que no depende\(n\). De ahí que la secuencia de sumas parciales de\(\sum \left\lvert {x_n} \right\rvert\) es acotada y\(\sum \left\lvert {x_n} \right\rvert\) convergente. Así\(\sum x_n\) es absolutamente convergente.

    La serie\[\sum_{n=1}^\infty \frac{2^n}{n!}\] converge absolutamente.

    Prueba: Escribimos\[\lim_{n\to\infty} \frac{2^{(n+1)}/(n+1)!}{2^n / n!} = \lim_{n\to\infty} \frac{2}{n+1} = 0 .\] Por lo tanto, la serie converge absolutamente por la prueba de ratio.

    Ejercicios

    Para\(r \not= 1\), probar\[\sum_{k=0}^{n-1} r^k = \frac{1-r^n}{1-r} .\] Pista: Dejar\(s := \sum_{k=0}^{n-1} r^k\), luego calcular\(s(1-r) = s-rs\), y resolver para\(s\).

    [geometric:exr] Demostrar que para\(-1 < r < 1\) tenemos\[\sum_{n=0}^\infty r^n = \frac{1}{1-r} .\] Sugerencia: Utilice el ejercicio anterior.

    Decidir la convergencia o divergencia de las siguientes series.

    a)\(\displaystyle \sum_{n=1}^\infty \frac{3}{9n+1}\) b)\(\displaystyle \sum_{n=1}^\infty \frac{1}{2n-1}\) c)\(\displaystyle \sum_{n=1}^\infty \frac{{(-1)}^n}{n^2}\)
    d)\(\displaystyle \sum_{n=1}^\infty \frac{1}{n(n+1)}\) e)\(\displaystyle \sum_{n=1}^\infty n e^{-n^2}\)

    1. Demostrar que si\(\displaystyle \sum_{n=1}^\infty x_n\) converge, entonces\(\displaystyle \sum_{n=1}^\infty ( x_{2n} + x_{2n+1} )\) también converge.
    2. Encuentra un ejemplo explícito donde no se sostiene lo contrario.

    Para\(j=1,2,\ldots,n\), vamos a\(\{ x_{j,k} \}_{k=1}^\infty\) denotar\(n\) secuencias. Supongamos que para cada uno\(j\)\[\sum_{k=1}^\infty x_{j,k}\] es convergente. A continuación, mostrar\[\sum_{j=1}^n \left( \sum_{k=1}^\infty x_{j,k} \right) = \sum_{k=1}^\infty \left( \sum_{j=1}^n x_{j,k} \right) .\]

    Demuestra la siguiente versión más fuerte de la prueba de ratio: Let\(\sum x_n\) be a series.

    1. Si hay un\(N\) y\(\rho < 1\) tal que por todo\(n \geq N\) lo que tenemos\(\frac{\left\lvert {x_{n+1}} \right\rvert}{\left\lvert {x_n} \right\rvert} < \rho\), entonces la serie converge absolutamente.
    2. Si hay\(N\) tal que por todo\(n \geq N\) lo que tenemos\(\frac{\left\lvert {x_{n+1}} \right\rvert}{\left\lvert {x_n} \right\rvert} \geq 1\), entonces la serie diverge.

    Dejar\(\{ x_n \}\) ser una secuencia decreciente tal que\(\sum x_n\) converja. \(\displaystyle \lim_{n\to\infty} n x_n = 0\)Demuéstralo.

    Demostrar que\(\displaystyle \sum_{n=1}^\infty \frac{{(-1)}^n}{n}\) converge. Pista: considere la suma de dos entradas posteriores.

    1. Demostrar que si\(\sum x_n\) y\(\sum y_n\) convergen absolutamente, entonces\(\sum x_ny_n\) converge absolutamente.
    2. Encuentra un ejemplo explícito donde no se sostiene lo contrario.
    3. Encuentra un ejemplo explícito donde las tres series son absolutamente convergentes, no son solo sumas finitas, y\((\sum x_n)(\sum y_n) \not= \sum x_ny_n\). Es decir, mostrar que las series no se multiplican término por término.

    Demostrar la desigualdad triangular para series, es decir, si\(\sum x_n\) converge absolutamente entonces\[\left\lvert {\sum_{n=1}^\infty x_n} \right\rvert \leq \sum_{n=1}^\infty \left\lvert {x_n} \right\rvert .\]

    Demostrar la prueba de comparación de límites. Es decir, demostrar que si\(a_n > 0\) y\(b_n > 0\) para todos\(n\), y\[0 < \lim_{n\to\infty} \frac{a_n}{b_n} < \infty ,\] entonces cualquiera\(\sum a_n\) y\(\sum b_n\) ambos convergen de ambos divergen.

    [ejercicio:badnocauchy] Vamos\(x_n = \sum_{j=1}^n \nicefrac{1}{j}\). Demuestre que por cada que\(k\) tenemos\(\displaystyle \lim_{n\to\infty} \left\lvert {x_{n+k}-x_n} \right\rvert = 0\), sin embargo no\(\{ x_n \}\) es Cauchy.

    \(s_k\)Sea la\(k\) th suma parcial de\(\sum x_n\).
    a) Supongamos que existe\(m \in {\mathbb{N}}\) tal que\(\displaystyle \lim_{k\to\infty} s_{mk}\) existe y\(\lim\, x_n = 0\). Demostrar que\(\sum x_n\) converge.
    b) Encontrar un ejemplo donde\(\displaystyle \lim_{k\to\infty} s_{2k}\) exista y\(\lim\, x_n \not= 0\) (y por lo tanto\(\sum x_n\) diverja).
    c) (Desafiante) Encuentra un ejemplo donde\(\lim\, x_n = 0\), y existe una subsecuencia\(\{ s_{k_j} \}\) tal que\(\displaystyle \lim_{j\to\infty} s_{k_j}\) existe, pero\(\sum x_n\) aún diverge.

    Más sobre series

    Nota: hasta 2—3 conferencias (opcional, puede omitirse o cubrirse parcialmente de forma segura)

    Prueba de raíz

    Ya hemos visto la prueba de ratio antes. Hay una prueba más similar llamada prueba raíz. De hecho, la prueba de esta prueba es similar y algo más fácil.

    Dejemos\(\sum x_n\) ser una serie y dejar\[L := \limsup_{n\to\infty} \, {\left\lvert {x_n} \right\rvert}^{1/n} .\] Entonces

    1. Si\(L < 1\) entonces\(\sum x_n\) converge absolutamente.
    2. Si\(L > 1\) entonces\(\sum x_n\) diverge.

    Si\(L > 1\), entonces existe una subsecuencia\(\{ x_{n_k} \}\) tal que\(L = \lim_{k\to\infty} \, {\left\lvert {x_{n_k}} \right\rvert}^{1/n_k}\). Que\(r\) sea tal que\(L > r > 1\). Existe\(M\) tal que para todos\(k \geq M\), tenemos\({\left\lvert {x_{n_k}} \right\rvert}^{1/n_k} > r > 1\), o en otras palabras\(\left\lvert {x_{n_k}} \right\rvert > r^{n_k} > 1\). La subsecuencia\(\{ \left\lvert {x_{n_k}} \right\rvert \}\), y por lo tanto también\(\{ \left\lvert {x_{n}} \right\rvert \}\), no puede converger a cero, por lo que la serie diverge.

    Ahora supongamos\(L < 1\). Escoge\(r\) tal que\(L < r < 1\). Por definición de límite supremum, escoja\(M\) tal que para todo lo que\(n \geq M\) tenemos\[\sup \{ {\left\lvert {x_k} \right\rvert}^{1/k} : k \geq n \} < r .\] Por lo tanto, para todos\(n \geq M\) tenemos\[{\left\lvert {x_n} \right\rvert}^{1/n} < r , \qquad \text{or in other words} \qquad \left\lvert {x_n} \right\rvert < r^n .\] Let\(k > M\) y vamos a estimar la\(k\) th suma parcial\[\sum_{n=1}^k \left\lvert {x_n} \right\rvert = \left( \sum_{n=1}^M \left\lvert {x_n} \right\rvert \right) + \left( \sum_{n=M+1}^k \left\lvert {x_n} \right\rvert \right) \leq \left( \sum_{n=1}^M \left\lvert {x_n} \right\rvert \right) + \left( \sum_{n=M+1}^k r^n \right) .\] As\(0 < r < 1\), la serie geométrica\(\sum_{n=M+1}^\infty r^n\) converge a\(\frac{r^{M+1}}{1-r}\). Como todo es positivo tenemos\[\sum_{n=1}^k \left\lvert {x_n} \right\rvert \leq \left( \sum_{n=1}^M \left\lvert {x_n} \right\rvert \right) + \frac{r^{M+1}}{1-r} .\] Así la secuencia de sumas parciales de\(\sum \left\lvert {x_n} \right\rvert\) es acotada, y así la serie converge. Por lo tanto\(\sum x_n\) converge absolutamente.

    Prueba en serie alterna

    Las pruebas que hasta ahora solo hemos abordado la convergencia absoluta. La siguiente prueba da un gran suministro de series condicionalmente convergentes.

    Dejar\(\{ x_n \}\) ser una secuencia decreciente monótona de números reales positivos tales que\(\lim\, x_n = 0\). Entonces\[\sum_{n=1}^\infty {(-1)}^n x_n\] converge.

    Escriba\(s_m := \sum_{k=1}^m {(-1)}^k x_k\) ser la suma parcial\(m\) th. Entonces escribe\[s_{2n} = \sum_{k=1}^{2n} {(-1)}^k x_k = (-x_1 + x_2) + \cdots + (-x_{2n-1} + x_{2n}) = \sum_{k=1}^{n} (-x_{2k-1} + x_{2k}) .\] La secuencia\(\{ x_k \}\) es decreciente y así\((-x_{2k-1}+x_{2k}) \leq 0\) para todos\(k\). Por lo tanto, la subsecuencia\(\{ s_{2n} \}\) de sumas parciales es una secuencia decreciente. De igual manera\((x_{2k}-x_{2k+1}) \geq 0\),, y así\[s_{2n} = - x_1 + ( x_2 - x_3 ) + \cdots + ( x_{2n-2} - x_{2n-1} ) + x_{2n} \geq -x_1 .\] La secuencia\(\{ s_{2n} \}\) es decreciente y delimitada por debajo, por lo que converge. Vamos\(a := \lim\, s_{2n}\).

    Deseamos mostrar eso\(\lim\, s_m = a\) (no solo para la subsecuencia). Aviso\[s_{2n+1} = s_{2n} + x_{2n+1} .\] Dado\(\epsilon > 0\), elige\(M\) tal que\(\left\lvert {s_{2n}-a} \right\rvert < \nicefrac{\epsilon}{2}\) siempre que\(2n \geq M\). Ya que\(\lim\, x_n = 0\), también hacemos\(M\) posiblemente más grandes para obtener\(x_{2n+1} < \nicefrac{\epsilon}{2}\) cuando sea\(2n \geq M\). Si\(2n \geq M\), tenemos\(\left\lvert {s_{2n}-a} \right\rvert < \nicefrac{\epsilon}{2} < \epsilon\), entonces solo necesitamos verificar la situación para\(s_{2n+1}\):\[\left\lvert {s_{2n+1}-a} \right\rvert = \left\lvert {s_{2n}-a + x_{2n+1}} \right\rvert \leq \left\lvert {s_{2n}-a} \right\rvert + x_{2n+1} < \nicefrac{\epsilon}{2}+ \nicefrac{\epsilon}{2} = \epsilon . \qedhere\]

    En particular, existen series condicionalmente convergentes donde los valores absolutos de los términos van a cero arbitrariamente lentamente. Por ejemplo,\[\sum_{n=1}^\infty \frac{{(-1)}^n}{n^p}\] converge por arbitrariamente pequeños\(p > 0\), pero no converge absolutamente cuando\(p \leq 1\).

    Reordenamientos

    Generalmente, las series absolutamente convergentes se comportan como imaginamos que deberían. Por ejemplo, las series absolutamente convergentes se pueden sumar en cualquier orden que sea. Nada de eso sirve para series condicionalmente convergentes (ver y).

    Tomar una serie\[\sum_{n=1}^\infty x_n .\] Dada una función biyectiva\(\sigma \colon {\mathbb{N}}\to {\mathbb{N}}\), el reordenamiento correspondiente es la siguiente serie: Simplemente\[\sum_{k=1}^\infty x_{\sigma(k)} .\] sumamos las series en un orden diferente.

    Let\(\sum x_n\) Ser una serie absolutamente convergente convergente convergente a un número\(x\). \(\sigma \colon {\mathbb{N}}\to {\mathbb{N}}\)Déjese ser una biyección. Entonces\(\sum x_{\sigma(n)}\) es absolutamente convergente y converge a\(x\).

    En otras palabras, un reordenamiento de una serie absolutamente convergente converge (absolutamente) al mismo número.

    Dejemos\(\epsilon > 0\) que se den. Entonces tomar\(M\) para ser tal que\[\left\lvert {\left(\sum_{n=1}^M x_n \right) - x} \right\rvert < \frac{\epsilon}{2} \qquad \text{and} \qquad \sum_{n=M+1}^\infty \left\lvert {x_n} \right\rvert < \frac{\epsilon}{2} .\] Como\(\sigma\) es una bijección, existe un número\(K\) tal que para cada uno\(n \leq M\), existe\(k \leq K\) tal que\(\sigma(k) = n\). En otras palabras\(\{ 1,2,\ldots,M \} \subset \sigma\bigl(\{ 1,2,\ldots,K \} \bigr)\).

    Entonces para cualquiera\(N \geq K\), vamos\(Q := \max \sigma(\{ 1,2,\ldots,K \})\) y computar\[\begin{split} \left\lvert {\left( \sum_{n=1}^N x_{\sigma(n)} \right) - x} \right\rvert & = \left\lvert { \left( \sum_{n=1}^M x_n + \sum_{\substack{n=1\\\sigma(n) > M}}^N x_{\sigma(n)} \right) - x} \right\rvert \\ & \leq \left\lvert { \left( \sum_{n=1}^M x_n \right) - x} \right\rvert + \sum_{\substack{n=1\\\sigma(n) > M}}^N \left\lvert {x_{\sigma(n)}} \right\rvert \\ & \leq \left\lvert { \left( \sum_{n=1}^M x_n \right) - x} \right\rvert + \sum_{n=M+1}^Q \left\lvert {x_{n}} \right\rvert \\ & < \nicefrac{\epsilon}{2} + \nicefrac{\epsilon}{2} = \epsilon . \end{split}\] Así\(\sum x_{\sigma(n)}\) converge a\(x\). Para ver que la convergencia es absoluta, aplicamos el argumento anterior\(\sum \left\lvert {x_n} \right\rvert\) para mostrar que\(\sum \left\lvert {x_{\sigma(n)}} \right\rvert\) converge.

    [ejemplo:armonsumanything] Demostremos que la serie armónica alterna\(\sum \frac

    ParseError: EOF expected (click for details)
    Callstack:
        at (Matematicas/Analisis/Introducción_al_Análisis_Real_(Lebl)/02:_Números_reales/2.01:_Propiedades_básicas), /content/body/div[14]/div/div[3]/p[7]/span[2]/span, line 1, column 5
    
    {n}\), que no converge absolutamente, se puede reorganizar para converger a cualquier cosa. Los términos impares y los términos pares divergen hasta el infinito (¡prueba esto!) :\[\sum_{n=1}^\infty \frac{1}{2n-1} = \infty, \qquad \text{and} \qquad \sum_{n=1}^\infty \frac{1}{2n} = \infty .\] Dejar\(a_n := \frac
    ParseError: EOF expected (click for details)
    Callstack:
        at (Matematicas/Analisis/Introducción_al_Análisis_Real_(Lebl)/02:_Números_reales/2.01:_Propiedades_básicas), /content/body/div[14]/div/div[3]/p[7]/span[4]/span, line 1, column 5
    
    {n}\)
    por simplicidad, dejar que\(L \in {\mathbb{R}}\) se dé un número arbitrario, y establezca\(\sigma(1) := 1\). Supongamos que hemos definido\(\sigma(n)\) para todos\(n \leq N\). Si\[\sum_{n=1}^N a_{\sigma(n)} \leq L ,\] entonces deja\(\sigma(N+1) := k\) ser el impar más pequeño\(k \in {\mathbb{N}}\) que aún no hemos usado, eso es\(\sigma(n) \not= k\) para todos\(n \leq N\). De lo contrario dejemos\(\sigma(N+1) := k\) ser los más pequeños incluso los\(k\) que aún no hemos usado.

    Por construcción\(\sigma \colon {\mathbb{N}}\to {\mathbb{N}}\) es uno a uno. También está en, porque si seguimos agregando cualquiera de los términos impares (resp. pares), eventualmente pasaremos\(L\) y cambiaremos a los pares (resp. odds). Entonces cambiamos infinitamente muchas veces.

    Por último, dejemos\(N\) ser el\(N\) donde acabamos de pasar\(L\) y cambiar. Por ejemplo, supongamos que acabamos de cambiar de impar a par (así que empezamos a restar), y dejar\(N' > N\) ser donde primero cambiamos de par a impar. Entonces\[L + \frac{1}{\sigma(N)} \geq \sum_{n=1}^{N-1} a_{\sigma(n)} > \sum_{n=1}^{N'-1} a_{\sigma(n)} > L- \frac{1}{\sigma(N')}.\] Y de manera similar para cambiar en la otra dirección. Por lo tanto, la suma hasta\(N'-1\) está dentro\(\frac{1}{\min \{ \sigma(N), \sigma(N') \}}\) de\(L\). A medida que cambiamos infinitamente muchas veces lo obtenemos\(\sigma(N) \to \infty\) y\(\sigma(N') \to \infty\), y por lo tanto\[\sum_{n=1}^\infty a_{\sigma(n)} = \sum_{n=1}^\infty \frac

    ParseError: EOF expected (click for details)
    Callstack:
        at (Matematicas/Analisis/Introducción_al_Análisis_Real_(Lebl)/02:_Números_reales/2.01:_Propiedades_básicas), /content/body/div[14]/div/div[3]/p[9]/span[11]/span, line 1, column 5
    
    {\sigma(n)} = L .\]

    Aquí hay un ejemplo para ilustrar la prueba. Supongamos\(L=1.2\), entonces el orden es\[1+\nicefrac{1}{3}-\nicefrac{1}{2}+\nicefrac{1}{5}+\nicefrac{1}{7}+\nicefrac{1}{9}-\nicefrac{1}{4}+\nicefrac{1}{11}+\nicefrac{1}{13}-\nicefrac{1}{6} +\nicefrac{1}{15}+\nicefrac{1}{17}+\nicefrac{1}{19} - \nicefrac{1}{8} + \cdots .\] En este punto no estamos más que\(\nicefrac{1}{8}\) del límite.

    Multiplicación de series

    Como ya hemos mencionado, la multiplicación de series es algo más dura que la suma. Si tenemos que al menos una de las series converge absolutamente, entonces podemos usar el siguiente teorema. Para este resultado es conveniente iniciar la serie a 0, en lugar de a 1.

    Supongamos\(\sum_{n=0}^\infty a_n\) y\(\sum_{n=0}^\infty b_n\) son dos series convergentes, convergentes hacia\(A\) y\(B\) respectivamente. Si al menos una de las series converge absolutamente, entonces la serie\(\sum_{n=0}^\infty c_n\) donde\[c_n = a_0 b_n + a_1 b_{n-1} + \cdots + a_n b_0 = \sum_{j=0}^n a_j b_{n-j} ,\] converge a\(AB\).

    La serie\(\sum c_n\) se llama el producto Cauchy de\(\sum a_n\) y\(\sum b_n\).

    Supongamos que\(\sum a_n\) converge absolutamente, y\(\epsilon > 0\) déjese dar. En esta prueba en lugar de escoger estimaciones complicadas sólo para hacer que la estimación final salga como menor que\(\epsilon\), simplemente obtengamos una estimación que\(\epsilon\) dependa y que pueda hacerse arbitrariamente pequeña.

    Escribir\[A_m := \sum_{n=0}^m a_n , \qquad B_m := \sum_{n=0}^m b_n .\] Reorganizamos la\(m\) th suma parcial de\(\sum c_n\): Seguramente\[\begin{split} \left\lvert {\left(\sum_{n=0}^m c_n \right) - AB} \right\rvert & = \left\lvert {\left( \sum_{n=0}^m \sum_{j=0}^n a_j b_{n-j} \right) - AB} \right\rvert \\ & = \left\lvert {\left( \sum_{n=0}^m B_n a_{m-n} \right) - AB} \right\rvert \\ & = \left\lvert {\left( \sum_{n=0}^m ( B_n - B ) a_{m-n} \right) + B A_m - AB} \right\rvert \\ & \leq \left( \sum_{n=0}^m \left\lvert { B_n - B } \right\rvert \left\lvert {a_{m-n}} \right\rvert \right) + \left\lvert {B} \right\rvert\left\lvert {A_m - A} \right\rvert \end{split}\] podemos hacer que el segundo término del lado derecho vaya a cero. El truco es manejar el primer término. Escoge\(K\) tal que por todo\(m \geq K\) lo que tenemos\(\left\lvert {A_m - A} \right\rvert < \epsilon\) y también\(\left\lvert {B_m - B} \right\rvert < \epsilon\). Por último, como\(\sum a_n\) converge absolutamente, asegúrate de que\(K\) sea lo suficientemente grande como para que para todos\(m \geq K\),\[\sum_{n=K}^m \left\lvert {a_n} \right\rvert < \epsilon .\] Como\(\sum b_n\) converja, entonces tenemos que\(B_{\text{max}} := \sup \{ \left\lvert { B_n - B } \right\rvert : n = 0,1,2,\ldots \}\) es finito. Tomemos\(m \geq 2K\), entonces en particular\(m-K+1 > K\). Entonces,\[\begin{split} %\left( \sum_{n=0}^m \left\lvert { B_n - B } \right\rvert \left\lvert {a_{m-n}} \right\rvert %\right) & = \left( \sum_{n=0}^{m-K} \left\lvert { B_n - B } \right\rvert \left\lvert {a_{m-n}} \right\rvert \right) + \left( \sum_{n=m-K+1}^m \left\lvert { B_n - B } \right\rvert \left\lvert {a_{m-n}} \right\rvert \right) \\ & < \left( \sum_{n=K}^m \left\lvert {a_{n}} \right\rvert \right) B_{\text{max}} + \left( \sum_{n=0}^{K-1} \epsilon \left\lvert {a_{n}} \right\rvert \right) \\ & < \epsilon B_{\text{max}} + \epsilon \left( \sum_{n=0}^\infty \left\lvert {a_{n}} \right\rvert \right) . \end{split}\] pues\(m \geq 2K\) tenemos\[\begin{split} \left\lvert {\left(\sum_{n=0}^m c_n \right) - AB} \right\rvert & \leq \left( \sum_{n=0}^m \left\lvert { B_n - B } \right\rvert \left\lvert {a_{m-n}} \right\rvert \right) + \left\lvert {B} \right\rvert\left\lvert {A_m - A} \right\rvert \\ & < \epsilon B_{\text{max}} + \epsilon \left( \sum_{n=0}^\infty \left\lvert {a_{n}} \right\rvert \right) + \left\lvert {B} \right\rvert\epsilon = \epsilon \left( B_{\text{max}} + \left( \sum_{n=0}^\infty \left\lvert {a_{n}} \right\rvert \right) + \left\lvert {B} \right\rvert \right) . \end{split}\] La expresión entre paréntesis del lado derecho es un número fijo. De ahí que podamos hacer el lado derecho arbitrariamente pequeño escogiendo uno lo suficientemente pequeño\(\epsilon> 0\). Así\(\sum_{n=0}^\infty c_n\) converge a\(AB\).

    Si ambas series son solo convergentes condicionalmente, la serie de productos Cauchy ni siquiera necesita converger. Supongamos que tomamos\(a_n = b_n = {(-1)}^n \frac{1}{\sqrt{n+1}}\). La serie\(\sum_{n=0}^\infty a_n = \sum_{n=0}^\infty b_n\) converge por la prueba de series alternas, sin embargo, no converge absolutamente como se puede ver en la\(p\) prueba -. Veamos el producto Cauchy. \[c_n = {(-1)}^n \left( \frac{1}{\sqrt{n+1}} + \frac{1}{\sqrt{2n}} + \frac{1}{\sqrt{3(n-1)}} + \cdots + %\frac{1}{\sqrt{2n}} + \frac{1}{\sqrt{n+1}} \right) = {(-1)}^n \sum_{j=0}^n \frac{1}{\sqrt{(j+1)(n-j+1)}} .\]Por lo tanto\[\left\lvert {c_n} \right\rvert = \sum_{j=0}^n \frac{1}{\sqrt{(j+1)(n-j+1)}} \geq \sum_{j=0}^n \frac{1}{\sqrt{(n+1)(n+1)}} = 1 .\] Los términos no van a cero y por lo tanto\(\sum c_n\) no pueden converger.

    Serie Power

    Arreglar\(x_0 \in {\mathbb{R}}\). Una serie de potencia sobre\(x_0\) es una serie de la forma\[\sum_{n=0}^\infty a_n {(x-x_0)}^n .\] Una serie de potencia es realmente una función de\(x\), y muchas funciones importantes en el análisis se pueden escribir como una serie de potencia.

    Decimos que una serie de poder es convergente si hay al menos una\(x \not= x_0\) que haga converger la serie. Tenga en cuenta que es trivial ver que si\(x=x_0\) entonces la serie siempre converge ya que todos los términos excepto el primero son cero. Si la serie no converge por ningún punto\(x \not= x_0\), decimos que la serie es divergente.

    [ps:expex] La serie\[\sum_{n=0}^\infty \frac{1}{n!} x^n\] es absolutamente convergente para todos\(x \in {\mathbb{R}}\). Esto se puede ver usando la prueba de ratio: Para cualquier\(x\) aviso que\[\lim_{n \to \infty} \frac{\bigl(1/(n+1)!\bigr) \, x^{n+1}}{(1/n!) \, x^{n}} = \lim_{n \to \infty} \frac{x}{n+1} = 0.\] De hecho, puede recordar del cálculo que esta serie converge a\(e^x\).

    [ps:1kex] La serie\[\sum_{n=1}^\infty \frac{1}{n} x^n\] converge absolutamente para todos\(x \in (-1,1)\) a través de la prueba de ratio:\[\lim_{n \to \infty} \left\lvert { \frac{\bigl(1/(n+1) \bigr) \, x^{n+1}}{(1/n) \, x^{n}} } \right\rvert = \lim_{n \to \infty} \left\lvert {x} \right\rvert \frac{n}{n+1} = \left\lvert {x} \right\rvert < 1 .\] Converge en\(x=-1\), como\(\sum_{n=1}^\infty \frac{{(-1)}^n}{n}\) converge por la prueba de series alternas. Pero la serie power no converge absolutamente en\(x=-1\), porque\(\sum_{n=1}^\infty \frac{1}{n}\) no converge. La serie diverge en\(x=1\). Cuando\(\left\lvert {x} \right\rvert > 1\), entonces la serie diverge a través de la prueba de ratio.

    [ps:divergeex] La serie\[\sum_{n=1}^\infty n^n x^n\] diverge para todos\(x \not= 0\). Apliquemos la prueba raíz\[\limsup_{n\to\infty} \, \left\lvert {n^n x^n} \right\rvert^{1/n} = \limsup_{n\to\infty} \, n \left\lvert {x} \right\rvert = \infty .\] Por lo tanto la serie diverge para todos\(x \not= 0\).

    De hecho, la convergencia de series de poder en general siempre funciona de manera similar a uno de los tres ejemplos anteriores.

    Let\(\sum a_n {(x-x_0)}^n\) Be a power series. Si la serie es convergente, entonces o converge en absoluto\(x \in {\mathbb{R}}\), o existe un número\(\rho\), tal que la serie converge absolutamente en el intervalo\((x_0-\rho,x_0+\rho)\) y diverge cuando\(x < x_0-\rho\) o\(x > x_0+\rho\).

    El número\(\rho\) se llama el radio de convergencia de la serie de potencia. Escribimos\(\rho = \infty\) si la serie converge para todos\(x\), y escribimos\(\rho = 0\) si la serie es divergente. Ver. En el radio de convergencia es\(\rho=1\). En el radio de convergencia es\(\rho=\infty\), y en el radio de convergencia es\(\rho=0\).

    Escribir\[R := \limsup_{n\to\infty} \, {\left\lvert {a_n} \right\rvert}^{1/n} .\] Utilizamos la prueba raíz para probar la proposición:\[L = \limsup_{n\to\infty} \, {\left\lvert {a_n{(x-x_0)}^n} \right\rvert}^{1/n} = \left\lvert {x-x_0} \right\rvert \limsup_{n\to\infty} \, {\left\lvert {a_n} \right\rvert}^{1/n} = \left\lvert {x-x_0} \right\rvert R .\] En particular si\(R = \infty\), entonces\(L=\infty\) para alguna\(x \not= x_0\), y la serie diverge por la prueba raíz. Por otro lado si\(R = 0\), entonces\(L=0\) para cualquiera\(x\), y la serie converge absolutamente para todos\(x\).

    Supongamos\(0 < R < \infty\). La serie converge absolutamente si\(1 > L = R \left\lvert {x-x_0} \right\rvert\), o en otras palabras cuando\[\left\lvert {x-x_0} \right\rvert < \nicefrac{1}{R} .\] La serie diverge cuando\(1 < L = R \left\lvert {x-x_0} \right\rvert\), o\[\left\lvert {x-x_0} \right\rvert > \nicefrac{1}{R} .\] Dejar\(\rho = \nicefrac{1}{R}\) completa la prueba.

    Puede ser útil reafirmar lo que hemos aprendido en la prueba como una proposición separada.

    Let\(\sum a_n {(x-x_0)}^n\) be a power series, y let\[R := \limsup_{n\to\infty} \, {\left\lvert {a_n} \right\rvert}^{1/n} .\] If\(R = \infty\), la serie power es divergente. Si\(R=0\), entonces la serie power converge en todas partes. De lo contrario el radio de convergencia\(\rho = \nicefrac{1}{R}\).

    A menudo, el radio de convergencia se escribe como\(\rho = \nicefrac{1}{R}\) en los tres casos, con la comprensión obvia de lo que\(\rho\) debería ser si\(R = 0\) o\(R = \infty\).

    Las series de potencia convergente se pueden sumar y multiplicar entre sí, y multiplicar por constantes. La propuesta tiene una prueba fácil usando lo que sabemos sobre series en general, y series de potencia en particular. Dejamos la prueba al lector.

    Let\(\sum_{n=0}^\infty a_n {(x-x_0)}^n\) y\(\sum_{n=0}^\infty b_n {(x-x_0)}^n\) ser dos series convergentes de potencia con radio de convergencia al menos\(\rho > 0\) y\(\alpha \in {\mathbb{R}}\). Entonces para todos\(x\) esos que\(\left\lvert {x-x_0} \right\rvert < \rho\), tenemos\[\left(\sum_{n=0}^\infty a_n {(x-x_0)}^n\right) + \left(\sum_{n=0}^\infty b_n {(x-x_0)}^n\right) = \sum_{n=0}^\infty (a_n+b_n) {(x-x_0)}^n ,\]\[\alpha \left(\sum_{n=0}^\infty a_n {(x-x_0)}^n\right) = \sum_{n=0}^\infty \alpha a_n {(x-x_0)}^n ,\] y\[\left(\sum_{n=0}^\infty a_n {(x-x_0)}^n\right) \, \left(\sum_{n=0}^\infty b_n {(x-x_0)}^n\right) = \sum_{n=0}^\infty c_n {(x-x_0)}^n ,\] donde\(c_n = a_0b_n + a_1 b_{n-1} + \cdots + a_n b_0\).

    Es decir, después de realizar las operaciones algebraicas, el radio de convergencia de la serie resultante es al menos\(\rho\). Para todos\(x\) con\(\left\lvert {x-x_0} \right\rvert < \rho\), tenemos dos series convergentes por lo que su término por término suma y multiplicación por constantes sigue por lo que aprendimos en la última sección. Para la multiplicación de dos series de potencia, las series son absolutamente convergentes dentro del radio de convergencia y es por eso que para esas\(x\) podemos aplicar el teorema de Mertens. Tenga en cuenta que después de aplicar una operación algebraica el radio de convergencia podría aumentar. Ver los ejercicios.

    Veamos algunos ejemplos de series de poder. Los polinomios son simplemente series de potencia finita. Es decir, un polinomio es una serie de potencias donde los\(a_n\) son cero para todos lo suficientemente\(n\) grandes. Ampliamos un polinomio como una serie de potencias sobre cualquier punto\(x_0\) escribiendo el polinomio como polinomio en\((x-x_0)\). Por ejemplo,\(2x^2-3x+4\) como una serie de potencia alrededor\(x_0 = 1\) es\[2x^2-3x+4 = 3 + (x-1) + 2{(x-1)}^2 .\]

    También podemos ampliar funciones racionales, es decir, proporciones de polinomios como series de potencia, aunque aquí no vamos a probar completamente este hecho. Observe que una serie para una función racional solo define la función en un intervalo incluso si la función se define en otro lugar. Por ejemplo, para la serie geométrica tenemos que para\(x \in (-1,1)\)\[\frac{1}{1-x} = \sum_{n=0}^\infty x^n .\] La serie diverge cuando\(\left\lvert {x} \right\rvert > 1\), aunque\(\frac{1}{1-x}\) se define para todos\(x \not= 1\).

    Podemos usar la serie geométrica junto con reglas para la suma y multiplicación de series de potencia para expandir las funciones racionales como series de potencia alrededor\(x_0\), siempre y cuando el denominador no sea cero en\(x_0\). Declaramos sin pruebas que esto siempre es posible, y damos un ejemplo de tal cómputo usando la serie geométrica.

    Vamos a expandirnos\(\frac{x}{1+2x+x^2}\) como una serie de potencias alrededor del origen (\(x_0 = 0\)) y encontrar el radio de convergencia.

    Escribe\(1+2x+x^2 = {(1+x)}^2 = {\bigl(1-(-x)\bigr)}^2\), y supongamos\(\left\lvert {x} \right\rvert < 1\). Calcular\[\begin{split} \frac{x}{1+2x+x^2} &= x \, {\left( \frac{1}{1-(-x)} \right)}^2 \\ &= x \, {\left( \sum_{n=0}^\infty {(-1)}^n x^n \right)}^2 \\ &= x \, \left( \sum_{n=0}^\infty c_n x^n \right) \\ &= \sum_{n=0}^\infty c_n x^{n+1} , \end{split}\] donde usando la fórmula para el producto de serie obtenemos,\(c_0 = 1\),\(c_1 = -1 -1 = -2\),\(c_2 = 1+1+1 = 3\), etc... Por lo tanto obtenemos eso para\(\left\lvert {x} \right\rvert < 1\),\[\frac{x}{1+2x+x^2} = \sum_{n=1}^\infty {(-1)}^{n+1} n x^n .\] El radio de convergencia es de al menos 1. Dejamos al lector verificar que el radio de convergencia es exactamente igual a 1.

    Puedes usar el método de fracciones parciales que conoces del cálculo. Por ejemplo, para encontrar la serie power para\(\frac{x^3+x}{x^2-1}\) en 0, escriba\[\frac{x^3+x}{x^2-1} = x + \frac{1}{1+x} - \frac{1}{1-x} = x + \sum_{n=0}^\infty {(-1)}^n x^n - \sum_{n=0}^\infty x^n .\]

    Ejercicios

    Decidir la convergencia o divergencia de las siguientes series.

    a)\(\displaystyle \sum_{n=1}^\infty \frac{1}{2^{2n+1}}\) b)\(\displaystyle \sum_{n=1}^\infty \frac{{(-1)}^{n}(n-1)}{n}\) c)\(\displaystyle \sum_{n=1}^\infty \frac
    ParseError: EOF expected (click for details)
    Callstack:
        at (Matematicas/Analisis/Introducción_al_Análisis_Real_(Lebl)/02:_Números_reales/2.01:_Propiedades_básicas), /content/body/div[15]/table[1]/tbody/tr/td[5]/span/span, line 1, column 5
    
    \)
    d)\(\displaystyle \sum_{n=1}^\infty \frac{n^n}
    ParseError: EOF expected (click for details)
    Callstack:
        at (Matematicas/Analisis/Introducción_al_Análisis_Real_(Lebl)/02:_Números_reales/2.01:_Propiedades_básicas), /content/body/div[15]/table[1]/tbody/tr/td[7]/span/span, line 1, column 6
    
    \)

    Supongamos ambas\(\sum_{n=0}^\infty a_n\) y\(\sum_{n=0}^\infty b_n\) convergen absolutamente. Demostrar que la serie de productos\(c_n = a_0 b_n + a_1 b_{n-1} + \cdots + a_n b_0\),\(\sum_{n=0}^\infty c_n\) donde, también converge absolutamente.

    [ejercicio:seriesconvergestocualquier cosa]\(\sum a_n\) Sea condicionalmente convergente. Demostrar que dado cualquier número\(x\) existe un reordenamiento de\(\sum a_n\) tal manera que la serie reordenada converge a\(x\). Pista: Ver.

    a) Demostrar que la serie armónica alterna\(\sum \frac

    ParseError: EOF expected (click for details)
    Callstack:
        at (Matematicas/Analisis/Introducción_al_Análisis_Real_(Lebl)/02:_Números_reales/2.01:_Propiedades_básicas), /content/body/div[15]/p[4]/span[1]/span, line 1, column 5
    
    {n}\) tiene un reordenamiento tal que para cualquiera\(x < y\), existe una suma parcial\(s_n\) de la serie reordenada tal que\(x < s_n < y\). b) Demuestre que el reordenamiento que encontró no converge. Ver. c) Demostrar que para cualquiera\(x \in {\mathbb{R}}\), existe una subsecuencia de sumas parciales\(\{ s_{n_k} \}\) de su reordenamiento tal que\(\lim \, s_{n_k} = x\).

    Para las siguientes series de potencia, encuentre si son convergentes o no, y de ser así encontrar su radio de convergencia.

    a)\(\displaystyle \sum_{n=0}^\infty 2^n x^n\) b)\(\displaystyle \sum_{n=0}^\infty n x^n\) c)\(\displaystyle \sum_{n=0}^\infty n! \, x^n\) d)\(\displaystyle \sum_{n=0}^\infty \frac{1}{(2k)!} {(x-10)}^n\) e)\(\displaystyle \sum_{n=0}^\infty x^{2n}\) f)\(\displaystyle \sum_{n=0}^\infty n! \, x^{n!}\)

    Supongamos que\(\sum a_n x^n\) converge para\(x=1\). a) ¿Qué se puede decir del radio de convergencia? b) Si además sabes que en\(x=1\) la convergencia no es absoluta, ¿qué puedes decir?

    Expandir\(\dfrac{x}{4-x^2}\) como una serie de potencia alrededor\(x_0 = 0\) y calcular su radio de convergencia.

    a) Encuentra un ejemplo donde el radio de convergencia de\(\sum a_n x^n\) y\(\sum b_n x^n\) son 1, pero el radio de convergencia de la suma de las dos series es infinito. b) (Trickier) Encuentra un ejemplo donde el radio de convergencia de\(\sum a_n x^n\) y\(\sum b_n x^n\) son 1, pero el radio de convergencia del producto del dos series es infinita.

    Averiguar cómo calcular el radio de convergencia usando la prueba de ratio. Es decir, supongamos que\(\sum a_n x^n\) es una serie de poder y\(R := \lim \, \frac{\left\lvert {a_{n+1}} \right\rvert}{\left\lvert {a_n} \right\rvert}\) existe o es\(\infty\). Encuentra el radio de convergencia y prueba tu reclamo.

    a) Demostrarlo\(\lim \, n^{1/n} = 1\). Pista: Escribir\(n^{1/n} = 1+b_n\) y anotar\(b_n > 0\). Entonces muestra eso\({(1+b_n)}^n \geq \frac{n(n-1)}{2}b_n^2\) y usa esto para demostrarlo\(\lim \, b_n = 0\). b) Usa el resultado de la parte a) para mostrar que si\(\sum a_n x^n\) es una serie de potencias convergentes con radio de convergencia\(R\), entonces también\(\sum n a_n x^n\) es convergente con el mismo radio de convergencia.

    Existen diferentes nociones de sumabilidad (convergencia) de una serie que solo la que hemos visto. Uno común es Ces à ro sumabilidad 14. Dejar\(\sum a_n\) ser una serie y dejar\(s_n\) ser la\(n\) ésima suma parcial. Se dice que la serie es Ces à ro sumable a\(a\) si\[a = \lim_{n\to \infty} \frac{s_1 + s_2 + \cdots + s_n}{n} .\]

    a) Si\(\sum a_n\) es convergente a\(a\) (en el sentido habitual), mostrar que\(\sum a_n\) es Ces à ro sumable a\(a\). b) Demostrar que en el sentido de Ces à ro\(\sum {(-1)}^n\) es sumable a\(\nicefrac{1}{2}\). c) Dejar\(a_n := k\) cuando\(n = k^3\) para algunos\(k \in {\mathbb{N}}\),\(a_n := -k\) cuando \(n = k^3+1\)para algunos\(k \in {\mathbb{N}}\), de lo contrario vamos\(a_n := 0\). Mostrar que\(\sum a_n\) diverge en el sentido habitual, (las sumas parciales son ilimitadas), pero es Ces à ro sumable a 0 (parece un poco paradójico a primera vista).

    Demostrar que la monotonicidad en la prueba de series alternas es necesaria. Es decir, encontrar una secuencia de números reales positivos\(\{ x_n \}\) con\(\lim\, x_n = 0\) pero tal que\(\sum {(-1)}^n x_n\) diverja.

    Funciones continuas

    Límites de funciones

    Nota: 2—3 conferencias

    Antes de definir la continuidad de las funciones, necesitamos visitar una noción algo más general de un límite. Es decir, dada una función\(f \colon S \to {\mathbb{R}}\), queremos ver cómo\(f(x)\) se comporta como\(x\) tiende hasta cierto punto.

    Puntos de clúster

    Primero, volvamos a un concepto que hemos visto anteriormente en un ejercicio.

    \(S \subset {\mathbb{R}}\)Déjese ser un conjunto. Un número\(x \in {\mathbb{R}}\) se llama punto de clúster de\(S\) si para cada\(\epsilon > 0\), el conjunto no\((x-\epsilon,x+\epsilon) \cap S \setminus \{ x \}\) está vacío.

    Es decir,\(x\) es un punto de cúmulo de\(S\) si hay puntos de\(S\) arbitrariamente cercanos a\(x\). Otra forma de formular la definición es decir que\(x\) es un punto de cúmulo de\(S\) si por cada\(\epsilon > 0\), existe\(y \in S\) tal que\(y \not= x\) y\(\left\lvert {x - y} \right\rvert < \epsilon\). Tenga en cuenta que un punto de clúster de no\(S\) es necesario que se encuentre en\(S\).

    Veamos algunos ejemplos.

    1. El conjunto\(\{ \nicefrac{1}{n} : n \in {\mathbb{N}}\}\) tiene un punto cero de clúster único.
    2. Los puntos de clúster del intervalo abierto\((0,1)\) son todos los puntos en el intervalo cerrado\([0,1]\).
    3. Para el conjunto\({\mathbb{Q}}\), el conjunto de puntos de clúster es toda la línea real\({\mathbb{R}}\).
    4. Para el conjunto\([0,1) \cup \{ 2 \}\), el conjunto de puntos de clúster es el intervalo\([0,1]\).
    5. El conjunto no\({\mathbb{N}}\) tiene puntos de clúster en\({\mathbb{R}}\).

    Vamos\(S \subset {\mathbb{R}}\). Entonces\(x \in {\mathbb{R}}\) es un punto de cúmulo de\(S\) si y sólo si existe una secuencia convergente de números\(\{ x_n \}\) tales que\(x_n \not= x\),\(x_n \in S\), y\(\lim\, x_n = x\).

    Primero supongamos que\(x\) es un punto de cúmulo de\(S\). Para cualquiera\(n \in {\mathbb{N}}\), elegimos\(x_n\) ser un punto arbitrario de\((x-\nicefrac{1}{n},x+\nicefrac{1}{n}) \cap S \setminus \{x\}\), que sabemos que no está vacío porque\(x\) es un punto de clúster de\(S\). Entonces\(x_n\) está dentro\(\nicefrac{1}{n}\) de\(x\), es decir,\[\left\lvert {x-x_n} \right\rvert < \nicefrac{1}{n} .\] As\(\{ \nicefrac{1}{n} \}\) converge a cero,\(\{ x_n \}\) converge a\(x\).

    Por otro lado, si empezamos con una secuencia de números\(\{ x_n \}\) en\(S\) converger a\(x\) tal que\(x_n \not= x\) para todos\(n\), entonces para cada\(\epsilon > 0\) hay\(M\) tal que en particular\(\left\lvert {x_M - x} \right\rvert < \epsilon\). Es decir,\(x_M \in (x-\epsilon,x+\epsilon) \cap S \setminus \{x\}\).

    Límites de funciones

    Si una función\(f\) se define en un conjunto\(S\) y\(c\) es un punto de clúster de\(S\), entonces podemos definir el límite de\(f(x)\) como\(x\) se acerca a\(c\). Tenga en cuenta que es irrelevante para la definición si\(f\) se define en\(c\) o no. Además, incluso si la función se define en\(c\), el límite de la función como\(x\) va a bien\(c\) podría ser diferente de\(f(c)\).

    Dejar\(f \colon S \to {\mathbb{R}}\) ser una función y\(c\) un punto de clúster de\(S\). Supongamos que existe una\(L \in {\mathbb{R}}\) y para cada\(\epsilon > 0\), existe\(\delta > 0\) tal que siempre\(x \in S \setminus \{ c \}\) y\(\left\lvert {x - c} \right\rvert < \delta\), entonces\[\left\lvert {f(x) - L} \right\rvert < \epsilon .\] en este caso decimos\(f(x)\) converge a\(L\) lo que\(x\) va a\(c\). Decimos que\(L\) es el límite de\(f(x)\) lo que\(x\) va a\(c\). Escribimos\[\lim_{x \to c} f(x) := L ,\] o\[f(x) \to L \quad\text{as}\quad x \to c .\] Si no\(L\) existe tal, entonces decimos que el límite no existe o que\(f\) diverge en\(c\).

    Nuevamente la notación y el lenguaje que estamos usando anteriormente asume que el límite es único aunque aún no lo hayamos probado. Hagámoslo ahora.

    Dejar\(c\) ser un punto de clúster de\(S \subset {\mathbb{R}}\) y dejar\(f \colon S \to {\mathbb{R}}\) ser una función tal que\(f(x)\) converja como\(x\) va a\(c\). Entonces el límite de\(f(x)\) lo que\(x\) va a\(c\) es único.

    Dejar\(L_1\) y\(L_2\) ser dos números que ambos satisfagan la definición. Toma una\(\epsilon > 0\) y encuentra\(\delta_1 > 0\) tal que\(\left\lvert {f(x)-L_1} \right\rvert < \nicefrac{\epsilon}{2}\) para todos\(x \in S \setminus \{c\}\) con\(\left\lvert {x-c} \right\rvert < \delta_1\). También encuentra\(\delta_2 > 0\) tal que\(\left\lvert {f(x)-L_2} \right\rvert < \nicefrac{\epsilon}{2}\) para todos\(x \in S \setminus \{c\}\) con\(\left\lvert {x-c} \right\rvert < \delta_2\). Poner\(\delta := \min \{ \delta_1, \delta_2 \}\). Supongamos\(x \in S\)\(\left\lvert {x-c} \right\rvert < \delta\),, y\(x \not= c\). Entonces\[\left\lvert {L_1 - L_2} \right\rvert = \left\lvert {L_1 - f(x) + f(x) - L_2} \right\rvert \leq \left\lvert {L_1 - f(x)} \right\rvert + \left\lvert {f(x) - L_2} \right\rvert < \frac{\epsilon}{2} + \frac{\epsilon}{2} = \epsilon.\] En\(\left\lvert {L_1-L_2} \right\rvert < \epsilon\) cuanto a arbitrario\(\epsilon > 0\), entonces\(L_1 = L_2\).

    Dejar que\(f \colon {\mathbb{R}}\to {\mathbb{R}}\) se defina como\(f(x) := x^2\). Entonces\[\lim_{x\to c} f(x) = \lim_{x\to c} x^2 = c^2 .\]

    Prueba: Primero deje\(c\) ser arreglado. Dejemos\(\epsilon > 0\) que se den. Toma\[\delta := \min \left\{ 1 , \, \frac{\epsilon}{2\left\lvert {c} \right\rvert+1} \right\} .\] Toma\(x \not= c\) tal que\(\left\lvert {x-c} \right\rvert < \delta\). En particular,\(\left\lvert {x-c} \right\rvert < 1\). Entonces por la desigualdad del triángulo inverso obtenemos\[\left\lvert {x} \right\rvert-\left\lvert {c} \right\rvert \leq \left\lvert {x-c} \right\rvert < 1 .\] Sumando\(2\left\lvert {c} \right\rvert\) a ambos lados obtenemos\(\left\lvert {x} \right\rvert + \left\lvert {c} \right\rvert < 2\left\lvert {c} \right\rvert + 1\). Calculamos\[\begin{split} \left\lvert {f(x) - c^2} \right\rvert &= \left\lvert {x^2-c^2} \right\rvert \\ &= \left\lvert {(x+c)(x-c)} \right\rvert \\ &= \left\lvert {x+c} \right\rvert\left\lvert {x-c} \right\rvert \\ &\leq (\left\lvert {x} \right\rvert+\left\lvert {c} \right\rvert)\left\lvert {x-c} \right\rvert \\ &< (2\left\lvert {c} \right\rvert+1)\left\lvert {x-c} \right\rvert \\ &< (2\left\lvert {c} \right\rvert+1)\frac{\epsilon}{2\left\lvert {c} \right\rvert+1} = \epsilon . \end{split}\]

    Definir\(f \colon [0,1) \to {\mathbb{R}}\) por\[f(x) := \begin{cases} x & \text{if $x > 0$} , \\ 1 & \text{if $x = 0$} . \end{cases}\] Entonces\[\lim_{x\to 0} f(x) = 0 ,\] aunque\(f(0) = 1\).

    Comprobante: Dejar\(\epsilon > 0\) ser dado. Vamos\(\delta := \epsilon\). Entonces para\(x \in [0,1)\)\(x \not= 0\),, y\(\left\lvert {x-0} \right\rvert < \delta\) obtenemos\[\left\lvert {f(x) - 0} \right\rvert = \left\lvert {x} \right\rvert < \delta = \epsilon .\]

    Límites secuenciales

    Conectemos el límite como se definió anteriormente con límites de secuencias.

    [seqflimit:lemma] Dejar\(S \subset {\mathbb{R}}\) y\(c\) ser un punto de cúmulo de\(S\). Dejar\(f \colon S \to {\mathbb{R}}\) ser una función.

    Entonces\(f(x) \to L\) como\(x \to c\), si y sólo si por cada secuencia\(\{ x_n \}\) de números tal que\(x_n \in S \setminus \{c\}\) para todos\(n\), y tal que\(\lim\, x_n = c\), tenemos que la secuencia\(\{ f(x_n) \}\) converge a\(L\).

    Supongamos\(f(x) \to L\) como\(x \to c\), y\(\{ x_n \}\) es una secuencia tal que\(x_n \in S \setminus \{c\}\) y\(\lim\, x_n = c\). Deseamos demostrar que\(\{ f(x_n) \}\) converge a\(L\). Dejemos\(\epsilon > 0\) que se den. Encuentra un\(\delta > 0\) tal que si\(x \in S \setminus \{c\}\) y\(\left\lvert {x-c} \right\rvert < \delta\), entonces\(\left\lvert {f(x) - L} \right\rvert < \epsilon\). Como\(\{ x_n \}\) converge a\(c\), encontrar un\(M\) tal que para\(n \geq M\) nosotros tenemos eso\(\left\lvert {x_n - c} \right\rvert < \delta\). Por lo tanto\(n \geq M\), para,\[\left\lvert {f(x_n) - L} \right\rvert < \epsilon .\] Así\(\{ f(x_n) \}\) converge a\(L\).

    Para la otra dirección, utilizamos prueba por contrapositivo. Supongamos que no es cierto que\(f(x) \to L\) como\(x \to c\). La negación de la definición es que existe\(\epsilon > 0\) tal que para cada uno\(\delta > 0\) existe un\(x \in S \setminus \{c\}\), dónde\(\left\lvert {x-c} \right\rvert < \delta\) y\(\left\lvert {f(x)-L} \right\rvert \geq \epsilon\).

    Vamos a utilizar\(\nicefrac{1}{n}\) for\(\delta\) en la declaración anterior para construir una secuencia\(\{ x_n \}\). Tenemos que existe\(\epsilon > 0\) tal que para cada\(n\), existe un punto\(x_n \in S \setminus \{c\}\), donde\(\left\lvert {x_n-c} \right\rvert < \nicefrac{1}{n}\) y\(\left\lvert {f(x_n)-L} \right\rvert \geq \epsilon\). La secuencia\(\{ x_n \}\) recién construida converge a\(c\), pero la secuencia\(\{ f(x_n) \}\) no converge a\(L\). Y ya terminamos.

    Es posible fortalecer la dirección inversa del lema simplemente afirmando que\(\{ f(x_n) \}\) converge sin requerir un límite específico. Ver.

    \(\displaystyle \lim_{x \to 0} \, \sin( \nicefrac{1}{x} )\)no existe, pero\(\displaystyle \lim_{x \to 0} \, x\sin( \nicefrac{1}{x} ) = 0\). Ver.

    Gráficas de\ sin (\ nicefrac {1} {x}) y x\ sin (\ nicefrac {1} {x}). Tenga en cuenta que la computadora no puede graficar correctamente\ sin (\ nicefrac {1} {x}) cerca de cero ya que oscila demasiado rápido. [figsin1x]Gráficas de\ sin (\ nicefrac {1} {x}) y x\ sin (\ nicefrac {1} {x}). Tenga en cuenta que la computadora no puede graficar correctamente\ sin (\ nicefrac {1} {x}) cerca de cero ya que oscila demasiado rápido. [figsin1x]

    Prueba: Trabajemos con\(\sin(\nicefrac{1}{x})\) primero. Definamos la secuencia\(x_n := \frac{1}{\pi n + \nicefrac{\pi}{2}}\). No es difícil de ver eso\(\lim\, x_n = 0\). Además,\[\sin ( \nicefrac{1}{x_n} ) = \sin (\pi n + \nicefrac{\pi}{2}) = {(-1)}^n .\] por lo tanto,\(\{ \sin ( \nicefrac{1}{x_n} ) \}\) no converge. Así, por,\(\lim_{x \to 0} \, \sin( \nicefrac{1}{x} )\) no existe.

    Ahora veamos\(x\sin(\nicefrac{1}{x})\). Que\(x_n\) sea una secuencia tal que\(x_n \not= 0\) para todos\(n\) y tal que\(\lim\, x_n = 0\). Observe que\(\left\lvert {\sin(t)} \right\rvert \leq 1\) para cualquier\(t \in {\mathbb{R}}\). Por lo tanto,\[\left\lvert {x_n\sin(\nicefrac{1}{x_n})-0} \right\rvert = \left\lvert {x_n} \right\rvert\left\lvert {\sin(\nicefrac{1}{x_n})} \right\rvert \leq \left\lvert {x_n} \right\rvert .\] As\(x_n\) va a 0, luego\(\left\lvert {x_n} \right\rvert\) va a cero, y de ahí\(\{ x_n\sin(\nicefrac{1}{x_n}) \}\) converge a cero. Por,\(\displaystyle \lim_{x \to 0} \, x\sin( \nicefrac{1}{x} ) = 0\).

    Ten en cuenta la frase “para cada secuencia” en el lema. Por ejemplo, toma\(\sin(\nicefrac{1}{x})\) y la secuencia\(x_n = \nicefrac{1}{\pi n}\). Entonces\(\{ \sin (\nicefrac{1}{x_n}) \}\) es la secuencia cero constante, y por lo tanto converge a cero.

    Usando, podemos comenzar a aplicar todo lo que sabemos sobre límites secuenciales a límites de funciones. Demos algunos ejemplos importantes.

    Dejar\(S \subset {\mathbb{R}}\) y\(c\) ser un punto de cúmulo de\(S\). Dejar\(f \colon S \to {\mathbb{R}}\) y\(g \colon S \to {\mathbb{R}}\) ser funciones. Supongamos que existen los límites de\(f(x)\) y\(g(x)\) como\(x\) va a\(c\) ambos, y que\[f(x) \leq g(x) \qquad \text{for all $x \in S$}.\] Entonces\[\lim_{x\to c} f(x) \leq \lim_{x\to c} g(x) .\]

    Tomar\(\{ x_n \}\) ser una secuencia de números en\(S \setminus \{ c \}\) que converja a\(c\). Let\[L_1 := \lim_{x\to c} f(x), \qquad \text{and} \qquad L_2 := \lim_{x\to c} g(x) .\] By sabemos que\(\{ f(x_n) \}\) converge hacia\(L_1\) y\(\{ g(x_n) \}\) converge hacia\(L_2\). También tenemos\(f(x_n) \leq g(x_n)\). Obtenemos\(L_1 \leq L_2\) usando.

    Al aplicar funciones constantes, obtenemos el siguiente corolario. La prueba se deja como ejercicio.

    [fconstineq:cor] Dejar\(S \subset {\mathbb{R}}\) y\(c\) ser un punto de cúmulo de\(S\). Dejar\(f \colon S \to {\mathbb{R}}\) ser una función. Y supongamos que\(c\) existe el límite de\(f(x)\) lo que\(x\) va a. Supongamos que hay dos números reales\(a\) y\(b\) tal que\[a \leq f(x) \leq b \qquad \text{for all $x \in S$}.\] Entonces\[a \leq \lim_{x\to c} f(x) \leq b .\]

    Utilizando de la misma manera que anteriormente también obtenemos los siguientes corolarios, cuyas pruebas se vuelven a dejar como ejercicio.

    [fsqueeze:cor] Dejar\(S \subset {\mathbb{R}}\) y\(c\) ser un punto de cúmulo de\(S\). Dejar\(f \colon S \to {\mathbb{R}}\),\(g \colon S \to {\mathbb{R}}\), y\(h \colon S \to {\mathbb{R}}\) ser funciones. Supongamos\[f(x) \leq g(x) \leq h(x) \qquad \text{for all $x \in S$},\] y los límites de\(f(x)\) y\(h(x)\) como\(x\) va a\(c\) ambos existir, y\[\lim_{x\to c} f(x) = \lim_{x\to c} h(x) .\] Entonces el límite de\(g(x)\) lo que\(x\) va a\(c\) existe y\[\lim_{x\to c} g(x) = \lim_{x\to c} f(x) = \lim_{x\to c} h(x) .\]

    [falg:cor] Dejar\(S \subset {\mathbb{R}}\) y\(c\) ser un punto de cúmulo de\(S\). Dejar\(f \colon S \to {\mathbb{R}}\) y\(g \colon S \to {\mathbb{R}}\) ser funciones. Supongamos que existen límites de\(f(x)\) y\(g(x)\) como\(x\) va a\(c\) ambos. Entonces

    1. \(\displaystyle \lim_{x\to c} \bigl(f(x)+g(x)\bigr) = \left(\lim_{x\to c} f(x)\right) + \left(\lim_{x\to c} g(x)\right) .\)
    2. \(\displaystyle \lim_{x\to c} \bigl(f(x)-g(x)\bigr) = \left(\lim_{x\to c} f(x)\right) - \left(\lim_{x\to c} g(x)\right) .\)
    3. \(\displaystyle \lim_{x\to c} \bigl(f(x)g(x)\bigr) = \left(\lim_{x\to c} f(x)\right) \left(\lim_{x\to c} g(x)\right) .\)
    4. [falg:cor:iv] Si\(\displaystyle \lim_{x\to c} g(x) \not= 0\), y\(g(x) \not= 0\) para todos\(x \in S \setminus \{ c \}\), entonces\[\lim_{x\to c} \frac{f(x)}{g(x)} = \frac{\lim_{x\to c} f(x)}{\lim_{x\to c} g(x)} .\]

    Límites de restricciones y límites unilaterales

    A veces trabajamos con la función definida en un subconjunto.

    Dejar\(f \colon S \to {\mathbb{R}}\) ser una función. Vamos\(A \subset S\). Definir la función\(f|_A \colon A \to {\mathbb{R}}\) por\[f|_A (x) := f(x) \qquad \text{for $x \in A$}.\] La función\(f|_A\) se llama la restricción de\(f\) a\(A\).

    La función\(f|_A\) es simplemente la función\(f\) tomada en un dominio más pequeño. La siguiente proposición es el análogo de tomar una cola de una secuencia.

    [prop:limrest] Dejar\(S \subset {\mathbb{R}}\),\(c \in {\mathbb{R}}\), y dejar\(f \colon S \to {\mathbb{R}}\) ser una función. Supongamos que\(A \subset S\) es tal que hay algunos\(\alpha > 0\) tales que\(A \cap (c-\alpha,c+\alpha) = S \cap (c-\alpha,c+\alpha)\).

    1. El punto\(c\) es un punto de clúster de\(A\) si y solo si\(c\) es un punto de clúster de\(S\).
    2. Supongamos\(c\) que es un punto de cúmulo de\(S\), entonces\(f(x) \to L\) como\(x \to c\) si y sólo si\(f|_A(x) \to L\) como\(x \to c\).

    Primero, dejemos\(c\) ser un punto de cúmulo de\(A\). Ya que\(A \subset S\), entonces si no\(( A \setminus \{ c\} ) \cap (c-\epsilon,c+\epsilon)\) está vacío para cada\(\epsilon > 0\), entonces no\(( S \setminus \{ c\} ) \cap (c-\epsilon,c+\epsilon)\) está vacío para cada uno\(\epsilon > 0\). Así\(c\) es un punto de cúmulo de\(S\). Segundo, supongamos que\(c\) es un punto de cúmulo de\(S\). Entonces para\(\epsilon > 0\) tal que\(\epsilon < \alpha\) conseguimos eso\(( A \setminus \{ c\} ) \cap (c-\epsilon,c+\epsilon) = ( S \setminus \{ c\} ) \cap (c-\epsilon,c+\epsilon)\), que no está vacío. Esto es cierto para todos\(\epsilon < \alpha\) y por lo tanto\(( A \setminus \{ c\} ) \cap (c-\epsilon,c+\epsilon)\) debe ser no vacío para todos\(\epsilon > 0\). Así\(c\) es un punto de cúmulo de\(A\).

    Ahora supongamos\(f(x) \to L\) como\(x \to c\). Es decir, para cada\(\epsilon > 0\) hay\(\delta > 0\) tal que si\(x \in S \setminus \{ c \}\) y\(\left\lvert {x-c} \right\rvert < \delta\), entonces\(\left\lvert {f(x)-L} \right\rvert < \epsilon\). Porque\(A \subset S\), si\(x\) está en\(A \setminus \{ c \}\), entonces\(x\) está adentro\(S \setminus \{ c \}\), y por lo tanto\(f|_A(x) \to L\) como\(x \to c\).

    Por último supongamos\(f|_A(x) \to L\) como\(x \to c\). De ahí que para cada\(\epsilon > 0\) haya\(\delta > 0\) tal que si\(x \in A \setminus \{ c \}\) y\(\left\lvert {x-c} \right\rvert < \delta\), entonces\(\bigl\lvert f|_A(x)-L \bigr\rvert < \epsilon\). Sin pérdida de generalidad asumir\(\delta \leq \alpha\). Si\(\left\lvert {x-c} \right\rvert < \delta\), entonces\(x \in S \setminus \{c \}\) si y solo si\(x \in A \setminus \{c \}\). Así\(\left\lvert {f(x)-L} \right\rvert = \bigl\lvert f|_A(x)-L \bigr\rvert < \epsilon\).

    La hipótesis de la proposición es necesaria. Para una restricción arbitraria generalmente solo obtenemos implicación en una sola dirección, véase.

    Un uso común de la restricción con respecto a los límites son los límites unilaterales.

    [defn:onesidedlimits] Let\(f \colon S \to {\mathbb{R}}\) be function y let\(c\) be a cluster point of\(S \cap (c,\infty)\). Entonces si el límite de la restricción de\(f\) a\(S \cap (c,\infty)\) como\(x \to c\) existe, definimos\[\lim_{x \to c^+} f(x) := \lim_{x\to c} f|_{S \cap (c,\infty)}(x) .\] De manera similar si\(c\) es un punto de clúster de\(S \cap (-\infty,c)\) y el límite de la restricción como\(x \to c\) existe, definimos\[\lim_{x \to c^-} f(x) := \lim_{x\to c} f|_{S \cap (-\infty,c)}(x) .\]

    La proposición anterior no se aplica a los límites unilaterales. Es posible tener límites unilaterales, pero ningún límite en un punto. Por ejemplo, definir\(f \colon {\mathbb{R}}\to {\mathbb{R}}\) por\(f(x) := 1\) para\(x < 0\) y\(f(x) := 0\) para\(x \geq 0\). Dejamos al lector verificar que\[\lim_{x \to 0^-} f(x) = 1, \qquad \lim_{x \to 0^+} f(x) = 0, \qquad \lim_{x \to 0} f(x) \quad \text{does not exist.}\] tenemos el siguiente reemplazo.

    [prop:onesidedlimits] Let\(S \subset {\mathbb{R}}\) be a set such that\(c\) is a cluster point of both\(S \cap (-\infty,c)\) and\(S \cap (c,\infty)\), and let\(f \colon S \to {\mathbb{R}}\) be a function. Entonces\[\lim_{x \to c} f(x) = L \qquad \text{if and only if} \qquad \lim_{x \to c^-} f(x) = \lim_{x \to c^+} f(x) = L .\]

    Es decir, existe un límite si ambos límites unilaterales existen y son iguales, y viceversa. La prueba es una aplicación directa de la definición de límite y se deja como ejercicio. El punto clave es ese\(\bigl( S \cap (-\infty,c) \bigr) \cup \bigl( S \cap (c,\infty) \bigr) = S \setminus \{ c \}\).

    Ejercicios

    Encontrar el límite o probar que el límite no existe

    a)\(\displaystyle \lim_{x\to c} \sqrt{x}\), para\(c \geq 0\) b)\(\displaystyle \lim_{x\to c} x^2+x+1\), para cualquier\(c \in {\mathbb{R}}\) c)\(\displaystyle \lim_{x\to 0} x^2 \cos (\nicefrac{1}{x})\)
    d)\(\displaystyle \lim_{x\to 0}\, \sin(\nicefrac{1}{x}) \cos (\nicefrac{1}{x})\) e)\(\displaystyle \lim_{x\to 0}\, \sin(x) \cos (\nicefrac{1}{x})\)

    Demostrar.

    Demostrar.

    Demostrar.

    Vamos\(A \subset S\). Mostrar que si\(c\) es un punto de clúster de\(A\), entonces\(c\) es un punto de clúster de\(S\). Tenga en cuenta la diferencia de.

    [ejercicio:restricciónlimitarejercicio] Vamos\(A \subset S\). Supongamos que\(c\) es un punto de clúster de\(A\) y también es un punto de clúster de\(S\). Dejar\(f \colon S \to {\mathbb{R}}\) ser una función. Demostrar que si\(f(x) \to L\) como\(x \to c\), entonces\(f|_A(x) \to L\) como\(x \to c\). Tenga en cuenta la diferencia de.

    Encuentra un ejemplo de una función\(f \colon [-1,1] \to {\mathbb{R}}\) tal que para\(A:=[0,1]\), la restricción\(f|_A(x) \to 0\) as\(x \to 0\), pero el límite de\(f(x)\) como\(x \to 0\) no existe. Tenga en cuenta por qué no puede aplicar.

    Encuentra funciones de ejemplo\(f\) y\(g\) tal que el límite de\(f(x)\) ni\(g(x)\) existe como\(x \to 0\), pero tal que el límite de\(f(x)+g(x)\) existe como\(x \to 0\).

    [exercise:contlimitcomposition] Dejar\(c_1\) ser un punto de cluster de\(A \subset {\mathbb{R}}\) y\(c_2\) ser un punto de cluster de\(B \subset {\mathbb{R}}\). Supongamos\(f \colon A \to B\) y\(g \colon B \to {\mathbb{R}}\) son funciones tales que\(f(x) \to c_2\) como\(x \to c_1\) y\(g(y) \to L\) como\(y \to c_2\). Si\(c_2 \in B\) también supongamos eso\(g(c_2) = L\). Dejar\(h(x) := g\bigl(f(x)\bigr)\) y mostrar\(h(x) \to L\) como\(x \to c_1\). Pista: nota que\(f(x)\) podría igualar\(c_2\) para muchos\(x \in A\), véase también.

    Dejar\(c\) ser un punto de clúster de\(A \subset {\mathbb{R}}\), y\(f \colon A \to {\mathbb{R}}\) ser una función. Supongamos por cada secuencia\(\{x_n\}\) en\(A\), tal que\(\lim\, x_n = c\), la secuencia\(\{ f(x_n) \}_{n=1}^\infty\) es Cauchy. Demostrar que\(\lim_{x\to c} f(x)\) existe.

    [exercise:seqflimitalt] Demostrar la siguiente versión más fuerte de una dirección de: Dejar\(S \subset {\mathbb{R}}\),\(c\) ser un punto de clúster de\(S\), y\(f \colon S \to {\mathbb{R}}\) ser una función. Supongamos que para cada secuencia\(\{x_n\}\) en\(S \setminus \{c\}\) tal que\(\lim\, x_n = c\) la secuencia\(\{ f(x_n) \}\) sea convergente. Entonces mostrar\(f(x) \to L\) como\(x \to c\) para algunos\(L \in {\mathbb{R}}\).

    Demostrar.

    Supongamos\(S \subset {\mathbb{R}}\) y\(c\) es un punto de cúmulo de\(S\). Supongamos que\(f \colon S \to {\mathbb{R}}\) está acotado. Demostrar que existe una secuencia\(\{ x_n \}\) con\(x_n \in S \setminus \{ c \}\) y\(\lim\, x_n = c\) tal que\(\{ f(x_n) \}\) converge.

    [exercise:contlimitbadcomposition] Mostrar que la hipótesis de que\(g(c_2) = L\) en es necesaria. Es decir, encontrar\(f\) y\(g\) tal que\(f(x) \to c_2\) como\(x \to c_1\) y\(g(y) \to L\) como\(y \to c_2\), pero\(g\bigl(f(x)\bigr)\) no vaya a\(L\) como\(x \to c_1\).

    Funciones continuas

    Nota: 2—2.5 conferencias

    Sin duda escuchaste de funciones continuas en tu escolaridad. Un criterio de secundaria para este concepto es que una función es continua si podemos dibujar su gráfica sin levantar la pluma del papel. Si bien ese concepto intuitivo puede ser útil en situaciones simples, requerimos rigor. La siguiente definición tomó a tres grandes matemáticos (Bolzano, Cauchy, y finalmente Weierstrass) para obtener correctamente y su forma final data sólo a finales del siglo XIX.

    Definición y propiedades básicas

    Dejar\(S \subset {\mathbb{R}}\),\(c \in S\), y dejar\(f \colon S \to {\mathbb{R}}\) ser una función. Decimos que\(f\) es continuo en\(c\) si por cada\(\epsilon > 0\) hay\(\delta > 0\) tal que cuando\(x \in S\) y\(\left\lvert {x-c} \right\rvert < \delta\), entonces\(\left\lvert {f(x)-f(c)} \right\rvert < \epsilon\).

    Cuando\(f \colon S \to {\mathbb{R}}\) es continuo en absoluto\(c \in S\), entonces simplemente decimos que\(f\) es una función continua.

    Si\(f\) es continuo para todos\(c \in A\), decimos que\(f\) es continuo en\(A \subset S\). Se deja como un ejercicio fácil para demostrar que esto implica que\(f|_A\) es continuo, aunque lo contrario no se sostiene.

    La continuidad puede ser la definición más importante a entender en el análisis, y no es fácil. Ver. Tenga en cuenta que\(\delta\) no sólo depende de\(\epsilon\), sino también de\(c\); no necesitamos escoger uno\(\delta\) para todos\(c \in S\). No es casualidad que la definición de continuidad sea similar a la definición de un límite de una función. La característica principal de las funciones continuas es que estas son precisamente las funciones que se comportan muy bien con límites.

    [contbasic:prop] Supongamos que\(f \colon S \to {\mathbb{R}}\) es una función y\(c \in S\). Entonces

    1. Si no\(c\) es un punto de clúster de\(S\), entonces\(f\) es continuo en\(c\).
    2. Si\(c\) es un punto de clúster de\(S\), entonces\(f\) es continuo en\(c\) si y solo si el límite de\(f(x)\) como\(x \to c\) existe y\[\lim_{x\to c} f(x) = f(c) .\]
    3. \(f\)es continuo en\(c\) si y solo si para cada secuencia\(\{ x_n \}\) donde\(x_n \in S\) y\(\lim\, x_n = c\), la secuencia\(\{ f(x_n) \}\) converge a\(f(c)\).

    Empecemos con el primer ítem. Supongamos que no\(c\) es un punto de clúster de\(S\). Entonces existe\(\delta > 0\) tal que\(S \cap (c-\delta,c+\delta) = \{ c \}\). Por lo tanto, para cualquiera\(\epsilon > 0\), simplemente escoja este delta dado. El único\(x \in S\) tal que\(\left\lvert {x-c} \right\rvert < \delta\) es\(x=c\). Entonces\(\left\lvert {f(x)-f(c)} \right\rvert = \left\lvert {f(c)-f(c)} \right\rvert = 0 < \epsilon\).

    Pasemos al segundo ítem. Supongamos que\(c\) es un punto de clúster de\(S\). Supongamos primero eso\(\lim_{x\to c} f(x) = f(c)\). Entonces para cada\(\epsilon > 0\) hay\(\delta > 0\) tal que si\(x \in S \setminus \{ c \}\) y\(\left\lvert {x-c} \right\rvert < \delta\), entonces\(\left\lvert {f(x)-f(c)} \right\rvert < \epsilon\). Como\(\left\lvert {f(c)-f(c)} \right\rvert = 0 < \epsilon\), entonces se satisface la definición de continuidad en\(c\). Por otro lado, supongamos que\(f\) es continuo en\(c\). Para cada\(\epsilon > 0\), existe\(\delta > 0\) tal que para\(x \in S\) donde\(\left\lvert {x-c} \right\rvert < \delta\) tenemos\(\left\lvert {f(x)-f(c)} \right\rvert < \epsilon\). Entonces la afirmación es, por supuesto, sigue siendo cierta si\(x \in S \setminus \{ c \} \subset S\). Por lo tanto\(\lim_{x\to c} f(x) = f(c)\).

    Para el tercer ítem, supongamos que\(f\) es continuo en\(c\). Que\(\{ x_n \}\) sea una secuencia tal que\(x_n \in S\) y\(\lim\, x_n = c\). Dejemos\(\epsilon > 0\) que se den. Encuentra un\(\delta > 0\) tal que\(\left\lvert {f(x)-f(c)} \right\rvert < \epsilon\) para todos\(x \in S\) donde\(\left\lvert {x-c} \right\rvert < \delta\). Encuentra un\(M \in {\mathbb{N}}\) tal que para\(n \geq M\) nosotros tenemos\(\left\lvert {x_n-c} \right\rvert < \delta\). Entonces porque\(n \geq M\) tenemos eso\(\left\lvert {f(x_n)-f(c)} \right\rvert < \epsilon\), así\(\{ f(x_n) \}\) converge a\(f(c)\).

    Demostremos lo contrario del tercer ítem por contrapositivo. Supongamos que no\(f\) es continuo en\(c\). Entonces existe\(\epsilon > 0\) tal que para todos\(\delta > 0\), existe\(x \in S\) tal que\(\left\lvert {x-c} \right\rvert < \delta\) y\(\left\lvert {f(x)-f(c)} \right\rvert \geq \epsilon\). Definamos una secuencia de la\(\{ x_n \}\) siguiente manera. Que\(x_n \in S\) sean tales que\(\left\lvert {x_n-c} \right\rvert < \nicefrac{1}{n}\) y\(\left\lvert {f(x_n)-f(c)} \right\rvert \geq \epsilon\). Ahora\(\{ x_n \}\) es una secuencia de números en\(S\) tal que\(\lim\, x_n = c\) y tal que\(\left\lvert {f(x_n)-f(c)} \right\rvert \geq \epsilon\) para todos\(n \in {\mathbb{N}}\). Así\(\{ f(x_n) \}\) no converge a\(f(c)\). Puede o no converger, pero definitivamente no converge a\(f(c)\).

    El último ítem de la proposición es particularmente poderoso. Nos permite aplicar rápidamente lo que sabemos sobre límites de secuencias a funciones continuas e incluso demostrar que ciertas funciones son continuas. También se puede fortalecer, ver.

    \(f \colon (0,\infty) \to {\mathbb{R}}\)definido por\(f(x) := \nicefrac{1}{x}\) es continuo.

    Prueba: Fijar\(c \in (0,\infty)\). Que\(\{ x_n \}\) sea una secuencia en\((0,\infty)\) tal que\(\lim\, x_n = c\). Entonces sabemos que\[f(c) = \frac{1}{c} = \frac{1}{\lim\, x_n} = \lim_{n \to \infty} \frac{1}{x_n} = \lim_{n \to \infty} f(x_n) .\] Así\(f\) es continuo en\(c\). Como\(f\) es continuo en absoluto\(c \in (0,\infty)\),\(f\) es continuo.

    Anteriormente hemos mostrado\(\lim_{x \to c} x^2 = c^2\) directamente. Por lo tanto la función\(x^2\) es continua. Podemos utilizar la continuidad de las operaciones algebraicas con respecto a los límites de secuencias, que probamos en el capítulo anterior, para probar un resultado mucho más general.

    \(f \colon {\mathbb{R}}\to {\mathbb{R}}\)Sea un polinomio. Eso es\[f(x) = a_d x^d + a_{d-1} x^{d-1} + \cdots + a_1 x + a_0 ,\] para algunas constantes\(a_0, a_1, \ldots, a_d\). Entonces\(f\) es continuo.

    Arreglar\(c \in {\mathbb{R}}\). Que\(\{ x_n \}\) sea una secuencia tal que\(\lim\, x_n = c\). Entonces\[\begin{split} f(c) &= a_d c^d + a_{d-1} c^{d-1} + \cdots + a_1 c + a_0 \\ &= a_d {(\lim\, x_n)}^d + a_{d-1} {(\lim\, x_n)}^{d-1} + \cdots + a_1 (\lim\, x_n) + a_0 \\ & = \lim_{n \to \infty} \left( a_d x_n^d + a_{d-1} x_n^{d-1} + \cdots + a_1 x_n + a_0 \right) = \lim_{n \to \infty} f(x_n) . \end{split}\] Así\(f\) es continuo en\(c\). Como\(f\) es continuo en absoluto\(c \in {\mathbb{R}}\),\(f\) es continuo.

    Por razonamiento similar, o apelando a, podemos probar lo siguiente. Los detalles de la prueba se dejan como ejercicio.

    [contalg:prop] Dejar\(f \colon S \to {\mathbb{R}}\) y\(g \colon S \to {\mathbb{R}}\) ser funciones continuas en\(c \in S\).

    1. La función\(h \colon S \to {\mathbb{R}}\) definida por\(h(x) := f(x)+g(x)\) es continua en\(c\).
    2. La función\(h \colon S \to {\mathbb{R}}\) definida por\(h(x) := f(x)-g(x)\) es continua en\(c\).
    3. La función\(h \colon S \to {\mathbb{R}}\) definida por\(h(x) := f(x)g(x)\) es continua en\(c\).
    4. Si\(g(x)\not=0\) para todos\(x \in S\), la función\(h \colon S \to {\mathbb{R}}\) definida por\(h(x) := \frac{f(x)}{g(x)}\) es continua en\(c\).

    [sincos:ejemplo] Las funciones\(\sin(x)\) y\(\cos(x)\) son continuas. En los siguientes cálculos utilizamos las identidades trigonométricas sumar-producto. También utilizamos los hechos simples que\(\left\lvert {\sin(x)} \right\rvert \leq \left\lvert {x} \right\rvert\),\(\left\lvert {\cos(x)} \right\rvert \leq 1\), y\(\left\lvert {\sin(x)} \right\rvert \leq 1\). \[\begin{split} \left\lvert {\sin(x)-\sin(c)} \right\rvert & = \left\lvert { 2 \sin \left( \frac{x-c}{2} \right) \cos \left( \frac{x+c}{2} \right) } \right\rvert \\ & = 2 \left\lvert { \sin \left( \frac{x-c}{2} \right) } \right\rvert \left\lvert { \cos \left( \frac{x+c}{2} \right) } \right\rvert \\ & \leq 2 \left\lvert { \sin \left( \frac{x-c}{2} \right) } \right\rvert \\ & \leq 2 \left\lvert { \frac{x-c}{2} } \right\rvert = \left\lvert {x-c} \right\rvert \end{split}\]\[\begin{split} \left\lvert {\cos(x)-\cos(c)} \right\rvert & = \left\lvert { -2 \sin \left( \frac{x-c}{2} \right) \sin \left( \frac{x+c}{2} \right) } \right\rvert \\ & = 2 \left\lvert { \sin \left( \frac{x-c}{2} \right) } \right\rvert \left\lvert { \sin \left( \frac{x+c}{2} \right) } \right\rvert \\ & \leq 2 \left\lvert { \sin \left( \frac{x-c}{2} \right) } \right\rvert \\ & \leq 2 \left\lvert { \frac{x-c}{2} } \right\rvert = \left\lvert {x-c} \right\rvert \end{split}\]

    La afirmación de que pecado y cos son continuos sigue tomando una secuencia arbitraria\(\{ x_n \}\) convergente a\(c\), o aplicando directamente la definición de continuidad. Los detalles se dejan al lector.

    Composición de funciones continuas

    Probablemente ya te hayas dado cuenta de que una de las herramientas básicas para construir funciones complicadas a partir de simples es la composición. Una propiedad útil de las funciones continuas es que las composiciones de funciones continuas vuelven a ser continuas. Recordemos que para dos funciones\(f\) y\(g\), la composición\(f \circ g\) se define por\((f \circ g)(x) := f\bigl(g(x)\bigr)\).

    Dejar\(A, B \subset {\mathbb{R}}\)\(f \colon B \to {\mathbb{R}}\) y\(g \colon A \to B\) ser funciones. Si\(g\) es continuo en\(c \in A\) y\(f\) es continuo en\(g(c)\), entonces\(f \circ g \colon A \to {\mathbb{R}}\) es continuo en\(c\).

    Que\(\{ x_n \}\) sea una secuencia en\(A\) tal que\(\lim\, x_n = c\). Entonces como\(g\) es continuo en\(c\), luego\(\{ g(x_n) \}\) converge a\(g(c)\). Como\(f\) es continuo en\(g(c)\), luego\(\{ f\bigl(g(x_n)\bigr) \}\) converge a\(f\bigl(g(c)\bigr)\). Así\(f \circ g\) es continuo en\(c\).

    Reclamación:\({\bigl(\sin(\nicefrac{1}{x})\bigr)}^2\) es una función continua sobre\((0,\infty)\).

    Prueba: Primera nota que\(\nicefrac{1}{x}\) es una función continua\((0,\infty)\) y\(\sin(x)\) es una función continua en\((0,\infty)\) (en realidad en todos\({\mathbb{R}}\), pero\((0,\infty)\) es el rango para\(\nicefrac{1}{x}\)). De ahí que la composición\(\sin(\nicefrac{1}{x})\) sea continua. También sabemos que\(x^2\) es continuo en el intervalo\((-1,1)\) (el rango de pecado). Por lo tanto, la composición también\({\bigl(\sin(\nicefrac{1}{x})\bigr)}^2\) es continua\((0,\infty)\).

    Funciones discontinuas

    Cuando no\(f\) es continuo en\(c\), decimos que\(f\) es discontinuo en\(c\), o que tiene una discontinuidad en\(c\). Si declaramos el contrapositivo del tercer ítem de como reclamo separado, obtenemos una prueba fácil de usar para detectar discontinuidades.

    Dejar\(f \colon S \to {\mathbb{R}}\) ser una función. Supongamos que para algunos\(c \in S\), existe una secuencia\(\{ x_n \}\)\(x_n \in S\),, y\(\lim\, x_n = c\) tal que\(\{ f(x_n) \}\) no converge a\(f(c)\) (o no converge en absoluto), entonces no\(f\) es continua en\(c\).

    [ejemplo:stepdiscont] La función\(f \colon {\mathbb{R}}\to {\mathbb{R}}\) definida por no\[f(x) := \begin{cases} -1 & \text{ if $x < 0$,} \\ 1 & \text{ if $x \geq 0$,} \end{cases}\] es continua a 0.

    Prueba: Toma la secuencia\(\{ - \nicefrac{1}{n} \}\). Entonces\(f(-\nicefrac{1}{n}) = -1\) y así\(\lim\, f(-\nicefrac{1}{n}) = -1\), pero\(f(0) = 1\).

    Para un ejemplo extremo tomamos la llamada función Dirichlet. \[f(x) := \begin{cases} 1 & \text{ if $x$ is rational,} \\ 0 & \text{ if $x$ is irrational.} \end{cases}\]La función\(f\) es discontinua en absoluto\(c \in {\mathbb{R}}\).

    Prueba: Supongamos que\(c\) es racional. Tomar una secuencia\(\{ x_n \}\) de números irracionales tal que\(\lim\, x_n = c\) (¿por qué podemos?). Entonces\(f(x_n) = 0\) y así\(\lim\, f(x_n) = 0\), pero\(f(c) = 1\). Si\(c\) es irracional, tomar una secuencia de números racionales\(\{ x_n \}\) que converja a\(c\) (¿por qué podemos?). Entonces\(\lim\, f(x_n) = 1\), pero\(f(c) = 0\).

    Volvamos a probar los límites de tu intuición. ¿Puede existir una función que sea continua en todos los números irracionales, pero discontinua en todos los números racionales? Hay números racionales arbitrariamente cercanos a cualquier número irracional. Quizás extrañamente, la respuesta es sí. El siguiente ejemplo se llama la función Thomae 15 o la función palomitas de maíz.

    [popcornfunction:ejemplo] Dejar\(f \colon (0,1) \to {\mathbb{R}}\) ser definido por\[f(x) := \begin{cases} \nicefrac{1}{k} & \text{ if $x=\nicefrac{m}{k}$ where $m,k \in {\mathbb{N}}$ and $m$ and $k$ have no common divisors,} \\ 0 & \text{ if $x$ is irrational}. \end{cases}\] Entonces\(f\) es continuo en absoluto irracional\(c \in (0,1)\) y discontinuo en todo racional\(c\). Ver la gráfica de\(f\) en.

    Gráfica de la “función palomitas de maíz”. [popcornfig]
    Gráfica de la “función palomitas de maíz”. [popcornfig]

    Prueba: Supongamos que\(c = \nicefrac{m}{k}\) es racional. Toma una secuencia de números irracionales\(\{ x_n \}\) tal que\(\lim\, x_n = c\). Entonces\(\lim\, f(x_n) = \lim \, 0 = 0\), pero\(f(c) = \nicefrac{1}{k} \not= 0\). Así\(f\) es discontinuo en\(c\).

    Ahora vamos a\(c\) ser irracionales, entonces\(f(c) = 0\). Tomar una secuencia\(\{ x_n \}\) de números en\((0,1)\) tal que\(\lim\, x_n = c\). Dado\(\epsilon > 0\), encuentra\(K \in {\mathbb{N}}\) tal que\(\nicefrac{1}{K} < \epsilon\) por el. Si\(\nicefrac{m}{k} \in (0,1)\) es términos más bajos (sin divisores comunes), entonces\(m < k\). Entonces solo hay finitamente muchos números racionales en\((0,1)\) cuyo denominador\(k\) en términos más bajos es menor que\(K\). De ahí que exista\(M\) tal que para\(n \geq M\), todos los números\(x_n\) que son racionales tienen un denominador mayor o igual a\(K\). Así para\(n \geq M\)\[\left\lvert {f(x_n) - 0} \right\rvert = f(x_n) \leq \nicefrac{1}{K} < \epsilon .\] Por lo tanto\(f\) es continuo en irracional\(c\).

    Terminemos con un ejemplo más fácil.

    Definir\(g \colon {\mathbb{R}}\to {\mathbb{R}}\) por\(g(x) := 0\) si\(x \not= 0\) y\(g(0) := 1\). Entonces no\(g\) es continuo a cero, sino continuo en todas partes (¿por qué?). El punto\(x=0\) se llama una discontinuidad removible. Eso es porque si cambiáramos la definición de\(g\), al insistir en que\(g(0)\) sea\(0\), obtendríamos una función continua. Por otro lado dejó\(f\) ser la función del ejemplo. Entonces\(f\) no tiene una discontinuidad removible en\(0\). No importa cómo definamos\(f(0)\) la función seguirá fallando en ser continua. La diferencia es que\(\lim_{x\to 0} g(x)\) existe mientras que\(\lim_{x\to 0} f(x)\) no.

    Quedémonos con este ejemplo pero mostremos otro fenómeno. Vamos\(A = \{ 0 \}\), entonces\(g|_A\) es continuo (¿por qué?) , mientras que no\(g\) es continuo encendido\(A\).

    Ejercicios

    El uso de la definición de continuidad demuestra directamente que\(f \colon {\mathbb{R}}\to {\mathbb{R}}\) definido por\(f(x) := x^2\) es continuo.

    El uso de la definición de continuidad demuestra directamente que\(f \colon (0,\infty) \to {\mathbb{R}}\) definido por\(f(x) := \nicefrac{1}{x}\) es continuo.

    Dejar\(f \colon {\mathbb{R}}\to {\mathbb{R}}\) definirse mediante el\[f(x) := \begin{cases} x & \text{ if $x$ is rational,} \\ x^2 & \text{ if $x$ is irrational.} \end{cases}\] uso de la definición de continuidad probar directamente que\(f\) es continuo en\(1\) y discontinuo en\(2\).

    Dejar\(f \colon {\mathbb{R}}\to {\mathbb{R}}\) ser definido por\[f(x) := \begin{cases} \sin(\nicefrac{1}{x}) & \text{ if $x \not= 0$,} \\ 0 & \text{ if $x=0$.} \end{cases}\] ¿Es\(f\) continuo? Demuestra tu aseveración.

    Dejar\(f \colon {\mathbb{R}}\to {\mathbb{R}}\) ser definido por\[f(x) := \begin{cases} x \sin(\nicefrac{1}{x}) & \text{ if $x \not= 0$,} \\ 0 & \text{ if $x=0$.} \end{cases}\] ¿Es\(f\) continuo? Demuestra tu aseveración.

    Demostrar.

    Demostrar la siguiente declaración. Dejar\(S \subset {\mathbb{R}}\) y\(A \subset S\). Dejar\(f \colon S \to {\mathbb{R}}\) ser una función continua. Entonces la restricción\(f|_A\) es continua.

    Supongamos\(S \subset {\mathbb{R}}\). Supongamos para algunos\(c \in {\mathbb{R}}\) y\(\alpha > 0\), tenemos\(A=(c-\alpha,c+\alpha) \subset S\). Dejar\(f \colon S \to {\mathbb{R}}\) ser una función. Demostrar que si\(f|_A\) es continuo en\(c\), entonces\(f\) es continuo en\(c\).

    Dar un ejemplo de funciones\(f \colon {\mathbb{R}}\to {\mathbb{R}}\) y\(g \colon {\mathbb{R}}\to {\mathbb{R}}\) tal que la función\(h\) definida por\(h(x) := f(x) + g(x)\) es continua, pero\(f\) y no\(g\) son continuas. ¿Se puede encontrar\(f\) y\(g\) que en ninguna parte son continuos, sino que\(h\) es una función continua?

    Dejar\(f \colon {\mathbb{R}}\to {\mathbb{R}}\) y\(g \colon {\mathbb{R}}\to {\mathbb{R}}\) ser funciones continuas. Supongamos que para todos los números racionales\(r\),\(f(r) = g(r)\). \(f(x) = g(x)\)Demuéstralo para todos\(x\).

    Dejar\(f \colon {\mathbb{R}}\to {\mathbb{R}}\) ser continuo. Supongamos\(f(c) > 0\). Demostrar que existe\(\alpha > 0\) tal que por todo\(x \in (c-\alpha,c+\alpha)\) lo que tenemos\(f(x) > 0\).

    Dejar\(f \colon {\mathbb{Z}}\to {\mathbb{R}}\) ser una función. Demostrar que\(f\) es continuo.

    [exercise:contseqalt] Dejar\(f \colon S \to {\mathbb{R}}\) ser una función y\(c \in S\), tal que para cada secuencia\(\{ x_n \}\) en\(S\) con\(\lim\, x_n = c\), la secuencia\(\{ f(x_n) \}\) converja. Espectáculo que\(f\) es continuo en\(c\).

    Supongamos\(f \colon [-1,0] \to {\mathbb{R}}\) y\(g \colon [0,1] \to {\mathbb{R}}\) son continuos y\(f(0) = g(0)\). Definir\(h \colon [-1,1] \to {\mathbb{R}}\) por\(h(x) := f(x)\) si\(x \leq 0\) y\(h(x) := g(x)\) si\(x > 0\). Demostrar que\(h\) es continuo.

    Supongamos que\(g \colon {\mathbb{R}}\to {\mathbb{R}}\) es una función continua tal que\(g(0) = 0\), y supppse\(f \colon {\mathbb{R}}\to {\mathbb{R}}\) es tal que\(\left\lvert {f(x)-f(y)} \right\rvert \leq g(x-y)\) para todos\(x\) y\(y\). Demostrar que\(f\) es continuo.

    Supongamos\(f(x+y) = f(x) + f(y)\) para algunos\(f \colon {\mathbb{R}}\to {\mathbb{R}}\) tales que\(f\) es continuo a 0. \(f(x) = ax\)Demuéstralo para algunos\(a \in {\mathbb{R}}\). Pista: Demuéstralo\(f(nx) = nf(x)\), luego\(f\) el show es continuo\({\mathbb{R}}\). Entonces\(\nicefrac{f(x)}{x} = f(1)\) demuéstralo para todos racionales\(x\).

    Teoremas de valor min-max e intermedio

    Nota: 1.5 conferencias

    Las funciones continuas definidas en intervalos cerrados y acotados tienen algunas propiedades interesantes y muy útiles.

    Teorema Min-Max

    Recordar una función\(f \colon [a,b] \to {\mathbb{R}}\) está acotada si existe\(B \in {\mathbb{R}}\) tal que\(\left\lvert {f(x)} \right\rvert \leq B\) para todos\(x \in [a,b]\). Tenemos el siguiente lema.

    Dejar\(f \colon [a,b] \to {\mathbb{R}}\) ser una función continua. Entonces\(f\) se acota.

    Demostremos esta afirmación por contrapositivo. Supongamos que no\(f\) está acotado, entonces para cada uno\(n \in {\mathbb{N}}\), hay una\(x_n \in [a,b]\), tal que\[\left\lvert {f(x_n)} \right\rvert \geq n .\] Ahora\(\{ x_n \}\) es una secuencia acotada como\(a \leq x_n \leq b\). Por el, hay una subsecuencia convergente\(\{ x_{n_i} \}\). Vamos\(x := \lim\, x_{n_i}\). Ya que\(a \leq x_{n_i} \leq b\) para todos\(i\), entonces\(a \leq x \leq b\). El límite\(\lim\, f(x_{n_i})\) no existe ya que la secuencia no está delimitada como\(\left\lvert {f(x_{n_i})} \right\rvert \geq n_i \geq i\). Por otro lado\(f(x)\) es un número finito y\[f(x) = f\left( \lim_{i\to\infty} x_{n_i} \right) .\] así no\(f\) es continuo en\(x\).

    De hecho, para un continuo\(f\), veremos que en realidad se logran el mínimo y el máximo. Recordemos del cálculo que\(f \colon S \to {\mathbb{R}}\) logra un mínimo absoluto\[f(x) \geq f(c) \qquad \text{ for all $x \in S$.}\] en\(c \in S\) si Por otro lado,\(f\) logra un máximo absoluto en\(c \in S\) si\[f(x) \leq f(c) \qquad \text{ for all $x \in S$.}\] Decimos\(f\) logra un mínimo absoluto o un máximo absoluto en\(S\) si tal \(c \in S\)existe. Si\(S\) es un intervalo cerrado y delimitado, entonces un continuo\(f\) debe tener un mínimo absoluto y un máximo absoluto activado\(S\).

    Dejar\(f \colon [a,b] \to {\mathbb{R}}\) ser una función continua. Después\(f\) logra tanto un mínimo absoluto como un máximo absoluto on\([a,b]\).

    Nosotros hemos demostrado que\(f\) está delimitado por el lema. Por lo tanto, el conjunto\(f([a,b]) = \{ f(x) : x \in [a,b] \}\) tiene un supremo y un infimum. Por lo que sabemos de suprema e infima, existen secuencias en el conjunto\(f([a,b])\) que se acercan a ellas. Es decir, hay secuencias\(\{ f(x_n) \}\) y\(\{ f(y_n) \}\), en dónde\(x_n, y_n\) están\([a,b]\), tales que aún no\[\lim_{n\to\infty} f(x_n) = \inf f([a,b]) \qquad \text{and} \qquad \lim_{n\to\infty} f(y_n) = \sup f([a,b]).\] hemos terminado, necesitamos encontrar dónde están los mínimos y los máximos. El problema es que las secuencias\(\{ x_n \}\) y no\(\{ y_n \}\) necesitan converger. Conocemos\(\{ x_n \}\) y\(\{ y_n \}\) estamos acotados (sus elementos pertenecen a un intervalo acotado\([a,b]\)). Aplicamos el. De ahí que existan subsecuencias convergentes\(\{ x_{n_i} \}\) y\(\{ y_{m_i} \}\). Que\[x := \lim_{i\to\infty} x_{n_i} \qquad \text{and} \qquad y := \lim_{i\to\infty} y_{m_i}.\] Entonces como\(a \leq x_{n_i} \leq b\), tenemos eso\(a \leq x \leq b\). Del mismo modo\(a \leq y \leq b\), así\(x\) y\(y\) están en\([a,b]\). Aplicamos que un límite de una subsecuencia es el mismo que el límite de la secuencia, y aplicamos la continuidad de\(f\) para obtener\[\inf f([a,b]) = \lim_{n\to\infty} f(x_n) = \lim_{i\to\infty} f(x_{n_i}) = f \left( \lim_{i\to\infty} x_{n_i} \right) = f(x) .\] De manera similar,\[\sup f([a,b]) = \lim_{n\to\infty} f(m_n) = \lim_{i\to\infty} f(y_{m_i}) = f \left( \lim_{i\to\infty} y_{m_i} \right) = f(y) .\] por lo tanto,\(f\) logra un mínimo absoluto en\(x\) y\(f\) logra un máximo absoluto en\(y\).

    La función\(f(x) := x^2+1\) definida en el intervalo\([-1,2]\) alcanza un mínimo en\(x=0\) cuando\(f(0) = 1\). Logra un máximo en\(x=2\) donde\(f(2) = 5\). Señalar que el dominio de la definición importa. Si en cambio tomáramos el dominio para ser\([-10,10]\), entonces ya no\(x=2\) sería un máximo de\(f\). En cambio el máximo se lograría en cualquiera\(x=10\) o\(x=-10\).

    Demostremos con ejemplos que las diferentes hipótesis del teorema son realmente necesarias.

    La función\(f(x) := x\), definida en toda la línea real, no logra ni un mínimo, ni un máximo. Por lo que es importante que estemos viendo un intervalo acotado.

    La función\(f(x) := \nicefrac{1}{x}\), definida en no\((0,1)\) logra ni un mínimo, ni un máximo. Los valores de la función no están acotados a medida que nos acercamos a 0. También a medida que nos acercamos\(x=1\), los valores de la función se acercan a 1, pero\(f(x) > 1\) para todos\(x \in (0,1)\). No hay\(x \in (0,1)\) tal que\(f(x) = 1\). Por lo que es importante que estemos viendo un intervalo cerrado.

    La continuidad es importante. Definir\(f \colon [0,1] \to {\mathbb{R}}\) por\(f(x) := \nicefrac{1}{x}\) para\(x > 0\) y dejar\(f(0) := 0\). Entonces la función no logra un máximo. El problema es que la función no es continua a 0.

    Teorema del valor intermedio de Bolzano

    El teorema del valor intermedio de Bolzano es una de las piedras angulares del análisis. A veces se le llama solamente teorema de valor intermedio, o simplemente teorema de Bolzano. Para probar el teorema de Bolzano probamos el siguiente lema más simple.

    [IVT:Lemma] Dejar\(f \colon [a,b] \to {\mathbb{R}}\) ser una función continua. Supongamos\(f(a) < 0\) y\(f(b) > 0\). Entonces existe un número\(c \in (a,b)\) tal que\(f(c) = 0\).

    Definimos dos secuencias\(\{ a_n \}\) e\(\{ b_n \}\) inductivamente:

    1. Dejar\(a_1 := a\) y\(b_1 := b\).
    2. Si\(f\left(\frac{a_n+b_n}{2}\right) \geq 0\), vamos\(a_{n+1} := a_n\) y\(b_{n+1} := \frac{a_n+b_n}{2}\).
    3. Si\(f\left(\frac{a_n+b_n}{2}\right) < 0\), vamos\(a_{n+1} := \frac{a_n+b_n}{2}\) y\(b_{n+1} := b_n\).

    Vea un ejemplo definiendo los cinco primeros pasos. De la definición de las dos secuencias es obvio que si\(a_n < b_n\), entonces\(a_{n+1} < b_{n+1}\). Así por\(a_n < b_n\) para todos\(n\). Además,\(a_n \leq a_{n+1}\) y\(b_n \geq b_{n+1}\) para todos\(n\), es decir, las secuencias son monótonas. Como\(a_n < b_n \leq b_1 = b\) y\(b_n > a_n \geq a_1 = a\) para todos\(n\), las secuencias también están acotadas. Por lo tanto, las secuencias convergen. Dejar\(c := \lim\, a_n\) y\(d := \lim\, b_n\). Ahora queremos demostrarlo\(c=d\). Nos damos cuenta\[b_{n+1} - a_{n+1} = \frac{b_n-a_n}{2}.\] Por vemos que\[b_n - a_n = \frac{b_1-a_1}{2^{n-1}} = 2^{1-n} (b-a) .\] Como\(2^{1-n}(b-a)\) converge a cero, tomamos el límite como\(n\) va al infinito para llegar\[d-c = \lim_{n\to\infty} (b_n - a_n) = \lim_{n\to\infty} 2^{1-n} (b-a) = 0.\] En otras palabras\(d=c\).

    Por construcción, por todo\(n\) lo que\[f(a_n) < 0 \qquad \text{and} \qquad f(b_n) \geq 0 .\] tenemos Utilizamos el hecho de que\(\lim\, a_n = \lim\, b_n = c\) y la continuidad de\(f\) tomar límites en esas desigualdades para conseguir\[f(c) = \lim\, f(a_n) \leq 0 \qquad \text{and} \qquad f(c) = \lim\, f(b_n) \geq 0 .\] As\(f(c) \geq 0\) y\(f(c) \leq 0\), concluimos\(f(c) = 0\). Obviamente,\(a < c < b\).

    Observe que la prueba nos dice cómo encontrar el\(c\). La prueba no sólo es útil para nosotros los matemáticos puros, sino que es una idea útil en las matemáticas aplicadas.

    [IVT:thm] Dejar\(f \colon [a,b] \to {\mathbb{R}}\) ser una función continua. Supongamos que existe\(y\) tal que\(f(a) < y < f(b)\) o\(f(a) > y > f(b)\). Entonces existe\(c \in (a,b)\) tal que\(f(c) = y\).

    El teorema dice que una función continua en un intervalo cerrado logra todos los valores entre los valores en los puntos finales.

    Si\(f(a) < y < f(b)\), entonces defina\(g(x) := f(x)-y\). Entonces vemos eso\(g(a) < 0\) y\(g(b) > 0\) y podemos aplicar a\(g\). Si\(g(c) = 0\), entonces\(f(c) = y\).

    Del mismo modo si\(f(a) > y > f(b)\), entonces defina\(g(x) := y-f(x)\). Entonces otra vez\(g(a) < 0\)\(g(b) > 0\) y ya podemos aplicar. Otra vez si\(g(c) = 0\), entonces\(f(c) = y\).

    Si una función es continua, entonces la restricción a un subconjunto es continua. Entonces si\(f \colon S \to {\mathbb{R}}\) es continuo y\([a,b] \subset S\), entonces también\(f|_{[a,b]}\) es continuo. Por lo tanto, generalmente aplicamos el teorema a una función continua en algún conjunto grande\(S\), pero restringimos la atención a un intervalo.

    El polinomio\(f(x) := x^3-2x^2+x-1\) tiene una raíz real en\((1,2)\). Simplemente notamos eso\(f(1) = -1\) y\(f(2) = 1\). De ahí que deba existir un punto\(c \in (1,2)\) tal que\(f(c) = 0\). Para encontrar una mejor aproximación de la raíz podríamos seguir la prueba de. Por ejemplo, a continuación miraríamos 1.5 y encontraríamos eso\(f(1.5) = -0.625\). Por lo tanto, hay una raíz de la ecuación en\((1.5,2)\). A continuación nos fijamos en 1.75 y notamos eso\(f(1.75) \approx -0.016\). De ahí que haya una raíz de\(f\) in\((1.75,2)\). A continuación nos fijamos en 1.875 y encontramos eso\(f(1.875) \approx 0.44\), así hay una raíz en\((1.75,1.875)\). Seguimos este procedimiento hasta obtener suficiente precisión.

    La técnica anterior es el método más simple de encontrar raíces de polinomios. Encontrar raíces de polinomios es quizás el problema más común en las matemáticas aplicadas. En general es difícil de hacer de forma rápida, precisa y automática. Podemos usar el teorema del valor intermedio para encontrar raíces para cualquier función continua, no solo un polinomio.

    Existen métodos mejores y más rápidos para encontrar raíces de ecuaciones, como el método de Newton. Una ventaja del método anterior es su simplicidad. En el momento en que encontramos un intervalo inicial donde se aplica el teorema del valor intermedio, se nos garantiza encontrar una raíz hasta una precisión deseada en finitamente muchos pasos. Además, el método solo requiere una función continua.

    El teorema garantiza al menos una\(c\) tal que\(f(c) = y\), pero puede haber muchas raíces diferentes de la ecuación\(f(c) = y\). Si seguimos el procedimiento de la prueba, se nos garantiza que encontraremos aproximaciones a una de esas raíces. Tenemos que trabajar más duro para encontrar otras raíces.

    Los polinomios de grado par pueden no tener raíces reales. Por ejemplo, no hay un número real\(x\) tal que\(x^2+1 = 0\). Los polinomios impares, por otro lado, siempre tienen al menos una raíz real.

    Dejar\(f(x)\) ser un polinomio de grado impar. Entonces\(f\) tiene una raíz real.

    Supongamos que\(f\) es un polinomio de grado impar\(d\). Escribimos\[f(x) = a_d x^d + a_{d-1} x^{d-1} + \cdots + a_1 x + a_0 ,\] dónde\(a_d \not= 0\). Dividimos por\(a_d\) para obtener un polinomio\[g(x) := x^d + b_{d-1} x^{d-1} + \cdots + b_1 x + b_0 ,\] donde\(b_k = \nicefrac{a_k}{a_d}\). Demostremos que\(g(n)\) es positivo para algunos grandes\(n \in {\mathbb{N}}\). \[\begin{split} \left\lvert {\frac{b_{d-1} n^{d-1} + \cdots + b_1 n + b_0}{n^d}} \right\rvert & = \frac{\left\lvert {b_{d-1} n^{d-1} + \cdots + b_1 n + b_0} \right\rvert}{n^d} \\ & \leq \frac{\left\lvert {b_{d-1}} \right\rvert n^{d-1} + \cdots + \left\lvert {b_1} \right\rvert n + \left\lvert {b_0} \right\rvert}{n^d} \\ & \leq \frac{\left\lvert {b_{d-1}} \right\rvert n^{d-1} + \cdots + \left\lvert {b_1} \right\rvert n^{d-1} + \left\lvert {b_0} \right\rvert n^{d-1}}{n^d} \\ & = \frac{n^{d-1}\bigl(\left\lvert {b_{d-1}} \right\rvert + \cdots + \left\lvert {b_1} \right\rvert + \left\lvert {b_0} \right\rvert\bigr)}{n^d} \\ & = \frac{1}{n} \bigl(\left\lvert {b_{d-1}} \right\rvert + \cdots + \left\lvert {b_1} \right\rvert + \left\lvert {b_0} \right\rvert\bigr) . \end{split}\]Por lo\[\lim_{n\to\infty} \frac{b_{d-1} n^{d-1} + \cdots + b_1 n + b_0}{n^d} = 0 .\] tanto, existe\(M \in {\mathbb{N}}\) tal lo\[\left\lvert {\frac{b_{d-1} M^{d-1} + \cdots + b_1 M + b_0}{M^d}} \right\rvert < 1 ,\] que implica\[-(b_{d-1} M^{d-1} + \cdots + b_1 M + b_0) < M^d .\] Por lo tanto\(g(M) > 0\).

    A continuación nos fijamos en\(g(-n)\) para\(n \in {\mathbb{N}}\). Por un argumento similar (ejercicio) encontramos que existe alguna\(K \in {\mathbb{N}}\) tal que\(b_{d-1} {(-K)}^{d-1} + \cdots + b_1 (-K) + b_0 < K^d\) y por lo tanto\(g(-K) < 0\) (¿por qué?). En la prueba asegúrate de usar el hecho que\(d\) es extraño. En particular, si\(d\) es impar entonces\({(-n)}^d = -(n^d)\).

    Apelamos al teorema del valor intermedio, para encontrar\(c \in [-K,M]\) tal que\(g(c) = 0\). Como\(g(x) = \frac{f(x)}{a_d}\), vemos eso\(f(c) = 0\), y se hace la prueba.

    Un dato interesante es que existen funciones discontinuas que tienen la propiedad de valor intermedio. La función no\[f(x) := \begin{cases} \sin(\nicefrac{1}{x}) & \text{ if $x \not= 0$,} \\ 0 & \text{ if $x=0$,} \end{cases}\] es continua a 0, sin embargo, tiene la propiedad de valor intermedio. Es decir, para cualquiera\(a < b\), y cualquiera\(y\) tal que\(f(a) < y < f(b)\) o\(f(a) > y > f(b)\), existe\(c\) tal que\(f(y) = c\). La prueba se deja como ejercicio.

    Ejercicios

    Encuentre un ejemplo de una función discontinua\(f \colon [0,1] \to {\mathbb{R}}\) donde falla el teorema del valor intermedio.

    Encuentre un ejemplo de una función discontinua acotada\(f \colon [0,1] \to {\mathbb{R}}\) que no tenga ni un mínimo absoluto ni un máximo absoluto.

    Que\(f \colon (0,1) \to {\mathbb{R}}\) sea una función continua tal que\(\displaystyle \lim_{x\to 0} f(x) = \displaystyle \lim_{x\to 1} f(x) = 0\). Demostrar que\(f\) logra ya sea un mínimo absoluto o un máximo absoluto en\((0,1)\) (pero quizás no ambos).

    Let\[f(x) := \begin{cases} \sin(\nicefrac{1}{x}) & \text{ if $x \not= 0$,} \\ 0 & \text{ if $x=0$.} \end{cases}\] Show que\(f\) tiene la propiedad de valor intermedio. Es decir, para cualquiera\(a < b\), si existe\(y\) tal que\(f(a) < y < f(b)\) o\(f(a) > y > f(b)\), entonces existe\(c \in (a,b)\) tal que\(f(c) = y\).

    Supongamos que\(g(x)\) es un polinomio de grado impar\(d\) tal que\[g(x) = x^d + b_{d-1} x^{d-1} + \cdots + b_1 x + b_0 ,\] para algunos números reales\(b_{0}, b_1, \ldots, b_{d-1}\). Demostrar que existe\(K \in {\mathbb{N}}\) tal que\(g(-K) < 0\). Pista: Asegúrate de usar el hecho que\(d\) es extraño. Tendrás que usar eso\({(-n)}^d = -(n^d)\).

    Supongamos que\(g(x)\) es un polinomio de grado par positivo\(d\) tal que\[g(x) = x^d + b_{d-1} x^{d-1} + \cdots + b_1 x + b_0 ,\] para algunos números reales\(b_{0}, b_1, \ldots, b_{d-1}\). Supongamos\(g(0) < 0\). Demostrar que\(g\) tiene al menos dos raíces reales distintas.

    [exercise:imageofinterval] Supongamos que\(f \colon [a,b] \to {\mathbb{R}}\) es una función continua. Demostrar que la imagen directa\(f([a,b])\) es un intervalo cerrado y delimitado o un solo número.

    Supongamos que\(f \colon {\mathbb{R}}\to {\mathbb{R}}\) es continuo y periódico con periodo\(P > 0\). Es decir,\(f(x+P) = f(x)\) para todos\(x \in {\mathbb{R}}\). Demostrar que\(f\) logra un mínimo absoluto y un máximo absoluto.

    Supongamos que\(f(x)\) es un polinomio acotado, es decir, hay\(M\) tal que\(\left\lvert {f(x)} \right\rvert \leq M\) para todos\(x \in {\mathbb{R}}\). \(f\)Demostrar que debe ser una constante.

    Supongamos que\(f \colon [0,1] \to [0,1]\) es continuo. Demostrar que\(f\) tiene un punto fijo, es decir, mostrar que existe\(x \in [0,1]\) tal eso\(f(x) = x\).

    Encuentre un ejemplo de una función acotada\(f \colon {\mathbb{R}}\to {\mathbb{R}}\) que no logre un mínimo absoluto ni un máximo absoluto on\({\mathbb{R}}\).

    Supongamos que\(f \colon {\mathbb{R}}\to {\mathbb{R}}\) es una función continua tal que\(x \leq f(x) \leq x+1\) para todos\(x \in {\mathbb{R}}\). Encuentra\(f({\mathbb{R}})\).

    Verdadero/Falso, probar o encontrar un contraejemplo. Si\(f \colon {\mathbb{R}}\to {\mathbb{R}}\) es una función continua tal que\(f|_

    ParseError: invalid DekiScript (click for details)
    Callstack:
        at (Matematicas/Analisis/Introducción_al_Análisis_Real_(Lebl)/02:_Números_reales/2.01:_Propiedades_básicas), /content/body/div[19]/p[13]/span[2]/span, line 1, column 1
    
    \) está acotada, entonces\(f\) es acotada.

    Continuidad uniforme

    Nota: 1.5—2 conferencias (la extensión continua y Lipschitz pueden ser opcionales)

    Continuidad uniforme

    Hicimos un alboroto al decir que el\(\delta\) en la definición de continuidad dependía del punto\(c\). Hay situaciones en las que es ventajoso tener un\(\delta\) independiente de cualquier punto. Demos un nombre a este concepto.

    Dejar\(S \subset {\mathbb{R}}\), y dejar que\(f \colon S \to {\mathbb{R}}\) sea una función. Supongamos que para cualquiera\(\epsilon > 0\) existe\(\delta > 0\) tal que cuando\(x, c \in S\) y\(\left\lvert {x-c} \right\rvert < \delta\), entonces\(\left\lvert {f(x)-f(c)} \right\rvert < \epsilon\). Entonces decimos que\(f\) es uniformemente continuo.

    No es difícil ver que una función uniformemente continua debe ser continua. La única diferencia en las definiciones es que para un dado\(\epsilon > 0\) elegimos una\(\delta > 0\) que funcione para todos\(c \in S\). Es decir, ya no\(\delta\) puede depender\(c\), sólo depende de\(\epsilon\). El dominio de definición de la función marca la diferencia ahora. Una función que no es uniformemente continua en un conjunto más grande, puede ser uniformemente continua cuando se restringe a un conjunto más pequeño.

    La función\(f \colon (0,1) \to {\mathbb{R}}\), definida por no\(f(x) := \nicefrac{1}{x}\) es uniformemente continua, sino continua.

    Prueba: Dado\(\epsilon > 0\), entonces\(\epsilon > \left\lvert {\nicefrac{1}{x}-\nicefrac{1}{y}} \right\rvert\) para sostenernos debemos tener\[\epsilon > \left\lvert {\nicefrac{1}{x}-\nicefrac{1}{y}} \right\rvert = \frac{\left\lvert {y-x} \right\rvert}{\left\lvert {xy} \right\rvert} = \frac{\left\lvert {y-x} \right\rvert}{xy} ,\] o\[\left\lvert {x-y} \right\rvert < xy \epsilon .\] Por lo tanto, para satisfacer la definición de continuidad uniforme tendríamos que tener\(\delta \leq xy \epsilon\) para todos\(x,y\) adentro\((0,1)\), pero eso significaría que\(\delta \leq 0\). Por lo tanto no hay una sola\(\delta > 0\).

    \(f \colon [0,1] \to {\mathbb{R}}\), definido por\(f(x) := x^2\) es uniformemente continuo.

    Prueba: Tenga en cuenta que\(0 \leq x,c \leq 1\). Entonces\[\left\lvert {x^2-c^2} \right\rvert = \left\lvert {x+c} \right\rvert\left\lvert {x-c} \right\rvert \leq (\left\lvert {x} \right\rvert+\left\lvert {c} \right\rvert) \left\lvert {x-c} \right\rvert \leq (1+1)\left\lvert {x-c} \right\rvert .\] Por lo tanto dado\(\epsilon > 0\), vamos\(\delta := \nicefrac{\epsilon}{2}\). Si\(\left\lvert {x-c} \right\rvert < \delta\), entonces\(\left\lvert {x^2-c^2} \right\rvert < \epsilon\).

    Por otro lado,\(f \colon {\mathbb{R}}\to {\mathbb{R}}\), definido por no\(f(x) := x^2\) es uniformemente continuo.

    Prueba: Supongamos que es uniformemente continuo, entonces para todos\(\epsilon > 0\), existiría\(\delta > 0\) tal que si\(\left\lvert {x-c} \right\rvert < \delta\), entonces\(\left\lvert {x^2 -c^2} \right\rvert < \epsilon\). Toma\(x > 0\) y deja\(c := x+\nicefrac{\delta}{2}\). Escribe\[\epsilon > \left\lvert {x^2-c^2} \right\rvert = \left\lvert {x+c} \right\rvert\left\lvert {x-c} \right\rvert = (2x+\nicefrac{\delta}{2})\nicefrac{\delta}{2} \geq \delta x .\] Por lo tanto\(x < \nicefrac{\epsilon}{\delta}\) para todos\(x > 0\), lo cual es una contradicción.

    Hemos visto que si\(f\) se define en un intervalo que o bien no está cerrado o no acotado, entonces\(f\) puede ser continuo, pero no uniformemente continuo. Para un intervalo cerrado y acotado\([a,b]\), podemos, sin embargo, hacer la siguiente declaración.

    [unifcont:thm] Dejar\(f \colon [a,b] \to {\mathbb{R}}\) ser una función continua. Entonces\(f\) es uniformemente continuo.

    Demostramos la afirmación por contrapositivo. Supongamos que no\(f\) es uniformemente continuo. Vamos a demostrar que hay algunos\(c \in [a,b]\) donde no\(f\) es continuo. Negemos la definición de uniformemente continuo. Existe\(\epsilon > 0\) tal que para cada\(\delta > 0\), existen puntos\(x, y\) en\(S\) con\(\left\lvert {x-y} \right\rvert < \delta\) y\(\left\lvert {f(x)-f(y)} \right\rvert \geq \epsilon\).

    Entonces por lo\(\epsilon > 0\) anterior, encontramos secuencias\(\{ x_n \}\) y\(\{ y_n \}\) tal que\(\left\lvert {x_n-y_n} \right\rvert < \nicefrac{1}{n}\) y tal que\(\left\lvert {f(x_n)-f(y_n)} \right\rvert \geq \epsilon\). Por, existe una subsecuencia convergente\(\{ x_{n_k} \}\). Vamos\(c := \lim\, x_{n_k}\). Como\(a \leq x_{n_k} \leq b\), entonces\(a \leq c \leq b\). Escribe\[\left\lvert {y_{n_k} - c} \right\rvert = \left\lvert {y_{n_k} - x_{n_k} + x_{n_k} - c} \right\rvert \leq \left\lvert {y_{n_k} - x_{n_k}} \right\rvert + \left\lvert {x_{n_k}-c} \right\rvert < \nicefrac{1}{n_k} + \left\lvert {x_{n_k}-c} \right\rvert .\] As\(\nicefrac{1}{n_k}\) y\(\left\lvert {x_{n_k}-c} \right\rvert\) ambos van a cero cuando\(k\) va al infinito,\(\{ y_{n_k} \}\) converge y el límite es\(c\). Ahora demostramos que no\(f\) es continuo en\(c\). Estimamos\[\begin{split} \left\lvert {f(x_{n_k}) - f(c)} \right\rvert & = \left\lvert {f(x_{n_k}) - f(y_{n_k}) + f(y_{n_k}) - f(c)} \right\rvert \\ & \geq \left\lvert {f(x_{n_k}) - f(y_{n_k})} \right\rvert - \left\lvert {f(y_{n_k}) - f(c)} \right\rvert \\ & \geq \epsilon - \left\lvert {f(y_{n_k})-f(c)} \right\rvert . \end{split}\] O en otras palabras\[\left\lvert {f(x_{n_k})-f(c)} \right\rvert + \left\lvert {f(y_{n_k})-f(c)} \right\rvert \geq \epsilon .\] Al menos una de las secuencias\(\{ f(x_{n_k}) \}\) o\(\{ f(y_{n_k}) \}\) no puede converger a\(f(c)\), de lo contrario el lado izquierdo de la desigualdad iría a cero mientras que el lado derecho es positivo. Por lo tanto,\(f\) no puede ser continuo en\(c\).

    Extensión continua

    Antes de llegar a la extensión continua, mostramos el siguiente lema útil. Dice que las funciones uniformemente continuas se comportan muy bien con respecto a las secuencias de Cauchy. El nuevo problema aquí es que para una secuencia de Cauchy ya no sabemos dónde termina el límite; puede que no termine en el dominio de la función.

    [unifcauchycauchy:lemma] Dejar\(f \colon S \to {\mathbb{R}}\) ser una función uniformemente continua. Deja\(\{ x_n \}\) ser una secuencia de Cauchy en\(S\). Entonces\(\{ f(x_n) \}\) está Cauchy.

    Dejemos\(\epsilon > 0\) que se den. Entonces hay\(\delta > 0\) tal que\(\left\lvert {f(x)-f(y)} \right\rvert < \epsilon\) siempre\(\left\lvert {x-y} \right\rvert < \delta\). Ahora encuentra un\(M \in {\mathbb{N}}\) tal que por todo\(n, k \geq M\) lo que tenemos\(\left\lvert {x_n-x_k} \right\rvert < \delta\). Entonces por todo\(n, k \geq M\) lo que tenemos\(\left\lvert {f(x_n)-f(x_k)} \right\rvert < \epsilon\).

    Una aplicación del lema anterior es el siguiente teorema. Dice que una función en un intervalo abierto es uniformemente continua si y sólo si se puede extender a una función continua en el intervalo cerrado.

    [context:thm] Una función\(f \colon (a,b) \to {\mathbb{R}}\) es uniformemente continua si y solo si\[L_a := \lim_{x \to a} f(x) \qquad \text{and} \qquad L_b := \lim_{x \to b} f(x)\] existen los límites y la función\(\widetilde{f} \colon [a,b] \to {\mathbb{R}}\) definida por\[\widetilde{f}(x) := \begin{cases} f(x) & \text{ if $x \in (a,b)$,} \\ L_a & \text{ if $x = a$,} \\ L_b & \text{ if $x = b$,} \end{cases}\] es continua.

    Una dirección no es difícil de probar. Si\(\widetilde{f}\) es continuo, entonces es uniformemente continuo por. Como\(f\) es la restricción de\(\widetilde{f}\) a\((a,b)\), entonces también\(f\) es uniformemente continuo (ejercicio fácil).

    Ahora supongamos que\(f\) es uniformemente continuo. Primero debemos demostrar que los límites\(L_a\) y\(L_b\) existen. Concentrémonos en\(L_a\). Toma una secuencia\(\{ x_n \}\) en\((a,b)\) tal que\(\lim\, x_n = a\). La secuencia es una secuencia de Cauchy y por lo tanto, por, la secuencia\(\{ f(x_n) \}\) es Cauchy y por lo tanto convergente. Tenemos algún número\(L_1 := \lim\, f(x_n)\). Toma otra secuencia\(\{ y_n \}\) en\((a,b)\) tal que\(\lim\, y_n = a\). Por el mismo razonamiento que obtenemos\(L_2 := \lim\, f(y_n)\). Si lo demostramos\(L_1 = L_2\), entonces\(L_a = \lim_{x\to a} f(x)\) existe el límite. \(\epsilon > 0\)Déjese dar, encontrar\(\delta > 0\) tal que\(\left\lvert {x-y} \right\rvert < \delta\) implique\(\left\lvert {f(x)-f(y)} \right\rvert < \nicefrac{\epsilon}{3}\). Encontrar\(M \in {\mathbb{N}}\) tal que para\(n \geq M\) nosotros tenemos\(\left\lvert {a-x_n} \right\rvert < \nicefrac{\delta}{2}\),\(\left\lvert {a-y_n} \right\rvert < \nicefrac{\delta}{2}\),\(\left\lvert {f(x_n)-L_1} \right\rvert < \nicefrac{\epsilon}{3}\), y\(\left\lvert {f(y_n)-L_2} \right\rvert < \nicefrac{\epsilon}{3}\). Entonces porque\(n \geq M\) tenemos\[\left\lvert {x_n-y_n} \right\rvert = \left\lvert {x_n-a+a-y_n} \right\rvert \leq \left\lvert {x_n-a} \right\rvert+\left\lvert {a-y_n} \right\rvert < \nicefrac{\delta}{2} + \nicefrac{\delta}{2} = \delta.\] Así\[\begin{split} \left\lvert {L_1-L_2} \right\rvert &= \left\lvert {L_1-f(x_n)+f(x_n)-f(y_n)+f(y_n)-L_2} \right\rvert \\ & \leq \left\lvert {L_1-f(x_n)} \right\rvert+\left\lvert {f(x_n)-f(y_n)} \right\rvert+\left\lvert {f(y_n)-L_2} \right\rvert \\ & \leq \nicefrac{\epsilon}{3} + \nicefrac{\epsilon}{3} + \nicefrac{\epsilon}{3} = \epsilon . \end{split}\] Por lo tanto\(L_1 = L_2\). Así\(L_a\) existe. Demostrar que\(L_b\) existe se deja como ejercicio.

    Ahora que sabemos que los límites\(L_a\) y\(L_b\) existen, estamos hechos. Si\(\lim_{x\to a} f(x)\) existe, entonces\(\lim_{x\to a} \widetilde{f}(x)\) existe (Ver). De igual manera con\(L_b\). De ahí\(\widetilde{f}\) que sea continuo en\(a\) y\(b\). Y ya que\(f\) es continuo en\(c \in (a,b)\), entonces\(\widetilde{f}\) es continuo en\(c \in (a,b)\).

    Funciones continuas de Lipschitz

    \(f \colon S \to {\mathbb{R}}\)Sea una función tal que exista un número\(K\) tal que para todos\(x\) y\(y\) en\(S\) tenemos\[\left\lvert {f(x)-f(y)} \right\rvert \leq K \left\lvert {x-y} \right\rvert .\] Entonces\(f\) se dice que es Lipschitz continuo 16.

    Una gran clase de funciones es Lipschitz continuo. Tenga cuidado, así como para las funciones uniformemente continuas, el dominio de definición de la función es importante. Consulta los ejemplos a continuación y los ejercicios. Primero justificamos el uso de la palabra continuo.

    Una función continua de Lipschitz es uniformemente continua.

    Que\(f \colon S \to {\mathbb{R}}\) sea una función y dejemos\(K\) ser una constante tal que para todos\(x, y\) en\(S\) tenemos\(\left\lvert {f(x)-f(y)} \right\rvert \leq K \left\lvert {x-y} \right\rvert\).

    Dejemos\(\epsilon > 0\) que se den. Tomar\(\delta := \nicefrac{\epsilon}{K}\). Para cualquiera\(x\) y\(y\) en\(S\) tal que\(\left\lvert {x-y} \right\rvert < \delta\) tenemos que\[\left\lvert {f(x)-f(y)} \right\rvert \leq K \left\lvert {x-y} \right\rvert < K \delta = K \frac{\epsilon}{K} = \epsilon .\] Por lo tanto\(f\) es uniformemente continuo.

    Interpretamos geométricamente la continuidad de Lipschitz. Si\(f\) es una función continua de Lipschitz con alguna constante\(K\). Reescribimos la desigualdad para decir que para\(x \not=y\) tenemos\[\left\lvert {\frac{f(x)-f(y)}{x-y}} \right\rvert \leq K .\] La cantidad\(\frac{f(x)-f(y)}{x-y}\) es la pendiente de la línea entre los puntos\(\bigl(x,f(x)\bigr)\) y\(\bigl(y,f(y)\bigr)\), es decir, una línea secante. Por lo tanto,\(f\) es Lipschitz continuo si y solo si cada línea que interseca la gráfica de\(f\) en al menos dos puntos distintos tiene pendiente menor o igual a\(K\). Ver.

    Las funciones\(\sin(x)\) y\(\cos(x)\) son Lipschitz continuas. Hemos visto () las siguientes dos desigualdades. \[\left\lvert {\sin(x)-\sin(y)} \right\rvert \leq \left\lvert {x-y} \right\rvert \qquad \text{and} \qquad \left\lvert {\cos(x)-\cos(y)} \right\rvert \leq \left\lvert {x-y} \right\rvert .\]

    De ahí que el pecado y los cos sean Lipschitz continuos con\(K=1\).

    La función\(f \colon [1,\infty) \to {\mathbb{R}}\) definida por\(f(x) := \sqrt{x}\) es Lipschitz continua. Prueba:\[\left\lvert {\sqrt{x}-\sqrt{y}} \right\rvert = \left\lvert {\frac{x-y}{\sqrt{x}+\sqrt{y}}} \right\rvert = \frac{\left\lvert {x-y} \right\rvert}{\sqrt{x}+\sqrt{y}} .\] Como\(x \geq 1\) y\(y \geq 1\), vemos eso\(\frac{1}{\sqrt{x}+\sqrt{y}} \leq \frac{1}{2}\). Por lo tanto\[\left\lvert {\sqrt{x}-\sqrt{y}} \right\rvert = \left\lvert {\frac{x-y}{\sqrt{x}+\sqrt{y}}} \right\rvert \leq \frac{1}{2} \left\lvert {x-y} \right\rvert.\]

    Por otro lado\(f \colon [0,\infty) \to {\mathbb{R}}\) definido por no\(f(x) := \sqrt{x}\) es Lipschitz continuo. Veamos por qué: Supongamos que tenemos\[\left\lvert {\sqrt{x}-\sqrt{y}} \right\rvert \leq K \left\lvert {x-y} \right\rvert ,\] para algunos\(K\). Dejar\(y=0\) obtener\(\sqrt{x} \leq K x\). Si\(K > 0\), entonces para\(x > 0\) nosotros entonces obtenemos\(\nicefrac{1}{K} \leq \sqrt{x}\). Esto no puede ser cierto para todos\(x > 0\). Así no\(K > 0\) existe tal y no\(f\) es Lipschitz continuo.

    El último ejemplo es una función que es uniformemente continua pero no continua de Lipschitz. Para ver que\(\sqrt{x}\) es uniformemente continuo en\([0,\infty)\) nota que es uniformemente continuo en\([0,1]\) por. También es Lipschitz (y por lo tanto uniformemente continuo) en\([1,\infty)\). No es difícil (ejercicio) demostrar que esto significa que\(\sqrt{x}\) es uniformemente continuo en\([0,\infty)\).

    Ejercicios

    Dejar\(f \colon S \to {\mathbb{R}}\) ser uniformemente continuo. Vamos\(A \subset S\). Entonces la restricción\(f|_A\) es uniformemente continua.

    Dejar\(f \colon (a,b) \to {\mathbb{R}}\) ser una función uniformemente continua. Terminar la prueba de demostrando que el límite\(\lim\limits_{x \to b} f(x)\) existe.

    Demostrar que\(f \colon (c,\infty) \to {\mathbb{R}}\) para algunos\(c > 0\) y definido por\(f(x) := \nicefrac{1}{x}\) es Lipschitz continuo.

    Mostrar que\(f \colon (0,\infty) \to {\mathbb{R}}\) definido por no\(f(x) := \nicefrac{1}{x}\) es Lipschitz continuo.

    Dejar\(A, B\) ser intervalos. Dejar\(f \colon A \to {\mathbb{R}}\) y\(g \colon B \to {\mathbb{R}}\) ser uniformemente continuas funciones tales que\(f(x) = g(x)\) para\(x \in A \cap B\). Definir la función\(h \colon A \cup B \to {\mathbb{R}}\) por\(h(x) := f(x)\) si\(x \in A\) y\(h(x) := g(x)\) si\(x \in B \setminus A\). a) Demostrar que si\(A \cap B \not= \emptyset\), entonces\(h\) es uniformemente continuo. b) Encuentra un ejemplo donde\(A \cap B = \emptyset\) y ni siquiera\(h\) es continuo.

    Dejar\(f \colon {\mathbb{R}}\to {\mathbb{R}}\) ser un polinomio de grado\(d \geq 2\). Demostrar que no\(f\) es Lipschitz continuo.

    Dejar\(f \colon (0,1) \to {\mathbb{R}}\) ser una función continua acotada. Demostrar que la función\(g(x) := x(1-x)f(x)\) es uniformemente continua.

    Mostrar que\(f \colon (0,\infty) \to {\mathbb{R}}\) definido por no\(f(x) := \sin (\nicefrac{1}{x})\) es uniformemente continuo.

    Dejar\(f \colon {\mathbb{Q}}\to {\mathbb{R}}\) ser una función uniformemente continua. Demostrar que existe una función uniformemente continua\(\widetilde{f} \colon {\mathbb{R}}\to {\mathbb{R}}\) tal que\(f(x) = \widetilde{f}(x)\) para todos\(x \in {\mathbb{Q}}\).

    a) Encontrar un continuo\(f \colon (0,1) \to {\mathbb{R}}\) y una secuencia\(\{ x_n \}\) en\((0,1)\) eso es Cauchy, pero tal que no\(\{ f(x_n) \}\) es Cauchy. b) Demostrar que si\(f \colon {\mathbb{R}}\to {\mathbb{R}}\) es continuo, y\(\{ x_n \}\) es Cauchy, entonces\(\{ f(x_n) \}\) es Cauchy.

    a) Si\(f \colon S \to {\mathbb{R}}\) y\(g \colon S \to {\mathbb{R}}\) son uniformemente continuos, entonces mostrar que\(h \colon S \to {\mathbb{R}}\) dado por\(h(x) := f(x) + g(x)\) es uniformemente continuo.
    b) Si\(f \colon S \to {\mathbb{R}}\) es uniformemente continuo y\(a \in {\mathbb{R}}\), luego mostrar que\(h \colon S \to {\mathbb{R}}\) dado por\(h(x) := a f(x)\) es uniformemente continuo.

    a) Si\(f \colon S \to {\mathbb{R}}\) y\(g \colon S \to {\mathbb{R}}\) son Lipschitz, entonces mostrar que\(h \colon S \to {\mathbb{R}}\) dado por\(h(x) := f(x) + g(x)\) es Lipschitz.
    b) Si\(f \colon S \to {\mathbb{R}}\) es Lipschitz y\(a \in {\mathbb{R}}\), luego mostrar que\(h \colon S \to {\mathbb{R}}\) dado por\(h(x) := a f(x)\) es Lipschitz.

    a) Si\(f \colon [0,1] \to {\mathbb{R}}\) se da por\(f(x) := x^m\) para un entero\(m \geq 0\), show\(f\) es Lipschitz y encontrar la mejor (la más pequeña) constante de Lipschitz\(K\) (dependiendo\(m\) de por supuesto). Pista:\((x-y)(x^{m-1} + x^{m-2}y + x^{m-3}y^2 + \cdots + x y^{m-2} + y^{m-1}) = x^m - y^m\).
    b) Utilizando el ejercicio anterior, demostrar que si\(f \colon [0,1] \to {\mathbb{R}}\) es un polinomio, es decir\(f(x) := a_m x^m + a_{m-1} x^{m-1} + \cdots + a_0\), entonces\(f\) es Lipschitz.

    Supongamos\(f \colon [0,1] \to {\mathbb{R}}\) que para nosotros tenemos\(\left\lvert {f(x)-f(y)} \right\rvert \leq K \left\lvert {x-y} \right\rvert\), y\(f(0) = f(1) = 0\). \(\left\lvert {f(x)} \right\rvert \leq \nicefrac{K}{2}\)Demuéstralo. Mostrar además con el ejemplo que\(\nicefrac{K}{2}\) es lo mejor posible, es decir, existe una función tan continua para la cual\(\left\lvert {f(x)} \right\rvert = \nicefrac{K}{2}\) para algunos\(x \in [0,1]\).

    Límites al infinito

    Nota: menos de 1 conferencia (opcional, se puede omitir de forma segura a menos que o también esté cubierta)

    Límites al infinito

    En cuanto a las secuencias, una variable continua también puede acercarse al infinito. Hagamos precisa esta noción.

    Decimos que\(\infty\) es un punto de cúmulo de\(S \subset {\mathbb{R}}\)\(M \in {\mathbb{R}}\), si por cada, existe un\(x \in S\) tal que\(x \geq M\). De igual manera\(- \infty\) es un punto de cúmulo de\(S \subset {\mathbb{R}}\)\(M \in {\mathbb{R}}\), si por cada, existe un\(x \in S\) tal que\(x \leq M\).

    Dejar\(f \colon S \to {\mathbb{R}}\) ser una función, donde\(\infty\) es un punto de clúster de\(S\). Si existe\(L \in {\mathbb{R}}\) tal que para cada uno\(\epsilon > 0\), hay\(M \in {\mathbb{R}}\) tal que\[\left\lvert {f(x) - L} \right\rvert < \epsilon\] siempre\(x \geq M\), entonces decimos\(f(x)\) converge a\(L\) lo que\(x\) va a\(\infty\). Llamamos\(L\) al límite y escribimos\[\lim_{x \to \infty} f(x) := L .\] Alternativamente escribimos\(f(x) \to L\) como\(x \to \infty\).

    De igual manera, si\(-\infty\) es un punto de cúmulo de\(S\) y existe un\(L \in {\mathbb{R}}\) tal que para cada uno\(\epsilon > 0\), hay\(M \in {\mathbb{R}}\) tal que\[\left\lvert {f(x) - L} \right\rvert < \epsilon\] siempre\(x \leq M\), entonces decimos\(f(x)\) converge a\(L\) como\(x\) va a\(-\infty\). Llamamos\(L\) al límite y escribimos\[\lim_{x \to -\infty} f(x) := L .\] Alternativamente escribimos\(f(x) \to L\) como\(x \to -\infty\).

    Volvimos a hacer trampa un poco y dijimos el límite. Lo dejamos como un ejercicio para que el lector pruebe la siguiente proposición.

    [liminfty:unique] El límite en\(\infty\) o\(-\infty\) como se definió anteriormente es único si existe.

    Vamos\(f(x) := \frac{1}{\left\lvert {x} \right\rvert+1}\). Entonces\[\lim_{x\to \infty} f(x) = 0 \qquad \text{and} \qquad \lim_{x\to -\infty} f(x) = 0 .\]

    Comprobante: Dejar\(\epsilon > 0\) ser dado. Encuentra lo suficientemente\(M > 0\) grande para que\(\frac{1}{M+1} < \epsilon\). Si\(x \geq M\), entonces\(\frac{1}{x+1} \leq \frac{1}{M+1} < \epsilon\). Ya que\(\frac{1}{\left\lvert {x} \right\rvert+1} > 0\) para todos\(x\) se prueba el primer límite. El comprobante para\(-\infty\) se deja al lector.

    Vamos\(f(x) := \sin(\pi x)\). Entonces\(\lim_{x\to\infty} f(x)\) no existe. Para probar este hecho tenga en cuenta que si\(x = 2n+\nicefrac{1}{2}\) para algunos\(n \in {\mathbb{N}}\) entonces\(f(x)=1\), mientras que si\(x = 2n+\nicefrac{3}{2}\) entonces\(f(x)=-1\), así no pueden estar ambos dentro\(\epsilon\) de un pequeño número real.

    Debemos tener cuidado de no confundir límites continuos con límites de secuencias. Porque\(f(x) = \sin(\pi x)\) podríamos decir Por\[\lim_{n \to \infty} f(n) = 0, \qquad \text{but} \qquad \lim_{x \to \infty} f(x) ~ \text{does not exist}.\] supuesto que la notación es ambigua. Simplemente estamos usando la convención que\(n \in {\mathbb{N}}\), mientras\(x \in {\mathbb{R}}\). Cuando la notación no está clara, es bueno mencionar explícitamente dónde vive la variable, o qué tipo de límite estás usando.

    Hay una conexión de límites continuos a límites de secuencias, pero debemos tomar todas las secuencias yendo al infinito, igual que antes en.

    [seqflimitinf:lemma] Supongamos que\(f \colon S \to {\mathbb{R}}\) es una función,\(\infty\) es un punto de clúster de\(S \subset {\mathbb{R}}\), y\(L \in {\mathbb{R}}\). Entonces\[\lim_{x\to\infty} f(x) = L % \qquad \text{if and only if} \qquad\] si y sólo si\[\lim_{n\to\infty} f(x_n) = L% ~~\text{for all sequences $\{ x_n \}$ such that $\lim\, x_n = \infty$} .\] para todas las secuencias\(\{ x_n \}\) tales que\(\lim\limits_{n\to\infty} x_n = \infty\).

    El lema se sostiene para el límite como\(x \to -\infty\). Su prueba es casi idéntica y se deja como ejercicio.

    Primero supongamos\(f(x) \to L\) como\(x \to \infty\). Dado un\(\epsilon > 0\), existe\(M\) tal que por todo\(x \geq M\) lo que tenemos\(\left\lvert {f(x)-L} \right\rvert < \epsilon\). Que\(\{ x_n \}\) sea una secuencia en\(S\) tal que\(\lim \, x_n = \infty\). Entonces existe\(N\) tal que por todo\(n \geq N\) lo que tenemos\(x_n \geq M\). Y así\(\left\lvert {f(x_n)-L} \right\rvert < \epsilon\).

    Demostramos lo contrario por contrapositivo. Supongamos que\(f(x)\) no va a\(L\) como\(x \to \infty\). Esto quiere decir que existe un\(\epsilon > 0\), tal que para cada\(M \in {\mathbb{N}}\), existe un\(x \in S\),\(x \geq M\), llamémoslo\(x_M\), tal que\(\left\lvert {f(x_M)-L} \right\rvert \geq \epsilon\). Considera la secuencia\(\{ x_n \}\). Claramente\(\{ f(x_n) \}\) no converge a\(L\). Queda por señalar que\(\lim\, x_n = \infty\), porque\(x_n \geq n\) para todos\(n\).

    Usando el lema, nuevamente traducimos resultados sobre límites secuenciales en resultados sobre límites continuos como\(x\) va al infinito. Es decir, tenemos análogos casi inmediatos de los corolarios en. Simplemente permitimos que el punto\(c\) de clúster sea\(\infty\) o\(-\infty\), además de un número real. Dejamos al alumno verificar estas declaraciones.

    Límite infinito

    Al igual que para las secuencias, a menudo es conveniente distinguir ciertas secuencias divergentes, y hablar de que los límites son infinitos casi como si los límites existieran.

    Dejar\(f \colon S \to {\mathbb{R}}\) ser una función y supongamos que\(S\) tiene\(\infty\) como punto de clúster. Decimos\(f(x)\) diverge al infinito como\(x\) va a\(\infty\), si por cada\(N \in {\mathbb{R}}\) existe un\(M \in {\mathbb{R}}\) tal que\[f(x) > N\] siempre\(x \in S\) y cuando\(x \geq M\). Escribimos\[\lim_{x \to \infty} f(x) := \infty ,\] o decimos eso\(f(x) \to \infty\) como\(x \to \infty\).

    Se puede hacer una definición similar para los límites como\(x \to -\infty\) o como\(x \to c\) para un finito\(c\). También se pueden hacer definiciones similares para los límites siendo\(-\infty\). Declarar estas definiciones se deja como un ejercicio. Tenga en cuenta que a veces converge al infinito se utiliza. Podemos volver a usar límites secuenciales, y un análogo de se deja como ejercicio.

    Demostremos eso\(\lim_{x \to \infty} \frac{1+x^2}{1+x} = \infty\).

    Comprobante: Porque\(x \geq 1\) hemos\[\frac{1+x^2}{1+x} \geq \frac{x^2}{x+x} = \frac{x}{2} .\] dado\(N \in {\mathbb{R}}\), tomar\(M = \max \{ 2N+1 , 1 \}\). Si\(x \geq M\), entonces\(x \geq 1\) y\(\nicefrac{x}{2} > N\). Entonces\[\frac{1+x^2}{1+x} \geq \frac{x}{2} > N .\]

    Composiciones

    Finalmente, al igual que para los límites en números finitos podemos componer funciones fácilmente.

    [prop:inflimcompositions] Supongamos que\(f \colon A \to B\)\(g \colon B \to {\mathbb{R}}\)\(A, B \subset {\mathbb{R}}\),,\(a \in {\mathbb{R}}\cup \{ -\infty, \infty\}\) es un punto de clúster de\(A\), y\(b \in {\mathbb{R}}\cup \{ -\infty, \infty\}\) es un punto de clúster de\(B\). Supongamos\[\lim_{x \to a} f(x) = b\qquad \text{and} \qquad \lim_{y \to b} g(y) = c\] para algunos\(c \in {\mathbb{R}}\cup \{ -\infty, \infty \}\). Si\(b \in B\), entonces supongamos\(g(b) = c\). Entonces\[\lim_{x \to a} g\bigl(f(x)\bigr) = c .\]

    La prueba es directa, y se deja como ejercicio. Ya conocemos la proposición cuándo\(a, b, c \in {\mathbb{R}}\), ver Ejercicios [ejercicio:contlimitcomposición] y [ejercicio:contlimitbadcomposición]. Nuevamente el requisito de que\(g\) sea continuo en\(b\), si\(b \in B\), es necesario.

    Vamos\(h(x) := e^{-x^2+x}\). Entonces\[\lim_{x\to \infty} h(x) = 0 .\]

    Prueba: El reclamo sigue una vez que conocemos\[\lim_{x\to \infty} -x^2+x = -\infty\] y\[\lim_{y\to -\infty} e^y = 0 ,\] que generalmente se prueba cuando se define la función exponencial.

    Ejercicios

    Demostrar.

    Dejar\(f \colon [1,\infty) \to {\mathbb{R}}\) ser una función. Definir\(g \colon (0,1] \to {\mathbb{R}}\) vía\(g(x) := f(\nicefrac{1}{x})\). Utilizando las definiciones de límites directamente, mostrar que\(\lim_{x\to 0^+} g(x)\) existe si y sólo si\(\lim_{x\to \infty} f(x)\) existe, en cuyo caso son iguales.

    Demostrar.

    Justificemos la terminología. Dejar\(f \colon {\mathbb{R}}\to {\mathbb{R}}\) ser una función tal que\(\lim_{x \to \infty} f(x) = \infty\) (diverge al infinito). Mostrar que\(f(x)\) diverge (es decir, no converge) como\(x \to \infty\).

    Llegar con las definiciones de límites de\(f(x)\) ir a\(-\infty\) como\(x \to \infty\),\(x \to -\infty\), y como\(x \to c\) para un finito\(c \in {\mathbb{R}}\). Entonces exponer las definiciones para los límites de\(f(x)\) ir a\(\infty\) como\(x \to -\infty\), y como\(x \to c\) para un finito\(c \in {\mathbb{R}}\).

    Supongamos que\(P(x) := x^n + a_{n-1} x^{n-1} + \cdots + a_1 x + a_0\) es un polinomio monico de grado\(n \geq 1\) (monico significa que el coeficiente de\(x^n\) es 1). a) Mostrar que si\(n\) es par entonces\(\lim_{x\to\infty} P(x) = \lim_{x\to-\infty} P(x) = \infty\). b) Mostrar que si\(n\) es impar entonces\(\lim_{x\to\infty} P(x) = \infty\) y\(\lim_{x\to-\infty} P(x) = -\infty\) (ver ejercicio anterior).

    Dejar\(\{ x_n \}\) ser una secuencia. Considerar\(S := {\mathbb{N}}\subset {\mathbb{R}}\), y\(f \colon S \to {\mathbb{R}}\) definido por\(f(n) := x_n\). Demostrar que las dos nociones de límite,\[\lim_{n\to\infty} x_n \qquad \text{and} \qquad \lim_{x\to\infty} f(x)\] son equivalentes. Es decir, mostrar que si uno existe también lo hace el otro, y en este caso son iguales.

    Extender de la siguiente manera. Supongamos que\(S \subset {\mathbb{R}}\) tiene un punto de clúster\(c \in {\mathbb{R}}\)\(c = \infty\),, o\(c = -\infty\). Dejar\(f \colon S \to {\mathbb{R}}\) ser una función y dejar\(L = \infty\) o\(L = -\infty\). Demostrar que\[\lim_{x\to c} f(x) = L \qquad \text{if and only if} \qquad \lim_{n\to\infty} f(x_n) = L ~~\text{for all sequences $\{ x_n \}$ such that $\lim\, x_n = c$} .\]

    Funciones y continuidad monótona

    Nota: 1 conferencia (opcional, se puede omitir de forma segura a menos que también esté cubierta, requiera)

    Vamos\(S \subset {\mathbb{R}}\). Decimos que\(f \colon S \to {\mathbb{R}}\) está aumentando (resp. estrictamente creciente) si\(x,y \in S\) con\(x < y\) implica\(f(x) \leq f(y)\) (resp. \(f(x) < f(y)\)). Definimos decreciente y estrictamente decreciente de la misma manera cambiando las desigualdades por\(f\).

    Si una función está aumentando o disminuyendo decimos que es monótona. Si es estrictamente creciente o estrictamente decreciente decimos que es estrictamente monótona.

    A veces no decreciente (resp. no creciente) se utiliza para aumentar (resp. decreciente) la función para enfatizar que no es estrictamente creciente (resp. estrictamente decreciente).

    Continuidad de funciones monótona

    Es fácil calcular límites unilaterales para funciones monótona.

    [prop:monotlimits] Dejar\(S \subset {\mathbb{R}}\),\(c \in {\mathbb{R}}\), y\(f \colon S \to {\mathbb{R}}\) estar aumentando. Si\(c\) es un punto de clúster de\(S \cap (-\infty,c)\), entonces\[\lim_{x \to c^-} f(x) = \sup \{ f(x) : x < c, x \in S \} ,\] y si\(c\) es un punto de clúster de\(S \cap (c,\infty)\), entonces\[\lim_{x \to c^+} f(x) = \inf \{ f(x) : x > c, x \in S \} .\]

    Del mismo modo, si\(f\) es decreciente y\(c\) es un punto de clúster de\(S \cap (-\infty,c)\), entonces\[\lim_{x \to c^-} f(x) = \inf \{ f(x) : x < c, x \in S \} ,\] y si\(c\) es un punto de clúster de\(S \cap (c,\infty)\), entonces\[\lim_{x \to c^+} f(x) = \sup \{ f(x) : x > c, x \in S \} .\]

    En particular, todos los límites unilaterales existen cada vez que tienen sentido. Si a partir de ahora decimos que digamos que\(x \to c^-\) existe el límite de la mano izquierda queremos decir que\(c\) es un punto de cúmulo de\(S \cap (-\infty,c)\).

    Supongamos que\(f\) va en aumento, y vamos a mostrar la primera igualdad. El resto de la prueba es muy similar y se deja como ejercicio.

    Vamos\(a := \sup \{ f(x) : x < c, x \in S \}\). Si\(a = \infty\), entonces para cada\(M \in {\mathbb{R}}\), existe\(x_M\) tal que\(f(x_M) > M\). A medida\(f\) que va aumentando,\(f(x) \geq f(x_M) > M\) para todos\(x \in S\) con\(x > x_M\). Si tomamos\(\delta = c-x_M\) obtenemos la definición del límite que va al infinito.

    Entonces asumamos\(a < \infty\). Dejemos\(\epsilon > 0\) que se den. Porque\(a\) es lo supremo, existe un\(x_\epsilon < c\),\(x_\epsilon \in S\), tal que\(f(x_\epsilon) > a-\epsilon\). A medida\(f\) que va aumentando, si\(x \in S\) y\(x_\epsilon < x < c\), tenemos\(a-\epsilon < f(x_\epsilon) \leq f(x) \leq a\). Vamos\(\delta := c-x_\epsilon\). Entonces para\(x \in S \cap (-\infty,c)\) con\(\left\lvert {x-c} \right\rvert < \delta\), tenemos\(\left\lvert {f(x)-a} \right\rvert < \epsilon\).

    Supongamos\(f \colon S \to {\mathbb{R}}\),\(c \in S\) y que ambos límites unilaterales existen. Ya\(f(x) \leq f(c) \leq f(y)\) que siempre\(x < c < y\), tomando los límites que obtenemos\[\lim_{x \to c^-} f(x) \leq f(c) \leq \lim_{x \to c^+} f(x) .\] Entonces\(f\) es continuo en\(c\) si y solo si ambos límites son iguales entre sí (y por lo tanto iguales a\(f(c)\)). Ver también. Ve para hacerte una idea de cómo se ve una discontinuidad.

    [cor:continterval] Si\(I \subset {\mathbb{R}}\) es un intervalo y\(f \colon I \to {\mathbb{R}}\) es monótona y no constante, entonces\(f(I)\) es un intervalo si y solo si\(f\) es continuo.

    Asumir que no\(f\) es constante es evitar el tecnicismo que\(f(I)\) es un punto único en ese caso;\(f(I)\) es un punto único si y sólo si\(f\) es constante. Una función constante es continua.

    Si\(f\) es continuo entonces\(f(I)\) ser un intervalo es consecuencia de. Ver también.

    Demostremos la dirección inversa por contrapositivo. Supongamos que no\(f\) es continuo en\(c \in I\), y que no\(c\) es un punto final de\(I\). Sin pérdida de generalidad supongamos que\(f\) va en aumento. Let\[a := \lim_{x \to c^-} f(x) = \sup \{ f(x) : x \in I, x < c \} , \qquad b := \lim_{x \to c^+} f(x) = \inf \{ f(x) : x \in I, x > c \} .\] As\(c\) es una discontinuidad,\(a < b\). Si\(x < c\), entonces\(f(x) \leq a\), y si\(x > c\), entonces\(f(x) \geq b\). Por lo tanto, cualquier punto en no\((a,b) \setminus \{ f(c) \}\) está en\(f(I)\). Sin embargo existe\(x_1 \in S\),\(x_1 < c\) así\(f(x_1) \leq a\), y existe\(x_2 \in S\),\(x_2 > c\) así\(f(x_2) \geq b\). Ambos\(f(x_1)\) y\(f(x_2)\) están en\(f(I)\), y así no\(f(I)\) es un intervalo. Ver.

    Cuando\(c \in I\) es un punto final, la prueba es similar y se deja como ejercicio.

    Una propiedad llamativa de las funciones monótonas es que no pueden tener demasiadas discontinuidades.

    [cor:monotcountcont] Dejar\(I \subset {\mathbb{R}}\) ser un intervalo y\(f \colon I \to {\mathbb{R}}\) ser monótono. Entonces\(f\) tiene a lo sumo contabilizadamente muchas discontinuidades.

    \(E \subset I\)Sea el conjunto de todas las discontinuidades que no son puntos finales de\(I\). Como sólo hay dos puntos finales, basta con mostrar que\(E\) es contable. Sin pérdida de generalidad, supongamos que\(f\) va en aumento. Vamos a definir una inyección\(h \colon E \to {\mathbb{Q}}\). Para cada uno\(c \in E\) los límites unilaterales de\(f\) ambos existen ya que no\(c\) es un punto final. Dejemos\[a := \lim_{x \to c^-} f(x) = \sup \{ f(x) : x \in I, x < c \} , \qquad b := \lim_{x \to c^+} f(x) = \inf \{ f(x) : x \in I, x > c \} .\] Como\(c\) es una discontinuidad, tenemos\(a < b\). Existe un número racional\(q \in (a,b)\), así que vamos\(h(c) := q\). Porque\(f\) va en aumento,\(q\) no puede corresponder a ninguna otra discontinuidad, así que después de hacer esta elección para todos\(c \in E\), tenemos que\(h\) es uno-a-uno (inyectivo). Por lo tanto,\(E\) es contable.

    [ejemplo:countdiscont] Por\(\lfloor x \rfloor\) denotar el entero más grande menor o igual a\(x\). Definir\(f \colon [0,1] \to {\mathbb{R}}\) por\[f(x) := x + \sum_{n=0}^{\lfloor 1/(1-x) \rfloor} 2^{-n} ,\] para\(x < 1\) y\(f(1) = 3\). Se deja como un ejercicio para demostrar que\(f\) es estrictamente creciente, acotado, y tiene una discontinuidad en todos los puntos\(1-\nicefrac{1}{k}\) para\(k \in {\mathbb{N}}\). En particular, hay contabilizadamente muchas discontinuidades, pero la función está delimitada y definida en un intervalo delimitado cerrado.

    Continuidad de funciones inversas

    Una función estrictamente monótona\(f\) es uno a uno (inyección). Para ver este aviso que si\(x \not= y\) entonces podemos asumir\(x < y\). Entonces ya sea\(f(x) < f(y)\) si\(f\) está aumentando estrictamente o\(f(x) > f(y)\) si\(f\) es estrictamente decreciente, entonces\(f(x) \not= f(y)\). De ahí que deba tener una inversa\(f^{-1}\) definida en su rango.

    [prop:invcont] Si\(I \subset {\mathbb{R}}\) es un intervalo y\(f \colon I \to {\mathbb{R}}\) es estrictamente monótona. Entonces la inversa\(f^{-1} \colon f(I) \to I\) es continua.

    Supongamos que\(f\) está aumentando estrictamente. La prueba es casi idéntica para una función estrictamente decreciente. Ya que\(f\) es estrictamente creciente, así es\(f^{-1}\). Es decir, si\(f(x) < f(y)\), entonces debemos tener\(x < y\) y por lo tanto\(f^{-1}\bigl(f(x)\bigr) < f^{-1}\bigl(f(y)\bigr)\).

    Tomar\(c \in f(I)\). Si no\(c\) es un punto de clúster de\(f(I)\), entonces\(f^{-1}\) es continuo\(c\) automáticamente. Así que vamos a\(c\) ser un punto de cúmulo de\(f(I)\). Supongamos que existen ambos límites unilaterales siguientes:\[\begin{aligned} x_0 & := \lim_{y \to c^-} f^{-1}(y) = \sup \{ f^{-1}(y) : y < c, y \in f(I) \} = \sup \{ x : f(x) < c, x \in I \} , \\ x_1 & := \lim_{y \to c^+} f^{-1}(y) = \inf \{ f^{-1}(y) : y > c, y \in f(I) \} = \inf \{ x : f(x) > c, x \in I \} .\end{aligned}\] Tenemos\(x_0 \leq x_1\) como\(f^{-1}\) va aumentando. Para todos\(x > x_0\) con\(x \in I\), tenemos\(f(x) \geq c\). Como\(f\) es estrictamente creciente, debemos tener\(f(x) > c\) para todos\(x > x_0\),\(x \in I\). Por lo tanto,\[\{ x : x > x_0, x \in I \} \subset \{ x : f(x) > c, x \in I \}.\] El infimum del conjunto de la mano izquierda es\(x_0\) y el infimum del conjunto de la mano derecha es\(x_1\), así obtenemos\(x_0 \geq x_1\). Entonces\(x_1 = x_0\), y\(f^{-1}\) es continuo en\(c\).

    Si uno de los límites unilaterales no existe el argumento es similar y se deja como ejercicio.

    La proposición no requiere\(f\) que sea continua. Por ejemplo, vamos\(f \colon {\mathbb{R}}\to {\mathbb{R}}\)\[f(x) := \begin{cases} x & \text{if $x < 0$}, \\ x+1 & \text{if $x \geq 0$}. \\ \end{cases}\] La función no\(f\) es continua en\(0\). La imagen de\(I = {\mathbb{R}}\) es el conjunto\((-\infty,0)\cup [1,\infty)\), no un intervalo. Entonces se\(f^{-1} \colon (-\infty,0)\cup [1,\infty) \to {\mathbb{R}}\) puede escribir como\[f^{-1}(x) = \begin{cases} x & \text{if $x < 0$}, \\ x-1 & \text{if $x \geq 1$}. \end{cases}\] No es difícil ver que\(f^{-1}\) es una función continua.

    Observe lo que sucede con la proposición si\(f(I)\) es un intervalo. En ese caso podríamos simplemente aplicar a ambos\(f\) y\(f^{-1}\). Es decir, si\(f \colon I \to J\) es una función estrictamente monótona y\(I\) y\(J\) son intervalos, entonces ambos\(f\) y\(f^{-1}\) son continuos. Además\(f(I)\) es un intervalo precisamente cuando\(f\) es continuo.

    Ejercicios

    Supongamos\(f \colon [0,1] \to {\mathbb{R}}\) es monótona Demostrar\(f\) está acotado.

    Terminar la prueba de.

    Terminar la prueba de.

    Demostrar las reclamaciones en.

    Terminar la prueba de.

    Supongamos\(S \subset {\mathbb{R}}\), y\(f \colon S \to {\mathbb{R}}\) es una función creciente. a) Si\(c\) es un punto de clúster de\(S \cap (c,\infty)\) mostrar eso\(\lim\limits_{x\to c^+} f(x) < \infty\). b) Si\(c\) es un punto de clúster de\(S \cap (-\infty,c)\) y\(\lim\limits_{x\to c^-} f(x) = \infty\), probarlo\(S \subset (-\infty,c)\).

    Supongamos que\(I \subset {\mathbb{R}}\)\(f \colon I \to {\mathbb{R}}\) es un intervalo y es una función tal que para cada uno\(c \in I\), existen\(a, b \in {\mathbb{R}}\) con\(a > 0\) tal que\(f(x) \geq a x + b\) para todos\(x \in I\) y\(f(c) = a c + b\). Demostrar que\(f\) está aumentando estrictamente.

    Supongamos que\(f \colon I \to J\) es una función continua, biyectiva (uno a uno y sobre) durante dos intervalos\(I\) y\(J\). Demostrar que\(f\) es estrictamente monótona.

    Considera una función monótona\(f \colon I \to {\mathbb{R}}\) en un intervalo\(I\). Demostrar que existe una función\(g \colon I \to {\mathbb{R}}\) tal que\(\lim\limits_{x \to c^-} g(x) = g(c)\) para todos\(c \in I\), excepto el punto final más pequeño (izquierdo) de\(I\), y tal que\(g(x) = f(x)\) para todos pero contablemente muchos\(x\).

    a) Dejar\(S \subset {\mathbb{R}}\) ser cualquier subconjunto. Si\(f \colon S \to {\mathbb{R}}\) está aumentando, entonces mostrar que existe un creciente\(F \colon {\mathbb{R}}\to {\mathbb{R}}\) tal que\(f(x) = F(x)\) para\(x \in S\) todos. b) Encuentra un ejemplo de un estrictamente creciente\(f \colon S \to {\mathbb{R}}\) tal que un incremento\(F\) como el anterior nunca es estrictamente creciente.

    [Exercise:IncrementingFuncDiscatQ] Encuentra un ejemplo de una función creciente\(f \colon [0,1] \to {\mathbb{R}}\) que tiene una discontinuidad en cada número racional. Después demuestre que la imagen no\(f([0,1])\) contiene ningún intervalo. Pista: Enumera los números racionales y define la función con una serie.

    La Derivada

    El derivado

    Nota: 1 conferencia

    La idea de un derivado es la siguiente. Supongamos que una gráfica de una función se ve localmente como una línea recta. Entonces podemos hablar de la pendiente de esta línea. La pendiente nos indica la velocidad a la que cambia el valor de la función en el punto particular. Por supuesto, estamos dejando fuera cualquier función que tenga esquinas o discontinuidades. Seamos precisos.

    Definición y propiedades básicas

    Dejar\(I\) ser un intervalo, dejar\(f \colon I \to {\mathbb{R}}\) ser una función, y dejar\(c \in I\). Si el límite\[L := \lim_{x \to c} \frac{f(x)-f(c)}{x-c}\] existe, entonces decimos que\(f\) es diferenciable en\(c\), que\(L\) es la derivada de\(f\) at\(c\), y escribir\(f'(c) := L\).

    Si\(f\) es diferenciable en absoluto\(c \in I\), entonces simplemente decimos que\(f\) es diferenciable, y luego obtenemos una función\(f' \colon I \to {\mathbb{R}}\).

    La expresión\(\frac{f(x)-f(c)}{x-c}\) se llama cociente de diferencia.

    La interpretación gráfica de la derivada se representa en. La gráfica de la izquierda da la línea a través\(\bigl(c,f(c)\bigr)\) y\(\bigl(x,f(x)\bigr)\) con pendiente\(\frac{f(x)-f(c)}{x-c}\), es decir, la llamada línea secante. Cuando tomamos el límite como\(x\) va a\(c\), obtenemos la gráfica de la derecha, donde vemos que la derivada de la función en el punto\(c\) es la pendiente de la línea tangente a la gráfica de\(f\) en el punto\(\bigl(c,f(c)\bigr)\).

    Permitimos\(I\) ser un intervalo cerrado y permitimos\(c\) ser un punto final de\(I\). Algunos libros de cálculo no permiten\(c\) ser un punto final de un intervalo, pero toda la teoría sigue funcionando al permitirlo, y facilita nuestro trabajo.

    Dejemos\(f(x) := x^2\) definido en toda la línea real. Encontramos que\[\lim_{x\to c} \frac{x^2-c^2}{x-c} = \lim_{x\to c} \frac{(x+c)(x-c)}{x-c} = \lim_{x\to c} (x+c) = 2c.\] Por lo tanto\(f'(c) = 2c\).

    La función no\(f(x) := \left\lvert {x} \right\rvert\) es diferenciable en el origen. Cuando\(x > 0\), entonces\[\frac{\left\lvert {x} \right\rvert-\left\lvert {0} \right\rvert}{x-0} = \frac{x-0}{x-0} = 1 ,\] y cuando\(x < 0\) tenemos\[\frac{\left\lvert {x} \right\rvert-\left\lvert {0} \right\rvert}{x-0} = \frac{-x-0}{x-0} = -1 .\]

    Un famoso ejemplo de Weierstrass muestra que existe una función continua que no es diferenciable en ningún momento. La construcción de esta función está más allá del alcance de este libro. Por otro lado, una función diferenciable es siempre continua.

    Dejar\(f \colon I \to {\mathbb{R}}\) ser diferenciable en\(c \in I\), entonces es continuo en\(c\).

    Sabemos que los límites\[\lim_{x\to c}\frac{f(x)-f(c)}{x-c} = f'(c) \qquad \text{and} \qquad \lim_{x\to c}(x-c) = 0\] existen. Además,\[f(x)-f(c) = \left( \frac{f(x)-f(c)}{x-c} \right) (x-c) .\] por lo tanto el límite de\(f(x)-f(c)\) existe y\[\lim_{x\to c} \bigl( f(x)-f(c) \bigr) = \left(\lim_{x\to c} \frac{f(x)-f(c)}{x-c} \right) \left(\lim_{x\to c} (x-c) \right) = f'(c) \cdot 0 = 0.\] Por lo tanto\(\lim\limits_{x\to c} f(x) = f(c)\), y\(f\) es continuo en\(c\).

    Una propiedad importante de la derivada es la linealidad. La derivada es la aproximación de una función por una línea recta. La pendiente de una línea a través de dos puntos cambia linealmente cuando las\(y\) coordenadas se cambian linealmente. Al tomar el límite, tiene sentido que la derivada sea lineal.

    Dejar\(I\) ser un intervalo, dejar\(f \colon I \to {\mathbb{R}}\) y\(g \colon I \to {\mathbb{R}}\) ser diferenciable en\(c \in I\), y dejar\(\alpha \in {\mathbb{R}}\).

    1. Definir\(h \colon I \to {\mathbb{R}}\) por\(h(x) := \alpha f(x)\). Entonces\(h\) es diferenciable en\(c\) y\(h'(c) = \alpha f'(c)\).
    2. Definir\(h \colon I \to {\mathbb{R}}\) por\(h(x) := f(x) + g(x)\). Entonces\(h\) es diferenciable en\(c\) y\(h'(c) = f'(c) + g'(c)\).

    Primero, vamos\(h(x) := \alpha f(x)\). Porque\(x \in I\),\(x \not= c\) tenemos\[\frac{h(x)-h(c)}{x-c} = \frac{\alpha f(x) - \alpha f(c)}{x-c} = \alpha \frac{f(x) - f(c)}{x-c} .\] El límite como\(x\) va a\(c\) existe a la derecha por. Obtenemos\[\lim_{x\to c}\frac{h(x)-h(c)}{x-c} = \alpha \lim_{x\to c} \frac{f(x) - f(c)}{x-c} .\] Por lo tanto\(h\) es diferenciable en\(c\), y la derivada se calcula como se da.

    A continuación, defina\(h(x) := f(x)+g(x)\). Porque\(x \in I\),\(x \not= c\) tenemos\[\frac{h(x)-h(c)}{x-c} = \frac{\bigl(f(x) + g(x)\bigr) - \bigl(f(c) + g(c)\bigr)}{x-c} = \frac{f(x) - f(c)}{x-c} + \frac{g(x) - g(c)}{x-c} .\] El límite como\(x\) va a\(c\) existe a la derecha por. Obtenemos\[\lim_{x\to c}\frac{h(x)-h(c)}{x-c} = \lim_{x\to c} \frac{f(x) - f(c)}{x-c} + \lim_{x\to c}\frac{g(x) - g(c)}{x-c} .\] Por lo tanto\(h\) es diferenciable en\(c\) y la derivada se calcula como se da.

    No es cierto que la derivada de un múltiplo de dos funciones sea el múltiplo de las derivadas. En cambio obtenemos la llamada regla del producto o la regla Leibniz 17.

    Dejar\(I\) ser un intervalo, dejar\(f \colon I \to {\mathbb{R}}\) y\(g \colon I \to {\mathbb{R}}\) ser funciones diferenciables en\(c\). Si\(h \colon I \to {\mathbb{R}}\) se define por\[h(x) := f(x) g(x) ,\] entonces\(h\) es diferenciable en\(c\) y\[h'(c) = f(c) g'(c) + f'(c) g(c) .\]

    El comprobante de la regla del producto se deja como ejercicio. La clave es usar la identidad\(f(x) g(x) - f(c) g(c) = f(x)\bigl( g(x) - g(c) \bigr) + g(c) \bigl( f(x) - f(c) \bigr)\).

    Dejar\(I\) ser un intervalo, dejar\(f \colon I \to {\mathbb{R}}\) y\(g \colon I \to {\mathbb{R}}\) ser diferenciable en\(c\) y\(g(x) \not= 0\) para todos\(x \in I\). Si\(h \colon I \to {\mathbb{R}}\) se define por\[h(x) := \frac{f(x)}{g(x)},\] entonces\(h\) es diferenciable en\(c\) y\[h'(c) = \frac{f'(c) g(c) - f(c) g'(c)}{{\bigl(g(c)\bigr)}^2} .\]

    Nuevamente la prueba se deja como ejercicio.

    Regla de la cadena

    Una regla útil para calcular derivados es la regla de la cadena.

    Dejar\(I_1, I_2\) ser intervalos, dejar\(g \colon I_1 \to I_2\) ser diferenciables en\(c \in I_1\), y\(f \colon I_2 \to {\mathbb{R}}\) ser diferenciables en\(g(c)\). Si\(h \colon I_1 \to {\mathbb{R}}\) se define por\[h(x) := (f \circ g) (x) = f\bigl(g(x)\bigr) ,\] entonces\(h\) es diferenciable en\(c\) y\[h'(c) = f'\bigl(g(c)\bigr)g'(c) .\]

    Vamos\(d := g(c)\). Definir\(u \colon I_2 \to {\mathbb{R}}\) y\(v \colon I_1 \to {\mathbb{R}}\) por\[\begin{aligned} & u(y) := \begin{cases} \frac{f(y) - f(d)}{y-d} & \text{ if $y \not=d$,} \\ f'(d) & \text{ if $y = d$,} \end{cases} \\ & v(x) := \begin{cases} \frac{g(x) - g(c)}{x-c} & \text{ if $x \not=c$,} \\ g'(c) & \text{ if $x = c$.} \end{cases}\end{aligned}\]\[f(y)-f(d) = u(y) (y-d) \qquad \text{and} \qquad g(x)-g(c) = v(x) (x-c) .\] Observamos que nos enchufamos para obtener\[h(x)-h(c) = f\bigl(g(x)\bigr)-f\bigl(g(c)\bigr) = u\bigl( g(x) \bigr) \bigl(g(x)-g(c)\bigr) = u\bigl( g(x) \bigr) \bigl(v(x) (x-c)\bigr) .\] Por lo tanto,\[\label{eq:chainruleeq} \frac{h(x)-h(c)}{x-c} = u\bigl( g(x) \bigr) v(x) .\] calculamos los límites\(\lim_{y \to d} u(y) = f'(d) = f'\bigl(g(c)\bigr)\) y\(\lim_{x \to c} v(x) = g'(c)\). Es decir, las funciones\(u\) y\(v\) son continuas en\(d = g(c)\) y\(c\) respectivamente. Además, la función\(g\) es continua en\(c\). De ahí el límite del lado derecho de [eq:chainruleeq] como\(x\) va a\(c\) existe y es igual a\(f'\bigl(g(c)\bigr) g'(c)\). Así\(h\) es diferenciable en\(c\) y el límite es\(f'\bigl(g(c)\bigr)g'(c)\).

    Ejercicios

    Demostrar la regla del producto. Pista: Uso\(f(x) g(x) - f(c) g(c) = f(x)\bigl( g(x) - g(c) \bigr) + g(c) \bigl( f(x) - f(c) \bigr)\).

    Demostrar la regla del cociente. Pista: Puedes hacer esto directamente, pero puede ser más fácil encontrar la derivada de\(\nicefrac{1}{x}\) y luego usar la regla de cadena y la regla del producto.

    [ejercicio:diffofxn] Para\(n \in {\mathbb{Z}}\), demostrar que\(x^n\) es diferenciable y encontrar la derivada, a menos que, por supuesto,\(n < 0\) y\(x=0\). Pista: Usa la regla del producto.

    Demostrar que un polinomio es diferenciable y encontrar la derivada. Pista: Usa el ejercicio anterior.

    Definir\(f \colon {\mathbb{R}}\to {\mathbb{R}}\) por\[f(x) := \begin{cases} x^2 & \text{ if $x \in {\mathbb{Q}}$,}\\ 0 & \text{ otherwise.} \end{cases}\] Demostrar que\(f\) es diferenciable en\(0\), pero discontinuo en todos los puntos excepto\(0\).

    Asumir la desigualdad\(\left\lvert {x-\sin(x)} \right\rvert \leq x^2\). Demostrar que el pecado es diferenciable en\(0\), y encontrar el derivado en\(0\).

    Utilizando el ejercicio anterior, demostrar que el pecado es diferenciable en absoluto\(x\) y que la derivada lo es\(\cos(x)\). Pista: Utilice la identidad trigonométrica suma-a-producto como hicimos antes.

    Dejemos\(f \colon I \to {\mathbb{R}}\) ser diferenciables. Dado\(n \in {\mathbb{Z}}\), definir\(f^n\) ser la función definida por\(f^n(x) := {\bigl( f(x) \bigr)}^n\). Si\(n < 0\) asumamos\(f(x) \not= 0\). \((f^n)'(x) = n {\bigl(f(x) \bigr)}^{n-1} f'(x)\)Demuéstralo.

    Supongamos que\(f \colon {\mathbb{R}}\to {\mathbb{R}}\) es una función continua diferenciable de Lipschitz. Demostrar que\(f'\) es una función acotada.

    Dejar\(I_1, I_2\) ser intervalos. Dejar\(f \colon I_1 \to I_2\) ser una función biyectiva y\(g \colon I_2 \to I_1\) ser la inversa. Supongamos que ambos\(f\) es diferenciable en\(c \in I_1\)\(f'(c) \not=0\) y y\(g\) es diferenciable en\(f(c)\). Use la regla de la cadena para encontrar una fórmula para\(g'\bigl(f(c)\bigr)\) (en términos de\(f'(c)\)).

    [exercise:bndmuldiff] Supongamos que\(f \colon I \to {\mathbb{R}}\) es una función delimitada y\(g \colon I \to {\mathbb{R}}\) es una función diferenciable en\(c \in I\) y\(g(c) = g'(c) = 0\). Demostrar que\(h(x) := f(x) g(x)\) es diferenciable en\(c\). Pista: Tenga en cuenta que no puede aplicar la regla del producto.

    [exercise:diffsqueeze] Supongamos\(f \colon I \to {\mathbb{R}}\)\(g \colon I \to {\mathbb{R}}\),, y\(h \colon I \to {\mathbb{R}}\), son funciones. Supongamos que\(c \in I\) es tal que\(f(c) = g(c) = h(c)\),\(g\) y\(h\) son diferenciables en\(c\), y\(g'(c) = h'(c)\). Además supongamos\(h(x) \leq f(x) \leq g(x)\) para todos\(x \in I\). Demostrar\(f\) es diferenciable en\(c\) y\(f'(c) = g'(c) = h'(c)\).

    Teorema del valor medio

    Nota: 2 conferencias (algunas aplicaciones pueden omitirse)

    Mínimos y máximos relativos

    Hablamos de máximos y mínimos absolutos. Estos son los picos más altos y valles más bajos de toda la cordillera. También podríamos hablar de picos de montañas y valles individuales.

    Dejar\(S \subset {\mathbb{R}}\) ser un conjunto y dejar\(f \colon S \to {\mathbb{R}}\) ser una función. \(f\)Se dice que la función tiene un máximo relativo en\(c \in S\) si existe\(\delta>0\) tal que para todos los\(x \in S\) tales que\(\left\lvert {x-c} \right\rvert < \delta\) tenemos\(f(x) \leq f(c)\). La definición de mínimo relativo es análoga.

    [relminmax:thm] Let\(f \colon [a,b] \to {\mathbb{R}}\) Ser una función diferenciable en\(c \in (a,b)\), y\(c\) es un mínimo relativo o un máximo relativo de\(f\). Entonces\(f'(c) = 0\).

    Demostramos la declaración por un máximo. Por un mínimo la sentencia sigue considerando la función\(-f\).

    Dejar\(c\) ser un máximo relativo de\(f\). En particular el tiempo\(\left\lvert {x-c} \right\rvert < \delta\) que tengamos\(f(x)-f(c) \leq 0\). Entonces nos fijamos en el cociente de diferencia. Si\(x > c\) notamos eso\[\frac{f(x)-f(c)}{x-c} \leq 0 ,\] y si\(y < c\) tenemos\[\frac{f(y)-f(c)}{y-c} \geq 0 .\] Ver para una ilustración.

    Ahora tomamos secuencias\(\{ x_n\}\) y\(\{ y_n \}\), tal que\(x_n > c\), y\(y_n < c\) para todos\(n \in {\mathbb{N}}\), y tal que\(\lim\, x_n = \lim\, y_n = c\). Ya que\(f\) es diferenciable en\(c\) sabemos\[0 \geq \lim_{n\to\infty} \frac{f(x_n)-f(c)}{x_n-c} = f'(c) = \lim_{n\to\infty} \frac{f(y_n)-f(c)}{y_n-c} \geq 0. \qedhere\]

    Para una función diferenciable, un punto donde\(f'(c) = 0\) se denomina punto crítico. Cuando no\(f\) es diferenciable en algunos puntos, es común decir también que\(c\) es un punto crítico si\(f'(c)\) no existe. El teorema dice que un mínimo o máximo relativo en un punto interior de un intervalo debe ser un punto crítico. Como recuerdas del cálculo, encontrar mínimos y máximos de una función se puede hacer encontrando todos los puntos críticos junto con los puntos finales del intervalo y simplemente verificando dónde está la función más grande o más pequeña.

    Teorema de Rolle

    Supongamos que una función es cero en ambos puntos finales de un intervalo. Intuitivamente debe alcanzar un mínimo o un máximo en el interior del intervalo, entonces a tal mínimo o máximo, la derivada debe ser cero. Consulta por la idea geométrica. Este es el contenido del llamado teorema de Rolle 18.

    [thm:rolle] Dejar\(f \colon [a,b] \to {\mathbb{R}}\) ser función continua diferenciable sobre\((a,b)\) tal que\(f(a) = f(b)\). Entonces existe\(c \in (a,b)\) tal que\(f'(c) = 0\).

    Como\(f\) es continuo en\([a,b]\) ella alcanza un mínimo absoluto y un máximo absoluto en\([a,b]\). Deseamos aplicar y por lo tanto necesitamos un mínimo o máximo en algunos\(c \in (a,b)\). Escribir\(K := f(a) = f(b)\). Si existe\(x\) tal que\(f(x) > K\), entonces el máximo absoluto es mayor que\(K\) y por lo tanto ocurre en\(c \in (a,b)\), y por lo tanto obtenemos\(f'(c) = 0\). Por otro lado si existe\(x\) tal que\(f(x) < K\), entonces el mínimo absoluto ocurre en algunos\(c \in (a,b)\) y tenemos eso\(f'(c) = 0\). Si no hay\(x\) tal que\(f(x) > K\) o\(f(x) < K\), entonces tenemos eso\(f(x) = K\) para todos\(x\) y luego\(f'(x) = 0\) para todos\(x \in [a,b]\), entonces cualquiera\(c\) va a funcionar.

    Tenga en cuenta que es absolutamente necesario que la derivada exista para todos\(x \in (a,b)\). Por ejemplo, tomar la función\(f(x) = \left\lvert {x} \right\rvert\) encendido\([-1,1]\). Claramente\(f(-1) = f(1)\), pero no tiene sentido dónde\(f'(c) = 0\).

    Teorema del valor medio

    Nos extendemos a funciones que alcanzan diferentes valores en los puntos finales.

    [thm:mvt] Dejar\(f \colon [a,b] \to {\mathbb{R}}\) ser una función continua diferenciable en\((a,b)\). Entonces existe un punto\(c \in (a,b)\) tal que\[f(b)-f(a) = f'(c)(b-a) .\]

    El teorema se desprende de. Definir la función\(g \colon [a,b] \to {\mathbb{R}}\) por\[g(x) := f(x)-f(b)+\bigl(f(b)-f(a)\bigr)\frac{b-x}{b-a}.\] La función\(g\) es un diferenciable en\((a,b)\), continuo en\([a,b]\), tal que\(g(a) = 0\) y\(g(b) = 0\). Así existe\(c \in (a,b)\) tal que\(g'(c) = 0\). \[0 = g'(c) = f'(c) + \bigl(f(b)-f(a)\bigr)\frac{-1}{b-a} .\]O en otras palabras\(f'(c)(b-a) = f(b)-f(a)\).

    Para una interpretación geométrica del teorema del valor medio, véase. La idea es que el valor\(\frac{f(b)-f(a)}{b-a}\) sea la pendiente de la línea entre los puntos\(\bigl(a,f(a)\bigr)\) y\(\bigl(b,f(b)\bigr)\). Entonces\(c\) es el punto tal que\(f'(c) = \frac{f(b)-f(a)}{b-a}\), es decir, la línea tangente en el punto\(\bigl(c,f(c)\bigr)\) tiene la misma pendiente que la línea entre\(\bigl(a,f(a)\bigr)\) y\(\bigl(b,f(b)\bigr)\).

    Aplicaciones

    Ahora resolvemos nuestra primera ecuación diferencial.

    [prop:derzeroconst] Dejar\(I\) ser un intervalo y dejar\(f \colon I \to {\mathbb{R}}\) ser una función diferenciable tal que\(f'(x) = 0\) para todos\(x \in I\). Entonces\(f\) es constante.

    Tomar arbitrario\(x,y \in I\) con\(x < y\). Entonces\(f\) restringido a\([x,y]\) satisface las hipótesis de la. Por lo tanto hay\(c \in (x,y)\) tal que\[f(y)-f(x) = f'(c)(y-x).\] como\(f'(c) = 0\), tenemos\(f(y) = f(x)\). Por lo tanto, la función es constante.

    Ahora que sabemos lo que significa que la función se mantenga constante, veamos las funciones crecientes y decrecientes. Decimos que\(f \colon I \to {\mathbb{R}}\) está aumentando (resp. estrictamente creciente) si\(x < y\) implica\(f(x) \leq f(y)\) (resp. \(f(x) < f(y)\)). Definimos decreciente y estrictamente decreciente de la misma manera cambiando las desigualdades por\(f\).

    [incdecdiffprop] Dejar\(I\) ser un intervalo y dejar\(f \colon I \to {\mathbb{R}}\) ser una función diferenciable.

    1. \(f\)está aumentando si y sólo si es\(f'(x) \geq 0\) por todos\(x \in I\).
    2. \(f\)es decreciente si y sólo si\(f'(x) \leq 0\) por todos\(x \in I\).

    Probemos el primer ítem. Supongamos que\(f\) va en aumento, entonces para todos\(x\) y\(c\) en\(I\) tenemos\[\frac{f(x)-f(c)}{x-c} \geq 0 .\] tomando un límite como\(x\) va a\(c\) ver eso\(f'(c) \geq 0\).

    Para la otra dirección, supongamos\(f'(x) \geq 0\) para todos\(x \in I\). Lleve a cualquier\(x, y \in I\) lugar\(x < y\). Por el hay algunos\(c \in (x,y)\) tales que\[f(x)-f(y) = f'(c)(x-y) .\] As\(f'(c) \geq 0\), y\(x-y < 0\), entonces\(f(x) - f(y) \leq 0\) o\(f(x) \leq f(y)\) así\(f\) va en aumento.

    Dejamos la parte decreciente al lector como ejercicio.

    Podemos hacer una declaración similar pero más débil sobre las funciones estrictamente crecientes y decrecientes. Si\(f'(x) > 0\) por todos\(x \in I\), entonces\(f\) está aumentando estrictamente. La prueba se deja como ejercicio. Lo contrario no es cierto. Por ejemplo,\(f(x) := x^3\) es una función estrictamente creciente, pero\(f'(0) = 0\).

    Otra aplicación de la es el siguiente resultado sobre ubicación de extrema. El teorema se establece para un mínimo y máximo absolutos, pero la forma en que se aplica para encontrar mínimos y máximos relativos es restringirse\(f\) a un intervalo\((c-\delta,c+\delta)\).

    [firstderminmaxtest] Dejar\(f \colon (a,b) \to {\mathbb{R}}\) ser continuo. Dejar\(c \in (a,b)\) y suponer\(f\) es diferenciable en\((a,c)\) y\((c,b)\).

    1. Si\(f'(x) \leq 0\) para\(x \in (a,c)\) y\(f'(x) \geq 0\) para\(x \in (c,b)\), entonces\(f\) tiene un mínimo absoluto en\(c\).
    2. Si\(f'(x) \geq 0\) para\(x \in (a,c)\) y\(f'(x) \leq 0\) para\(x \in (c,b)\), entonces\(f\) tiene un máximo absoluto en\(c\).

    Probemos el primer ítem. El segundo se deja al lector. Dejar entrar\(x\)\((a,c)\) y\(\{ y_n\}\) una secuencia tal que\(x < y_n < c\) y\(\lim\, y_n = c\). Por la proposición anterior, la función está disminuyendo en\((a,c)\) así\(f(x) \geq f(y_n)\). La función es continua en\(c\) así que podemos tomar el límite para obtener\(f(x) \geq f(c)\) para todos\(x \in (a,c)\).

    Del mismo modo tomar\(x \in (c,b)\) y\(\{ y_n\}\) una secuencia tal que\(c < y_n < x\) y\(\lim\, y_n = c\). La función está aumentando\((c,b)\) así\(f(x) \geq f(y_n)\). Por continuidad de\(f\) obtenemos\(f(x) \geq f(c)\) para todos\(x \in (c,b)\). Así\(f(x) \geq f(c)\) para todos\(x \in (a,b)\).

    Lo contrario de la proposición no se sostiene. Ver abajo.

    Continuidad de las derivadas y el teorema del valor intermedio

    Derivadas de funciones satisfacen una propiedad de valor intermedio. El resultado suele llamarse teorema de Darboux.

    [thm:darboux] Dejar\(f \colon [a,b] \to {\mathbb{R}}\) ser diferenciables. Supongamos que existe\(y \in {\mathbb{R}}\) tal que\(f'(a) < y < f'(b)\) o\(f'(a) > y > f'(b)\). Entonces existe\(c \in (a,b)\) tal que\(f'(c) = y\).

    Supongamos sin pérdida de generalidad eso\(f'(a) < y < f'(b)\). Definir\[g(x) := yx - f(x) .\] Como\(g\) es continuo en\([a,b]\), luego\(g\) alcanza un máximo en algunos\(c \in [a,b]\).

    Ahora computa\(g'(x) = y-f'(x)\). Así\(g'(a) > 0\). Como la derivada es el límite de los cocientes de diferencia y es positiva, debe haber algún cociente de diferencia que sea positivo. Es decir, debe existir\(x > a\) tal que\[\frac{g(x)-g(a)}{x-a} > 0 ,\] o\(g(x) > g(a)\). Por lo tanto,\(a\) no puede ser un máximo de\(g\). De igual manera\(g'(b) < 0\), encontramos un\(x < b\) (un diferente\(x\)) tal que\(\frac{g(x)-g(b)}{x-b} < 0\) o aquello\(g(x) > g(b)\), así\(b\) no puede ser posiblemente un máximo.

    Por lo tanto\(c \in (a,b)\). Entonces como\(c\) es un máximo de\(g\) nos encontramos\(g'(c) = 0\) y\(f'(c) = y\).

    Ya hemos visto que existen funciones discontinuas que tienen la propiedad de valor intermedio. Si bien es difícil de imaginar al principio, también existen funciones que son diferenciables en todas partes y la derivada no es continua.

    [baddifffunc:example]\(f \colon {\mathbb{R}}\to {\mathbb{R}}\) Sea la función definida por\[f(x) := \begin{cases} {\bigl( x \sin(\nicefrac{1}{x}) \bigr)}^2 & \text{ if $x \not= 0$,} \\ 0 & \text{ if $x = 0$.} \end{cases}\] Afirmamos que\(f\) es diferenciable en todas partes, pero no\(f' \colon {\mathbb{R}}\to {\mathbb{R}}\) es continua en el origen. Además,\(f\) tiene un mínimo en 0, pero la derivada cambia signo infinitamente a menudo cerca del origen. Ver.

    Una función con una derivada discontinua. La función f está a la izquierda y f' está a la derecha. Observe que f (x)\ leq x^2 en la gráfica izquierda. [fig:nonc1diff]Una función con una derivada discontinua. La función f está a la izquierda y f' está a la derecha. Observe que f (x)\ leq x^2 en la gráfica izquierda. [fig:nonc1diff]

    Prueba: Es fácil ver a partir de la definición que\(f\) tiene un mínimo absoluto en 0: sabemos\(f(x) \geq 0\) para todos\(x\) y\(f(0) = 0\).

    La función\(f\) es diferenciable para\(x\not=0\) y la derivada es\(2 \sin (\nicefrac{1}{x}) \bigl( x \sin (\nicefrac{1}{x}) - \cos(\nicefrac{1}{x}) \bigr)\). Como ejercicio muestran que para\(x_n = \frac{4}{(8n+1)\pi}\) nosotros tenemos\(\lim\, f'(x_n) = -1\), y para\(y_n = \frac{4}{(8n+3)\pi}\) nosotros tenemos\(\lim\, f'(y_n) = 1\). De ahí\(f'\) que si existe en\(0\), entonces no puede ser continuo.

    Demostremos que\(f'\) existe a 0. Afirmamos que la derivada es cero. En otras palabras\(\left\lvert {\frac{f(x)-f(0)}{x-0} - 0} \right\rvert\) va a cero como\(x\) va a cero. Porque\(x \not= 0\) tenemos\[\left\lvert {\frac{f(x)-f(0)}{x-0} - 0} \right\rvert = \left\lvert {\frac{x^2 \sin^2(\nicefrac{1}{x})}{x}} \right\rvert = \left\lvert {x \sin^2(\nicefrac{1}{x})} \right\rvert \leq \left\lvert {x} \right\rvert .\] Y, por supuesto, como\(x\) tiende a cero, luego\(\left\lvert {x} \right\rvert\) tiende a cero y de ahí\(\left\lvert {\frac{f(x)-f(0)}{x-0} - 0} \right\rvert\) va a cero. Por lo tanto,\(f\) es diferenciable a 0 y la derivada a 0 es 0. Un punto clave en el cálculo anterior es que es que\(\left\lvert {f(x)} \right\rvert \leq x^2\), véase también Ejercicios [ejercicio:bndmuldiff] y [ejercicio:difsqueeze].

    A veces es útil asumir que la derivada de una función diferenciable es continua. Si\(f \colon I \to {\mathbb{R}}\) es diferenciable y la derivada\(f'\) es continua\(I\), entonces decimos que\(f\) es continuamente diferenciable. Es común escribir\(C^1(I)\) para el conjunto de funciones continuamente diferenciables en\(I\).

    Ejercicios

    Terminar la prueba de.

    Terminar la prueba de.

    Supongamos que\(f \colon {\mathbb{R}}\to {\mathbb{R}}\) es una función diferenciable tal que\(f'\) es una función acotada. Probar\(f\) es una función continua de Lipschitz.

    Supongamos que\(f \colon [a,b] \to {\mathbb{R}}\) es diferenciable y\(c \in [a,b]\). Entonces mostrar existe una secuencia\(\{ x_n \}\) convergente a\(c\),\(x_n \not= c\) para todos\(n\), tal que\[f'(c) = \lim_{n\to \infty} f'(x_n).\] Notar esto no implica que\(f'\) sea continuo (¿por qué?).

    Supongamos que\(f \colon {\mathbb{R}}\to {\mathbb{R}}\) es una función tal que\(\left\lvert {f(x)-f(y)} \right\rvert \leq \left\lvert {x-y} \right\rvert^2\) para todos\(x\) y\(y\). \(f(x) = C\)Demuéstralo por alguna constante\(C\). Pista: Mostrar que\(f\) es diferenciable en todos los puntos y computa la derivada.

    [exercise:posderincr] Supongamos que\(I\) es un intervalo y\(f \colon I \to {\mathbb{R}}\) es una función diferenciable. Si\(f'(x) > 0\) por todos\(x \in I\), mostrar que\(f\) es estrictamente creciente.

    Supongamos que\(f \colon (a,b) \to {\mathbb{R}}\) es una función diferenciable tal que\(f'(x) \not= 0\) para todos\(x \in (a,b)\). Supongamos que existe un punto\(c \in (a,b)\) tal que\(f'(c) > 0\). Demostrar\(f'(x) > 0\) para todos\(x \in (a,b)\).

    [ejercicio:samediffconst] Supongamos\(f \colon (a,b) \to {\mathbb{R}}\) y\(g \colon (a,b) \to {\mathbb{R}}\) son funciones diferenciables tales que\(f'(x) = g'(x)\) para todos\(x \in (a,b)\), luego muestran que existe una constante\(C\) tal que\(f(x) = g(x) + C\).

    Demostrar la siguiente versión de la regla de L'Hopital. Supongamos\(f \colon (a,b) \to {\mathbb{R}}\) y\(g \colon (a,b) \to {\mathbb{R}}\) son funciones diferenciables. Supongamos que en\(c \in (a,b)\)\(f(c) = 0\),\(g(c)=0\),, y que\(c\) existe el límite de\(\nicefrac{f'(x)}{g'(x)}\) lo que\(x\) va a. Demostrar que\[\lim_{x \to c} \frac{f(x)}{g(x)} = \lim_{x \to c} \frac{f'(x)}{g'(x)} .\]

    Dejar\(f \colon (a,b) \to {\mathbb{R}}\) ser una función diferenciable sin límites. \(f' \colon (a,b) \to {\mathbb{R}}\)El espectáculo no tiene límites.

    Demostrar el teorema Rolle realmente demostró en 1691: Si\(f\) es un polinomio\(a < b\),\(f'(a) = f'(b) = 0\) para algunos, y no hay\(c \in (a,b)\) tal que\(f'(c) = 0\), entonces hay como mucho una raíz de\(f\) adentro\((a,b)\), que es a lo sumo uno\(x \in (a,b)\) tal que\(f(x) = 0\). En otras palabras, entre dos raíces consecutivas cualquiera de\(f'\) es a lo sumo una raíz de\(f\). Pista: supongamos que hay dos raíces y ver qué pasa.

    Supongamos\(a,b \in {\mathbb{R}}\) y\(f \colon {\mathbb{R}}\to {\mathbb{R}}\) es diferenciable,\(f'(x) = a\) para todos\(x\), y\(f(0) = b\). Encuentra\(f\) y demuestra que es la función diferenciable única con esta propiedad.

    Teorema de Taylor

    Nota: media conferencia (sección opcional)

    Derivados de órdenes superiores

    Cuando\(f \colon I \to {\mathbb{R}}\) es diferenciable, obtenemos una función\(f' \colon I \to {\mathbb{R}}\). La función\(f'\) se llama la primera derivada de\(f\). Si\(f'\) es diferenciable, denotamos por\(f'' \colon I \to {\mathbb{R}}\) la derivada de\(f'\). La función\(f''\) se llama la segunda derivada de\(f\). De manera similar obtenemos\(f'''\),\(f''''\), y así sucesivamente. Con un mayor número de derivadas la notación se saldría de control; denotamos por\(f^{(n)}\) la derivada\(n\) th de\(f\).

    Cuando\(f\) posee\(n\) derivados, decimos que\(f\) es \(n\)tiempos diferenciables.

    Teorema de Taylor

    El teorema de Taylor 19 es una generalización de la. El teorema del valor medio dice que hasta un pequeño error\(f(x)\) para\(x\) cerca se\(x_0\) puede aproximar por\(f(x_0)\), es decir\[f(x) = f(x_0) + f'(c)(x-x_0),\] donde se mide el “error” en términos de la primera derivada en algún punto\(c\) entre\(x\) y\(x_0\). El teorema de Taylor generaliza este resultado a derivados superiores. Nos dice que hasta un pequeño error, cualquier función diferenciable\(n\) veces puede ser aproximada en un punto\(x_0\) por un polinomio. El error de esta aproximación se comporta como\({(x-x_0)}^{n}\) cerca del punto\(x_0\). Para ver por qué esta es una buena aproximación note que para una gran\(n\),\({(x-x_0)}^n\) es muy pequeña en un pequeño intervalo alrededor\(x_0\).

    Para una función diferenciable de\(n\) tiempos\(f\) definida cerca de un punto\(x_0 \in {\mathbb{R}}\), defina el polinomio de Taylor\(n\) th para\(f\) at\(x_0\) as\[\begin{split} P_n^{x_0}(x) & := \sum_{k=0}^n \frac{f^{(k)}(x_0)}{k!}{(x-x_0)}^k \\ & = f(x_0) + f'(x_0)(x-x_0) + \frac{f''(x_0)}{2}{(x-x_0)}^2 + \frac{f^{(3)}(x_0)}{6}{(x-x_0)}^3 + \cdots + \frac{f^{(n)}(x_0)}{n!}{(x-x_0)}^n . \end{split}\]

    El teorema de Taylor dice que una función se comporta como su polinomio\(n\) th Taylor. El es realmente el teorema de Taylor para la primera derivada.

    [thm:taylor] Supongamos que\(f \colon [a,b] \to {\mathbb{R}}\) es una función con derivadas\(n\) continuas sobre\([a,b]\) y tal que\(f^{(n+1)}\) existe en\((a,b)\). Dados distintos puntos\(x_0\) y\(x\) en\([a,b]\), podemos encontrar un punto\(c\) entre\(x_0\) y\(x\) tal que\[f(x)=P_{n}^{x_0}(x)+\frac{f^{(n+1)}(c)}{(n+1)!}{(x-x_0)}^{n+1} .\]

    El término\(R_n^{x_0}(x):=\frac{f^{(n+1)}(c)}{(n+1)!}{(x-x_0)}^{n+1}\) se denomina término restante. Esta forma del término restante se llama la forma Lagrange del resto. Hay otras formas de escribir el término restante, pero nos saltamos esas. Tenga en cuenta que\(c\) depende de ambos\(x\) y\(x_0\).

    Encontrar un número\(M_{x,x_0}\) (dependiendo de\(x\) y\(x_0\)) resolviendo la ecuación\[f(x)=P_{n}^{x_0}(x)+M_{x,x_0}{(x-x_0)}^{n+1} .\] Definir una función\(g(s)\) por\[g(s) := f(s)-P_n^{x_0}(s)-M_{x,x_0}{(s-x_0)}^{n+1} .\] Calculamos la derivada\(k\) th en\(x_0\) del polinomio de Taylor\({(P_n^{x_0})}^{(k)}(x_0) = f^{(k)}(x_0)\) para\(k=0,1,2,\ldots,n\) (la derivada cero corresponde a la función misma) . Por lo tanto,\[g(x_0) = g'(x_0) = g''(x_0) = \cdots = g^{(n)}(x_0) = 0 .\] en particular\(g(x_0) = 0\). Por otro lado\(g(x) = 0\). Por el existe un\(x_1\) entre\(x_0\) y\(x\) tal que\(g'(x_1) = 0\). Aplicando el a\(g'\) obtenemos que existe\(x_2\) entre\(x_0\) y\(x_1\) (y por lo tanto entre\(x_0\) y\(x\)) tal que\(g''(x_2) = 0\). Repetimos los\(n+1\) tiempos de argumento para obtener un número\(x_{n+1}\) entre\(x_0\) y\(x_n\) (y por lo tanto entre\(x_0\) y\(x\)) tal que\(g^{(n+1)}(x_{n+1}) = 0\).

    Vamos\(c:=x_{n+1}\). Calculamos el\((n+1)\) th derivado de\(g\) para encontrar\[g^{(n+1)}(s) = f^{(n+1)}(s)-(n+1)!\,M_{x,x_0} .\] Plugging in\(c\) para\(s\) que obtengamos\(M_{x,x_0} = \frac{f^{(n+1)}(c)}{(n+1)!}\), y ya terminamos.

    En la prueba\({(P_n^{x_0})}^{(k)}(x_0) = f^{(k)}(x_0)\) para la que hemos calculado\(k=0,1,2,\ldots,n\). Por lo tanto, el polinomio de Taylor tiene los mismos\(f\) derivados que\(x_0\) hasta el\(n\) th derivado. Es por ello que el polinomio Taylor es una buena aproximación a\(f\).

    La definición de derivada dice que una función es diferenciable si se aproxima localmente por una línea. De igual manera mencionamos de pasada que existe una inversa al teorema de Taylor, que no vamos a declarar ni probar, diciendo que si una función es localmente aproximada de cierta manera por un polinomio de grado\(d\), entonces tiene\(d\) derivados.

    Ejercicios

    Calcula el polinomio de Taylor\(n\) th en\(0\) para la función exponencial.

    Supongamos que\(p\) es un polinomio de grado\(d\). Dado cualquiera\(x_0 \in {\mathbb{R}}\), mostrar que el polinomio\((d+1)\) th Taylor para\(p\) at\(x_0\) es igual a\(p\).

    Vamos\(f(x) := \left\lvert {x} \right\rvert^3\). Compute\(f'(x)\) y\(f''(x)\) para todos\(x\), pero demuestre que\(f^{(3)}(0)\) no existe.

    Supongamos que\(f \colon {\mathbb{R}}\to {\mathbb{R}}\) tiene derivadas\(n\) continuas. Demostrar que para cualquiera\(x_0 \in {\mathbb{R}}\), existen polinomios\(P\) y\(Q\) de grado\(n\) y\(\epsilon > 0\) tal que\(P(x) \leq f(x) \leq Q(x)\) para todos\(x \in [x_0-\epsilon,x_0+\epsilon]\) y\(Q(x)-P(x) = \lambda {(x-x_0)}^n\) para algunos\(\lambda \geq 0\).

    Si\(f \colon [a,b] \to {\mathbb{R}}\) tiene derivados\(n+1\) continuos y\(x_0 \in [a,b]\), probar\(\lim\limits_{x\to x_0} \frac{R_n^{x_0}(x)}{{(x-x_0)}^n} = 0\).

    Supongamos que\(f \colon [a,b] \to {\mathbb{R}}\) tiene derivadas\(n+1\) continuas y\(x_0 \in (a,b)\). Demostrar que\(f^{(k)}(x_0) = 0\) para todos\(k = 0, 1, 2, \ldots, n\) si y solo si\(g(x) := \frac{f(x)}

    ParseError: EOF expected (click for details)
    Callstack:
        at (Matematicas/Analisis/Introducción_al_Análisis_Real_(Lebl)/02:_Números_reales/2.01:_Propiedades_básicas), /content/body/div[26]/p[6]/span[6]/span, line 1, column 8
    
    \) es continuo en\(x_0\).

    Supongamos\(a,b,c \in {\mathbb{R}}\) y\(f \colon {\mathbb{R}}\to {\mathbb{R}}\) es diferenciable,\(f''(x) = a\) para todos\(x\),\(f'(0) = b\), y\(f(0) = c\). Encuentra\(f\) y demuestra que es la función diferenciable única con esta propiedad.

    Demostrar que una simple conversa al teorema de Taylor no se sostiene. Encuentra una función\(f \colon {\mathbb{R}}\to {\mathbb{R}}\) sin segunda derivada en\(x=0\) tal que\(\left\lvert {f(x)} \right\rvert \leq \left\lvert {x^3} \right\rvert\), es decir,\(f\) vaya a cero a 0 más rápido que\(x^3\), y mientras\(f'(0)\) exista,\(f''(0)\) no.

    Teorema de la función inversa

    Nota: menos de 1 conferencia (sección opcional, necesaria para, requiere)

    Teorema de la función inversa

    La idea principal de diferenciar funciones inversas es el siguiente lema.

    [lemma:ift] Dejar\(I,J \subset {\mathbb{R}}\) ser intervalos. Si\(f \colon I \to J\) es estrictamente monótona (de ahí uno a uno\(f(I) = J\)), onto (), diferenciable en\(x\)\(f'(x) \not= 0\), y, entonces la inversa\(f^{-1}\) es diferenciable en\(y = f(x)\) y\[(f^{-1})'(y) = \frac{1}{f'\bigl( f^{-1}(y) \bigr)} = \frac{1}{f'(x)} .\] Si\(f\) es continuamente diferenciable y nunca\(f'\) es cero, entonces\(f^{-1}\) es continuamente diferenciable.

    Por\(f\) tiene un inverso continuo. Llamemos a la inversa\(g \colon J \to I\) por conveniencia. Seamos\(x,y\) como en la declaración, tomemos\(t \in I\) para ser arbitrarios y dejemos\(s := f(t)\). Entonces\[\frac{g(s)-g(y)}{s-y} = \frac{g\bigl(f(t)\bigr)-g\bigl(f(x)\bigr)}{f(t)-f(x)} = \frac{t-x}{f(t)-f(x)} .\] As\(f\) es diferenciable en\(x\) y\(f'(x) \not= 0\), entonces\(\frac{t-x}{f(t)-f(x)} \to \nicefrac{1}{f'(x)}\) como\(t \to x\). Porque\(g(s) \to g(y)\) como\(s \to y\), podemos enchufar\(g(s)\) para\(t\), y\(g(y)\) para\(x\) y tomar el límite como\(s\) va a\(y\), es decir, el límite existe. En otras palabras,\[\lim_{s \to y} \frac{g(s)-g(y)}{s-y} = \lim_{t \to x} \frac{t-x}{f(t)-f(x)} = \frac{1}{f'(x)} = \frac{1}{f'\bigl(g(y)\bigr)}\] Ve por la idea geométrica.

    Si ambos\(f'\) y\(g\) son continuos,\(f'\) es distinto de cero en absoluto\(x\), entonces el lema se aplica en todos los puntos\(x \in I\) y la función resultante\(g'(y) = \frac{1}{f'\bigl(g(t)\bigr)}\) debe ser continua.

    Lo que suele llamarse el teorema de la función inversa es el siguiente resultado.

    Dejar\(f \colon (a,b) \to {\mathbb{R}}\) ser una función continuamente diferenciable,\(x_0 \in (a,b)\) un punto donde\(f'(x_0) \not= 0\). Luego existe un intervalo\(I \subset (a,b)\) con\(x_0 \in I\), la restricción\(f|_{I}\) es inyectiva con una inversa\(g \colon J \to I\) definida en\(J := f(I)\), que es continuamente diferenciable y\[g'(y) = \frac{1}{f'\bigl( g(y) \bigr)} , \qquad \text{for all $y \in J$}.\]

    Sin pérdida de generalidad, supongamos\(f'(x_0) > 0\). Como\(f'\) es continuo, debe existir un intervalo\(I\) con\(x_0 \in I\) tal que\(f'(x) > 0\) para todos\(x_0 \in I\).

    Por\(f\) está aumentando estrictamente en\(I\), y de ahí la restricción\(f|_{I}\) biyectiva sobre\(J: = f(I)\). Como\(f\) es continuo, entonces por el (ver también),\(f(I)\) está en intervalo. Ahora aplica.

    Si intentaste demostrar la existencia de raíces directamente como en es posible que hayas visto lo difícil que es ese esfuerzo. Sin embargo, con la maquinaria que hemos construido para funciones inversas se convierte en un ejercicio casi trivial, y con y el teorema de la función inversa demostramos mucho más que la mera existencia.

    Dado cualquiera\(n \in {\mathbb{N}}\) y cualquiera\(x \geq 0\) existe un número único\(y \geq 0\) (denotado\(x^{1/n} := y\)), tal que\(y^n = x\). Además, la función\(g \colon (0,\infty) \to (0,\infty)\) definida por\(g(x) := x^{1/n}\) es continuamente diferenciable y\[g'(x) = \frac{1}{nx^{(n-1)/n}} = \frac{1}{n} \, x^{(1-n)/n} ,\] utilizando la convención\(x^{n/m} := {(x^{1/m})}^{n}\).

    Porque\(x=0\) la existencia de una raíz única es trivial.

    Vamos\(f(x) := x^n\). El uso de regla de producto,\(f\) es continuamente diferenciable y\(f'(x) = nx^{n-1}\), ver. Para\(x > 0\) la derivada\(f'\) es estrictamente positiva y así de nuevo por,\(f\) es estrictamente creciente (esto también se puede probar directamente). También es fácil ver que la imagen de\(f\) es todo el intervalo\((0,\infty)\). Obtenemos un inverso único\(g\) y así la existencia y singularidad de\(n\) raíces positivas. Aplicamos para obtener el derivado.

    El corolario proporciona un buen ejemplo de donde el teorema de la función inversa nos da un intervalo menor que\((a,b)\). Toma\(f \colon {\mathbb{R}}\to {\mathbb{R}}\) definida por\(f(x) := x^2\). Entonces\(f'(x) \not= 0\), siempre y cuando\(x \not= 0\). Si\(x_0 > 0\), podemos tomar\(I=(0,\infty)\), pero no más grande.

    Otro ejemplo útil es\(f(x) := x^3\). La función\(f \colon {\mathbb{R}}\to {\mathbb{R}}\) es uno a uno y on, por lo que\(f^{-1}(x) = x^{1/3}\) existe en toda la línea real incluyendo cero y negativo\(x\). La función\(f\) tiene una derivada continua, pero no\(f^{-1}\) tiene derivada en el origen. El punto es que\(f'(0) = 0\). Ver también.

    Ejercicios

    Supongamos que\(f \colon {\mathbb{R}}\to {\mathbb{R}}\) es continuamente diferenciable de tal manera que\(f'(x) > 0\) para todos\(x\). Mostrar que\(f\) es invertible en el intervalo\(J = f({\mathbb{R}})\), la inversa es continuamente diferenciable, y\({(f^{-1})}'(y) > 0\) para todos\(y \in f({\mathbb{R}})\).

    Supongamos que\(I,J\) son intervalos y un monótona sobre\(f \colon I \to J\) tiene una inversa\(g \colon J \to I\). Supongamos que ya sabe que ambos\(f\) y\(g\) son diferenciables en todas partes y nunca\(f'\) es cero. Usando regla de cadena pero no probar la fórmula\(g'(y) = \nicefrac{1}{f'\bigl(g(y)\bigr)}\).

    \(n\in {\mathbb{N}}\)Déjese igualar. Demostrar que cada uno\(x > 0\) tiene una raíz única negativa\(n\) th. Es decir, existe un número negativo\(y\) tal que\(y^n = x\). Calcula la derivada de la función\(g(x) := y\).

    [ejercicio:oddroot] Dejar\(n \in {\mathbb{N}}\) ser impar y\(n \geq 3\). Demostrar que cada uno\(x\) tiene una\(n\) raíz única. Es decir, existe un número\(y\) tal que\(y^n = x\). Demostrar que la función definida por\(g(x) := y\) es diferenciable excepto en\(x=0\) y computar la derivada. Demostrar que no\(g\) es diferenciable en\(x=0\).

    § 3] Mostrar que si en el teorema de la función inversa\(f\) tiene derivadas\(k\) continuas, entonces la función inversa\(g\) también tiene derivadas\(k\) continuas.

    Dejar\(f(x) := x + 2 x^2 \sin(\nicefrac{1}{x})\) para\(x \not= 0\) y\(f(0) = 0\). Mostrar que\(f\) es diferenciable en absoluto\(x\), eso\(f'(0) > 0\), pero eso no\(f\) es invertible en ningún intervalo que contenga el origen.

    a) Dejar\(f \colon {\mathbb{R}}\to {\mathbb{R}}\) ser una función continuamente diferenciable y\(k > 0\) ser un número tal que\(f'(x) \geq k\) para todos\(x \in {\mathbb{R}}\). Show\(f\) es uno a uno y onto, y tiene una inversa continuamente diferenciable\(f^{-1} \colon {\mathbb{R}}\to {\mathbb{R}}\). b) Encuentra un ejemplo\(f \colon {\mathbb{R}}\to {\mathbb{R}}\) donde\(f'(x) > 0\) para todos\(x\), pero no\(f\) es on.

    Supongamos que\(I,J\) son intervalos y un monótona sobre\(f \colon I \to J\) tiene una inversa\(g \colon J \to I\). Supongamos\(x \in I\) y\(y := f(x) \in J\), y eso\(g\) es diferenciable en\(y\). Demostrar:
    a) Si\(g'(y) \not= 0\), entonces\(f\) es diferenciable en\(x\).
    b) Si\(g'(y) = 0\), entonces no\(f\) es diferenciable en\(x\).

    La Integral de Riemann

    La integral de Riemann

    Nota: 1.5 conferencias

    Ahora llegamos al concepto fundamental de integración. A menudo existe confusión entre los estudiantes de cálculo entre integral y antiderivado. La integral es (informalmente) el área bajo la curva, nada más. Que podamos calcular un antiderivado usando la integral es un resultado no trivial que tenemos que probar. En este capítulo definimos la integral de Riemann 20 utilizando la integral Darboux 21, la cual es técnicamente más simple que (pero equivalente a) la definición tradicional que hace Riemann.

    Particiones e integrales inferiores y superiores

    Queremos integrar una función delimitada definida en un intervalo\([a,b]\). Primero definimos dos integrales auxiliares que se pueden definir para todas las funciones delimitadas. Sólo entonces podremos hablar de la integral de Riemann y de las funciones integrables de Riemann.

    Una partición\(P\) del intervalo\([a,b]\) es un conjunto finito de números\(\{ x_0,x_1,x_2,\ldots,x_n \}\) tal que\[a = x_0 < x_1 < x_2 < \cdots < x_{n-1} < x_n = b .\] Escribimos\[\Delta x_i := x_i - x_{i-1} .\]

    Dejar\(f \colon [a,b] \to {\mathbb{R}}\) ser una función acotada. Dejar\(P\) ser una partición de\([a,b]\). Definir\[\begin{aligned} & m_i := \inf \{ f(x) : x_{i-1} \leq x \leq x_i \} , \\ & M_i := \sup \{ f(x) : x_{i-1} \leq x \leq x_i \} , \\ & L(P,f) := \sum_{i=1}^n m_i \Delta x_i , \\ & U(P,f) := \sum_{i=1}^n M_i \Delta x_i .\end{aligned}\] Llamamos a\(L(P,f)\) la suma inferior de Darboux y\(U(P,f)\) a la suma superior de Darboux.

    La idea geométrica de las sumas Darboux se indica en. La suma inferior es el área de los rectángulos sombreados, y la suma superior es el área de los rectángulos enteros, sombreados más partes sin sombrear. El ancho del rectángulo\(i\) th es\(\Delta x_i\), la altura del rectángulo sombreado es\(m_i\) y la altura de todo el rectángulo es\(M_i\).

    [sumulbound:prop] Dejar\(f \colon [a,b] \to {\mathbb{R}}\) ser una función acotada. Seamos tales\(m, M \in {\mathbb{R}}\) que por todo\(x\) lo que tenemos\(m \leq f(x) \leq M\). Para cualquier partición\(P\) de\([a,b]\) tenemos\[\label{sumulbound:eq} m(b-a) \leq L(P,f) \leq U(P,f) \leq M(b-a) .\]

    Dejar\(P\) ser una partición. Entonces tenga en cuenta que\(m \leq m_i\) para todos\(i\) y\(M_i \leq M\) para todos\(i\). También\(m_i \leq M_i\) para todos\(i\). Por último\(\sum_{i=1}^n \Delta x_i = (b-a)\). Por lo\[\begin{gathered} m(b-a) = m \left( \sum_{i=1}^n \Delta x_i \right) = \sum_{i=1}^n m \Delta x_i \leq \sum_{i=1}^n m_i \Delta x_i \leq \\ \leq \sum_{i=1}^n M_i \Delta x_i \leq \sum_{i=1}^n M \Delta x_i = M \left( \sum_{i=1}^n \Delta x_i \right) = M(b-a) .\end{gathered}\] tanto, de ahí obtenemos [sumulbound:eq]. En otras palabras, el conjunto de sumas inferior y superior son conjuntos acotados.

    A medida que se acoplan los conjuntos de sumas inferior y superior de Darboux,\[\begin{aligned} & \underline{\int_a^b} f(x)~dx := \sup \{ L(P,f) : P \text{ a partition of $[a,b]$} \} , \\ & \overline{\int_a^b} f(x)~dx := \inf \{ U(P,f) : P \text{ a partition of $[a,b]$} \} .\end{aligned}\] definimos Nosotros llamamos\(\underline{\int}\) integral inferior de Darboux y\(\overline{\int}\) integral superior de Darboux. Para evitar preocuparnos por la variable de integración, a menudo simplemente escribimos\[\underline{\int_a^b} f := \underline{\int_a^b} f(x)~dx \qquad \text{and} \qquad \overline{\int_a^b} f := \overline{\int_a^b} f(x)~dx .\]

    Si la integración va a tener sentido, entonces las integrales inferior y superior de Darboux deberían ser el mismo número, ya que queremos que un solo número llame a la integral. Sin embargo, estas dos integrales pueden, de hecho, diferir para algunas funciones.

    Toma la función Dirichlet\(f \colon [0,1] \to {\mathbb{R}}\), donde\(f(x) := 1\) si\(x \in {\mathbb{Q}}\) y\(f(x) := 0\) si\(x \notin {\mathbb{Q}}\). Entonces\[\underline{\int_0^1} f = 0 \qquad \text{and} \qquad \overline{\int_0^1} f = 1 .\] La razón es que por cada\(i\) que tenemos\(m_i = \inf \{ f(x) : x \in [x_{i-1},x_i] \} = 0\) y\(M_i = \sup \{ f(x) : x \in [x_{i-1},x_i] \} = 1\). Así\[\begin{aligned} & L(P,f) = \sum_{i=1}^n 0 \cdot \Delta x_i = 0 , \\ & U(P,f) = \sum_{i=1}^n 1 \cdot \Delta x_i = \sum_{i=1}^n \Delta x_i = 1 .\end{aligned}\]

    La misma definición de\(\underline{\int_a^b} f\) y\(\overline{\int_a^b} f\) se usa cuando\(f\) se define en un conjunto más grande\(S\) tal que\([a,b] \subset S\). En ese caso, utilizamos la restricción de\(f\) a\([a,b]\) y debemos asegurarnos de que la restricción esté acotada en\([a,b]\).

    Para calcular la integral a menudo tomamos una partición\(P\) y la hacemos más fina. Es decir, cortamos intervalos en la partición en trozos aún más pequeños.

    Dejar\(P = \{ x_0, x_1, \ldots, x_n \}\) y\(\widetilde{P} = \{ \widetilde{x}_0, \widetilde{x}_1, \ldots, \widetilde{x}_m \}\) ser particiones de\([a,b]\). Decimos que\(\widetilde{P}\) es un refinamiento de\(P\) si como conjuntos\(P \subset \widetilde{P}\).

    Es decir,\(\widetilde{P}\) es un refinamiento de una partición si contiene todos los números en\(P\) y quizás algunos otros números intermedios. Por ejemplo,\(\{ 0, 0.5, 1, 2 \}\) es una partición de\([0,2]\) y\(\{ 0, 0.2, 0.5, 1, 1.5, 1.75, 2 \}\) es un refinamiento. La razón principal para introducir refinamientos es la siguiente propuesta.

    [prop:refinement] Dejar\(f \colon [a,b] \to {\mathbb{R}}\) ser una función acotada, y dejar\(P\) ser una partición de\([a,b]\). Que\(\widetilde{P}\) sea un refinamiento de\(P\). Entonces\[L(P,f) \leq L(\widetilde{P},f) \qquad \text{and} \qquad U(\widetilde{P},f) \leq U(P,f) .\]

    La parte complicada de esta prueba es conseguir que la notación sea correcta. Que\(\widetilde{P} := \{ \widetilde{x}_0, \widetilde{x}_1, \ldots, \widetilde{x}_m \}\) sea un refinamiento de\(P := \{ x_0, x_1, \ldots, x_n \}\). Entonces\(x_0 = \widetilde{x}_0\) y\(x_n = \widetilde{x}_m\). De hecho, podemos encontrar enteros\(k_0 < k_1 < \cdots < k_n\) tales que\(x_j = \widetilde{x}_{k_j}\) para\(j=0,1,2,\ldots,n\).

    Vamos\(\Delta \widetilde{x}_j = \widetilde{x}_{j-1} - \widetilde{x}_j\). Obtenemos\[\Delta x_j = \sum_{p=k_{j-1}+1}^{k_j} \Delta \widetilde{x}_p .\]

    Dejar\(m_j\) ser como antes y corresponder a la partición\(P\). Vamos\(\widetilde{m}_j := \inf \{ f(x) : \widetilde{x}_{j-1} \leq x \leq \widetilde{x}_j \}\). Ahora,\(m_j \leq \widetilde{m}_p\) para\(k_{j-1} < p \leq k_j\). Por lo\[m_j \Delta x_j = m_j \sum_{p=k_{j-1}+1}^{k_j} \Delta \widetilde{x}_p = \sum_{p=k_{j-1}+1}^{k_j} m_j \Delta \widetilde{x}_p \leq \sum_{p=k_{j-1}+1}^{k_j} \widetilde{m}_p \Delta \widetilde{x}_p .\] tanto,\[L(P,f) = \sum_{j=1}^n m_j \Delta x_j \leq \sum_{j=1}^n \sum_{p=k_{j-1}+1}^{k_j} \widetilde{m}_p \Delta \widetilde{x}_p = \sum_{j=1}^m \widetilde{m}_j \Delta \widetilde{x}_j = L(\widetilde{P},f).\]

    El comprobante de\(U(\widetilde{P},f) \leq U(P,f)\) se deja como ejercicio.

    Armados con refinamientos demostramos lo siguiente. El punto clave de esta proposición siguiente es que la integral inferior de Darboux es menor o igual que la integral superior de Darboux.

    [intulbound:prop] Dejar\(f \colon [a,b] \to {\mathbb{R}}\) ser una función acotada. Seamos tales\(m, M \in {\mathbb{R}}\) que por todo\(x \in [a,b]\) lo que tenemos\(m \leq f(x) \leq M\). Entonces\[\label{intulbound:eq} m(b-a) \leq \underline{\int_a^b} f \leq \overline{\int_a^b} f \leq M(b-a) .\]

    Por tenemos para cualquier partición\(P\)\[m(b-a) \leq L(P,f) \leq U(P,f) \leq M(b-a).\] La desigualdad\(m(b-a) \leq L(P,f)\) implica\(m(b-a) \leq \underline{\int_a^b} f\). También\(U(P,f) \leq M(b-a)\) implica\(\overline{\int_a^b} f \leq M(b-a)\).

    El punto clave de esta proposición es la desigualdad media en [intulbound:eq]. Dejar\(P_1, P_2\) ser particiones de\([a,b]\). Definir\(\widetilde{P} := P_1 \cup P_2\). El conjunto\(\widetilde{P}\) es una partición de\([a,b]\). Además,\(\widetilde{P}\) es un refinamiento de\(P_1\) y también es un refinamiento de\(P_2\). Por tenemos\(L(P_1,f) \leq L(\widetilde{P},f)\) y\(U(\widetilde{P},f) \leq U(P_2,f)\). Poniéndolo todo junto tenemos\[L(P_1,f) \leq L(\widetilde{P},f) \leq U(\widetilde{P},f) \leq U(P_2,f) .\] En otras palabras, para dos particiones arbitrarias\(P_1\) y\(P_2\) tenemos\(L(P_1,f) \leq U(P_2,f)\). Ahora recordamos. Tomando lo supremo y el infimum sobre todas las particiones obtenemos\[\sup \{ L(P,f) : \text{$P$ a partition of $[a,b]$} \} \leq \inf \{ U(P,f) : \text{$P$ a partition of $[a,b]$} \} .\] En otras palabras\(\underline{\int_a^b} f \leq \overline{\int_a^b} f\).

    Integral de Riemann

    Finalmente podemos definir la integral de Riemann. Sin embargo, la integral de Riemann solo se define en una cierta clase de funciones, llamadas funciones integrables de Riemann.

    Que\(f \colon [a,b] \to {\mathbb{R}}\) sea una función acotada tal que\[\underline{\int_a^b} f(x)~dx = \overline{\int_a^b} f(x)~dx .\] Entonces\(f\) se dice que es Riemann integrable. El conjunto de funciones integrables de Riemann en\([a,b]\) se denota por\({\mathcal{R}}[a,b]\). Cuando\(f \in {\mathcal{R}}[a,b]\) definimos\[\int_a^b f(x)~dx := \underline{\int_a^b} f(x)~dx = \overline{\int_a^b} f(x)~dx .\] Como antes, muchas veces simplemente escribimos\[\int_a^b f := \int_a^b f(x)~dx.\] El número\(\int_a^b f\) se llama la integral de Riemann\(f\), o a veces simplemente la integral de\(f\).

    Por definición, cualquier función integrable de Riemann está delimitada. Al apelar a obtenemos de inmediato la siguiente proposición.

    [intbound:prop] Dejar\(f \colon [a,b] \to {\mathbb{R}}\) ser una función integrable de Riemann. Seamos\(m, M \in {\mathbb{R}}\) tal que\(m \leq f(x) \leq M\) para todos\(x \in [a,b]\). Entonces\[m(b-a) \leq \int_a^b f \leq M(b-a) .\]

    A menudo usamos una forma más débil de esta proposición. Es decir, si es\(\left\lvert {f(x)} \right\rvert \leq M\) por todos\(x \in [a,b]\), entonces\[\left\lvert {\int_a^b f} \right\rvert \leq M(b-a) .\]

    Integramos funciones constantes utilizando. Si\(f(x) := c\) por alguna constante\(c\), entonces tomamos\(m = M = c\). En la desigualdad [intulbound:eq] todas las desigualdades deben ser igualdades. Por lo tanto,\(f\) es integrable en\([a,b]\) y\(\int_a^b f = c(b-a)\).

    \(f \colon [0,2] \to {\mathbb{R}}\)Déjese definir por\[f(x) := \begin{cases} 1 & \text{ if $x < 1$,}\\ \nicefrac{1}{2} & \text{ if $x = 1$,}\\ 0 & \text{ if $x > 1$.} \end{cases}\] Afirmamos\(f\) es Riemann integrable y\(\int_0^2 f = 1\).

    Prueba:\(0 < \epsilon < 1\) Sea arbitrario. Dejar\(P := \{0, 1-\epsilon, 1+\epsilon, 2\}\) ser una partición. Utilizamos la notación de la definición de las sumas Darboux. Entonces\[\begin{aligned} m_1 &= \inf \{ f(x) : x \in [0,1-\epsilon] \} = 1 , & M_1 &= \sup \{ f(x) : x \in [0,1-\epsilon] \} = 1 , \\ m_2 &= \inf \{ f(x) : x \in [1-\epsilon,1+\epsilon] \} = 0 , & M_2 &= \sup \{ f(x) : x \in [1-\epsilon,1+\epsilon] \} = 1 , \\ m_3 &= \inf \{ f(x) : x \in [1+\epsilon,2] \} = 0 , & M_3 &= \sup \{ f(x) : x \in [1+\epsilon,2] \} = 0 .\end{aligned}\] Además,\(\Delta x_1 = 1-\epsilon\),\(\Delta x_2 = 2\epsilon\) y\(\Delta x_3 = 1-\epsilon\). Nosotros computamos\[\begin{aligned} & L(P,f) = \sum_{i=1}^3 m_i \Delta x_i = 1 \cdot (1-\epsilon) + 0 \cdot 2\epsilon + 0 \cdot (1-\epsilon) = 1-\epsilon , \\ & U(P,f) = \sum_{i=1}^3 M_i \Delta x_i = 1 \cdot (1-\epsilon) + 1 \cdot 2\epsilon + 0 \cdot (1-\epsilon) = 1+\epsilon .\end{aligned}\] Así,\[\overline{\int_0^2} f - \underline{\int_0^2} f \leq U(P,f) - L(P,f) = (1+\epsilon) - (1-\epsilon) = 2 \epsilon .\] Por tenemos\(\underline{\int_0^2} f \leq \overline{\int_0^2} f\). Como\(\epsilon\) fue arbitrario vemos\(\overline{\int_0^2} f = \underline{\int_0^2} f\). Así\(f\) es Riemann integrable. Por último,\[1-\epsilon = L(P,f) \leq \int_0^2 f \leq U(P,f) = 1+\epsilon.\] De ahí,\(\bigl\lvert \int_0^2 f - 1 \bigr\rvert \leq \epsilon\). Como\(\epsilon\) fue arbitrario, tenemos\(\int_0^2 f = 1\).

    Puede valer la pena extraer parte de la técnica del ejemplo en una proposición.

    Dejar\(f \colon [a,b] \to {\mathbb{R}}\) ser una función acotada. Entonces Riemann\(f\) es integrable si por cada\(\epsilon > 0\), existe una partición\(P\) tal que\[U(P,f) - L(P,f) < \epsilon .\]

    Si por cada\(\epsilon > 0\), a\(P\) existe tenemos:\[0 \leq \overline{\int_a^b} f - \underline{\int_a^b} f \leq U(P,f) - L(P,f) < \epsilon .\] Por lo tanto,\(\overline{\int_a^b} f = \underline{\int_a^b} f\), y\(f\) es integrable.

    Vamos a mostrar\(\frac{1}{1+x}\) es integrable en\([0,b]\) para cualquier\(b > 0\). Veremos más adelante que todas las funciones continuas son integrables, pero vamos a demostrar cómo lo hacemos directamente.

    Dejemos\(\epsilon > 0\) que se den. Tomar\(n \in {\mathbb{N}}\) y recoger\(x_j := \nicefrac{jb}{n}\), para formar la partición\(P := \{ x_0,x_1,\ldots,x_n \}\) de\([0,b]\). Tenemos\(\Delta x_j = \nicefrac{b}{n}\) para todos\(j\). Como\(f\) es decreciente, para cualquier subintervalo\([x_{j-1},x_j]\) obtenemos\[m_j = \inf \left\{ \frac{1}{1+x} : x \in [x_{j-1},x_j] \right\} = \frac{1}{1+x_j} , \qquad M_j = \sup \left\{ \frac{1}{1+x} : x \in [x_{j-1},x_j] \right\} = \frac{1}{1+x_{j-1}} .\] Entonces tenemos\[\begin{gathered} U(P,f)-L(P,f) = \sum_{j=1}^n \Delta x_j (M_j-m_j) = \\ = \frac{b}{n} \sum_{j=1}^n \left( \frac{1}{1+\nicefrac{(j-1)b}{n}} - \frac{1}{1+\nicefrac{jb}{n}} \right) = \frac{b}{n} \left( \frac{1}{1+\nicefrac{0b}{n}} - \frac{1}{1+\nicefrac{nb}{n}} \right) = \frac{b^2}{n(b+1)} .\end{gathered}\] La suma telescopios, los términos se cancelan sucesivamente entre sí, algo que hemos visto antes. Picking\(n\) para ser tal que\(\frac{b^2}{n(b+1)} < \epsilon\) la propuesta esté satisfecha y la función sea integrable.

    Más notación

    Cuando\(f \colon S \to {\mathbb{R}}\) se define en un conjunto más grande\(S\) y\([a,b] \subset S\), decimos\(f\) es Riemann integrable en\([a,b]\) si la restricción de\(f\) a\([a,b]\) es Riemann integrable. En este caso, decimos\(f \in {\mathcal{R}}[a,b]\), y escribimos\(\int_a^b f\) para significar la integral Riemann de la restricción de\(f\) a\([a,b]\).

    Es útil definir la integral\(\int_a^b f\) aunque\(a \not< b\). Supongamos\(b < a\) y\(f \in {\mathcal{R}}[b,a]\), luego definimos\[\int_a^b f := - \int_b^a f .\] Para cualquier función\(f\) definimos\[\int_a^a f := 0 .\]

    En ocasiones, la variable\(x\) puede que ya tenga algún otro significado. Cuando necesitamos anotar la variable de integración, podemos simplemente usar una letra diferente. Por ejemplo,\[\int_a^b f(s)~ds := \int_a^b f(x)~dx .\]

    Ejercicios

    Dejar\(f \colon [0,1] \to {\mathbb{R}}\) ser definido por\(f(x) := x^3\) y dejar\(P := \{ 0, 0.1, 0.4, 1 \}\). Cómputos\(L(P,f)\) y\(U(P,f)\).

    Dejar\(f \colon [0,1] \to {\mathbb{R}}\) ser definido por\(f(x) := x\). \(f \in {\mathcal{R}}[0,1]\)Demostrarlo y computar\(\int_0^1 f\) usando la definición de la integral (pero siéntase libre de usar las proposiciones de esta sección).

    Dejar\(f \colon [a,b] \to {\mathbb{R}}\) ser una función acotada. Supongamos que existe una secuencia de particiones\(\{ P_k \}\) de\([a,b]\) tal manera que\[\lim_{k \to \infty} \bigl( U(P_k,f) - L(P_k,f) \bigr) = 0 .\] Show que\(f\) es Riemann integrable y que\[\int_a^b f = \lim_{k \to \infty} U(P_k,f) = \lim_{k \to \infty} L(P_k,f) .\]

    Terminar la prueba de.

    Supongamos que\(f \colon [-1,1] \to {\mathbb{R}}\) se define como\[f(x) := \begin{cases} 1 & \text{ if $x > 0$,} \\ 0 & \text{ if $x \leq 0$.} \end{cases}\] Probar eso\(f \in {\mathcal{R}}[-1,1]\) y computar\(\int_{-1}^1 f\) usando la definición de la integral (pero siéntase libre de usar las proposiciones de esta sección).

    Dejar\(c \in (a,b)\) y dejar\(d \in {\mathbb{R}}\). Definir\(f \colon [a,b] \to {\mathbb{R}}\) como\[f(x) := \begin{cases} d & \text{ if $x = c$,} \\ 0 & \text{ if $x \not= c$.} \end{cases}\] Demostrarlo\(f \in {\mathcal{R}}[a,b]\) y computar\(\int_a^b f\) usando la definición de la integral (pero siéntase libre de usar las proposiciones de esta sección).

    [exercise:taggedpartition] Supongamos que Riemann\(f \colon [a,b] \to {\mathbb{R}}\) es integrable. Dejemos\(\epsilon > 0\) que se den. Entonces muestra que existe una partición\(P = \{ x_0, x_1, \ldots, x_n \}\) tal que si escogemos algún conjunto de números\(\{ c_1, c_2, \ldots, c_n \}\) con\(c_k \in [x_{k-1},x_k]\) para todos\(k\), entonces\[\left\lvert {\int_a^b f - \sum_{k=1}^n f(c_k) \Delta x_k} \right\rvert < \epsilon .\]

    Dejar\(f \colon [a,b] \to {\mathbb{R}}\) ser una función integrable de Riemann. Dejar\(\alpha > 0\) y\(\beta \in {\mathbb{R}}\). Después defina\(g(x) := f(\alpha x + \beta)\) en el intervalo\(I = [\frac{a-\beta}{\alpha}, \frac{b-\beta}{\alpha}]\). Demostrar que\(g\) es Riemann integrable en\(I\).

    Supongamos\(f \colon [0,1] \to {\mathbb{R}}\) y\(g \colon [0,1] \to {\mathbb{R}}\) somos tales que por todo\(x \in (0,1]\) lo que tenemos\(f(x) = g(x)\). Supongamos\(f\) que Riemann es integrable. Probar\(g\) es Riemann integrable y\(\int_{0}^1 f = \int_{0}^1 g\).

    Dejar\(f \colon [0,1] \to {\mathbb{R}}\) ser una función acotada. Que\(P_n = \{ x_0,x_1,\ldots,x_n \}\) sea una partición uniforme de\([0,1]\), es decir,\(x_j := \nicefrac{j}{n}\). ¿\(\{ L(P_n,f) \}_{n=1}^\infty\)Siempre es monótona? Sí/No: Probar o encontrar un contraejemplo.

    Para una función delimitada\(f \colon [0,1] \to {\mathbb{R}}\) let\(R_n := (\nicefrac{1}{n})\sum_{j=1}^n f(\nicefrac{j}{n})\) (la regla uniforme de la mano derecha). a) Si\(f\) es Riemann integrable mostrar\(\int_0^1 f = \lim \, R_n\). b) Encontrar un\(f\) que no sea Riemann integrable, pero que\(\lim \, R_n\) exista.

    [exercise:riemannintdarboux] Generalizar el ejercicio anterior. Demostrar que\(f \in {\mathcal{R}}[a,b]\) si y sólo si existe una\(I \in {\mathbb{R}}\), tal que por cada\(\epsilon > 0\) existe una\(\delta > 0\) tal que si\(P\) es una partición con\(\Delta x_i < \delta\) para todos\(i\), entonces\(\left\lvert {L(P,f) - I} \right\rvert < \epsilon\) y\(\left\lvert {U(P,f) - I} \right\rvert < \epsilon\). Si\(f \in {\mathcal{R}}[a,b]\), entonces\(I = \int_a^b f\).

    [exercise:riemannintdarboux2] El uso y la idea de la prueba en, muestran que la integral de Darboux es la misma que la definición estándar de integral de Riemann, que probablemente hayas visto en el cálculo. Es decir, mostrar que\(f \in {\mathcal{R}}[a,b]\) si y sólo si existe una\(I \in {\mathbb{R}}\), tal que por cada\(\epsilon > 0\) existe una\(\delta > 0\) tal que si\(P = \{ x_0,x_1,\ldots,x_n \}\) es una partición con\(\Delta x_i < \delta\) para todos\(i\), entonces\(\left\lvert {\sum_{i=1}^n f(c_i) \Delta x_i - I} \right\rvert < \epsilon\) para cualquier conjunto\(\{ c_1,c_2,\ldots,c_n \}\) con\(c_i \in [x_{i-1},x_i]\). Si\(f \in {\mathcal{R}}[a,b]\), entonces\(I = \int_a^b f\).

    Encuentra un ejemplo de funciones\(f \colon [0,1] \to {\mathbb{R}}\) que es Riemann integrable, y\(g \colon [0,1] \to [0,1]\) que es uno a uno y sobre, tal que la composición no\(f \circ g\) es Riemann integrable.

    Propiedades de la integral

    Nota: 2 conferencias, la integrabilidad de funciones con discontinuidades se puede omitir de manera segura

    Aditividad

    El siguiente resultado que probamos generalmente se conoce como la propiedad aditiva de la integral. Primero probamos la propiedad de aditividad para las integrales inferiores y superiores de Darboux.

    [lemma:darbouxadd] Supongamos\(a < b < c\) y\(f \colon [a,c] \to {\mathbb{R}}\) es una función acotada. Entonces\[\underline{\int_a^c} f = \underline{\int_a^b} f + \underline{\int_b^c} f\] y\[\overline{\int_a^c} f = \overline{\int_a^b} f + \overline{\int_b^c} f .\]

    Si tenemos particiones\(P_1 = \{ x_0,x_1,\ldots,x_k \}\) de\([a,b]\) y\(P_2 = \{ x_k, x_{k+1}, \ldots, x_n \}\) de\([b,c]\), entonces el conjunto\(P := P_1 \cup P_2 = \{ x_0, x_1, \ldots, x_n \}\) es una partición de\([a,c]\). Entonces\[L(P,f) = \sum_{j=1}^n m_j \Delta x_j = \sum_{j=1}^k m_j \Delta x_j + \sum_{j=k+1}^n m_j \Delta x_j = L(P_1,f) + L(P_2,f) .\] cuando tomamos el supremo del lado derecho sobre todo\(P_1\) y\(P_2\), estamos tomando un supremo del lado izquierdo sobre todas las particiones\(P\) de\([a,c]\) que contienen\(b\). Si\(Q\) hay alguna partición de\([a,c]\) y\(P = Q \cup \{ b \}\), entonces\(P\) es un refinamiento de\(Q\) y así\(L(Q,f) \leq L(P,f)\). Por lo tanto, tomar un supremo sólo sobre los\(P\) que contienen\(b\) es suficiente para encontrar el supremo de\(L(P,f)\) sobre todas las particiones\(P\), ver. Finalmente recordar a computar\[\begin{split} \underline{\int_a^c} f & = \sup \{ L(P,f) : \text{$P$ a partition of $[a,c]$} \} \\ & = \sup \{ L(P,f) : \text{$P$ a partition of $[a,c]$, $b \in P$} \} \\ & = \sup \{ L(P_1,f) + L(P_2,f) : \text{$P_1$ a partition of $[a,b]$, $P_2$ a partition of $[b,c]$} \} \\ & = \sup \{ L(P_1,f) : \text{$P_1$ a partition of $[a,b]$} \} + \sup \{ L(P_2,f) : \text{$P_2$ a partition of $[b,c]$} \} \\ &= \underline{\int_a^b} f + \underline{\int_b^c} f . \end{split}\]

    De igual manera\(P\), para\(P_1\),, y\(P_2\) como antes\[U(P,f) = \sum_{j=1}^n M_j \Delta x_j = \sum_{j=1}^k M_j \Delta x_j + \sum_{j=k+1}^n M_j \Delta x_j = U(P_1,f) + U(P_2,f) .\] obtenemos Deseamos tomar el infimum a la derecha sobre todo\(P_1\) y\(P_2\), y así estamos tomando el infimum sobre todas las particiones\(P\) de\([a,c]\) que contienen\(b\). Si\(Q\) hay alguna partición de\([a,c]\) y\(P = Q \cup \{ b \}\), entonces\(P\) es un refinamiento de\(Q\) y así\(U(Q,f) \geq U(P,f)\). Por lo tanto, tomar un infimum sólo sobre el\(P\) que contienen\(b\) es suficiente para encontrar el infimum de\(U(P,f)\) para todos\(P\). Obtenemos\[\overline{\int_a^c} f = \overline{\int_a^b} f + \overline{\int_b^c} f . \qedhere\]

    Vamos\(a < b < c\). Una función\(f \colon [a,c] \to {\mathbb{R}}\) es Riemann integrable si y solo si Riemann\(f\) es integrable en\([a,b]\) y\([b,c]\). Si Riemann\(f\) es integrable, entonces\[\int_a^c f = \int_a^b f + \int_b^c f .\]

    Supongamos\(f \in {\mathcal{R}}[a,c]\), entonces\(\overline{\int_a^c} f = \underline{\int_a^c} f = \int_a^c f\). Aplicamos el lema para obtener\[\int_a^c f = \underline{\int_a^c} f = \underline{\int_a^b} f + \underline{\int_b^c} f \leq \overline{\int_a^b} f + \overline{\int_b^c} f = \overline{\int_a^c} f = \int_a^c f .\] Así la desigualdad es una igualdad y\[\underline{\int_a^b} f + \underline{\int_b^c} f = \overline{\int_a^b} f + \overline{\int_b^c} f .\] como también sabemos\(\underline{\int_a^b} f \leq \overline{\int_a^b} f\) y\(\underline{\int_b^c} f \leq \overline{\int_b^c} f\), concluimos\[\underline{\int_a^b} f = \overline{\int_a^b} f \qquad \text{and} \qquad \underline{\int_b^c} f = \overline{\int_b^c} f .\] Así\(f\) es Riemann integrable en\([a,b]\) y\([b,c]\) y la fórmula deseada sostiene.

    Ahora asuma las restricciones de\(f\) a\([a,b]\) y a\([b,c]\) son Riemann integrables. Nuevamente aplicamos el lema para obtener\[\underline{\int_a^c} f = \underline{\int_a^b} f + \underline{\int_b^c} f = \int_a^b f + \int_b^c f = \overline{\int_a^b} f + \overline{\int_b^c} f = \overline{\int_a^c} f .\] Por lo tanto, Riemann\(f\) es integrable en\([a,c]\), y la integral se calcula como se indica.

    Una consecuencia fácil de la aditividad es el siguiente corolario. Dejamos los detalles al lector como ejercicio.

    [intsubcor] Si\(f \in {\mathcal{R}}[a,b]\) y\([c,d] \subset [a,b]\), entonces la restricción\(f|_{[c,d]}\) está en\({\mathcal{R}}[c,d]\).

    Linealidad y monotonicidad

    Dejar\(f\) y\(g\) estar dentro\({\mathcal{R}}[a,b]\) y\(\alpha \in {\mathbb{R}}\).

    1. \(\alpha f\)está en\({\mathcal{R}}[a,b]\) y\[\int_a^b \alpha f(x) ~dx = \alpha \int_a^b f(x) ~dx .\]
    2. \(f+g\)está en\({\mathcal{R}}[a,b]\) y\[\int_a^b \bigl( f(x)+g(x) \bigr) ~dx = \int_a^b f(x) ~dx + \int_a^b g(x) ~dx .\]

    Probemos el primer ítem. Primero supongamos\(\alpha \geq 0\). Dejar\(P\) ser una partición de\([a,b]\). Dejar\(m_i := \inf \{ f(x) : x \in [x_{i-1},x_i] \}\) como de costumbre. Ya que no\(\alpha\) es negativo, podemos mover la multiplicación por más\(\alpha\) allá del infimum,\[\inf \{ \alpha f(x) : x \in [x_{i-1},x_i] \} = \alpha \inf \{ f(x) : x \in [x_{i-1},x_i] \} = \alpha m_i .\] Por lo tanto\[L(P,\alpha f) = \sum_{i=1}^n \alpha m_i \Delta_i = \alpha \sum_{i=1}^n m_i \Delta_i = \alpha L(P,f).\] De manera similar\[U(P,\alpha f) = \alpha U(P,f) .\] Otra vez, ya\(\alpha \geq 0\) que podemos mover multiplicación por\(\alpha\) más allá del supremo. De ahí,\[\begin{split} \underline{\int_a^b} \alpha f(x)~dx & = \sup \{ L(P,\alpha f) : \text{$P$ a partition of $[a,b]$} \} \\ & = \sup \{ \alpha L(P,f) : \text{$P$ a partition of $[a,b]$} \} \\ & = \alpha \sup \{ L(P,f) : \text{$P$ a partition of $[a,b]$} \} \\ & = \alpha \underline{\int_a^b} f(x)~dx . \end{split}\] Del mismo modo mostramos\[\overline{\int_a^b} \alpha f(x)~dx = \alpha \overline{\int_a^b} f(x)~dx .\] La conclusión ahora sigue para\(\alpha \geq 0\).

    Para terminar la prueba del primer ítem, necesitamos demostrar que\(-f\) es Riemann integrable y\(\int_a^b - f(x)~dx = - \int_a^b f(x)~dx\). La prueba de este hecho se deja como ejercicio.

    También se deja como ejercicio el comprobante del segundo ítem de la proposición. No es tan trivial como puede parecer a primera vista.

    Debemos señalar que el segundo ítem de la proposición no se sostiene con igualdad para las integrales de Darboux. Para funciones acotadas arbitrarias\(f\) y solo\(g\) obtenemos\[%\overline{\int_a^b} \bigl(f(x)+g(x)\bigr)~dx \leq %\overline{\int_a^b}f(x)~dx+\overline{\int_a^b}g(x)~dx \overline{\int_a^b} (f+g) \leq \overline{\int_a^b}f+\overline{\int_a^b}g , \qquad \text{and} \qquad \underline{\int_a^b} (f+g) \geq \underline{\int_a^b}f+\underline{\int_a^b}g %\underline{\int_a^b} \bigl(f(x)+g(x)\bigr)~dx \geq %\underline{\int_a^b}f(x)~dx+\underline{\int_a^b}g(x)~dx .\] Ver.

    \(f\)Dejad y\(g\) entrad\({\mathcal{R}}[a,b]\) y\(f(x) \leq g(x)\) dejad para todos\(x \in [a,b]\). Entonces\[\int_a^b f \leq \int_a^b g .\]

    Dejar\(P = \{ x_0, x_1, \ldots, x_n \}\) ser una partición de\([a,b]\). Entonces deja\[m_i := \inf \{ f(x) : x \in [x_{i-1},x_i] \} \qquad \text{and} \qquad \widetilde{m}_i := \inf \{ g(x) : x \in [x_{i-1},x_i] \} .\] As\(f(x) \leq g(x)\), entonces\(m_i \leq \widetilde{m}_i\). Por lo tanto,\[L(P,f) = \sum_{i=1}^n m_i \Delta x_i \leq \sum_{i=1}^n \widetilde{m}_i \Delta x_i = L(P,g) .\] Tomamos lo supremo sobre todo\(P\) (ver) para obtener\[\underline{\int_a^b} f \leq \underline{\int_a^b} g .\] As\(f\) y\(g\) son Riemann integrables, sigue la conclusión.

    Funciones continuas

    Demostremos que las funciones continuas son integrables por Riemann. De hecho mostraremos que incluso podemos permitir algunas discontinuidades. Comenzamos con una función continua en todo el intervalo cerrado\([a,b]\).

    [lemma:contint] Si\(f \colon [a,b] \to {\mathbb{R}}\) es una función continua, entonces\(f \in {\mathcal{R}}[a,b]\).

    Como\(f\) es continuo en un intervalo delimitado cerrado, es uniformemente continuo. Dejemos\(\epsilon > 0\) que se den. Encontrar un\(\delta > 0\) tal que\(\left\lvert {x-y} \right\rvert < \delta\) implique\(\left\lvert {f(x)-f(y)} \right\rvert < \frac{\epsilon}{b-a}\).

    Que\(P = \{ x_0, x_1, \ldots, x_n \}\) sea una partición de\([a,b]\) tal que\(\Delta x_i < \delta\) para todos\(i = 1,2, \ldots, n\). Por ejemplo, toma\(n\) tal que\(\frac{b-a}{n} < \delta\) y deja\(x_i := \frac{i}{n}(b-a) + a\). Entonces por todo lo que\(x, y \in [x_{i-1},x_i]\) tenemos\(\left\lvert {x-y} \right\rvert \leq \Delta x_i < \delta\) y así\[f(x)-f(y) \leq \left\lvert {f(x)-f(y)} \right\rvert < \frac{\epsilon}{b-a} .\] As\(f\) es continuo encendido\([x_{i-1},x_i]\), alcanza un máximo y un mínimo en este intervalo. Dejar\(x\) ser un punto\(f\) donde alcance el máximo y\(y\) ser un punto\(f\) donde alcance el mínimo. Entonces\(f(x) = M_i\) y\(f(y) = m_i\) en la notación a partir de la definición de la integral. Por lo tanto,\[M_i-m_i = f(x)-f(y) < \frac{\epsilon}{b-a} .\] Y así\[\begin{split} \overline{\int_a^b} f - \underline{\int_a^b} f & \leq U(P,f) - L(P,f) \\ & = \left( \sum_{i=1}^n M_i \Delta x_i \right) - \left( \sum_{i=1}^n m_i \Delta x_i \right) \\ & = \sum_{i=1}^n (M_i-m_i) \Delta x_i \\ & < \frac{\epsilon}{b-a} \sum_{i=1}^n \Delta x_i \\ & = \frac{\epsilon}{b-a} (b-a) = \epsilon . \end{split}\] As\(\epsilon > 0\) fue arbitrario,\[\overline{\int_a^b} f = \underline{\int_a^b} f ,\] y\(f\) es Riemann integrable en\([a,b]\).

    El segundo lema dice que necesitamos que la función solo sea “Riemann integrable dentro del intervalo”, siempre y cuando esté acotada. También nos dice cómo computar la integral.

    [lemma:boundedimpriemann] Dejar\(f \colon [a,b] \to {\mathbb{R}}\) ser una función acotada que es Riemann integrable en\([a',b']\) para todos\(a',b'\) tales que\(a < a' < b' < b\). Entonces\(f \in {\mathcal{R}}[a,b]\). Además, si\(a < a_n < b_n < b\) son tales que\(\lim \, a_n = a\) y\(\lim \, b_n = b\), entonces\[\int_a^b f = \lim_{n \to \infty} \int_{a_n}^{b_n} f .\]

    Dejemos\(M > 0\) ser un número real tal que\(\left\lvert {f(x)} \right\rvert \leq M\). Escoge dos secuencias de números de\(a < a_n < b_n < b\) tal manera que\(\lim\, a_n = a\) y\(\lim\, b_n = b\). Nota\(M > 0\) y\((b-a) \geq (b_n-a_n)\). Así\[-M(b-a) \leq -M(b_n-a_n) \leq \int_{a_n}^{b_n} f \leq M(b_n-a_n) \leq M(b-a) .\] pues, la secuencia de números\(\{ \int_{a_n}^{b_n} f \}_{n=1}^\infty\) está delimitada y por tiene una subsecuencia convergente indexada por\(n_k\). Llamemos\(L\) al límite de la subsecuencia\(\{ \int_{a_{n_k}}^{b_{n_k}} f \}_{k=1}^\infty\).

    dice que las integrales inferior y superior son aditivas y la hipótesis dice que\(f\) es integrable en\([a_{n_k},b_{n_k}]\). Por lo tanto\[\underline{\int_a^b} f = \underline{\int_a^{a_{n_k}}} f + \int_{a_{n_k}}^{b_{n_k}} f + \underline{\int_{b_{n_k}}^b} f \geq -M(a_{n_k}-a) + \int_{a_{n_k}}^{b_{n_k}} f - M(b-b_{n_k}) .\] tomamos el límite como\(k\) va a\(\infty\) en el lado derecho,\[\underline{\int_a^b} f \geq -M\cdot 0 + L - M\cdot 0 = L .\]

    A continuación utilizamos la aditividad de la integral superior,\[\overline{\int_a^b} f = \overline{\int_a^{a_{n_k}}} f + \int_{a_{n_k}}^{b_{n_k}} f + \overline{\int_{b_{n_k}}^b} f \leq M(a_{n_k}-a) + \int_{a_{n_k}}^{b_{n_k}} f + M(b-b_{n_k}) .\] tomamos la misma subsecuencia\(\{ \int_{a_{n_k}}^{b_{n_k}} f \}_{k=1}^\infty\) y tomamos el límite para obtener\[\overline{\int_a^b} f \leq M\cdot 0 + L + M\cdot 0 = L .\] Así\(\overline{\int_a^b} f = \underline{\int_a^b} f = L\) y por lo tanto\(f\) es Riemann integrable y\(\int_a^b f = L\). En particular, no importa con qué secuencias\(\{ a_n \}\) y\(\{b_n\}\) empezamos y qué subsecuencia elegimos, la\(L\) es el mismo número.

    Para acreditar la declaración final del lema que utilizamos. Hemos demostrado que cada subsecuencia convergente convergente\(\{ \int_{a_{n_k}}^{b_{n_k}} f \}\) converge a\(L = \int_a^b f\). Por lo tanto, la secuencia\(\{ \int_{a_n}^{b_n} f \}\) es convergente y converge a\(L\).

    Decimos que una función\(f \colon [a,b] \to {\mathbb{R}}\) tiene finitamente muchas discontinuidades si existe un conjunto finito\(S := \{ x_1, x_2, \ldots, x_n \} \subset [a,b]\), y\(f\) es continua en todos los puntos de\([a,b] \setminus S\).

    \(f \colon [a,b] \to {\mathbb{R}}\)Sea una función acotada con finitamente muchas discontinuidades. Entonces\(f \in {\mathcal{R}}[a,b]\).

    Dividimos el intervalo en finitamente muchos intervalos\([a_i,b_i]\) para que\(f\) sea continuo en el interior\((a_i,b_i)\). Si\(f\) es continuo encendido\((a_i,b_i)\), entonces es continuo y por lo tanto integrable en\([c_i,d_i]\) cualquier momento\(a_i < c_i < d_i < b_i\). Por la restricción de\(f\) a\([a_i,b_i]\) es integrable. Por aditividad de la integral (y)\(f\) es integrable en la unión de los intervalos.

    A veces es conveniente (o necesario) cambiar ciertos valores de una función y luego integrarse. El siguiente resultado dice que si cambiamos los valores sólo en finitamente muchos puntos, la integral no cambia.

    Que\(f \colon [a,b] \to {\mathbb{R}}\) sea Riemann integrable. Dejar\(g \colon [a,b] \to {\mathbb{R}}\) ser una función tal que\(f(x) = g(x)\) para todos\(x \in [a,b] \setminus S\), donde\(S\) es un conjunto finito. Entonces\(g\) es una función integrable de Riemann y\[\int_a^b g = \int_a^b f.\]

    Usando la aditividad de la integral, dividimos el intervalo\([a,b]\) en intervalos más pequeños de tal manera que se\(f(x) = g(x)\) mantiene para todos\(x\) excepto en los puntos finales (los detalles se dejan al lector).

    Por lo tanto, sin pérdida de generalidad supongamos\(f(x) = g(x)\) para todos\(x \in (a,b)\). A la prueba le sigue, y se deja como ejercicio.

    Ejercicios

    \(f\)Déjese entrar\({\mathcal{R}}[a,b]\). Demostrar que\(-f\) está en\({\mathcal{R}}[a,b]\) y\[\int_a^b - f(x) ~dx = - \int_a^b f(x) ~dx .\]

    Deja\(f\) y\(g\) entra\({\mathcal{R}}[a,b]\). Demostrar que\(f+g\) está en\({\mathcal{R}}[a,b]\) y\[\int_a^b \bigl( f(x)+g(x) \bigr) ~dx = \int_a^b f(x) ~dx + \int_a^b g(x) ~dx .\] Pista: Utilícelo para encontrar una sola partición\(P\) tal que\(U(P,f)-L(P,f) < \nicefrac{\epsilon}{2}\) y\(U(P,g)-L(P,g) < \nicefrac{\epsilon}{2}\).

    Que\(f \colon [a,b] \to {\mathbb{R}}\) sea Riemann integrable. Que\(g \colon [a,b] \to {\mathbb{R}}\) sea una función tal que\(f(x) = g(x)\) para todos\(x \in (a,b)\). Demostrar que\(g\) es Riemann integrable y que\[\int_a^b g = \int_a^b f.\]

    Demostrar el teorema del valor medio para integrales. Es decir, probar que si\(f \colon [a,b] \to {\mathbb{R}}\) es continuo, entonces existe\(c \in [a,b]\) tal que\(\int_a^b f = f(c)(b-a)\).

    Si\(f \colon [a,b] \to {\mathbb{R}}\) es una función continua tal que\(f(x) \geq 0\) para todos\(x \in [a,b]\) y\(\int_a^b f = 0\). \(f(x) = 0\)Demuéstralo para todos\(x\).

    Si\(f \colon [a,b] \to {\mathbb{R}}\) es una función continua para todos\(x \in [a,b]\) y\(\int_a^b f = 0\). Demostrar que existe\(c \in [a,b]\) tal que\(f(c) = 0\) (Comparar con el ejercicio anterior).

    Si\(f \colon [a,b] \to {\mathbb{R}}\) y\(g \colon [a,b] \to {\mathbb{R}}\) son funciones continuas tales que\(\int_a^b f = \int_a^b g\). Entonces demuéstrense que existe\(c \in [a,b]\) tal eso\(f(c) = g(c)\).

    Vamos\(f \in {\mathcal{R}}[a,b]\). Dejar\(\alpha, \beta, \gamma\) ser números arbitrarios en\([a,b]\) (no necesariamente ordenados de ninguna manera). Demostrar\[\int_\alpha^\gamma f = \int_\alpha^\beta f + \int_\beta^\gamma f .\] Recordar lo\(\int_a^b f\) que significa si\(b \leq a\).

    Demostrar.

    [ejercicio:easyabsint] Supongamos que\(f \colon [a,b] \to {\mathbb{R}}\) está acotado y tiene finitamente muchas discontinuidades. Demostrar que como una función de\(x\) la expresión\(\left\lvert {f(x)} \right\rvert\) está delimitada con finitamente muchas discontinuidades y por lo tanto es Riemann integrable. A continuación, mostrar\[\left\lvert {\int_a^b f(x)~dx} \right\rvert \leq \int_a^b \left\lvert {f(x)} \right\rvert~dx .\]

    Demostrar que la función Thomae o palomitas de maíz (ver) es Riemann integrable. Por lo tanto, existe una función discontinua en todos los números racionales (un conjunto denso) que es Riemann integrable.

    En particular, definir\(f \colon [0,1] \to {\mathbb{R}}\) por\[f(x) := \begin{cases} \nicefrac{1}{k} & \text{ if $x=\nicefrac{m}{k}$ where $m,k \in {\mathbb{N}}$ and $m$ and $k$ have no common divisors,} \\ 0 & \text{ if $x$ is irrational}. \end{cases}\] Show\(\int_0^1 f = 0\).

    Si\(I \subset {\mathbb{R}}\) es un intervalo limitado, entonces la función\[\varphi_I(x) := \begin{cases} 1 & \text{if $x \in I$,} \\ 0 & \text{otherwise,} \end{cases}\] se llama una función de paso elemental.

    Dejar\(I\) ser un intervalo acotado arbitrario (debe considerar todos los tipos de intervalos: cerrado, abierto, semiabierto) y\(a < b\), luego usando solo la definición de la integral mostrar que la función de paso elemental\(\varphi_I\) es integrable\([a,b]\), y encontrar la integral en términos de\(a\), \(b\), y los puntos finales de\(I\).

    Cuando una función se\(f\) puede escribir como\[f(x) = \sum_{k=1}^n \alpha_k \varphi_{I_k} (x)\] para algunos números reales\(\alpha_1,\alpha_2, \ldots, \alpha_n\) y algunos intervalos acotados\(I_1,I_2,\ldots,I_n\), entonces\(f\) se llama una función de paso.

    Usando el ejercicio anterior, mostrar que una función de paso es integrable en cualquier intervalo\([a,b]\). Además, encontrar la integral en términos de\(a\),\(b\), los puntos finales de\(I_k\) y el\(\alpha_k\).

    [exercise:boundedvariationintegrable] Let\(f \colon [a,b] \to {\mathbb{R}}\) be incrementando. a) Demostrar que\(f\) es Riemann integrable. Pista: Usa una partición uniforme; cada subintervalo de la misma longitud. b) Usa la parte a para mostrar que una función decreciente es integrable. c) Supongamos\(h = f-g\) dónde\(f\) y\(g\) están aumentando las funciones\([a,b]\). Demostrar que\(h\) es Riemann integrable 22.

    [exercise:hardabsint] Supongamos\(f \in {\mathcal{R}}[a,b]\), entonces la función que lleva\(x\) a\(\left\lvert {f(x)} \right\rvert\) es también Riemann integrable en\([a,b]\). Entonces mostrar la misma desigualdad que.

    [exercise:upperlowerlinineq] Supongamos\(f \colon [a,b] \to {\mathbb{R}}\) y\(g \colon [a,b] \to {\mathbb{R}}\) son reboteados. a) Mostrar\(\overline{\int_a^b} (f+g) \leq \overline{\int_a^b}f+\overline{\int_a^b}g\) y\(\underline{\int_a^b} (f+g) \geq \underline{\int_a^b}f+\underline{\int_a^b}g\). b) Encontrar ejemplo\(f\) y\(g\) donde la desigualdad es estricta. Pista:\(f\) y no\(g\) debe ser Riemann integrable.

    Teorema fundamental del cálculo

    Nota: 1.5 conferencias

    En este capítulo discutimos y probamos el teorema fundamental del cálculo. La totalidad del cálculo integral se construye sobre este teorema, ergo el nombre. El teorema relata los conceptos aparentemente no relacionados de integral y derivado. Nos dice cómo calcular la antiderivada de una función usando la integral y viceversa.

    Primera forma del teorema

    Dejar\(F \colon [a,b] \to {\mathbb{R}}\) ser una función continua, diferenciable en\((a,b)\). \(f \in {\mathcal{R}}[a,b]\)Sea tal que\(f(x) = F'(x)\) para\(x \in (a,b)\). Entonces\[\int_a^b f = F(b)-F(a) .\]

    No es difícil generalizar el teorema para permitir un número finito de puntos en\([a,b]\) donde no\(F\) es diferenciable, siempre y cuando sea continuo. Esta generalización se deja como ejercicio.

    Dejar\(P = \{ x_0, x_1, \ldots, x_n \}\) ser una partición de\([a,b]\). Para cada intervalo\([x_{i-1},x_i]\), usa el para encontrar un\(c_i \in (x_{i-1},x_i)\) tal que\[f(c_i) \Delta x_i = F'(c_i) (x_i - x_{i-1}) = F(x_i) - F(x_{i-1}) .\] Usando la notación de la definición de la integral, tenemos\(m_i \leq f(c_i) \leq M_i\) y así\[m_i \Delta x_i \leq F(x_i) - F(x_{i-1}) \leq M_i \Delta x_i .\] sumamos\(i = 1,2, \ldots, n\) para obtener\[\sum_{i=1}^n m_i \Delta x_i \leq \sum_{i=1}^n \bigl(F(x_i) - F(x_{i-1}) \bigr) \leq \sum_{i=1}^n M_i \Delta x_i .\] En la suma media, todos los términos excepto el primero y el último cancelan y terminamos con \(F(x_n)-F(x_0) = F(b)-F(a)\). Las sumas a la izquierda y a la derecha son la suma inferior y superior respectivamente. Entonces\[L(P,f) \leq F(b)-F(a) \leq U(P,f) .\] Tomamos lo supremo de\(L(P,f)\) sobre todo\(P\) y la izquierda la desigualdad rinde\[\underline{\int_a^b} f \leq F(b)-F(a) .\] Del mismo modo, tomando el infimum de\(U(P,f)\) sobre todas las particiones\(P\) rendimientos\[F(b)-F(a) \leq \overline{\int_a^b} f .\] Como\(f\) es Riemann integrable, tenemos\[\int_a^b f = \underline{\int_a^b} f \leq F(b)-F(a) \leq \overline{\int_a^b} f = \int_a^b f .\] Las desigualdades deben ser igualdades y ya terminamos.

    El teorema se utiliza para calcular integrales. Supongamos que sabemos que la función\(f(x)\) es una derivada de alguna otra función\(F(x)\), entonces podemos encontrar una expresión explícita para\(\int_a^b f\).

    Supongamos que estamos tratando de computar\[\int_0^1 x^2 ~dx .\] Notamos que\(x^2\) es el derivado de\(\frac{x^3}{3}\). Utilizamos el teorema fundamental para escribir\[\int_0^1 x^2 ~dx = \frac{1^3}{3} - \frac{0^3}{3} = \frac{1}{3}.\]

    Segunda forma del teorema

    La segunda forma del teorema fundamental nos da una manera de resolver la ecuación diferencial\(F'(x) = f(x)\), donde\(f\) se encuentra una función conocida y estamos tratando de encontrar una\(F\) que satisfaga la ecuación.

    Dejar\(f \colon [a,b] \to {\mathbb{R}}\) ser una función integrable de Riemann. Definir\[F(x) := \int_a^x f .\] Primero,\(F\) es continuo en\([a,b]\). Segundo, si\(f\) es continuo en\(c \in [a,b]\), entonces\(F\) es diferenciable en\(c\) y\(F'(c) = f(c)\).

    Como\(f\) está acotado, hay\(M > 0\) tal que\(\left\lvert {f(x)} \right\rvert \leq M\) para todos\(x \in [a,b]\). Supongamos\(x,y \in [a,b]\) con\(x > y\). Entonces\[\left\lvert {F(x)-F(y)} \right\rvert = \left\lvert {\int_a^x f - \int_a^y f} \right\rvert = \left\lvert {\int_y^x f} \right\rvert \leq M\left\lvert {x-y} \right\rvert .\] Por simetría, lo mismo también sostiene si\(x < y\). Así\(F\) es Lipschitz continuo y por lo tanto continuo.

    Ahora supongamos que\(f\) es continuo en\(c\). Dejemos\(\epsilon > 0\) que se den. \(\delta > 0\)Sea tal que por\(x \in [a,b]\)\(\left\lvert {x-c} \right\rvert < \delta\) implica\(\left\lvert {f(x)-f(c)} \right\rvert < \epsilon\). En particular, para tales\(x\) tenemos\[f(c)-\epsilon \leq f(x) \leq f(c) + \epsilon.\] Así si\(x > c\), entonces\[\bigl(f(c)-\epsilon\bigr) (x-c) \leq \int_c^x f \leq \bigl(f(c) + \epsilon\bigr)(x-c).\] Cuando\(c > x\), entonces se invierten las desigualdades. Por lo tanto, asumiendo\[\frac{F(x)-F(c)}{x-c} = \frac{\int_a^{x} f - \int_a^{c} f}{x-c} = \frac{\int_c^{x} f}{x-c} ,\] que\(c \not= x\) obtenemos\[f(c)-\epsilon \leq \frac{\int_c^{x} f}{x-c} \leq f(c)+\epsilon .\] Como tenemos\[\left\lvert {\frac{F(x)-F(c)}{x-c} - f(c)} \right\rvert \leq \epsilon .\] El resultado sigue. Se deja al lector ver por qué está bien que solo tengamos una desigualdad no estricta.

    Por supuesto, si\(f\) es continuo encendido\([a,b]\), entonces es automáticamente Riemann integrable,\(F\) es diferenciable en todos\([a,b]\) y\(F'(x) = f(x)\) para todos\(x \in [a,b]\).

    La segunda forma del teorema fundamental del cálculo aún se mantiene si dejamos\(d \in [a,b]\) y definimos Es\[F(x) := \int_d^x f .\] decir, podemos usar cualquier punto de\([a,b]\) como nuestro punto base. La prueba se deja como ejercicio.

    Veamos lo que puede hacer una simple discontinuidad. Tomar\(f(x) := -1\) si\(x < 0\), y\(f(x) := 1\) si\(x \geq 0\). Vamos\(F(x) := \int_0^x f\). No es difícil verlo\(F(x) = \left\lvert {x} \right\rvert\). Observe que\(f\) es discontinuo en\(0\) y no\(F\) es diferenciable en\(0\). Sin embargo, lo contrario no se sostiene. Hagamos otro ejemplo rápido. Que\(g(x) := 0\) si\(x \not= 0\), y\(g(0) = 1\). Dejando\(G(x) := \int_0^x g\), nos encontramos con eso\(G(x) = 0\) para todos\(x\). Así\(g\) es discontinuo en\(0\), pero\(G'(0)\) existe y es igual a 0.

    Un malentendido común de la integral para los estudiantes de cálculo es pensar en integrales cuya solución no se puede dar en forma cerrada como de alguna manera deficientes. Este no es el caso. La mayoría de las integrales que escribimos no son computables en forma cerrada. Incluso algunas integrales que consideramos en forma cerrada no son realmente tales. Por ejemplo, ¿cómo encuentra una computadora el valor de\(\ln x\)? Una forma de hacerlo es señalar que definimos el tronco natural como el antiderivado de\(\nicefrac{1}{x}\) tal que\(\ln 1 = 0\). Por lo tanto,\[\ln x := \int_1^x \nicefrac{1}{s}~ds .\] entonces podemos aproximar numéricamente la integral. Moralmente, en realidad no “simplificamos”\(\int_1^x \nicefrac{1}{s}~ds\) al anotar\(\ln x\). Simplemente le dimos un nombre a la integral. Si requerimos respuestas numéricas, es posible que terminemos haciendo el cálculo aproximando una integral de todos modos.

    Otra función común definida por una integral que no se puede evaluar simbólicamente es la función erf, definida como\[\operatorname{erf}(x) := \frac{2}{\sqrt{\pi}} \int_0^x e^{-s^2} ~ds .\] Esta función surge a menudo en las matemáticas aplicadas. Es simplemente la antiderivada de\(\left(\nicefrac{2}{\sqrt{\pi}}\right) e^{-x^2}\) que es cero a cero. La segunda forma del teorema fundamental nos dice que podemos escribir la función como una integral. Si queremos calcular algún valor en particular, aproximamos numéricamente la integral.

    Cambio de variables

    Un teorema de uso frecuente en el cálculo para resolver integrales es el teorema del cambio de variables. Demostrémoslo ahora. Recordar una función es continuamente diferenciable si es diferenciable y la derivada es continua.

    Dejar\(g \colon [a,b] \to {\mathbb{R}}\) ser una función continuamente diferenciable. Si\(g([a,b]) \subset [c,d]\) y\(f \colon [c,d] \to {\mathbb{R}}\) es continuo, entonces\[\int_a^b f\bigl(g(x)\bigr)\, g'(x)~ dx = \int_{g(a)}^{g(b)} f(s)~ ds .\]

    Como\(g\),\(g'\), y\(f\) son continuos, sabemos que\(f\bigl(g(x)\bigr)\,g'(x)\) es una función continua en\([a,b]\), por lo tanto es Riemann integrable.

    Definir\[F(y) := \int_{g(a)}^{y} f(s)~ds .\] Por la segunda forma del teorema fundamental del cálculo (utilizando a continuación)\(F\) es una función diferenciable y\(F'(y) = f(y)\). Aplicamos la regla de la cadena y\[\bigl( F \circ g \bigr)' (x) = F'\bigl(g(x)\bigr) g'(x) = f\bigl(g(x)\bigr) g'(x) .\] escribimos Observamos que\(F\bigl(g(a)\bigr) = 0\) y utilizamos la primera forma del teorema fundamental para obtener\[\int_{g(a)}^{g(b)} f(s)~ds = F\bigl(g(b)\bigr) = F\bigl(g(b)\bigr)-F\bigl(g(a)\bigr) = \int_a^b \bigl( F \circ g \bigr)' (x) ~dx = \int_a^b f\bigl(g(x)\bigr) g'(x) ~dx . %FIXME \qedhere\]

    El teorema del cambio de variables suele utilizarse para resolver integrales cambiándolas a integrales que conocemos o que podemos resolver utilizando el teorema fundamental del cálculo.

    A partir de un ejercicio, sabemos que la derivada de\(\sin(x)\) es\(\cos(x)\). Por lo tanto resolvemos\[\int_0^{\sqrt{\pi}} x \cos(x^2) ~ dx = \int_0^\pi \frac{\cos(s)}{2} ~ ds = \frac{1}{2} \int_0^\pi \cos(s) ~ ds = \frac{ \sin(\pi) - \sin(0) }{2} = 0 .\]

    No obstante, ten en cuenta que debemos satisfacer las hipótesis del teorema. El siguiente ejemplo demuestra por qué no debemos simplemente mover símbolos sin pensar. Debemos tener cuidado de que esos símbolos realmente tengan sentido.

    Supongamos\[\int_{-1}^{1} \frac{\ln \left\lvert {x} \right\rvert}{x} ~dx .\] que escribimos Puede ser tentador tomar\(g(x) := \ln \left\lvert {x} \right\rvert\). Entonces toma\(g'(x) = \frac{1}{x}\) e intenta escribir\[\int_{g(-1)}^{g(1)} s ~ds = \int_{0}^{0} s ~ds = 0.\] Esta “solución” es incorrecta, y no dice que podamos resolver la integral dada. El primer problema\(\frac{\ln \left\lvert {x} \right\rvert}{x}\) es que no es continuo\([-1,1]\). Segundo, ni siquiera Riemann\(\frac{\ln \left\lvert {x} \right\rvert}{x}\) es integrable en\([-1,1]\) (es ilimitada). La integral que escribimos simplemente no tiene sentido. Por último, no\(g\) es continuo en\([-1,1]\) ninguno de los dos.

    Ejercicios

    Computar\(\displaystyle \frac{d}{dx} \biggl( \int_{-x}^x e^{s^2}~ds \biggr)\).

    Computar\(\displaystyle \frac{d}{dx} \biggl( \int_{0}^{x^2} \sin(s^2)~ds \biggr)\).

    Supongamos que\(F \colon [a,b] \to {\mathbb{R}}\) es continuo y diferenciable en\([a,b] \setminus S\), donde\(S\) es un conjunto finito. Supongamos que existe\(f \in {\mathcal{R}}[a,b]\) tal que\(f(x) = F'(x)\) para\(x \in [a,b] \setminus S\). \(\int_a^b f = F(b)-F(a)\)Demuéstralo.

    [secondftc:exercise] Dejar\(f \colon [a,b] \to {\mathbb{R}}\) ser una función continua. Que\(c \in [a,b]\) sea arbitrario. Definir\[F(x) := \int_c^x f .\] Demostrar que\(F\) es diferenciable y eso\(F'(x) = f(x)\) para todos\(x \in [a,b]\).

    Demostrar integración por partes. Es decir, supongamos\(F\) y\(G\) son funciones continuamente diferenciables en\([a,b]\). Entonces prueba\[\int_a^b F(x)G'(x)~dx = F(b)G(b)-F(a)G(a) - \int_a^b F'(x)G(x)~dx .\]

    Supongamos\(F\) y\(G\) son continuamente 23 funciones diferenciables definidas sobre\([a,b]\) tal que\(F'(x) = G'(x)\) para todos\(x \in [a,b]\). Utilizando el teorema fundamental del cálculo, mostrar eso\(F\) y\(G\) diferir por una constante. Es decir, mostrar que existe\(C \in {\mathbb{R}}\) tal que\(F(x)-G(x) = C\).

    El siguiente ejercicio muestra cómo podemos usar la integral para “suavizar” una función no diferenciable.

    [ejercicio:suavizar] Dejar\(f \colon [a,b] \to {\mathbb{R}}\) ser una función continua. Que\(\epsilon > 0\) sea una constante. Para\(x \in [a+\epsilon,b-\epsilon]\), definir\[g(x) := \frac{1}{2\epsilon} \int_{x-\epsilon}^{x+\epsilon} f .\] a) Mostrar que\(g\) es diferenciable y encontrar la derivada.
    b) Dejar\(f\) ser diferenciable y fijar\(x \in (a,b)\) (dejar que\(\epsilon\) sea lo suficientemente pequeño). ¿Qué pasa a\(g'(x)\) medida que\(\epsilon\) se hace más pequeño?
    c) Encontrar\(g\) para\(f(x) := \left\lvert {x} \right\rvert\),\(\epsilon = 1\) (se puede asumir que\([a,b]\) es lo suficientemente grande).

    Supongamos que\(f \colon [a,b] \to {\mathbb{R}}\) es continuo y\(\int_a^x f = \int_x^b f\) para todos\(x \in [a,b]\). \(f(x) = 0\)Demuéstralo para todos\(x \in [a,b]\).

    Supongamos que\(f \colon [a,b] \to {\mathbb{R}}\) es continuo y\(\int_a^x f = 0\) para todos racional\(x\) en\([a,b]\). \(f(x) = 0\)Demuéstralo para todos\(x \in [a,b]\).

    Una función\(f\) es una función impar if\(f(x) = -f(-x)\), y\(f\) es una función par if\(f(x) = f(-x)\). Vamos\(a > 0\). Supongamos\(f\) que es continuo. Demostrar: a) Si\(f\) es impar,\(\int_{-a}^a f = 0\) entonces. b) Si\(f\) es par, entonces\(\int_{-a}^a f = 2 \int_0^a f\).

    a) Mostrar que\(f(x) := \sin(\nicefrac{1}{x})\) es integrable en cualquier intervalo (se puede\(f(0)\) definir como cualquier cosa). b) Calcular\(\int_{-1}^1 \sin(\nicefrac{1}{x})\,dx\). (Tenga en cuenta la discontinuidad)

    § 6] a) Supongamos que\(f \colon [a,b] \to {\mathbb{R}}\) está aumentando, por,\(f\) es Riemann integrable. Supongamos que\(f\) tiene una discontinuidad en\(c \in (a,b)\), espectáculo que no\(F(x) := \int_a^x f\) es diferenciable en\(c\).
    b) En, se ha construido una función creciente\(f \colon [0,1] \to {\mathbb{R}}\) que es discontinua en cada\(x \in [0,1] \cap {\mathbb{Q}}\). Usa esto\(f\) para construir una función\(F(x)\) que sea continua\([0,1]\), pero no diferenciable en cada uno\(x \in [0,1] \cap {\mathbb{Q}}\).

    El logaritmo y el exponencial

    Nota: 1 conferencia (opcional, requiere las secciones opcionales,,)

    Ahora tenemos todo lo que se requiere para definir finalmente adecuadamente el exponencial y el logaritmo que tan bien conoces del cálculo. Primero recordemos que tenemos una buena idea de lo que\(x^n\) significa siempre y cuando\(n\) sea un entero positivo. \[x^n := \underbrace{x \cdot x \cdot \cdots \cdot x}_{\text{$n$ times}} .\]Simplemente, tiene sentido definir\(x^0 := 1\). Para enteros negativos definimos\(x^{-n} := \nicefrac{1}{x^n}\). Si\(x > 0\), mencionamos antes que\(x^{1/n}\) se define como la raíz única positiva\(n\) th. Por último para cualquier número racional\(\nicefrac{n}{m}\), definimos\[x^{n/m} := {\bigl(x^{1/m}\bigr)}^n .\] Sin embargo, ¿qué queremos decir con\(\sqrt{2}^{\sqrt{2}}\)? ¿O\(x^y\) en general? En particular, ¿qué es\(e^x\) para todos\(x\)? ¿Y cómo solucionamos\(y=e^x\) para\(x\)? Esta sección responde a estas preguntas y más.

    El logaritmo

    Es conveniente comenzar con el logaritmo. Demostremos que existe una función única con las propiedades correctas, y sólo entonces la llamaremos logaritmo.

    Existe una función única\(L \colon (0,\infty) \to {\mathbb{R}}\) tal que

    1. [es:log:i]\(L(1) = 0\).
    2. [it:log:ii]\(L\) es diferenciable y\(L'(x) = \nicefrac{1}{x}\).
    3. [it:log:iii]\(L\) es estrictamente creciente, biyectiva, y\[\lim_{x\to 0} L(x) = -\infty , \qquad \text{and} \qquad \lim_{x\to \infty} L(x) = \infty .\]
    4. [it:log:iv]\(L(xy) = L(x)+L(y)\) para todos\(x,y \in (0,\infty)\).
    5. [it:log:v] Si\(q\) es un número racional y\(x > 0\), entonces\(L(x^q) = q L(x)\).

    Para probar la existencia, definamos a un candidato y demostremos que satisface todas las propiedades. Definir\[L(x) := \int_1^x \frac{1}{t}~dt .\]

    Obviamente [it:log:i] sostiene. Propiedad [it:log:ii] sostiene vía el teorema fundamental del cálculo.

    Para probar la propiedad [it:log:iv], cambiamos las variables\(u=yt\) para obtener\[L(x) = \int_1^{x} \frac{1}{t}~dt = \int_y^{xy} \frac{1}{u}~du = \int_1^{xy} \frac{1}{u}~du - \int_1^{y} \frac{1}{u}~du = L(xy)-L(y) .\]

    Propiedad [it:log:ii] junto con el hecho de que\(L'(x) = \nicefrac{1}{x} > 0\) para\(x > 0\), implica que\(L\) es estrictamente creciente y de ahí uno-a-uno. Vamos a mostrar\(L\) es sobre. Como\(\nicefrac{1}{t} \geq \nicefrac{1}{2}\) cuando\(t \in [1,2]\),\[L(2) = \int_1^2 \frac{1}{t} ~dt \geq \nicefrac{1}{2} .\] Por inducción, [it:log:iv] implica que para\(n \in {\mathbb{N}}\)\[L(2^n) = L(2) + L(2) + \cdots + L(2) = n L(2) .\] Dado cualquiera\(y > 0\), por el de los números reales (aviso\(L(2) > 0\)), hay\(n \in {\mathbb{N}}\) tal que\(L(2^n) > y\). Por el hay\(x_1 \in (1,2^n)\) tal que\(L(x_1) = y\). Obtenemos\((0,\infty)\) está en la imagen de\(L\). A medida\(L\) que va aumentando,\(L(x) > y\) para todos\(x > 2^n\), y así\[\lim_{x\to\infty} L(x) = \infty .\] Siguiente\(0 = L(\nicefrac{x}{x}) = L(x) + L(\nicefrac{1}{x})\), y así\(L(x) = - L(\nicefrac{1}{x})\). Utilizando\(x=2^{-n}\), obtenemos como arriba que\(L\) logra todos los números negativos. Y\[\lim_{x \to 0} L(x) = \lim_{x \to 0} -L(\nicefrac{1}{x}) = \lim_{x \to \infty} -L(x) = - \infty .\] en los límites, tenga en cuenta que sólo\(x > 0\) están en el dominio de\(L\).

    Ahora probemos [it:log:v]. Como antes, [it:log:iv] implica\(n \in {\mathbb{N}}\) porque tenemos\(L(x^n) = n L(x)\). Ya lo vimos\(L(x) = - L(\nicefrac{1}{x})\) así\(L(x^{-n}) = - L(x^n) = -n L(x)\). Entonces para\(m \in {\mathbb{N}}\)\[L(x) = L\Bigl({(x^{1/m})}^m\Bigr) = m L(x^{1/m}) .\] armar todo para\(n \in {\mathbb{Z}}\) y\(m \in {\mathbb{N}}\) tenemos\(L(x^{n/m}) = n L(x^{1/m}) = (\nicefrac{n}{m}) L(x)\).

    Finalmente por singularidad, usemos las propiedades [it:log:i] y [it:log:ii]. Vía el teorema fundamental del cálculo\[L(x) = \int_1^x \frac{1}{t}~dt\] es la función única tal que\(L(1) = 0\) y\(L'(x) = \nicefrac{1}{x}\).

    Habiendo demostrado que existe una función única con estas propiedades simplemente definimos el logaritmo o a veces llamado el logaritmo natural: A\[\ln(x) := L(x) .\] menudo los matemáticos escriben\(\log(x)\) en lugar de\(\ln(x)\), lo cual es más familiar para los estudiantes de cálculo.

    El exponencial

    Al igual que con el logaritmo definimos el exponencial a través de una lista de propiedades.

    Existe una función única\(E \colon {\mathbb{R}}\to (0,\infty)\) tal que

    1. [es:exp:i]\(E(0) = 1\).
    2. [it:exp:ii]\(E\) es diferenciable y\(E'(x) = E(x)\).
    3. [it:exp:iii]\(E\) es estrictamente creciente, biyectiva, y\[\lim_{x\to -\infty} E(x) = 0 , \qquad \text{and} \qquad \lim_{x\to \infty} E(x) = \infty .\]
    4. [it:exp:iv]\(E(x+y) = E(x)E(y)\) para todos\(x,y \in {\mathbb{R}}\).
    5. [it:exp:v] Si\(q \in {\mathbb{Q}}\), entonces\(E(qx) = {E(x)}^q\).

    Nuevamente, probemos la existencia de tal función definiendo a un candidato, y probemos que satisface todas las propiedades. Lo\(L\) definido anteriormente es invertible. Dejar\(E\) ser la función inversa de\(L\). Propiedad [it:exp:i] es inmediata.

    Property [it:exp:ii] sigue a través del teorema de la función inversa, en particular:\(L\) satisface todas las hipótesis del lema, y por lo tanto\[E'(x) = \frac{1}{L'\bigl(E(x)\bigr)} = E(x) .\]

    Echemos un vistazo a la propiedad [it:exp:iii]. La función\(E\) está aumentando estrictamente desde\(E(x) > 0\) y\(E'(x) = E(x) > 0\). Como\(E\) es la inversa de\(L\), también debe ser biyectiva. Para encontrar los límites, usamos eso\(E\) es estrictamente creciente y hacia\((0,\infty)\). Para cada\(M > 0\), hay\(x_0\) tal que\(E(x_0) = M\) y\(E(x) \geq M\) para todos\(x \geq x_0\). De igual manera para cada\(\epsilon > 0\), hay\(x_0\) tal que\(E(x_0) = \epsilon\) y\(E(x) < \epsilon\) para todos\(x < x_0\). Por lo tanto,\[\lim_{n\to -\infty} E(x) = 0 , \qquad \text{and} \qquad \lim_{n\to \infty} E(x) = \infty .\]

    Para probar la propiedad [it:exp:iv] utilizamos la propiedad correspondiente para el logaritmo. Tomar\(x, y \in {\mathbb{R}}\). Como\(L\) es biyectiva, encontrar\(a\) y\(b\) tal que\(x = L(a)\) y\(y = L(b)\). Entonces\[E(x+y) = E\bigl(L(a)+L(b)\bigr) = E\bigl(L(ab)\bigr) = ab = E(x)E(y) .\]

    Propiedad [it:exp:v] también se desprende de la propiedad correspondiente de\(L\). Dado\(x \in {\mathbb{R}}\), que\(a\) sea tal que\(x = L(a)\) y\[E(qx) = E\bigl(qL(a)\bigr) E\bigl(L(a^q)\bigr) = a^q = {E(x)}^q .\]

    Por último, la singularidad se desprende de [it:exp:i] y [it:exp:ii]. Dejar\(E\) y\(F\) ser dos funciones satisfaciendo [it:exp:i] y [it:exp:ii]. \[\frac{d}{dx} \Bigl( F(x)E(-x) \Bigr) = F'(x)E(-x) - E'(-x)F(x) = F(x)E(-x) - E(-x)F(x) = 0 .\]Por lo tanto por,\(F(x)E(-x) = F(0)E(-0) = 1\) para todos\(x \in {\mathbb{R}}\). Haciendo el cómputo con\(F = E\), obtenemos\(E(x)E(-x) = 1\). Entonces\[0 = 1-1 = F(x)E(-x) - E(x)E(-x) = \bigl(F(x)-E(x)\bigr) E(-x) .\] Desde\(E(x)E(-x) = 1\), entonces\(E(-x) \not= 0\) para todos\(x\). Entonces\(F(x)-E(x) = 0\) para todos\(x\), y ya terminamos.

    Habiendo demostrado que\(E\) es único, definimos la función exponencial como\[\exp(x) := E(x) .\]

    Ahora podemos darle sentido a la exponenciación\(x^y\) para números arbitrarios cuando\(x > 0\). Primero supongamos\(y \in {\mathbb{Q}}\). Entonces\[x^y = \exp\bigl(\ln(x^y)\bigr) = \exp\bigl(y\ln(x)\bigr) .\] Por lo tanto cuando\(x > 0\) y\(y\) es irracional definamos\[x^y := \exp\bigl(y\ln(x)\bigr) .\] Como\(\exp\) es continuo entonces\(x^y\) es una función continua de\(y\). Por lo tanto, obtendríamos el mismo resultado si hubiéramos tomado una secuencia de números racionales\(\{ y_n \}\) acercándose\(y\) y definidos\(x^y = \lim\, x^{y_n}\).

    Definir el número\(e\) como\[e := \exp(1) .\] El número a veces\(e\) se llama número de Euler o la base del logaritmo natural. Nos damos cuenta\[e^x = \exp\bigl(x \ln(e) \bigr) = \exp(x) .\] Hemos justificado la notación\(e^x\) para\(\exp(x)\).

    Finalmente, extendamos las propiedades del logaritmo y exponencial a los poderes irracionales. La prueba es inmediata.

    Vamos\(x, y \in {\mathbb{R}}\).

    1. \(\exp(xy) = {\bigl(\exp(x)\bigr)}^y\).
    2. Si\(x > 0\) entonces\(\ln(x^y) = y \ln (x)\).

    Ejercicios

    Dejar\(y\) ser cualquier número real y\(b > 0\). Definir\(f \colon (0,\infty) \to {\mathbb{R}}\) y\(g \colon {\mathbb{R}}\to {\mathbb{R}}\) como,\(f(x) := x^y\) y\(g(x) := b^x\). Demostrar eso\(f\) y\(g\) son diferenciables y encuentran su derivada.

    Dejemos\(b > 0\) que se den.
    a) Demostrar que para cada\(y > 0\), existe un número único\(x\) tal que\(y = b^x\). Definir la base logaritmo\(b\),\(\log_b \colon (0,\infty) \to {\mathbb{R}}\), por\(\log_b(y) := x\).
    b) Demostrar eso\(\log_b(x) = \frac{\ln(x)}{\ln(b)}\).
    c) Demostrar que si\(c > 0\), entonces\(\log_b(x) = \frac{\log_c(x)}{\log_c(b)}\).
    d) Demostrar\(\log_b(xy) = \log_b(x)+\log_b(y)\), y\(\log_b(x^y) = y \log_b(x)\).

    § 3] Úselo para estudiar el término restante y demostrar que para todos\(x \in {\mathbb{R}}\)\[e^x = \sum_{n=0}^\infty \frac{x^n}{n!} .\] Pista: No diferenciar el término de la serie por término (a menos que demuestre que funciona).

    Usa la fórmula de suma geométrica para mostrar (for\(t\not= -1\))\[1-t+t^2-\cdots+{(-1)}^n t^n = \frac{1}{1+t} - \frac

    ParseError: EOF expected (click for details)
    Callstack:
        at (Matematicas/Analisis/Introducción_al_Análisis_Real_(Lebl)/02:_Números_reales/2.01:_Propiedades_básicas), /content/body/div[32]/p[4]/span[2]/span, line 1, column 5
    
    {1+t}.\] Usando este show de hechos\[\ln (1+x) = \sum_{n=1}^\infty \frac{{(-1)}^{n+1}x^n}{n}\] para todos\(x \in (-1,1]\) (nota que\(x=1\) está incluida). Finalmente, encuentra el límite de la serie armónica alterna\[\sum_{n=1}^\infty \frac
    ParseError: EOF expected (click for details)
    Callstack:
        at (Matematicas/Analisis/Introducción_al_Análisis_Real_(Lebl)/02:_Números_reales/2.01:_Propiedades_básicas), /content/body/div[32]/p[4]/span[6]/span, line 1, column 5
    
    {n} = 1 - \nicefrac{1}{2} + \nicefrac{1}{3} - \nicefrac{1}{4} + \cdots % = \ln 2 .\]

    Mostrar\[e^x = \lim_{n\to\infty} {\left( 1 + \frac{x}{n} \right)}^n .\] pista: Toma el logaritmo.
    Nota: La expresión\({\left( 1 + \frac{x}{n} \right)}^n\) surge en los cálculos de interés compuesto. Es la cantidad de dinero en una cuenta bancaria después de 1 año si 1 dólar se depositó inicialmente a intereses\(x\) y el interés se compuso\(n\) veces durante el año. Por lo tanto,\(e^x\) es el resultado de la composición continua.

    a) Demostrar que para\(n \in {\mathbb{N}}\) nosotros tenemos\[\sum_{k=2}^{n} \frac{1}{k} \leq \ln (n) \leq \sum_{k=1}^{n-1} \frac{1}{k} .\]
    b) Demostrar que el límite\[\gamma := \lim_{n\to\infty} \left( \sum_{k=1}^{n} \frac{1}{k} - \ln (n) \right)\] existe. Esta constante se conoce como la constante de Euler-Mascheroni 24. No se sabe si esta constante es racional o no, es aproximadamente\(\gamma \approx 0.5772\).

    Mostrar\[\lim_{x\to\infty} \frac{\ln(x)}{x} = 0 .\]

    Mostrar que\(e^x\) es convexo, en otras palabras, mostrar que si\(a \leq x \leq b\) entonces\(e^x \leq e^a \frac{b-x}{b-a} + e^b \frac{x-a}{b-a}\).

    Usando el hallazgo logaritmo\[%\lim_{n\to\infty} {\left( 1 + \nicefrac{1}{n} \right)}^n = e . \lim_{n\to\infty} n^{1/n} .\]

    Demostrar que\(E(x) = e^x\) es la función continua única tal que\(E(x+y) = E(x)E(y)\) y\(E(1) = e\). De igual manera demostrar que\(L(x) = \ln(x)\) es la función continua única definida en positivo\(x\) tal que\(L(xy) = L(x)+L(y)\) y\(L(e) = 1\).

    Integrales inaceptables

    Nota: 2—3 conferencias (sección opcional, se puede omitir de forma segura, requiere la opcional)

    A menudo es necesario integrar sobre toda la línea real, o un intervalo infinito de la forma\([a,\infty)\) o\((\infty,b]\). También, es posible que deseemos integrar funciones definidas en un intervalo finito\((a,b)\) pero no delimitadas. Tales funciones no son Riemann integrables, pero quizá queramos anotar la integral de todos modos en el espíritu de. Estas integrales se llaman integrales inadecuadas, y son límites de integrales más que integrales en sí mismas.

    Supongamos que\(f \colon [a,b) \to {\mathbb{R}}\) es una función (no necesariamente acotada) que es Riemann integrable\([a,c]\) para todos\(c < b\). Definimos\[\int_a^b f := \lim_{c \to b^-} \int_a^{c} f ,\] si existe el límite.

    Supongamos que\(f \colon [a,\infty) \to {\mathbb{R}}\) es una función tal que\(f\) es Riemann integrable\([a,c]\) para todos\(c < \infty\). Definimos\[\int_a^\infty f := \lim_{c \to \infty} \int_a^c f ,\] si existe el límite.

    Si el límite existe, decimos que la integral impropia converge. Si el límite no existe, decimos que la integral impropia diverge.

    De manera similar definimos integrales inadecuadas para el punto final de la mano izquierda, dejamos esto al lector.

    Para un punto final finito\(b\), usando vemos que si\(f\) está acotado, entonces no hemos definido nada nuevo. Lo nuevo es que podemos aplicar esta definición a funciones no acotadas. El siguiente conjunto de ejemplos es tan útil que lo declaramos como una proposición.

    [impropriemann:ptest] La integral impropia\[\int_1^\infty \frac{1}{x^p} ~dx\] converge a\(\frac{1}{p-1}\) if\(p > 1\) y diverge if\(0 < p \leq 1\).

    La integral impropia\[\int_0^1 \frac{1}{x^p} ~dx\] converge a\(\frac{1}{1-p}\) si\(0 < p < 1\) y diverge si\(p \geq 1\).

    La prueba sigue por aplicación del teorema fundamental del cálculo. Hagamos la prueba\(p > 1\) para el extremo derecho infinito, y dejamos el resto al lector. Pista: Se debe manejar\(p=1\) por separado.

    Supongamos\(p > 1\). Entonces\[\int_1^b \frac{1}{x^p} ~dx = \int_1^b x^{-p} ~dx = \frac{b^{-p+1}}{-p+1} - \frac{1^{-p+1}}{-p+1} = - \frac{1}{(p-1)b^{p-1}} + \frac{1}{p-1} .\] As\(p > 1\), entonces\(p-1 > 0\). Tomando el límite a medida que\(b \to \infty\) obtenemos que\(\frac{1}{b^{p-1}}\) va a 0, y el resultado sigue.

    Declaramos la siguiente proposición para un solo tipo de integral impropia, aunque la prueba es sencilla y la misma para otros tipos de integrales impropias.

    [impropriemann:tail] Dejar\(f \colon [a,\infty) \to {\mathbb{R}}\) ser una función que sea Riemann integrable\([a,b]\) para todos\(b > a\). Dado cualquiera\(b > a\),\(\int_b^\infty f\) converge si y sólo si\(\int_a^\infty f\) converge, en cuyo caso\[\int_a^\infty f = \int_a^b f + \int_b^\infty f .\]

    Vamos\(c > b\). Entonces\[\int_a^c f = \int_a^b f + \int_b^c f .\] Tomando el límite\(c \to \infty\) termina la prueba.

    Las funciones no negativas son más fáciles de trabajar como lo demuestra la siguiente proposición. Los ejercicios demostrarán que esta proposición se sostiene únicamente para funciones no negativas. Existen análogos de esta proposición para que todos los demás tipos de límites impropios se dejen al alumno.

    [impropriemann:possimp] Supongamos que\(f \colon [a,\infty) \to {\mathbb{R}}\) es no negativo (\(f(x) \geq 0\)para todos\(x\)) y tal que\(f\) es Riemann integrable\([a,b]\) para todos\(b > a\).

    1. \[\int_a^\infty f = \sup \left\{ \int_a^x f : x \geq a \right\} .\]
    2. Supongamos que\(\{ x_n \}\) es una secuencia con\(\lim\, x_n = \infty\). Entonces\(\int_a^\infty f\) converge si y solo si\(\lim\, \int_a^{x_n} f\) existe, en cuyo caso\[\int_a^\infty f = \lim_{n\to\infty} \int_a^{x_n} f .\]

    En el primer ítem permitimos el valor de\(\infty\) en lo supremo indicando que la integral diverge hasta el infinito.

    Empecemos con el primer ítem. Observe que como no\(f\) es negativo, entonces\(\int_a^x f\) va aumentando en función de\(x\). Si lo supremo es infinito, entonces para cada uno\(M \in {\mathbb{R}}\) encontramos\(N\) tal que\(\int_a^N f \geq M\). Como\(\int_a^x f\) va aumentando entonces\(\int_a^x f \geq M\) para todos\(x \geq N\). Así\(\int_a^\infty f\) diverge hasta el infinito.

    Siguiente supongamos que el supremo es finito, digamos\(A = \sup \left\{ \int_a^x f : x \geq a \right\}\). Por cada\(\epsilon > 0\), nos encontramos con un\(N\) tal que\(A - \int_a^N f < \epsilon\). Como\(\int_a^x f\) va aumentando, entonces\(A - \int_a^x f < \epsilon\) para todos\(x \geq N\) y por lo tanto\(\int_a^\infty f\) converge a\(A\).

    Veamos el segundo ítem. Si\(\int_a^\infty f\) converge entonces cada secuencia\(\{ x_n \}\) que va al infinito funciona. El truco es probar la otra dirección. Supongamos que\(\{ x_n \}\) es tal que\(\lim\, x_n = \infty\) y\[\lim_{n\to\infty} \int_a^{x_n} f = A\] converge. Dado\(\epsilon > 0\), escoge\(N\) tal que por todo\(n \geq N\) lo que tenemos\(A - \epsilon < \int_a^{x_n} f < A + \epsilon\). Porque\(\int_a^x f\) va aumentando en función de\(x\), tenemos eso para todos\(x \geq x_N\)\[A - \epsilon < \int_a^{x_N} \leq \int_a^x f .\] Como\(\{ x_n \}\) va a\(\infty\), entonces para cualquier dado\(x\), hay\(x_m\) tal que\(m \geq N\) y\(x \leq x_m\). Entonces\[\int_a^{x} f \leq \int_a^{x_m} f < A + \epsilon .\] en particular, por todo\(x \geq x_N\) lo que tenemos\(\left\lvert {\int_a^{x} f - A} \right\rvert < \epsilon\).

    Dejar\(f \colon [a,\infty) \to {\mathbb{R}}\) y\(g \colon [a,\infty) \to {\mathbb{R}}\) ser funciones que sean Riemann integrables\([a,b]\) para todos\(b > a\). Supongamos que por todo\(x \geq a\) lo que tenemos\[\left\lvert {f(x)} \right\rvert \leq g(x) .\]

    1. Si\(\int_a^\infty g\) converge, entonces\(\int_a^\infty f\) converge, y en este caso\(\left\lvert {\int_a^\infty f} \right\rvert \leq \int_a^\infty g\).
    2. Si\(\int_a^\infty f\) diverge, entonces\(\int_a^\infty g\) diverge.

    Empecemos con el primer ítem. Para cualquier\(b\) y\(c\), tal que\(a \leq b \leq c\), tenemos\(-g(x) \leq f(x) \leq g(x)\), y así\[\int_b^c -g \leq \int_b^c f \leq \int_b^c g .\] En otras palabras,\(\left\lvert {\int_b^c f} \right\rvert \leq \int_b^c g\).

    Dejemos\(\epsilon > 0\) que se den. Debido a que tenemos\[\int_a^\infty g = \int_a^b g + \int_b^\infty g .\] As\(\int_a^b g\) va a\(\int_a^\infty g\) como\(b\) va al infinito, luego\(\int_b^\infty g\) va a 0 como\(b\) va al infinito. Elige\(B\) tal que\[\int_B^\infty g < \epsilon .\] As no\(g\) sea negativo, entonces si\(B \leq b < c\), entonces\(\int_b^c g < \epsilon\) también. \(\{ x_n \}\)Déjese ser una secuencia que va al infinito. \(M\)Sea tal que\(x_n \geq B\) para todos\(n \geq M\). Tomar\(n, m \geq M\), con\(x_n \leq x_m\),\[\left\lvert {\int_a^{x_m} f - \int_a^{x_n} f} \right\rvert = \left\lvert {\int_{x_n}^{x_m} f} \right\rvert \leq \int_{x_n}^{x_m} g < \epsilon .\] Por lo tanto la secuencia\(\{ \int_a^{x_n} f \}_{n=1}^\infty\) es Cauchy y de ahí converge.

    Tenemos que demostrar que el límite es único. Supongamos que\(\{ x_n \}\) es una secuencia que converge al infinito tal que\(\{ \int_a^{x_n} f \}\) converge a\(L_1\), y\(\{ y_n \}\) es una secuencia que converge al infinito es tal que\(\{ \int_a^{y_n} f \}\) converge a\(L_2\). Entonces debe haber alguna\(n\) tal que\(\left\lvert {\int_a^{x_n} f - L_1} \right\rvert < \epsilon\) y\(\left\lvert {\int_a^{y_n} f - L_2} \right\rvert < \epsilon\). También podemos suponer\(x_n \geq B\) y\(y_n \geq B\). Entonces\[\left\lvert {L_1 - L_2} \right\rvert \leq \left\lvert {L_1 - \int_a^{x_n} f} \right\rvert + \left\lvert {\int_a^{x_n} f- \int_a^{y_n} f} \right\rvert + \left\lvert {\int_a^{y_n} f - L_2} \right\rvert < \epsilon + \left\lvert {\int_{x_n}^{y_n} f} \right\rvert + \epsilon < 3 \epsilon.\] As\(\epsilon > 0\) fue arbitrario,\(L_1 = L_2\), y por lo tanto\(\int_a^\infty f\) converge. Arriba hemos demostrado eso\(\left\lvert {\int_a^c f} \right\rvert \leq \int_a^c g\) para todos\(c > a\). Al tomar el límite\(c \to \infty\), se prueba el primer ítem.

    El segundo ítem es simplemente un contrapositivo del primer ítem.

    La integral impropia\[\int_0^\infty \frac{\sin(x^2)(x+2)}{x^3+1} ~dx\] converge.

    Prueba: Primero observamos simplemente necesitamos demostrar que la integral converge al pasar del 1 al infinito. Para\(x \geq 1\) obtenemos\[\left\lvert {\frac{\sin(x^2)(x+2)}{x^3+1}} \right\rvert \leq \frac{x+2}{x^3+1} \leq \frac{x+2}{x^3} \leq \frac{x+2x}{x^3} \leq \frac{3}{x^2} .\] Entonces\[3 \int_1^\infty \frac{1}{x^2}~dx = \lim_{c\to\infty} \int_1^c \frac{3}{x^2} ~dx.\] Así converge la integral.

    Se debe tener cuidado al hacer manipulaciones formales con integrales impropias. Por ejemplo,\[\int_2^\infty \frac{2}{x^2-1}~dx\] converge a través de la prueba de comparación nuevamente usando\(\frac{1}{x^2}\). No obstante, si sucumbes a la tentación de escribir\[\frac{2}{x^2-1} = \frac{1}{x-1} - \frac{1}{x+1}\] y tratas de integrar cada parte por separado, no vas a tener éxito. No es cierto que se pueda dividir la integral impropia en dos; no se puede dividir el límite. \[\begin{split} \int_2^\infty \frac{2}{x^2-1} ~dx &= \lim_{b\to \infty} \int_2^b \frac{2}{x^2-1} ~dx \\ &= \lim_{b\to \infty} \left( \int_2^b \frac{1}{x-1}~dx - \int_2^b \frac{1}{x+1}~dx \right) \\ &\not= \int_2^\infty \frac{1}{x-1}~dx - \int_2^\infty \frac{1}{x+1}~dx . \end{split}\]La última línea en el cómputo ni siquiera tiene sentido. Ambas integrales allí divergen hasta el infinito ya que podemos aplicar la prueba de comparación apropiadamente con\(\nicefrac{1}{x}\). Obtenemos\(\infty - \infty\).

    Ahora supongamos que necesitamos tomar límites en ambos puntos finales.

    Supongamos que\(f \colon (a,b) \to {\mathbb{R}}\) es una función que es Riemann integrable\([c,d]\) para todos\(c\),\(d\) tal que\(a < c < d < b\), entonces definimos\[\int_a^b f := \lim_{c \to a^+} \, \lim_{d \to b^-} \, \int_{c}^{d} f ,\] si existen los límites.

    Supongamos que\(f \colon {\mathbb{R}}\to {\mathbb{R}}\) es una función tal que\(f\) es Riemann integrable en todos los intervalos finitos\([a,b]\). Entonces definimos\[\int_{-\infty}^\infty f := \lim_{c \to -\infty} \, \lim_{d \to \infty} \, \int_c^d f ,\] si existen los límites.

    De igual manera definimos integrales impropias con un punto final infinito y uno finito impropio, dejamos esto al lector.

    Siempre hay que tener cuidado con los dobles límites. La definición dada anteriormente dice que primero tomamos el límite como\(d\) va a\(b\) o\(\infty\) para un fijo\(c\), y luego tomamos el límite adentro\(c\). Habrá que demostrar que en este caso no importa qué límite calculemos primero.

    Veamos un ejemplo:\[\int_{-\infty}^\infty \frac{1}{1+x^2} ~ dx = \lim_{a \to -\infty} \, \lim_{b \to \infty} \, \int_{a}^b \frac{1}{1+x^2} ~ dx = \lim_{a \to -\infty} \, \lim_{b \to \infty} \bigl( \arctan(b) - \arctan(a) \bigr) = \pi .\]

    En la definición el orden de los límites siempre se puede cambiar si existen. Demostremos este hecho sólo por los límites infinitos.

    Si\(f \colon {\mathbb{R}}\to {\mathbb{R}}\) es una función integrable en cada intervalo. Entonces,\[\lim_{a \to -\infty} \, \lim_{b \to \infty} \, \int_a^b f \quad \text{converges if and only if} \qquad \lim_{b \to \infty} \, \lim_{a \to -\infty} \, \int_a^b f \quad \text{converges,}\] en cuyo caso las dos expresiones son iguales. Si alguna de las expresiones converge entonces la integral impropia converge y\[\lim_{a\to\infty} \int_{-a}^a f = \int_{-\infty}^\infty f .\]

    Sin pérdida de generalidad asumir\(a < 0\) y\(b > 0\). Supongamos que la primera expresión converge. Entonces Cálculo\[\begin{split} \lim_{a \to -\infty} \, \lim_{b \to \infty} \, \int_a^b f & = \lim_{a \to -\infty} \, \lim_{b \to \infty} \left( \int_a^0 f + \int_0^b f \right) = \left( \lim_{a \to -\infty} \int_a^0 f \right) + \left( \lim_{b \to \infty} \int_0^b f \right) \\ & = \lim_{b \to \infty} \left( \left( \lim_{a \to -\infty} \int_a^0 f \right) + \int_0^b f \right) = \lim_{b \to \infty} \, \lim_{a \to -\infty} \left( \int_a^0 f + \int_0^b f \right) . \end{split}\] similar muestra la otra dirección. Por lo tanto, si cualquiera de las expresiones converge entonces la integral impropia converge y\[\begin{split} \int_{-\infty}^\infty f = \lim_{a \to -\infty} \, \lim_{b \to \infty} \, \int_a^b f & = \left( \lim_{a \to -\infty} \int_a^0 f \right) + \left( \lim_{b \to \infty} \int_0^b f \right) \\ & = \left( \lim_{a \to \infty} \int_{-a}^0 f \right) + \left( \lim_{a \to \infty} \int_0^a f \right) = \lim_{a \to \infty} \left( \int_{-a}^0 f + \int_0^a f \right) = \lim_{a \to \infty} \int_{-a}^a f . \end{split}\]

    Por otro lado, hay que tener cuidado de tomar los límites de manera independiente antes de conocer la convergencia. Dejar\(f(x) = \frac{x}{\left\lvert {x} \right\rvert}\) para\(x \not= 0\) y\(f(0) = 0\). Si\(a < 0\) y\(b > 0\), entonces\[\int_{a}^b f = \int_{a}^0 f + \int_{0}^b f = a+b .\] Para cualquier fijo\(a < 0\) el límite como\(b \to \infty\) es infinito, por lo que ni siquiera el primer límite existe, y de ahí la integral impropia\(\int_{-\infty}^\infty f\) no converge. Por otra parte si\(a > 0\), entonces\[\int_{-a}^{a} f = (-a)+a = 0 .\] Por lo tanto,\[\lim_{a\to\infty} \int_{-a}^{a} f = 0 .\]

    Un ejemplo a tener en cuenta para integrales inadecuadas es la llamada función sinc 25. Esta función surge con bastante frecuencia tanto en matemáticas puras como aplicadas. Definir\[\operatorname{sinc}(x) = \begin{cases} \frac{\sin(x)}{x} & \text{if $x \not= 0$} , \\ 0 & \text{if $x = 0$} . \end{cases}\]

    No es difícil demostrar que la función sinc es continua a cero, pero eso no es importante en estos momentos. Lo importante es que\[\int_{-\infty}^\infty \operatorname{sinc}(x) ~dx = \pi , \qquad \text{while} \qquad \int_{-\infty}^\infty \left\lvert {\operatorname{sinc}(x)} \right\rvert ~dx = \infty .\] La integral de la función sinc es un análogo continuo de la serie armónica alterna\(\sum \nicefrac{{(-1)}^n}{n}\), mientras que el valor absoluto es como la serie armónica regular\(\sum \nicefrac{1}{n}\). En particular, el hecho de que la integral converja debe hacerse directamente en lugar de usar prueba de comparación.

    No vamos a probar exactamente la primera declaración. Simplemente probemos que la integral de la función sinc converge, pero no nos preocuparemos por el límite exacto. Porque\(\frac{\sin(-x)}{-x} = \frac{\sin(x)}{x}\), basta con mostrar que\[\int_{2\pi}^\infty \frac{\sin(x)}{x}~dx\] converge. También evitamos\(x=0\) esta forma de hacer nuestra vida más sencilla.

    Para cualquiera\(n \in {\mathbb{N}}\), tenemos eso para\(x \in [\pi 2n, \pi (2n+1)]\)\[\frac{\sin(x)}{\pi (2n+1)} \leq \frac{\sin(x)}{x} \leq \frac{\sin(x)}{\pi 2n} ,\] como\(\sin(x) \geq 0\). En\(x \in [\pi (2n+1), \pi (2n+2)]\)\[\frac{\sin(x)}{\pi (2n+1)} \leq \frac{\sin(x)}{x} \leq \frac{\sin(x)}{\pi (2n+2)} ,\] como\(\sin(x) \leq 0\).

    Vía el teorema fundamental del cálculo,\[\frac{2}{\pi (2n+1)} = \int_{\pi 2n}^{\pi (2n+1)} \frac{\sin(x)}{\pi (2n+1)} ~dx \leq \int_{\pi 2n}^{\pi (2n+1)} \frac{\sin(x)}{x} ~dx \leq \int_{\pi 2n}^{\pi (2n+1)} \frac{\sin(x)}{\pi 2n} ~dx = \frac{1}{\pi n} .\] Similarmente\[\frac{-2}{\pi (2n+1)} \leq \int_{\pi (2n+1)}^{\pi (2n+2)} \frac{\sin(x)}{x} ~dx \leq \frac{-1}{\pi (n+1)} .\] Poniendo los dos juntos tenemos\[0 = \frac{2}{\pi (2n+1)} - \frac{2}{\pi (2n+1)} + \leq \int_{2\pi n}^{2\pi (n+1)} \frac{\sin(x)}{x} ~dx \leq \frac{1}{\pi n} - \frac{1}{\pi (n+1)} = \frac{1}{\pi n(n+1)} .\] Let\(M > 2\pi\) ser arbitrarios, y dejar\(k \in {\mathbb{N}}\) ser el entero más grande tal que\(2k\pi \leq M\). Entonces\[\int_{2\pi}^M \frac{\sin(x)}{x}~dx = \int_{2\pi}^{2k\pi} \frac{\sin(x)}{x} ~dx + \int_{2k\pi}^{M} \frac{\sin(x)}{x} ~dx .\] Porque\(x \in [2k\pi,M]\) tenemos\(\frac{-1}{2k\pi} \leq \frac{\sin(x)}{x} \leq \frac{1}{2k\pi}\), y así\[\left\lvert {\int_{2k\pi}^{M} \frac{\sin(x)}{x} ~dx } \right\rvert \leq \frac{M-2k\pi}{2k\pi} \leq \frac{1}{k} .\] As\(k\) es el mayor\(k\) tal que\(2k\pi \leq M\), este término va a cero como\(M\) va al infinito.

    Siguiente\[0 \leq \int_{2\pi}^{2k\pi} \frac{\sin(x)}{x} \leq \sum_{n=1}^{k-1} \frac{1}{\pi n(n+1)} ,\] y esta serie converge como\(k \to \infty\).

    Al juntar las dos declaraciones obtenemos\[\int_{2\pi}^\infty \frac{\sin(x)}{x} ~dx \leq \sum_{n=1}^{\infty} \frac{1}{\pi n(n+1)} < \infty .\]

    La integral de doble cara de sinc también existe como se señaló anteriormente. Dejamos la otra afirmación —que la integral del valor absoluto de la función sinc diverge— como ejercicio.

    Prueba integral para series

    Puede ser muy útil aplicar el teorema fundamental del cálculo para demostrar que una serie es sumable y para estimar su suma.

    Supongamos que\(f \colon [k,\infty) \to {\mathbb{R}}\) es una función decreciente no negativa donde\(k \in {\mathbb{Z}}\). Entonces\[\sum_{n=k}^\infty f(n) \quad \text{converges if and only if} \qquad \int_k^\infty f \quad \text{converges}.\] en este caso\[\int_k^\infty f \leq \sum_{n=k}^\infty f(n) \leq f(k)+ \int_k^\infty f .\]

    Por,\(f\) es integrable en cada intervalo\([k,b]\) para todos\(b > k\), por lo que la afirmación del teorema tiene sentido sin hipótesis adicionales de integrabilidad.

    Dejemos\(\epsilon > 0\) que se den. Y supongamos que\(\int_k^\infty f\) converge. Que\(\ell, m \in {\mathbb{Z}}\) sea tal que\(m > \ell \geq k\). Porque\(f\) es decreciente tenemos\(\int_{n}^{n+1} f \leq f(n) \leq \int_{n-1}^{n} f\). Por lo tanto\[\label{impropriemann:eqseries} \int_\ell^m f = \sum_{n=\ell}^{m-1} \int_{n}^{n+1} f \leq \sum_{n=\ell}^{m-1} f(n) \leq f(\ell) + \sum_{n=\ell+1}^{m-1} \int_{n-1}^{n} f \leq f(\ell)+ \int_\ell^{m-1} f .\] como antes, ya que\(f\) es positivo entonces existe\(L \in {\mathbb{N}}\) tal que si\(\ell \geq L\), entonces\(\int_\ell^{m} f < \nicefrac{\epsilon}{2}\) para todos\(m \geq \ell\). Observamos que\(f\) debe disminuir a cero (¿por qué?). Entonces supongamos también eso porque\(\ell \geq L\) tenemos\(f(\ell) < \nicefrac{\epsilon}{2}\). Para tal\(\ell\) y\(m\) tenemos vía [impropriemann:eqseries]\[\sum_{n=\ell}^{m} f(n) \leq f(\ell)+ \int_\ell^{m} f < \nicefrac{\epsilon}{2} + \nicefrac{\epsilon}{2} = \epsilon .\] La serie es por lo tanto Cauchy y así converge. La estimación en la proposición se obtiene dejando\(m\) ir al infinito en [impropriemann:eqseries] con\(\ell = k\).

    Por el contrario supongamos\(\int_k^\infty f\) divergencias. Como\(f\) es positivo entonces por, la secuencia\(\{ \int_k^m f \}_{m=k}^\infty\) diverge hasta el infinito. Usando [impropriemann:eqseries] con\(\ell = k\) encontramos\[\int_k^m f \leq \sum_{n=k}^{m-1} f(n) .\] Como el lado izquierdo va al infinito como\(m \to \infty\), también lo hace el lado derecho.

    Demostremos\(\sum_{n=1}^\infty \frac{1}{n^2}\) que existe y estimaremos su suma dentro de 0.01. Como esta serie es la\(p\) -serie para\(p=2\), ya sabemos que converge, pero sólo hemos estimado muy aproximadamente su suma.

    Utilizando el teorema fundamental del cálculo encontramos que para nosotros\(k \in {\mathbb{N}}\) tenemos\[\int_{k}^\infty \frac{1}{x^2}~dx = \frac{1}{k} .\] En particular, la serie debe converger. Pero también tenemos eso\[\frac{1}{k} = \int_k^\infty \frac{1}{x^2}~dx \leq \sum_{n=k}^\infty \frac{1}{n^2} \leq \frac{1}{k^2} + \int_k^\infty \frac{1}{x^2}~dx = \frac{1}{k^2} + \frac{1}{k} .\] Sumando la suma parcial hasta que\(k-1\) obtenemos\[\frac{1}{k} + \sum_{n=1}^{k-1} \frac{1}{n^2} \leq \sum_{n=1}^\infty \frac{1}{n^2} \leq \frac{1}{k^2} + \frac{1}{k} + \sum_{n=1}^{k-1} \frac{1}{n^2} .\] En otras palabras,\(\nicefrac{1}{k} + \sum_{n=1}^{k-1} \nicefrac{1}{n^2}\) es una estimación para la suma a dentro\(\nicefrac{1}{k^2}\). Por lo tanto, si deseamos encontrar la suma dentro de 0.01, señalamos\(\nicefrac{1}{{10}^2} = 0.01\). Obtenemos\[1.6397\ldots \approx \frac{1}{10} + \sum_{n=1}^{9} \frac{1}{n^2} \leq \sum_{n=1}^\infty \frac{1}{n^2} \leq \frac{1}{100} + \frac{1}{10} + \sum_{n=1}^{9} \frac{1}{n^2} \approx 1.6497\ldots .\] La suma real es\(\nicefrac{\pi^2}{6} \approx 1.6449\ldots\).

    Ejercicios

    Terminar la prueba de.

    Entérate para\(a \in {\mathbb{R}}\) cuál\(\sum\limits_{n=1}^\infty e^{an}\) converge. Cuando la serie converge, encuentra un límite superior para la suma.

    a) Estimar\(\sum\limits_{n=1}^\infty \frac{1}{n(n+1)}\) correctamente dentro de 0.01 usando la prueba integral. b) Calcule el límite de la serie exactamente y compare. Pista: los telescopios de suma.

    \[\int_{-\infty}^\infty \left\lvert {\operatorname{sinc}(x)} \right\rvert~dx = \infty .\]Demostrar pista: de nuevo, basta con mostrar esto en un solo lado.

    ¿Se puede interpretar\[\int_{-1}^1 \frac{1}{\sqrt{\left\lvert {x} \right\rvert}}~dx\] como una integral impropia? Si es así, computa su valor.

    Tomemos\(f \colon [0,\infty) \to {\mathbb{R}}\), Riemann integrable en cada intervalo\([0,b]\), y tal que exista\(M\),\(a\), y\(T\), tal que\(\left\lvert {f(t)} \right\rvert \leq M e^{at}\) para todos\(t \geq T\). Demostrar que la transformación de Laplace de\(f\) existe. Es decir, para cada converge\(s > a\) la siguiente integral:\[F(s) := \int_{0}^\infty f(t) e^{-st} ~dt .\]

    Dejar\(f \colon {\mathbb{R}}\to {\mathbb{R}}\) ser una función integrable de Riemann en cada intervalo\([a,b]\), y tal que\(\int_{-\infty}^\infty \left\lvert {f(x)} \right\rvert~dx < \infty\). Mostrar que existen las transformadas de seno y coseno de Fourier. Es decir, para cada convergen\(\omega \geq 0\) las siguientes integrales\[F^s(\omega) := \frac{1}{\pi} \int_{-\infty}^\infty f(t) \sin(\omega t) ~dt , \qquad F^c(\omega) := \frac{1}{\pi} \int_{-\infty}^\infty f(t) \cos(\omega t) ~dt .\] Además, muestran que\(F^s\) y\(F^c\) son funciones acotadas.

    Supongamos que Riemann\(f \colon [0,\infty) \to {\mathbb{R}}\) es integrable en cada intervalo\([0,b]\). Demostrar que\(\int_0^\infty f\) converge si y sólo si por cada\(\epsilon > 0\) existe un\(M\) tal que si\(M \leq a < b\) entonces\(\left\lvert {\int_a^b f} \right\rvert < \epsilon\).

    Supongamos que no\(f \colon [0,\infty) \to {\mathbb{R}}\) es negativo y decreciente. a) Demostrar que si\(\int_0^\infty f < \infty\), entonces\(\lim\limits_{x\to\infty} f(x) = 0\). b) Demostrar que lo contrario no se sostiene.

    Encuentre un ejemplo de una función continua no acotada\(f \colon [0,\infty) \to {\mathbb{R}}\) que sea no negativa y tal que\(\int_0^\infty f < \infty\). Tenga en cuenta que esto\(\lim_{x\to\infty} f(x)\) significa que no existe; compare ejercicio anterior. Pista: en cada intervalo\([k,k+1]\),\(k \in {\mathbb{N}}\), definir una función cuya integral sobre este intervalo es menor que decir\(2^{-k}\).

    Encuentra un ejemplo de una función\(f \colon [0,\infty) \to {\mathbb{R}}\) integrable en todos los intervalos tal que\(\lim_{n\to\infty} \int_0^n f\) converja como límite de una secuencia, pero tal que\(\int_0^\infty f\) no existe. Sugerencia: para todos\(n\in {\mathbb{N}}\), divídalo\([n,n+1]\) en dos mitades. En una mitad hacer que la función sea negativa, en la otra hacer que la función sea positiva.

    Demostrar que si\(f \colon [1,\infty) \to {\mathbb{R}}\) es tal que\(g(x) := x^2 f(x)\) es una función acotada, entonces\(\int_1^\infty f\) converge.

    A veces es deseable asignar un valor a integrales que normalmente no se pueden interpretar como incluso integrales inadecuadas, e.g\(\int_{-1}^1 \nicefrac{1}{x}~dx\). Supongamos que\(f \colon [a,b] \to {\mathbb{R}}\) es una función y\(a < c < b\), donde\(f\) es Riemann integrable en todos los intervalos\([a,c-\epsilon]\) y\([c+\epsilon,b]\) para todos\(\epsilon > 0\). Definir el valor principal de Cauchy\(\int_a^b f\) como\[p.v.\!\int_a^b f := \lim_{\epsilon\to 0^+} \left( \int_a^{c-\epsilon} f + \int_{c+\epsilon}^b f \right) ,\] si el límite existiera.

    a) Calentar\(p.v.\!\int_{-1}^1 \nicefrac{1}{x}~dx\).
    b) Calcular\(\lim_{\epsilon\to 0^+} ( \int_{-1}^{-\epsilon} \nicefrac{1}{x}~dx + \int_{2\epsilon}^1 \nicefrac{1}{x}~dx )\) y mostrar que no es igual al valor principal.
    c) Demostrar que si\(f\) es integrable en\([a,b]\), entonces\(p.v.\!\int_a^b f = \int_a^b f\).
    d) Encontrar un ejemplo de una\(f\) con una singularidad en\(c\) como arriba tal que\(p.v.\!\int_a^b f\) existe, pero las integrales impropias\(\int_a^c f\) y\(\int_c^b f\) divergen.
    e) Supongamos que\(f \colon [-1,1] \to {\mathbb{R}}\) es continuo. Demostrar que\(p.v.\!\int_{-1}^1 \frac{f(x)}{x}~dx\) existe.

    Dejar\(f \colon {\mathbb{R}}\to {\mathbb{R}}\) y\(g \colon {\mathbb{R}}\to {\mathbb{R}}\) ser funciones continuas, donde\(g(x) = 0\) para todos\(x \notin [a,b]\) por algún intervalo\([a,b]\).
    a) Demostrar que la convolución\[(g * f)(x) := \int_{-\infty}^\infty f(t)g(x-t)~dt\] está bien definida para todos\(x \in {\mathbb{R}}\).
    b) Supongamos\(\int_{-\infty}^\infty \left\lvert {f(x)} \right\rvert~dx < \infty\). Demostrar que\[\lim_{x \to -\infty} (g * f)(x) = 0, \qquad \text{and} \qquad \lim_{x \to \infty} (g * f)(x) = 0 .\]

    Secuencias de funciones

    Convergencia puntual y uniforme

    Nota: 1—1.5 conferencia

    Hasta ahora cuando hablábamos de secuencias siempre hablábamos de secuencias de números. Sin embargo, un concepto muy útil en el análisis es utilizar una secuencia de funciones. Por ejemplo, se podría encontrar una solución a alguna ecuación diferencial encontrando solo soluciones aproximadas. Entonces la solución real es algún tipo de límite de esas soluciones aproximadas.

    Cuando se habla de secuencias de funciones, la parte complicada es que hay múltiples nociones de un límite. Describamos dos nociones comunes de un límite de una secuencia de funciones.

    Convergencia puntual

    Por cada\(n \in {\mathbb{N}}\) let\(f_n \colon S \to {\mathbb{R}}\) ser una función. Decimos que la secuencia\(\{ f_n \}_{n=1}^\infty\) converge puntualmente a\(f \colon S \to {\mathbb{R}}\), si por cada\(x \in S\) que tenemos\[f(x) = \lim_{n\to\infty} f_n(x) .\]

    Es común decir que\(f_n \colon S \to {\mathbb{R}}\) converge a\(f\) on\(T \subset {\mathbb{R}}\) para algunos\(f \colon T \to {\mathbb{R}}\). En ese caso nosotros, por supuesto, significamos\(f(x) = \lim\, f_n(x)\) para cada uno\(x \in T\). Simplemente queremos decir que las restricciones de\(f_n\) para\(T\) converger puntualmente a\(f\).

    La secuencia de funciones definidas por\(f_n(x) := x^{2n}\) converge a\(f \colon [-1,1] \to {\mathbb{R}}\) on\([-1,1]\), donde\[f(x) = \begin{cases} 1 & \text{if $x=-1$ or $x=1$,} \\ 0 & \text{otherwise.} \end{cases}\] Ver.

    Para ver esto es así, primero toma\(x \in (-1,1)\). Entonces\(0 \leq x^2 < 1\). Nosotros hemos visto antes que\[\left\lvert {x^{2n} - 0} \right\rvert = {(x^2)}^n \to 0 \quad \text{as} \quad n \to \infty .\] Por lo tanto\(\lim\,f_n(x) = 0\).

    Cuando\(x = 1\) o\(x=-1\), entonces\(x^{2n} = 1\) para todos\(n\) y por lo tanto\(\lim\,f_n(x) = 1\). También notamos que\(\{ f_n(x) \}\) no converge para todos los demás\(x\).

    A menudo, las funciones se dan como una serie. En este caso, utilizamos la noción de convergencia puntual para encontrar los valores de la función.

    Escribimos\[\sum_{k=0}^\infty x^k\] para denotar el límite de las funciones\[f_n(x) := \sum_{k=0}^n x^k .\] Al estudiar series, hemos visto que en\(x \in (-1,1)\) el\(f_n\) converger pointwise a\[\frac{1}{1-x} .\]

    El punto sutil aquí es que si bien\(\frac{1}{1-x}\) se define para todos\(x \not=1\), y\(f_n\) se definen para todos\(x\) (incluso en\(x=1\)), la convergencia solo ocurre en\((-1,1)\).

    Por lo tanto, cuando\[f(x) := \sum_{k=0}^\infty x^k\] escribimos queremos decir que\(f\) se define sobre\((-1,1)\) y es el límite puntual de las sumas parciales.

    Vamos\(f_n(x) := \sin(xn)\). Entonces\(f_n\) no converge puntualmente a ninguna función en ningún intervalo. Puede converger en ciertos puntos, como cuándo\(x=0\) o\(x=\pi\). Se deja como ejercicio que en cualquier intervalo\([a,b]\), existe un\(x\) tal que\(\sin(xn)\) no tiene límite como\(n\) va al infinito.

    Antes de pasar a la convergencia uniforme, reformulemos la convergencia puntual de una manera diferente. Dejamos la prueba al lector, es una simple aplicación de la definición de convergencia de una secuencia de números reales.

    [ptwsconv:prop] Let\(f_n \colon S \to {\mathbb{R}}\) and\(f \colon S \to {\mathbb{R}}\) be funciones. Entonces\(\{ f_n \}\) converge puntualmente a\(f\) si y solo si por cada\(x \in S\), y cada\(\epsilon > 0\), existe\(N \in {\mathbb{N}}\) tal que\[\left\lvert {f_n(x)-f(x)} \right\rvert < \epsilon\] para todos\(n \geq N\).

    El punto clave aquí es que\(N\) puede depender\(x\), no sólo de\(\epsilon\). Es decir, para cada uno\(x\) podemos elegir una diferente\(N\). Si podemos escoger uno\(N\) para todos\(x\), tenemos lo que se llama convergencia uniforme.

    Convergencia uniforme

    Dejar\(f_n \colon S \to {\mathbb{R}}\) ser funciones. Decimos que la secuencia\(\{ f_n \}\) converge uniformemente a\(f \colon S \to {\mathbb{R}}\), si por cada\(\epsilon > 0\) existe un\(N \in {\mathbb{N}}\) tal que por todos\(n \geq N\) tenemos\[\left\lvert {f_n(x) - f(x)} \right\rvert < \epsilon \qquad \text{for all $x \in S$.}\]

    Tenga en cuenta que\(N\) ahora no puede depender\(x\). Dado\(\epsilon > 0\) debemos encontrar una\(N\) que funcione para todos\(x \in S\). Debido a, vemos que la convergencia uniforme implica convergencia puntual.

    Dejar\(\{ f_n \}\) ser una secuencia de funciones\(f_n \colon S \to {\mathbb{R}}\). Si\(\{ f_n \}\) converge uniformemente a\(f \colon S \to {\mathbb{R}}\), entonces\(\{ f_n \}\) converge puntualmente a\(f\).

    Lo contrario no se sostiene.

    Las funciones\(f_n(x) := x^{2n}\) no convergen uniformemente\([-1,1]\), aunque convergen puntualmente. Para ver esto, supongamos por contradicción que la convergencia es uniforme. Para\(\epsilon := \nicefrac{1}{2}\), habría que existir un\(N\) tal que\(x^{2N} = \left\lvert {x^{2N} - 0} \right\rvert < \nicefrac{1}{2}\) para todos\(x \in (-1,1)\) (como\(f_n(x)\) converja a 0 on\((-1,1)\)). Pero eso significa que para cualquier secuencia\(\{ x_k \}\) en\((-1,1)\) tal que\(\lim\, x_k = 1\) tenemos\(x_k^{2N} < \nicefrac{1}{2}\) para todos\(k\). Por otro lado\(x^{2N}\) es una función continua de\(x\) (es un polinomio), por lo tanto obtenemos una contradicción\[1 = 1^{2N} = \lim_{k\to\infty} x_k^{2N} \leq \nicefrac{1}{2} .\]

    Sin embargo, si restringimos nuestro dominio a\([-a,a]\) donde\(0 < a < 1\), entonces\(\{ f_n \}\) converge uniformemente a 0 on\([-a,a]\). Primero tenga en cuenta que\(a^{2n} \to 0\) como\(n \to \infty\). Así dado\(\epsilon > 0\), escoge\(N \in {\mathbb{N}}\) tal que\(a^{2n} < \epsilon\) para todos\(n \geq N\). Entonces para cualquiera\(x \in [-a,a]\) que tengamos\(\left\lvert {x} \right\rvert \leq a\). Por lo tanto, para\(n \geq N\)\[\left\lvert {x^{2N}} \right\rvert = \left\lvert {x} \right\rvert^{2N} \leq a^{2N} < \epsilon .\]

    Convergencia en norma uniforme

    Para las funciones acotadas hay otra manera más abstracta de pensar en la convergencia uniforme. A cada función delimitada le asignamos un cierto número no negativo (llamado norma uniforme). Este número mide la “distancia” de la función desde 0. Entonces podemos “medir” qué tan lejos están dos funciones entre sí. Simplemente traducimos una declaración sobre la convergencia uniforme en una declaración sobre una cierta secuencia de números reales que convergen a cero.

    [def:unifnorm] Dejar\(f \colon S \to {\mathbb{R}}\) ser una función acotada. Definir\[\left\lVert {f} \right\rVert_u := \sup \bigl\{ \left\lvert {f(x)} \right\rvert : x \in S \bigr\} .\]\(\left\lVert {\cdot} \right\rVert_u\) se llama la norma uniforme.

    Para utilizar esta notación 26 y este concepto, se\(S\) debe fijar el dominio. Algunos autores utilizan la notación\(\left\lVert {f} \right\rVert_S\) para enfatizar la dependencia de\(S\).

    Una secuencia de funciones\(f_n \colon S \to {\mathbb{R}}\) delimitadas converge uniformemente a\(f \colon S \to {\mathbb{R}}\), si y solo si\[\lim_{n\to\infty} \left\lVert {f_n - f} \right\rVert_u = 0 .\]

    Primero supongamos\(\lim \left\lVert {f_n - f} \right\rVert_u = 0\). Dejemos\(\epsilon > 0\) que se den. Entonces existe\(N\) tal que para\(n \geq N\) nosotros tenemos\(\left\lVert {f_n - f} \right\rVert_u < \epsilon\). Como\(\left\lVert {f_n-f} \right\rVert_u\) es lo supremo de\(\left\lvert {f_n(x)-f(x)} \right\rvert\), vemos eso por todo\(x\) lo que tenemos\(\left\lvert {f_n(x)-f(x)} \right\rvert < \epsilon\).

    Por otro lado, supongamos que\(\{ f_n \}\) converge uniformemente a\(f\). Dejemos\(\epsilon > 0\) que se den. Entonces encuentra\(N\) tal que\(\left\lvert {f_n(x)-f(x)} \right\rvert < \epsilon\) para todos\(x \in S\). Tomando lo supremo vemos eso\(\left\lVert {f_n - f} \right\rVert_u < \epsilon\). De ahí\(\lim \left\lVert {f_n-f} \right\rVert_u = 0\).

    A veces se dice que \(\{ f_n \}\)converge\(f\) en norma uniforme en vez de converge uniformemente. Dice la proposición que las dos nociones son lo mismo.

    Dejar\(f_n \colon [0,1] \to {\mathbb{R}}\) ser definido por\(f_n(x) := \frac{nx+ \sin(nx^2)}{n}\). Entonces afirmamos\(\{ f_n \}\) converge de manera uniforme a\(f(x) := x\). Vamos a calcular:\[\begin{split} \left\lVert {f_n-f} \right\rVert_u & = \sup \left\{ \left\lvert {\frac{nx+ \sin(nx^2)}{n} - x} \right\rvert : x \in [0,1] \right\} \\ & = \sup \left\{ \frac{\left\lvert {\sin(nx^2)} \right\rvert}{n} : x \in [0,1] \right\} \\ & \leq \sup \{ \nicefrac{1}{n} : x \in [0,1] \} \\ & = \nicefrac{1}{n}. \end{split}\]

    Usando norma uniforme, definimos secuencias de Cauchy de manera similar a como definimos secuencias de Cauchy de números reales.

    Dejar\(f_n \colon S \to {\mathbb{R}}\) ser funciones acotadas. La secuencia es Cauchy en la norma uniforme o uniformemente Cauchy si por cada\(\epsilon > 0\), existe\(N \in {\mathbb{N}}\) tal que para\(m,k \geq N\) nosotros tenemos\[\left\lVert {f_m-f_k} \right\rVert_u < \epsilon .\]

    [prop:uniformcauchy] Dejan\(f_n \colon S \to {\mathbb{R}}\) ser funciones acotadas. Entonces\(\{ f_n \}\) está Cauchy en la norma uniforme si y sólo si existe una\(f \colon S \to {\mathbb{R}}\) y\(\{ f_n \}\) converge uniformemente a\(f\).

    Supongamos primero que\(\{ f_n \}\) es Cauchy en la norma uniforme. Definamos\(f\). Fijar\(x\), entonces la secuencia\(\{ f_n(x) \}\) es Cauchy porque\[\left\lvert {f_m(x)-f_k(x)} \right\rvert \leq \left\lVert {f_m-f_k} \right\rVert_u .\] Así\(\{ f_n(x) \}\) converge a algún número real. Definir\(f \colon S \to {\mathbb{R}}\) por\[f(x) := \lim_{n \to \infty} f_n(x) .\] La secuencia\(\{ f_n \}\) converge puntualmente a\(f\). Para demostrar que la convergencia es uniforme,\(\epsilon > 0\) déjese dar. Encuentra un\(N\) tal que para\(m, k \geq N\) nosotros tenemos\(\left\lVert {f_m-f_k} \right\rVert_u < \nicefrac{\epsilon}{2}\). En otras palabras, por todo\(x\) lo que tenemos\(\left\lvert {f_m(x)-f_k(x)} \right\rvert < \nicefrac{\epsilon}{2}\). Tomamos el límite como\(k\) va al infinito. Después\(\left\lvert {f_m(x)-f_k(x)} \right\rvert\) va a\(\left\lvert {f_m(x)-f(x)} \right\rvert\). En consecuencia para todos\(x\) obtenemos\[\left\lvert {f_m(x)-f(x)} \right\rvert \leq \nicefrac{\epsilon}{2} < \epsilon .\] Y de ahí\(\{ f_n \}\) converge de manera uniforme.

    Para la otra dirección, supongamos que\(\{ f_n \}\) converge uniformemente a\(f\). Dado\(\epsilon > 0\), encontramos\(N\) tal que por todos\(n \geq N\) tenemos\(\left\lvert {f_n(x)-f(x)} \right\rvert < \nicefrac{\epsilon}{4}\) para todos\(x \in S\). Por lo tanto para todos\(m, k \geq N\) tenemos\[\left\lvert {f_m(x)-f_k(x)} \right\rvert = \left\lvert {f_m(x)-f(x)+f(x)-f_k(x)} \right\rvert \leq \left\lvert {f_m(x)-f(x)} \right\rvert+\left\lvert {f(x)-f_k(x)} \right\rvert < \nicefrac{\epsilon}{4} + \nicefrac{\epsilon}{4} .\] tomar supremum sobre todos\(x\) para obtener\[\left\lVert {f_m-f_k} \right\rVert_u \leq \nicefrac{\epsilon}{2} < \epsilon . \qedhere\]

    Ejercicios

    Dejar\(f\) y\(g\) estar delimitado funciones encendidas\([a,b]\). Demostrar\[\left\lVert {f+g} \right\rVert_u \leq \left\lVert {f} \right\rVert_u + \left\lVert {g} \right\rVert_u .\]

    a) Encontrar el límite puntual\(\dfrac{e^{x/n}}{n}\) para\(x \in {\mathbb{R}}\).
    b) ¿El límite es uniforme\({\mathbb{R}}\)?
    c) ¿El límite es uniforme\([0,1]\)?

    Supongamos que\(f_n \colon S \to {\mathbb{R}}\) son funciones que convergen uniformemente a\(f \colon S \to {\mathbb{R}}\). Supongamos\(A \subset S\). Demostrar que la secuencia de restricciones\(\{ f_n|_A \}\) converge de manera uniforme a\(f|_A\).

    Supongamos\(\{ f_n \}\) y\(\{ g_n \}\) definidos en algún conjunto\(A\) convergen a\(f\) y\(g\) respectivamente puntualmente. Demostrar que\(\{ f_n+g_n \}\) converge puntualmente a\(f+g\).

    Supongamos\(\{ f_n \}\) y\(\{ g_n \}\) definido en algún conjunto\(A\) convergen a\(f\) y\(g\) respectivamente uniformemente en\(A\). Demostrar que\(\{ f_n+g_n \}\) converge de manera uniforme a\(f+g\) encendido\(A\).

    Encuentra un ejemplo de una secuencia de funciones\(\{ f_n \}\) y\(\{ g_n \}\) que convergen uniformemente a algunos\(f\) y\(g\) en algún conjunto\(A\), pero de tal manera que\(\{ f_ng_n \}\) (el múltiplo) no converge uniformemente a\(fg\) on\(A\). Pista: Vamos\(A := {\mathbb{R}}\), vamos\(f(x):=g(x) := x\). Incluso puedes escoger\(f_n = g_n\).

    Supongamos que existe una secuencia de funciones que convergen\(\{ g_n \}\) uniformemente a\(0\) on\(A\). Ahora supongamos que tenemos una secuencia de funciones\(\{ f_n \}\) y una función\(f\) sobre\(A\) tal que\[\left\lvert {f_n(x) - f(x)} \right\rvert \leq g_n(x)\] para todos\(x \in A\). Espectáculo que\(\{ f_n \}\) converge uniformemente a\(f\) encendido\(A\).

    Dejar\(\{ f_n \}\),\(\{ g_n \}\) y\(\{ h_n \}\) ser secuencias de funciones en\([a,b]\). Supongamos\(\{ f_n \}\) y\(\{ h_n \}\) convergen uniformemente a alguna función\(f \colon [a,b] \to {\mathbb{R}}\) y supongamos\(f_n(x) \leq g_n(x) \leq h_n(x)\) para todos\(x \in [a,b]\). Demostrar que\(\{ g_n \}\) converge de manera uniforme a\(f\).

    Dejar\(f_n \colon [0,1] \to {\mathbb{R}}\) ser una secuencia de funciones crecientes (es decir,\(f_n(x) \geq f_n(y)\) cuando sea\(x \geq y\)). Supongamos\(f_n(0) = 0\) y\(\lim\limits_{n \to \infty} f_n(1) = 0\). Demostrar que\(\{ f_n \}\) converge uniformemente a\(0\).

    Dejar\(\{f_n\}\) ser una secuencia de funciones definidas en\([0,1]\). Supongamos que existe una secuencia de números distintos\(x_n \in [0,1]\) tal que\[f_n(x_n) = 1 .\] Probar o desacredite las siguientes afirmaciones:
    a) Verdadero o falso: Existe\(\{ f_n \}\) como arriba que converge a\(0\) pointwise.
    b) Verdadero o falso: Existe\(\{ f_n \}\) como arriba que converge de manera\(0\) uniforme sobre\([0,1]\).

    Fijar un continuo\(h \colon [a,b] \to {\mathbb{R}}\). Dejemos\(f(x) := h(x)\) para\(x \in [a,b]\),\(f(x) := h(a)\) para\(x < a\) y\(f(x) := h(b)\) para todos\(x > b\). Primer espectáculo que\(f \colon {\mathbb{R}}\to {\mathbb{R}}\) es continuo. Ahora deja\(f_n\) ser la función\(g\) de con\(\epsilon = \nicefrac{1}{n}\), definida en el intervalo\([a,b]\). Espectáculo que\(\{ f_n \}\) converge uniformemente a\(h\) encendido\([a,b]\).

    Intercambio de límites

    Nota: 1—1.5 conferencias

    Gran parte del análisis moderno se ocupa principalmente de la cuestión del intercambio de dos operaciones limitantes. Cuando tenemos una cadena de dos límites, no siempre podemos simplemente intercambiar los límites. Por ejemplo,\[0 = \lim_{n\to\infty} \left( \lim_{k\to\infty} \frac{\nicefrac{n}{k}}{\nicefrac{n}{k} + 1} \right) \not= \lim_{k\to\infty} \left( \lim_{n\to\infty} \frac{\nicefrac{n}{k}}{\nicefrac{n}{k} + 1} \right) = 1 .\]

    Cuando se habla de secuencias de funciones, el intercambio de límites surge con bastante frecuencia. Tratamos dos casos. Primero miramos la continuidad del límite, y segundo miramos la integral del límite.

    Continuidad del límite

    Si tenemos una secuencia\(\{ f_n \}\) de funciones continuas, ¿el límite es continuo? Supongamos que\(f\) es el límite (puntual) de\(\{ f_n \}\). Si\(\lim\, x_k = x\) nos interesa el siguiente intercambio de límites. La igualdad que tenemos que demostrar (no siempre es cierta) está marcada con un signo de interrogación. De hecho, los límites a la izquierda del signo de interrogación tal vez ni siquiera existan. \[\lim_{k \to \infty} f(x_k) = \lim_{k \to \infty} \Bigl( \lim_{n \to \infty} f_n(x_k) \Bigr) \overset{\text{\textbf{?}}}{=} \lim_{n \to \infty} \Bigl( \lim_{k \to \infty} f_n(x_k) \Bigr) = \lim_{n \to \infty} f_n(x) = f(x) .\]En particular, deseamos encontrar condiciones sobre la secuencia\(\{ f_n \}\) para que se mantenga la ecuación anterior. Resulta que si solo requerimos convergencia puntual, entonces el límite de una secuencia de funciones no necesita ser continuo, y la ecuación anterior no necesita sostenerse.

    Dejar\(f_n \colon [0,1] \to {\mathbb{R}}\) definirse como\[f_n(x) := \begin{cases} 1-nx & \text{if $x < \nicefrac{1}{n}$,}\\ 0 & \text{if $x \geq \nicefrac{1}{n}$.} \end{cases}\] Ver.

    Cada función\(f_n\) es continua. Arreglar un\(x \in (0,1]\). Si\(n \geq \nicefrac{1}{x}\), entonces\(x \geq \nicefrac{1}{n}\). Por lo tanto para\(n \geq \nicefrac{1}{x}\) tenemos\(f_n(x) = 0\), y así\[\lim_{n \to \infty} f_n(x) = 0.\] Por otro lado si\(x=0\), entonces\[\lim_{n \to \infty} f_n(0) = \lim_{n \to \infty} 1 = 1.\] Así el límite puntual de\(f_n\) es la función\(f \colon [0,1] \to {\mathbb{R}}\) definida por\[f(x) := \begin{cases} 1 & \text{if $x = 0$,}\\ 0 & \text{if $x > 0$.} \end{cases}\] La función no\(f\) es continua a 0.

    Si, sin embargo, requerimos que la convergencia sea uniforme, los límites pueden ser intercambiados.

    Dejar\(\{ f_n \}\) ser una secuencia de funciones continuas que\(f_n \colon S \to {\mathbb{R}}\) convergen uniformemente a\(f \colon S \to {\mathbb{R}}\). Entonces\(f\) es continuo.

    Dejar que\(x \in S\) se arregle. Dejar\(\{ x_n \}\) ser una secuencia en\(S\) converger a\(x\).

    Dejemos\(\epsilon > 0\) que se den. Como\(\{ f_k \}\) converge uniformemente a\(f\), nos encontramos con un\(k \in {\mathbb{N}}\) tal que\[\left\lvert {f_k(y)-f(y)} \right\rvert < \nicefrac{\epsilon}{3}\] para todos\(y \in S\). Como\(f_k\) es continuo en\(x\), encontramos un\(N \in {\mathbb{N}}\) tal que para\(m \geq N\) nosotros tenemos\[\left\lvert {f_k(x_m)-f_k(x)} \right\rvert < \nicefrac{\epsilon}{3} .\] Así para nosotros\(m \geq N\) tenemos Por\[\begin{split} \left\lvert {f(x_m)-f(x)} \right\rvert & = \left\lvert {f(x_m)-f_k(x_m)+f_k(x_m)-f_k(x)+f_k(x)-f(x)} \right\rvert \\ & \leq \left\lvert {f(x_m)-f_k(x_m)} \right\rvert+ \left\lvert {f_k(x_m)-f_k(x)} \right\rvert+ \left\lvert {f_k(x)-f(x)} \right\rvert \\ & < \nicefrac{\epsilon}{3} + \nicefrac{\epsilon}{3} + \nicefrac{\epsilon}{3} = \epsilon . \end{split}\] lo tanto\(\{ f(x_m) \}\) converge a\(f(x)\) y por lo tanto\(f\) es continuo en\(x\). Como\(x\) fue arbitrario,\(f\) es continuo en todas partes.

    Integral del límite

    Nuevamente, si simplemente requerimos convergencia puntual, entonces la integral de un límite de una secuencia de funciones no necesita ser igual al límite de las integrales.

    Dejar\(f_n \colon [0,1] \to {\mathbb{R}}\) definirse como\[f_n(x) := \begin{cases} 0 & \text{if $x = 0$,}\\ n-n^2x & \text{if $0 < x < \nicefrac{1}{n}$,}\\ 0 & \text{if $x \geq \nicefrac{1}{n}$.} \end{cases}\] Ver.

    Cada uno\(f_n\) es Riemann integrable (es continuo\((0,1]\) y acotado), y es fácil de ver\[\int_0^1 f_n = \int_0^{\nicefrac{1}{n}} (n-n^2x)~dx = \nicefrac{1}{2} .\] Vamos a calcular el límite puntual de\(\{ f_n \}\). Arreglar un\(x \in (0,1]\). Porque\(n \geq \nicefrac{1}{x}\) tenemos\(x \geq \nicefrac{1}{n}\) y así\(f_n(x) = 0\). Por lo tanto también\[\lim_{n \to \infty} f_n(x) = 0.\] tenemos\(f_n(0) = 0\) para todos\(n\). Por lo tanto, el límite puntual de\(\{ f_n \}\) es la función cero. Así\[\nicefrac{1}{2} = \lim_{n\to\infty} \int_0^1 f_n (x)~dx \not= \int_0^1 \left( \lim_{n\to\infty} f_n(x)\right)~dx = \int_0^1 0~dx = 0 .\]

    Pero si nuevamente requerimos que la convergencia sea uniforme, se pueden intercambiar los límites.

    [integralintercambio:thm] Dejar\(\{ f_n \}\) ser una secuencia de funciones integrables de Riemann que\(f_n \colon [a,b] \to {\mathbb{R}}\) convergen uniformemente a\(f \colon [a,b] \to {\mathbb{R}}\). Entonces Riemann\(f\) es integrable y\[\int_a^b f = \lim_{n\to\infty} \int_a^b f_n .\]

    Dejemos\(\epsilon > 0\) que se den. Como\(f_n\) va a\(f\) uniformemente, nos encontramos con un\(M \in {\mathbb{N}}\) tal que por todo\(n \geq M\) lo que tenemos\(\left\lvert {f_n(x)-f(x)} \right\rvert < \frac{\epsilon}{2(b-a)}\) para todos\(x \in [a,b]\). En particular, por triángulo inverso la desigualdad\(\left\lvert {f(x)} \right\rvert < \frac{\epsilon}{2(b-a)} + \left\lvert {f_n(x)} \right\rvert\) para todos\(x\), de ahí\(f\) que esté delimitada como\(f_n\) está acotada. Obsérvese que\(f_n\) es integrable y computo\[\begin{split} \overline{\int_a^b} f - \underline{\int_a^b} f & = \overline{\int_a^b} \bigl( f(x) - f_n(x) + f_n(x) \bigr)~dx - \underline{\int_a^b} \bigl( f(x) - f_n(x) + f_n(x) \bigr)~dx \\ & \leq \overline{\int_a^b} \bigl( f(x) - f_n(x) \bigr)~dx + \overline{\int_a^b} f_n(x) ~dx - \underline{\int_a^b} \bigl( f(x) - f_n(x) \bigr)~dx - \underline{\int_a^b} f_n(x) ~dx \\ & = \overline{\int_a^b} \bigl( f(x) - f_n(x) \bigr)~dx + \int_a^b f_n(x) ~dx - \underline{\int_a^b} \bigl( f(x) - f_n(x) \bigr)~dx - \int_a^b f_n(x) ~dx \\ & = \overline{\int_a^b} \bigl( f(x) - f_n(x) \bigr)~dx - \underline{\int_a^b} \bigl( f(x) - f_n(x) \bigr)~dx \\ & \leq \frac{\epsilon}{2(b-a)} (b-a) + \frac{\epsilon}{2(b-a)} (b-a) = \epsilon . \end{split}\] La primera desigualdad es (sigue como supremum de una suma es menor o igual a la suma de suprema y de manera similar para infima, ver). La segunda desigualdad se desprende de y el hecho de que por todo\(x \in [a,b]\) lo que tenemos\(\frac{-\epsilon}{2(b-a)} < f(x)-f_n(x) < \frac{\epsilon}{2(b-a)}\). Como\(\epsilon > 0\) fue arbitrario,\(f\) es Riemann integrable.

    Finalmente calculamos\(\int_a^b f\). Aplicamos en el cálculo. Nuevamente, para\(n \geq M\) (donde\(M\) es lo mismo que arriba) tenemos\[\begin{split} \left\lvert {\int_a^b f - \int_a^b f_n} \right\rvert & = \left\lvert { \int_a^b \bigl(f(x) - f_n(x)\bigr)~dx} \right\rvert \\ & \leq \frac{\epsilon}{2(b-a)} (b-a) = \frac{\epsilon}{2} < \epsilon . \end{split}\] Por lo tanto\(\{ \int_a^b f_n \}\) converge a\(\int_a^b f\).

    Supongamos que deseamos calcular\[\lim_{n\to\infty} \int_0^1 \frac{nx+ \sin(nx^2)}{n} ~dx .\] Es imposible calcular las integrales para cualquier particular\(n\) usando cálculo ya que no\(\sin(nx^2)\) tiene antiderivada de forma cerrada. Sin embargo, podemos calcular el límite. Hemos demostrado antes que\(\frac{nx+ \sin(nx^2)}{n}\) converge uniformemente sobre\([0,1]\)\(x\). Por, el límite existe y\[\lim_{n\to\infty} \int_0^1 \frac{nx+ \sin(nx^2)}{n} ~dx = \int_0^1 x ~dx = \nicefrac{1}{2} .\]

    Si la convergencia es solo puntual, el límite ni siquiera necesita ser integrable por Riemann. On\([0,1]\) define\[f_n(x) := \begin{cases} 1 & \text{if $x = \nicefrac{p}{q}$ in lowest terms and $q \leq n$,} \\ 0 & \text{otherwise.} \end{cases}\] La función\(f_n\) difiere de la función cero en finitamente muchos puntos; solo hay finitamente muchas fracciones en\([0,1]\) con denominador menor o igual a\(n\). Así\(f_n\) es integrable y\(\int_0^1 f_n = \int_0^1 0 = 0\). Es un ejercicio fácil de mostrar que\(\{ f_n \}\) converge puntualmente a la función Dirichlet\[f(x) := \begin{cases} 1 & \text{if $x \in {\mathbb{Q}}$,} \\ 0 & \text{otherwise,} \end{cases}\] que no es integrable por Riemann.

    De hecho, si la convergencia es solo puntual, el límite de funciones acotadas ni siquiera está necesariamente limitado. Definir\(f_n \colon [0,1] \to {\mathbb{R}}\)\[f_n(x) := \begin{cases} 0 & \text{ if $x < \nicefrac{1}{n}$,}\\ \nicefrac{1}{x} & \text{ else.} \end{cases}\] por Por cada\(n\) obtenemos eso\(\left\lvert {f_n(x)} \right\rvert \leq n\) para todos\(x \in [0,1]\) así las funciones están acotadas. Sin embargo\(f_n\) convergen puntualmente a\[f(x) := \begin{cases} 0 & \text{ if $x = 0$,}\\ \nicefrac{1}{x} & \text{ else,} \end{cases}\] lo que no tiene límites.

    Comentemos que si bien la convergencia uniforme es suficiente para intercambiar límites con integrales, no es, sin embargo, suficiente intercambiar límites con derivados, a menos que también se tenga convergencia uniforme de los propios derivados. Consulta los ejercicios a continuación.

    Ejercicios

    Si bien la convergencia uniforme preserva la continuidad, no preserva la diferenciabilidad. Encuentra un ejemplo explícito de una secuencia de funciones diferenciables sobre las\([-1,1]\) que convergen uniformemente a una función\(f\) tal que no\(f\) es diferenciable. Pista: Considerar\(\left\lvert {x} \right\rvert^{1+1/n}\), mostrar que estas funciones son diferenciables, convergen uniformemente, y luego mostrar que el límite no es diferenciable.

    Vamos\(f_n(x) = \frac{x^n}{n}\). Mostrar que\(\{ f_n \}\) converge uniformemente a una función diferenciable\(f\) on\([0,1]\) (find\(f\)). No obstante, demuéstralo\(f'(1) \not= \lim\limits_{n\to\infty} f_n'(1)\).

    Nota: Los dos ejercicios anteriores muestran que no podemos simplemente intercambiar límites con derivados, aunque la convergencia sea uniforme. Ver también a continuación.

    Let\(f \colon [0,1] \to {\mathbb{R}}\) Ser una función integrable de Riemann (por lo tanto acotada). Encuentra\(\displaystyle \lim_{n\to\infty} \int_0^1 \frac{f(x)}{n} ~dx\).

    Espectáculo\(\displaystyle \lim_{n\to\infty} \int_1^2 e^{-nx^2} ~dx = 0\). Siéntase libre de usar lo que sepa sobre la función exponencial del cálculo.

    Encuentra un ejemplo de una secuencia de funciones continuas en\((0,1)\) que converge puntualmente a una función continua en\((0,1)\), pero la convergencia no es uniforme.

    Nota: En el ejercicio anterior,\((0,1)\) se escogió por simplicidad. Para un ejercicio más desafiante, reemplace\((0,1)\) con\([0,1]\).

    Verdadero/Falso; probar o encontrar un contraejemplo a la siguiente declaración: Si\(\{ f_n \}\) es una secuencia de funciones discontinuas en todas partes\([0,1]\) que convergen uniformemente a una función\(f\), entonces\(f\) es en todas partes discontinuo.

    [c1uniflim:exercise] Para una función continuamente diferenciable\(f \colon [a,b] \to {\mathbb{R}}\), definir\[\left\lVert {f} \right\rVert_{C^1} := \left\lVert {f} \right\rVert_u + \left\lVert {f'} \right\rVert_u .\] Supongamos que\(\{ f_n \}\) es una secuencia de funciones continuamente diferenciables tal que para cada\(\epsilon >0\), existe una\(M\) tal que para todos\(n,k \geq M\) tenemos\[\left\lVert {f_n-f_k} \right\rVert_{C^1} < \epsilon .\] Mostrar eso \(\{ f_n \}\)converge uniformemente a alguna función continuamente diferenciable\(f \colon [a,b] \to {\mathbb{R}}\).

    Para los dos ejercicios siguientes definamos para una función integrable de Riemann\(f \colon [0,1] \to {\mathbb{R}}\) el siguiente número\[\left\lVert {f} \right\rVert_{L^1} := \int_0^1 \left\lvert {f(x)} \right\rvert~dx .\] Es cierto que\(\left\lvert {f} \right\rvert\) es integrable siempre que\(f\) sea, ver. Esta norma define otro tipo de convergencia muy común llamado\(L^1\) -convergencia, que sin embargo es un poco más sutil.

    Supongamos que\(\{ f_n \}\) es una secuencia de funciones integrables de Riemann sobre la\([0,1]\) que converge uniformemente a\(0\). Demostrar que\[\lim_{n\to\infty} \left\lVert {f_n} \right\rVert_{L^1} = 0 .\]

    Encuentre una secuencia de funciones\(\{ f_n \}\) integrables de Riemann\([0,1]\) que converja puntualmente a\(0\), pero\[\lim_{n\to\infty} \left\lVert {f_n} \right\rVert_{L^1} \text{ does not exist (is $\infty$).}\]

    Demostrar el teorema de Dini: Let\(f_n \colon [a,b] \to {\mathbb{R}}\) be a sequence of continuous functions such that\[0 \leq f_{n+1}(x) \leq f_n(x) \leq \cdots \leq f_1(x) \qquad \text{for all $n \in {\mathbb{N}}$.}\] Supongamos\(\{ f_n \}\) converge pointwise to\(0\). Espectáculo que\(\{ f_n \}\) converge a cero de manera uniforme.

    Supongamos que\(f_n \colon [a,b] \to {\mathbb{R}}\) es una secuencia de funciones continuas que converge puntualmente a una continua\(f \colon [a,b] \to {\mathbb{R}}\). Supongamos que para cualquiera\(x \in [a,b]\) la secuencia\(\{ \left\lvert {f_n(x)-f(x)} \right\rvert \}\) es monótona. Demostrar que la secuencia\(\{f_n\}\) converge de manera uniforme.

    Encontrar una secuencia de funciones integrables de Riemann\(f_n \colon [0,1] \to {\mathbb{R}}\) tal que\(\{ f_n \}\) converja a cero puntualmente, y tal que a)\(\bigl\{ \int_0^1 f_n \bigr\}_{n=1}^\infty\) aumenta sin ataduras, b)\(\bigl\{ \int_0^1 f_n \bigr\}_{n=1}^\infty\) es la secuencia\(-1,1,-1,1,-1,1, \ldots\).

    Es posible definir un límite conjunto de una secuencia doble\(\{ x_{n,m} \}\) de números reales (es decir, una función de\({\mathbb{N}}\times {\mathbb{N}}\) a\({\mathbb{R}}\)). Decimos\(L\) es el límite conjunto de\(\{ x_{n,m} \}\) y escribimos\[\lim_{\substack{n\to\infty\\m\to\infty}} x_{n,m} = L , \qquad \text{or} \qquad \lim_{(n,m) \to \infty} x_{n,m} = L ,\] si por cada\(\epsilon > 0\), existe\(M\) tal que si\(n \geq M\) y\(m \geq M\), entonces\(\left\lvert {x_{n,m} - L} \right\rvert < \epsilon\).

    Supongamos que el límite conjunto de\(\{ x_{n,m} \}\) es\(L\), y supongamos que para todos\(n\),\(\lim\limits_{m \to \infty} x_{n,m}\) existe, y para todos\(m\),\(\lim\limits_{n \to \infty} x_{n,m}\) existe. Después mostrar\(\lim\limits_{n\to\infty}\lim\limits_{m \to \infty} x_{n,m} = \lim\limits_{m\to\infty}\lim\limits_{n \to \infty} x_{n,m} = L\).

    Un límite conjunto no significa que los límites iterados existan. Considerar\(x_{n,m} := \frac

    ParseError: EOF expected (click for details)
    Callstack:
        at (Matematicas/Analisis/Introducción_al_Análisis_Real_(Lebl)/02:_Números_reales/2.01:_Propiedades_básicas), /content/body/div[36]/p[18]/span[1]/span, line 1, column 5
    
    {\min \{n,m \}}\).
    a) Demostrar que para\(n\) no\(\lim\limits_{m \to \infty} x_{n,m}\) existe, y para\(m\) no\(\lim\limits_{n \to \infty} x_{n,m}\) existe. Entonces,\(\lim\limits_{n\to\infty}\lim\limits_{m \to \infty} x_{n,m}\) ni tiene ningún\(\lim\limits_{m\to\infty}\lim\limits_{n \to \infty} x_{n,m}\) sentido en absoluto.
    b) Demostrar que el límite conjunto de\(\{ x_{n,m} \}\) existe y es 0.

    Teorema de Picard

    Nota: 1—2 conferencias (se pueden omitir de forma segura)

    Un curso de primer semestre en análisis debe tener un teorema de calibre pièce de résistance. Escogemos un teorema cuya prueba combina todo lo que hemos aprendido. Es más sofisticado que el teorema fundamental del cálculo, el primer teorema más destacado de este curso. El teorema del que estamos hablando es el teorema 27 de Picard sobre la existencia y singularidad de una solución a una ecuación diferencial ordinaria. Tanto la declaración como la prueba son hermosos ejemplos de lo que se puede hacer con todo lo que hemos aprendido. También es un buen ejemplo de cómo se aplica el análisis ya que las ecuaciones diferenciales son indispensables en la ciencia.

    Ecuación diferencial ordinaria de primer orden

    La ciencia moderna se describe en el lenguaje de las ecuaciones diferenciales. Es decir, ecuaciones que involucran no sólo lo desconocido, sino también sus derivadas. La forma no trivial más simple de una ecuación diferencial es la denominada ecuación diferencial ordinaria de primer orden\[y' = F(x,y) .\] Generalmente también especificamos\(y(x_0)=y_0\). La solución de la ecuación es una función\(y(x)\) tal que\(y(x_0)=y_0\) y\(y'(x) = F\bigl(x,y(x)\bigr)\).

    Cuando\(F\) involucra sólo la\(x\) variable, la solución viene dada por el teorema fundamental del cálculo. Por otro lado, cuando\(F\) depende de ambos\(x\) y\(y\) necesitamos mucha más potencia de fuego. No siempre es cierto que existe una solución, y si lo hace, que es la solución única. El teorema de Picard nos da ciertas condiciones suficientes para la existencia y la singularidad.

    El teorema

    Necesitamos una definición de continuidad en dos variables. Primero, un punto en el plano\({\mathbb{R}}^2 = {\mathbb{R}}\times {\mathbb{R}}\) es denotado por un par ordenado\((x,y)\). Para simplificar las cosas, demos la siguiente definición secuencial de continuidad.

    Dejar\(U \subset {\mathbb{R}}^2\) ser un conjunto y\(F \colon U \to {\mathbb{R}}\) ser una función. \((x,y) \in U\)Sea un punto. La función\(F\) es continua en\((x,y)\) si por cada secuencia\(\{ (x_n,y_n) \}_{n=1}^\infty\) de puntos en\(U\) tal que\(\lim\, x_n = x\) y\(\lim\, y_n = y\), tenemos\[\lim_{n \to \infty} F(x_n,y_n) = F(x,y) .\] Decimos\(F\) es continua si es continua en todos los puntos en\(U\).

    Dejar que\(I, J \subset {\mathbb{R}}\) se cierren intervalos acotados, dejar\(I_0\) y\(J_0\) ser sus interiores, y dejar\((x_0,y_0) \in I_0 \times J_0\). Supongamos que\(F \colon I \times J \to {\mathbb{R}}\) es continuo y Lipschitz en la segunda variable, es decir, existe un número\(L\) tal que\[\left\lvert {F(x,y) - F(x,z)} \right\rvert \leq L \left\lvert {y-z} \right\rvert \ \ \ \text{ for all $y,z \in J$, $x \in I$} .\] Entonces existe una\(h > 0\) y una función diferenciable única\(f \colon [x_0 - h, x_0 + h] \to J \subset {\mathbb{R}}\), tal que\[\label{picard:diffeq} f'(x) = F\bigl(x,f(x)\bigr) \qquad \text{and} \qquad f(x_0) = y_0.\]

    Supongamos que podríamos encontrar una solución\(f\). Usando el teorema fundamental del cálculo integramos la ecuación\(f'(x) = F\bigl(x,f(x)\bigr)\),\(f(x_0) = y_0\), y escribimos [picard:diffeq] como la ecuación integral\[\label{picard:inteq} f(x) = y_0 + \int_{x_0}^x F\bigl(t,f(t)\bigr)~dt .\] La idea de nuestra prueba es que tratamos de enchufar aproximaciones a una solución al lado derecho de [picard:inteq] para obtener mejores aproximaciones en el lado izquierdo de [picard:inteq]. Esperamos que al final la secuencia converja y resuelva [picard:inteq] y de ahí [picard:diffeq]. La siguiente técnica se llama iteración Picard, y las funciones individuales\(f_k\) se llaman iteraciones Picard.

    Sin pérdida de generalidad, supongamos\(x_0 = 0\) (ejercicio a continuación). Otro ejercicio nos dice que\(F\) está acotado ya que es continuo. Por lo tanto escoge algunos\(M > 0\)\(\left\lvert {F(x,y)} \right\rvert \leq M\) para que para todos\((x,y) \in I\times J\). Escoge\(\alpha > 0\) tal que\([-\alpha,\alpha] \subset I\) y\([y_0-\alpha, y_0 + \alpha] \subset J\). Definir\[h := \min \left\{ \alpha, \frac{\alpha}{M+L\alpha} \right\} .\] Observar\([-h,h] \subset I\).

    Set\(f_0(x) := y_0\). Definimos\(f_k\) inductivamente. Asumiendo\(f_{k-1}([-h,h]) \subset [y_0-\alpha,y_0+\alpha]\), vemos\(F\bigl(t,f_{k-1}(t)\bigr)\) es una función bien definida de\(t\) for\(t \in [-h,h]\). Además si\(f_{k-1}\) es continuo\([-h,h]\), entonces\(F\bigl(t,f_{k-1}(t)\bigr)\) es continuo en función de\(t\) on\([-h,h]\) (izquierda como ejercicio). Definir\[f_k(x) := y_0+ \int_{0}^x F\bigl(t,f_{k-1}(t)\bigr)~dt ,\] y\(f_k\) continuar\([-h,h]\) por el teorema fundamental del cálculo. Para ver que se\(f_k\) mapea\([-h,h]\) a\([y_0-\alpha,y_0+\alpha]\),\(x \in [-h,h]\)\[\left\lvert {f_k(x) - y_0} \right\rvert = \left\lvert {\int_{0}^x F\bigl(t,f_{k-1}(t)\bigr)~dt } \right\rvert \leq M\left\lvert {x} \right\rvert \leq Mh \leq M \frac{\alpha}{M+L\alpha} \leq \alpha .\] calculamos para Ahora definimos\(f_{k+1}\) y así sucesivamente, y hemos definido una secuencia\(\{ f_k \}\) de funciones. Necesitamos demostrar que converge a una función\(f\) que resuelva la ecuación [picard:inteq] y por lo tanto [picard:diffeq].

    Deseamos demostrar que la secuencia\(\{ f_k \}\) converge de manera uniforme a alguna función en\([-h,h]\). Primero, porque\(t \in [-h,h]\) tenemos el siguiente encuadernado útil\[\left\lvert {F\bigl(t,f_{n}(t)\bigr) - F\bigl(t,f_{k}(t)\bigr)} \right\rvert \leq L \left\lvert {f_n(t)-f_k(t)} \right\rvert \leq L \left\lVert {f_n-f_k} \right\rVert_u ,\] donde\(\left\lVert {f_n-f_k} \right\rVert_u\) está la norma uniforme, esa es la suprema de\(\left\lvert {f_n(t)-f_k(t)} \right\rvert\) para\(t \in [-h,h]\). Ahora tenga en cuenta eso\(\left\lvert {x} \right\rvert \leq h \leq \frac{\alpha}{M+L\alpha}\). Por lo tanto,\[\begin{split} \left\lvert {f_n(x) - f_k(x)} \right\rvert & = \left\lvert {\int_{0}^x F\bigl(t,f_{n-1}(t)\bigr)~dt - \int_{0}^x F\bigl(t,f_{k-1}(t)\bigr)~dt} \right\rvert \\ & = \left\lvert {\int_{0}^x F\bigl(t,f_{n-1}(t)\bigr)- F\bigl(t,f_{k-1}(t)\bigr)~dt} \right\rvert \\ & \leq L\left\lVert {f_{n-1}-f_{k-1}} \right\rVert_u \left\lvert {x} \right\rvert \\ & \leq \frac{L\alpha}{M+L\alpha} \left\lVert {f_{n-1}-f_{k-1}} \right\rVert_u . \end{split}\]\(C := \frac{L\alpha}{M+L\alpha}\) tomemos nota de eso\(C < 1\). Tomando supremum por el lado izquierdo obtenemos\[\left\lVert {f_n-f_k} \right\rVert_u \leq C \left\lVert {f_{n-1}-f_{k-1}} \right\rVert_u .\] Sin pérdida de generalidad, supongamos\(n \geq k\). Entonces por podemos mostrar Porque\(x \in [-h,h]\) tenemos\[\left\lVert {f_n-f_k} \right\rVert_u \leq C^{k} \left\lVert {f_{n-k}-f_{0}} \right\rVert_u .\] Por\[\left\lvert {f_{n-k}(x)-f_{0}(x)} \right\rvert = \left\lvert {f_{n-k}(x)-y_0} \right\rvert \leq \alpha .\] lo tanto,\[\left\lVert {f_n-f_k} \right\rVert_u \leq C^{k} \left\lVert {f_{n-k}-f_{0}} \right\rVert_u \leq C^{k} \alpha .\] As\(C < 1\),\(\{f_n\}\) es uniformemente Cauchy y por obtenemos que\(\{ f_n \}\) converge uniformemente sobre\([-h,h]\) alguna función\(f \colon [-h,h] \to {\mathbb{R}}\). La función\(f\) es el límite uniforme de funciones continuas y por lo tanto continuas. Más lejos desde todos los\(f_n([-h,h]) \subset [y_0-\alpha,y_0+\alpha]\), entonces\(f([-h,h]) \subset [y_0-\alpha,y_0+\alpha]\) (¿por qué?).

    Ahora necesitamos mostrar que\(f\) resuelve [picard:inteq]. Primero, como antes notamos\[\left\lvert {F\bigl(t,f_{n}(t)\bigr) - F\bigl(t,f(t)\bigr)} \right\rvert \leq L \left\lvert {f_n(t)-f(t)} \right\rvert \leq L \left\lVert {f_n-f} \right\rVert_u .\] As\(\left\lVert {f_n-f} \right\rVert_u\) converge a 0, luego\(F\bigl(t,f_n(t)\bigr)\) converge uniformemente a\(F\bigl(t,f(t)\bigr)\) for\(t \in [-h,h]\). De ahí\(x \in [-h,h]\) que para la convergencia sea uniforme para\(t \in [0,x]\) (o\([x,0]\) si\(x < 0\)). Por lo tanto,\[\begin{aligned} y_0 + \int_0^{x} F(t,f(t)\bigr)~dt & = y_0 + \int_0^{x} F\bigl(t,\lim_{n\to\infty} f_n(t)\bigr)~dt & & \\ & = y_0 + \int_0^{x} \lim_{n\to\infty} F\bigl(t,f_n(t)\bigr)~dt & & \text{(by continuity of $F$)} \\ & = \lim_{n\to\infty} \left( y_0 + \int_0^{x} F\bigl(t,f_n(t)\bigr)~dt \right) & & \text{(by uniform convergence)} \\ & = \lim_{n\to\infty} f_{n+1}(x) = f(x) . & &\end{aligned}\] aplicamos el teorema fundamental del cálculo para mostrar que\(f\) es diferenciable y su derivada es\(F\bigl(x,f(x)\bigr)\). Es obvio que\(f(0) = y_0\).

    Por último, lo que queda por hacer es mostrar singularidad. Supongamos que\(g \colon [-h,h] \to J \subset {\mathbb{R}}\) es otra solución. Como antes usamos el hecho de que\(\left\lvert {F\bigl(t,f(t)\bigr) - F\bigl(t,g(t)\bigr)} \right\rvert \leq L \left\lVert {f-g} \right\rVert_u\). Entonces\[\begin{split} \left\lvert {f(x)-g(x)} \right\rvert & = \left\lvert { y_0 + \int_0^{x} F\bigl(t,f(t)\bigr)~dt - \left( y_0 + \int_0^{x} F\bigl(t,g(t)\bigr)~dt \right) } \right\rvert \\ & = \left\lvert { \int_0^{x} F\bigl(t,f(t)\bigr) - F\bigl(t,g(t)\bigr)~dt } \right\rvert \\ & \leq L\left\lVert {f-g} \right\rVert_u\left\lvert {x} \right\rvert \leq Lh\left\lVert {f-g} \right\rVert_u \leq \frac{L\alpha}{M+L\alpha}\left\lVert {f-g} \right\rVert_u . \end{split}\] como antes,\(C = \frac{L\alpha}{M+L\alpha} < 1\). Al tomar supremum\(x \in [-h,h]\) sobre el lado izquierdo obtenemos\[\left\lVert {f-g} \right\rVert_u \leq C \left\lVert {f-g} \right\rVert_u .\] Esto sólo es posible si\(\left\lVert {f-g} \right\rVert_u = 0\). Por lo tanto\(f=g\),, y la solución es única.

    Ejemplos

    Veamos algunos ejemplos. La prueba del teorema nos da una manera explícita de encontrar una\(h\) que funcione. No nos da, sin embargo, lo mejor\(h\). A menudo es posible encontrar una mucho más grande\(h\) para la que se sostiene la conclusión del teorema.

    La prueba también nos da las iteraciones de Picard como aproximaciones a la solución. Entonces la prueba en realidad nos dice cómo obtener la solución, no sólo que la solución existe.

    Considera Es\[f'(x) = f(x), \qquad f(0) = 1 .\] decir\(F(x,y) = y\), dejamos, y estamos buscando una función\(f\) tal que\(f'(x) = f(x)\). Escogemos cualquiera\(I\) que contenga 0 en el interior. Escogemos una arbitraria\(J\) que contiene 1 en su interior. Podemos usar\(L = 1\). El teorema garantiza\(h > 0\) tal que existe una solución única\(f \colon [-h,h] \to {\mathbb{R}}\). Esta solución suele ser denotada por\[e^x := f(x) .\] Dejamos al lector verificar que al escoger\(I\) y lo suficientemente\(J\) grande la prueba del teorema garantiza que somos capaces de escoger de\(\alpha\) tal manera que obtengamos cualquiera que\(h\) queramos siempre y cuando\(h < \nicefrac{1}{2}\). Omitimos el cálculo.

    Por supuesto, sabemos que esta función existe como una función para todos\(x\), así que una arbitraria\(h\) debería funcionar. Por el mismo razonamiento anterior, no importa qué\(x_0\) y\(y_0\) sean, la prueba garantiza una arbitraria\(h\) siempre y cuando\(h < \nicefrac{1}{2}\). Arreglar tal\(h\). Obtenemos una función única definida en\([x_0-h,x_0+h]\). Después de definir la función en\([-h,h]\) encontramos una solución en el intervalo\([0,2h]\) y notamos que las dos funciones deben coincidir\([0,h]\) por singularidad. Así construimos iterativamente lo exponencial para todos\(x \in {\mathbb{R}}\). Por lo tanto, el teorema de Picard podría utilizarse para probar la existencia y singularidad de lo exponencial.

    Calculemos las iteraciones de Picard. Comenzamos con la función constante\(f_0(x) := 1\). Entonces\[\begin{aligned} f_1(x) & = 1 + \int_0^x f_0(s)~ds = 1+x, \\ f_2(x) & = 1 + \int_0^x f_1(s)~ds = 1 + \int_0^x (1+s)~ds = 1 + x + \frac{x^2}{2}, \\ f_3(x) & = 1 + \int_0^x f_2(s)~ds = 1 + \int_0^x \left(1+ s + \frac{s^2}{2} \right)~ds = 1 + x + \frac{x^2}{2} + \frac{x^3}{6} .\end{aligned}\] Reconocemos el inicio de la serie Taylor por lo exponencial.

    Supongamos que tenemos la ecuación\[f'(x) = {\bigl(f(x)\bigr)}^2 \qquad \text{and} \qquad f(0)=1.\] A partir de ecuaciones diferenciales elementales sabemos que\[f(x) = \frac{1}{1-x}\] es la solución. La solución sólo se define en\((-\infty,1)\). Es decir, somos capaces de usar\(h < 1\), pero nunca uno más grande\(h\). La función que lleva\(y\) a no\(y^2\) es Lipschitz como una función en todos\({\mathbb{R}}\). A medida que nos acercamos\(x=1\) desde la izquierda, la solución se hace cada vez más grande. El derivado de la solución crece como\(y^2\), y por lo tanto el\(L\) requerido tendrá que ser más y más grande a\(y_0\) medida que crezca. Así si aplicamos el teorema con\(x_0\) cerca de 1 y\(y_0 = \frac{1}{1-x_0}\) encontramos que el\(h\) que la prueba garantiza será cada vez más pequeño a medida que se\(x_0\) aproxime a 1.

    Al escoger\(\alpha\) correctamente, la prueba del teorema garantiza\(h=1-\nicefrac{\sqrt{3}}{2} \approx 0.134\) (omitimos el cálculo) para\(x_0=0\) y\(y_0=1\), aunque vimos anteriormente que cualquiera\(h < 1\) debería funcionar.

    Considera la ecuación\[f'(x) = 2 \sqrt{\left\lvert {f(x)} \right\rvert}, \qquad f(0) = 0 .\] La función\(F(x,y) = 2 \sqrt{\left\lvert {y} \right\rvert}\) es continua, pero no Lipschitz en\(y\) (¿por qué?). La ecuación no satisface las hipótesis del teorema. La función\[f(x) = \begin{cases} x^2 & \text{ if $x \geq 0$,}\\ -x^2 & \text{ if $x < 0$,} \end{cases}\] es una solución, pero también\(f(x) = 0\) es una solución. Existe una solución, pero no es única.

    Considera\(y' = \varphi(x)\) dónde\(\varphi(x) := 0\) si\(x \in {\mathbb{Q}}\) y\(\varphi(x):=1\) si\(x \not\in {\mathbb{Q}}\). La ecuación no tiene solución independientemente de las condiciones iniciales. Una solución tendría derivada\(\varphi\), pero\(\varphi\) no tiene la propiedad de valor intermedio en ningún momento (¿por qué?). No existe solución por. Por lo tanto, para obtener la existencia de una solución,\(F\) es necesaria alguna hipótesis de continuidad.

    Ejercicios

    Dejar\(I, J \subset {\mathbb{R}}\) ser intervalos. Dejar\(F \colon I \times J \to {\mathbb{R}}\) ser una función continua de dos variables y supongamos\(f \colon I \to J\) ser una función continua. Demostrar que\(F\bigl(x,f(x)\bigr)\) es una función continua en\(I\).

    Dejar que\(I, J \subset {\mathbb{R}}\) se cierren intervalos acotados. Demostrar que si\(F \colon I \times J \to {\mathbb{R}}\) es continuo, entonces\(F\) está acotado.

    Demostramos el teorema de Picard bajo el supuesto de que\(x_0 = 0\). Demostrar la declaración completa del teorema de Picard para una arbitraria\(x_0\).

    \(f'(x)=x f(x)\)Sea nuestra ecuación. Comience con la condición inicial\(f(0)=2\) y encuentre las iteraciones de Picard\(f_0,f_1,f_2,f_3,f_4\).

    Supongamos que\(F \colon I \times J \to {\mathbb{R}}\) es una función que es continua en la primera variable, es decir, para cualquier fijo\(y\) la función que lleva\(x\) a\(F(x,y)\) es continua. Además, supongamos que\(F\) es Lipschitz en la segunda variable, es decir, existe un número\(L\) tal que\[\left\lvert {F(x,y) - F(x,z)} \right\rvert \leq L \left\lvert {y-z} \right\rvert \ \ \ \text{ for all $y,z \in J$, $x \in I$} .\] Show que\(F\) es continuo en función de dos variables. Por lo tanto, las hipótesis en el teorema podrían hacerse aún más débiles.

    Un tipo común de ecuación que uno encuentra son ecuaciones diferenciales lineales de primer orden, es decir, ecuaciones de la forma\[y' + p(x) y = q(x) , \qquad y(x_0) = y_0 .\] Probar el teorema de Picard para ecuaciones lineales. Supongamos que\(I\) es un intervalo\(x_0 \in I\),,\(p \colon I \to {\mathbb{R}}\) y\(q \colon I \to {\mathbb{R}}\) son continuos. Demostrar que existe un diferenciable único\(f \colon I \to {\mathbb{R}}\), tal que\(y = f(x)\) satisface la ecuación y la condición inicial. Pista: Asumir la existencia de la función exponencial y utilizar la fórmula del factor integrador para la existencia de\(f\) (probar que funciona):\[f(x) := e^{-\int_{x_0}^x p(s)\, ds} \left( \int_{x_0}^x e^{\int_{x_0}^t p(s)\, ds} q(t) ~dt + y_0 \right).\]

    Espacios métricos

    Espacios métricos

    Nota: 1.5 conferencias

    Como se mencionó en la introducción, la idea principal en el análisis es tomar límites. En aprendimos a tomar límites de secuencias de números reales. Y en aprendimos a tomar límites de funciones ya que un número real se acercaba a algún otro número real.

    Queremos tomar límites en contextos más complicados. Por ejemplo, queremos tener secuencias de puntos en el espacio tridimensional. Deseamos definir funciones continuas de varias variables. Incluso queremos definir funciones en espacios que son un poco más difíciles de describir, como la superficie de la tierra. Todavía queremos hablar de límites ahí.

    Por último, hemos visto el límite de una secuencia de funciones en. Queremos unificar todas estas nociones para que no tengamos que reprobar teoremas una y otra vez en cada contexto. El concepto de un espacio métrico es una herramienta elemental pero poderosa en el análisis. Y si bien no es suficiente describir cada tipo de límite que encontramos en el análisis moderno, realmente nos lleva muy lejos.

    Dejar\(X\) ser un conjunto, y dejar\(d \colon X \times X \to {\mathbb{R}}\) ser una función tal que

    1. [métrica:pos]\(d(x,y) \geq 0\) para todos\(x, y\) en\(X\),
    2. [métrica:cero]\(d(x,y) = 0\) si y sólo si\(x = y\),
    3. [métrica:com]\(d(x,y) = d(y,x)\),
    4. [métrica:triang]\(d(x,z) \leq d(x,y)+ d(y,z)\) (desigualdad triangular).

    Entonces el par\((X,d)\) se llama un espacio métrico. La función\(d\) se llama la función métrica o, a veces, la función de distancia. A veces solo decimos que\(X\) es un espacio métrico si la métrica es clara a partir del contexto.

    La idea geométrica es que\(d\) es la distancia entre dos puntos. Los ítems [metric:pos][metric:com] tienen una interpretación geométrica obvia: la distancia siempre es no negativa, el único punto que está lejos de la distancia 0\(x\) es\(x\) en sí mismo, y finalmente que la distancia de\(x\) a\(y\) es la misma que la distancia desde \(y\)a\(x\). La desigualdad triangular [métrica:triang] tiene la interpretación dada en.

    Para fines de dibujar, es conveniente dibujar figuras y diagramas en el plano y tener la métrica sea la distancia estándar. Sin embargo, ese es solo un espacio métrico en particular. El hecho de que un hecho determinado parezca quedar claro a partir de dibujar un cuadro no significa que sea cierto. Es posible que se esté desviando por la intuición de la geometría euclidiana, mientras que el concepto de un espacio métrico es mucho más general.

    Demos algunos ejemplos de espacios métricos.

    El conjunto de números reales\({\mathbb{R}}\) es un espacio métrico con la métrica\[d(x,y) := \left\lvert {x-y} \right\rvert .\] Items [metric:pos][metric:com] de la definición son fáciles de verificar. La desigualdad del triángulo [métrica:triang] sigue inmediatamente de la desigualdad triangular estándar para números reales:\[d(x,z) = \left\lvert {x-z} \right\rvert = \left\lvert {x-y+y-z} \right\rvert \leq \left\lvert {x-y} \right\rvert+\left\lvert {y-z} \right\rvert = d(x,y)+ d(y,z) .\] Esta métrica es la métrica estándar en\({\mathbb{R}}\). Si hablamos de\({\mathbb{R}}\) como un espacio métrico sin mencionar una métrica específica, nos referimos a esta métrica en particular.

    También podemos poner una métrica diferente en el conjunto de números reales. Por ejemplo, tomar el conjunto de números reales\({\mathbb{R}}\) junto con la métrica\[d(x,y) := \frac{\left\lvert {x-y} \right\rvert}{\left\lvert {x-y} \right\rvert+1} .\] Los elementos [metric:pos][metric:com] vuelven a ser fáciles de verificar. La desigualdad triangular [métrica:triang] es un poco más difícil. Tenga en cuenta que\(d(x,y) = \varphi(\left\lvert {x-y} \right\rvert)\) donde\(\varphi(t) = \frac{t}{t+1}\) y\(\varphi\) es una función creciente (derivada positiva). De ahí\[\begin{split} d(x,z) & = \varphi(\left\lvert {x-z} \right\rvert) = \varphi(\left\lvert {x-y+y-z} \right\rvert) \leq \varphi(\left\lvert {x-y} \right\rvert+\left\lvert {y-z} \right\rvert) \\ & = \frac{\left\lvert {x-y} \right\rvert+\left\lvert {y-z} \right\rvert}{\left\lvert {x-y} \right\rvert+\left\lvert {y-z} \right\rvert+1} = \frac{\left\lvert {x-y} \right\rvert}{\left\lvert {x-y} \right\rvert+\left\lvert {y-z} \right\rvert+1} + \frac{\left\lvert {y-z} \right\rvert}{\left\lvert {x-y} \right\rvert+\left\lvert {y-z} \right\rvert+1} \\ & \leq \frac{\left\lvert {x-y} \right\rvert}{\left\lvert {x-y} \right\rvert+1} + \frac{\left\lvert {y-z} \right\rvert}{\left\lvert {y-z} \right\rvert+1} = d(x,y)+ d(y,z) . \end{split}\] Aquí tenemos un ejemplo de una métrica no estándar en\({\mathbb{R}}\). Con esta métrica vemos por ejemplo eso\(d(x,y) < 1\) para todos\(x,y \in {\mathbb{R}}\). Es decir, dos puntos cualesquiera están separados a menos de 1 unidad.

    Un espacio métrico importante es el espacio euclidiano\(n\) -dimensional\({\mathbb{R}}^n = {\mathbb{R}} \times {\mathbb{R}}\times \cdots \times {\mathbb{R}}\). Utilizamos la siguiente notación para los puntos:\(x =(x_1,x_2,\ldots,x_n) \in {\mathbb{R}}^n\). También simplemente escribimos\(0 \in {\mathbb{R}}^n\) para significar el vector\((0,0,\ldots,0)\). Antes de hacer\({\mathbb{R}}^n\) un espacio métrico, probemos una desigualdad importante, la llamada desigualdad Cauchy-Schwarz.

    Tomar\(x =(x_1,x_2,\ldots,x_n) \in {\mathbb{R}}^n\) y\(y =(y_1,y_2,\ldots,y_n) \in {\mathbb{R}}^n\). Entonces\[{\biggl( \sum_{j=1}^n x_j y_j \biggr)}^2 \leq \biggl(\sum_{j=1}^n x_j^2 \biggr) \biggl(\sum_{j=1}^n y_j^2 \biggr) .\]

    Cualquier cuadrado de un número real no es negativo. De ahí que cualquier suma de cuadrados no sea negativa:\[\begin{split} 0 & \leq \sum_{j=1}^n \sum_{k=1}^n {(x_j y_k - x_k y_j)}^2 \\ & = \sum_{j=1}^n \sum_{k=1}^n \bigl( x_j^2 y_k^2 + x_k^2 y_j^2 - 2 x_j x_k y_j y_k \bigr) \\ & = \biggl( \sum_{j=1}^n x_j^2 \biggr) \biggl( \sum_{k=1}^n y_k^2 \biggr) + \biggl( \sum_{j=1}^n y_j^2 \biggr) \biggl( \sum_{k=1}^n x_k^2 \biggr) - 2 \biggl( \sum_{j=1}^n x_j y_j \biggr) \biggl( \sum_{k=1}^n x_k y_k \biggr) \end{split}\] Reetiquetamos y dividimos por 2 para obtener\[0 \leq \biggl( \sum_{j=1}^n x_j^2 \biggr) \biggl( \sum_{j=1}^n y_j^2 \biggr) - {\biggl( \sum_{j=1}^n x_j y_j \biggr)}^2 ,\] que es precisamente lo que queríamos.

    Construyamos la métrica estándar para\({\mathbb{R}}^n\). Define\[d(x,y) := \sqrt{ {(x_1-y_1)}^2 + {(x_2-y_2)}^2 + \cdots + {(x_n-y_n)}^2 } = \sqrt{ \sum_{j=1}^n {(x_j-y_j)}^2 } .\] For\(n=1\), la línea real, esta métrica concuerda con lo que hicimos anteriormente. Nuevamente, la única parte complicada de la definición a verificar es la desigualdad triangular. Es menos desordenado trabajar con el cuadrado de la métrica. A continuación, anotar el uso de la desigualdad Cauchy-Schwarz. \[\begin{split} {\bigl(d(x,z)\bigr)}^2 & = \sum_{j=1}^n {(x_j-z_j)}^2 \\ & = \sum_{j=1}^n {(x_j-y_j+y_j-z_j)}^2 \\ & = \sum_{j=1}^n \Bigl( {(x_j-y_j)}^2+{(y_j-z_j)}^2 + 2(x_j-y_j)(y_j-z_j) \Bigr) \\ & = \sum_{j=1}^n {(x_j-y_j)}^2 + \sum_{j=1}^n {(y_j-z_j)}^2 + \sum_{j=1}^n 2(x_j-y_j)(y_j-z_j) \\ & \leq \sum_{j=1}^n {(x_j-y_j)}^2 + \sum_{j=1}^n {(y_j-z_j)}^2 + 2 \sqrt{ \sum_{j=1}^n {(x_j-y_j)}^2 \sum_{j=1}^n {(y_j-z_j)}^2 } \\ & = {\left( \sqrt{ \sum_{j=1}^n {(x_j-y_j)}^2 } + \sqrt{ \sum_{j=1}^n {(y_j-z_j)}^2 } \right)}^2 = {\bigl( d(x,y) + d(y,z) \bigr)}^2 . \end{split}\]Tomando la raíz cuadrada de ambos lados obtenemos la desigualdad correcta, porque la raíz cuadrada es una función creciente.

    Un ejemplo a tener en cuenta es la llamada métrica discreta. \(X\)Sea cualquier conjunto y defina Es\[d(x,y) := \begin{cases} 1 & \text{if $x \not= y$}, \\ 0 & \text{if $x = y$}. \end{cases}\] decir, todos los puntos están igualmente distantes entre sí. Cuando\(X\) es un conjunto finito, podemos dibujar un diagrama, ver por ejemplo. Las cosas se vuelven sutiles cuando\(X\) hay un conjunto infinito como los números reales.

    Si bien este ejemplo en particular rara vez aparece en la práctica, da una útil “prueba de olfato”. Si haces una declaración sobre los espacios métricos, pruébalo con la métrica discreta. Para mostrar que efectivamente\((X,d)\) es un espacio métrico se deja como ejercicio.

    [Ejemplo:MSC01] Let\(C([a,b],{\mathbb{R}})\) Ser el conjunto de funciones continuas de valor real en el intervalo\([a,b]\). Definir la métrica on\(C([a,b],{\mathbb{R}})\) como\[d(f,g) := \sup_{x \in [a,b]} \left\lvert {f(x)-g(x)} \right\rvert .\] Comprobemos las propiedades. Primero,\(d(f,g)\) es finito al igual\(\left\lvert {f(x)-g(x)} \right\rvert\) que es una función continua en un intervalo delimitado cerrado\([a,b]\), y así es acotado. Es claro que\(d(f,g) \geq 0\), es el supremo de los números no negativos. Si\(f = g\) entonces\(\left\lvert {f(x)-g(x)} \right\rvert = 0\) para todos\(x\) y por lo tanto\(d(f,g) = 0\). Por el contrario si\(d(f,g) = 0\), entonces para cualquiera\(x\) tenemos\(\left\lvert {f(x)-g(x)} \right\rvert \leq d(f,g) = 0\) y por lo tanto\(f(x) = g(x)\) para todos\(x\) y\(f=g\). Eso\(d(f,g) = d(g,f)\) es igualmente trivial. Para mostrar la desigualdad triangular utilizamos la desigualdad estándar del triángulo. \[\begin{split} d(f,g) & = \sup_{x \in [a,b]} \left\lvert {f(x)-g(x)} \right\rvert = \sup_{x \in [a,b]} \left\lvert {f(x)-h(x)+h(x)-g(x)} \right\rvert \\ & \leq \sup_{x \in [a,b]} ( \left\lvert {f(x)-h(x)} \right\rvert+\left\lvert {h(x)-g(x)} \right\rvert ) \\ & \leq \sup_{x \in [a,b]} \left\lvert {f(x)-h(x)} \right\rvert+ \sup_{x \in [a,b]} \left\lvert {h(x)-g(x)} \right\rvert = d(f,h) + d(h,g) . \end{split}\]Al tratar\(C([a,b],{\mathbb{R}})\) como un espacio métrico sin mencionar una métrica, nos referimos a esta métrica en particular. Observe que\(d(f,g) = \left\lVert {f-g} \right\rVert_u\), la norma uniforme de.

    Este ejemplo puede parecer esotérico al principio, pero resulta que trabajar con espacios como\(C([a,b],{\mathbb{R}})\) es realmente la carne de gran parte del análisis moderno. Tratar conjuntos de funciones como espacios métricos nos permite abstraer gran parte de los detalles sucios y demostrar resultados poderosos como el teorema de Picard con menos trabajo.

    A menudo es útil considerar un subconjunto de un espacio métrico más grande como un espacio métrico en sí mismo. Obtenemos la siguiente proposición, la cual tiene una prueba trivial.

    Dejar\((X,d)\) ser un espacio métrico y\(Y \subset X\), entonces la restricción\(d|_{Y \times Y}\) es una métrica en\(Y\).

    Si\((X,d)\) es un espacio métrico,\(Y \subset X\), y\(d' := d|_{Y \times Y}\), entonces\((Y,d')\) se dice que es un subespacio de\((X,d)\).

    Es común escribir simplemente\(d\) para la métrica on\(Y\), ya que es la restricción de la métrica on\(X\). A veces decimos que\(d'\) es la métrica subespacial y\(Y\) tiene la topología subespacial.

    Un subconjunto de los números reales se limita siempre que todos sus elementos estén como mucho a cierta distancia fija de 0. También definimos conjuntos acotados en un espacio métrico. Cuando se trata de un espacio métrico arbitrario, puede que no haya algún punto fijo natural 0. A los efectos de la generosidad no importa.

    Dejar\((X,d)\) ser un espacio métrico. \(S \subset X\)Se dice que un subconjunto está acotado si existe un\(p \in X\) y\(B \in {\mathbb{R}}\) tal que\[d(p,x) \leq B \quad \text{for all $x \in S$}.\] Nosotros decimos\((X,d)\) está acotado si\(X\) en sí mismo es un subconjunto acotado.

    Por ejemplo, el conjunto de números reales con la métrica estándar no es un espacio métrico acotado. No es difícil ver que un subconjunto de los números reales está acotado en el sentido de si y sólo si está delimitado como un subconjunto del espacio métrico de los números reales con la métrica estándar.

    Por otro lado, si tomamos los números reales con la métrica discreta, entonces obtenemos un espacio métrico acotado. De hecho, cualquier conjunto con la métrica discreta está acotado.

    Ejercicios

    Mostrar que para cualquier conjunto\(X\), la métrica discreta (\(d(x,y) = 1\)if\(x\not=y\) y\(d(x,x) = 0\)) da un espacio métrico\((X,d)\).

    \(X := \{ 0 \}\)Déjese ser un conjunto. ¿Puedes convertirlo en un espacio métrico?

    \(X := \{ a, b \}\)Déjese ser un conjunto. ¿Puedes convertirlo en dos espacios métricos distintos? (definir dos métricas distintas en él)

    Que el conjunto\(X := \{ A, B, C \}\) represente 3 edificios en el campus. Supongamos que deseamos que nuestra distancia sea el tiempo que lleva caminar de un edificio a otro. Tarda 5 minutos de cualquier manera entre edificios\(A\) y\(B\). Sin embargo, el edificio\(C\) está en una colina y se tarda 10 minutos\(A\) y 15 minutos de llegar\(B\) a\(C\). Por otro lado se necesitan 5 minutos para ir de\(C\) a\(A\) y 7 minutos para ir de\(C\) a\(B\), ya que vamos cuesta abajo. ¿Estas distancias definen una métrica? Si es así, demuéstralo, si no, di por qué no.

    Supongamos que\((X,d)\) es un espacio métrico y\(\varphi \colon [0,\infty) \to {\mathbb{R}}\) es una función tal que\(\varphi(t) \geq 0\) para todos\(t\) y\(\varphi(t) = 0\) si y sólo si\(t=0\). También supongamos que\(\varphi\) es subaditivo, es decir,\(\varphi(s+t) \leq \varphi(s)+\varphi(t)\). Demostrar que con\(d'(x,y) := \varphi\bigl(d(x,y)\bigr)\), obtenemos un nuevo espacio métrico\((X,d')\).

    [ejercicio:mscross] Dejar\((X,d_X)\) y\((Y,d_Y)\) ser espacios métricos.
    a) Demostrar que\((X \times Y,d)\) con\(d\bigl( (x_1,y_1), (x_2,y_2) \bigr) := d_X(x_1,x_2) + d_Y(y_1,y_2)\) es un espacio métrico.
    b) Demostrar que\((X \times Y,d)\) con\(d\bigl( (x_1,y_1), (x_2,y_2) \bigr) := \max \{ d_X(x_1,x_2) , d_Y(y_1,y_2) \}\) es un espacio métrico.

    Dejar\(X\) ser el conjunto de funciones continuas encendidas\([0,1]\). Dejar\(\varphi \colon [0,1] \to (0,\infty)\) ser continuo. Definir\[d(f,g) := \int_0^1 \left\lvert {f(x)-g(x)} \right\rvert\varphi(x)~dx .\] Mostrar que\((X,d)\) es un espacio métrico.

    [exercise:mshausdorffpseudo] Dejar\((X,d)\) ser un espacio métrico. Para subconjuntos acotados no vacíos\(A\) y\(B\) vamos\[d(x,B) := \inf \{ d(x,b) : b \in B \} \qquad \text{and} \qquad d(A,B) := \sup \{ d(a,B) : a \in A \} .\] Ahora definir la métrica de Hausdorff como\[d_H(A,B) := \max \{ d(A,B) , d(B,A) \} .\] Nota: se\(d_H\) puede definir para subconjuntos no vacíos arbitrarios si permitimos los reales extendidos.
    a) Let\(Y \subset {\mathcal{P}}(X)\) Ser el conjunto de subconjuntos delimitados no vacíos. Demostrar que\((Y,d_H)\) es un llamado espacio pseudométrico:\(d_H\) satisface las propiedades métricas [metric:pos], [metric:com], [metric:triang], y además\(d_H(A,A) = 0\) para todos\(A \in Y\).
    b) Mostrar con el ejemplo que\(d\) en sí mismo no es simétrico, es decir\(d(A,B) \not= d(B,A)\).
    c) Encontrar un espacio métrico\(X\) y dos subconjuntos delimitados no vacíos diferentes\(A\) y\(B\) tal que\(d_H(A,B) = 0\).

    Conjuntos abiertos y cerrados

    Nota: 2 conferencias

    Topología

    Es útil definir una llamada topología. Es decir, definimos conjuntos cerrados y abiertos en un espacio métrico. Antes de hacerlo, definamos dos conjuntos especiales.

    Dejar\((X,d)\) ser un espacio métrico,\(x \in X\) y\(\delta > 0\). Luego definimos la bola abierta o simplemente bola de radio\(\delta\) alrededor\(x\) como\[B(x,\delta) := \{ y \in X : d(x,y) < \delta \} .\] De manera similar definimos la bola cerrada como\[C(x,\delta) := \{ y \in X : d(x,y) \leq \delta \} .\]

    Cuando se trata de diferentes espacios métricos, a veces es conveniente enfatizar en qué espacio métrico se encuentra la pelota. Esto lo hacemos por escrito\(B_X(x,\delta) := B(x,\delta)\) o\(C_X(x,\delta) := C(x,\delta)\).

    Tome el espacio métrico\({\mathbb{R}}\) con la métrica estándar. Para\(x \in {\mathbb{R}}\), y\(\delta > 0\) obtenemos\[B(x,\delta) = (x-\delta,x+\delta) \qquad \text{and} \qquad C(x,\delta) = [x-\delta,x+\delta] .\]

    Tenga cuidado al trabajar en un subespacio. Supongamos que tomamos el espacio métrico\([0,1]\) como subespacio de\({\mathbb{R}}\). Entonces en\([0,1]\) obtenemos\[B(0,\nicefrac{1}{2}) = B_{[0,1]}(0,\nicefrac{1}{2}) = [0,\nicefrac{1}{2}) .\] Esto es diferente de\(B_

    ParseError: invalid DekiScript (click for details)
    Callstack:
        at (Matematicas/Analisis/Introducción_al_Análisis_Real_(Lebl)/02:_Números_reales/2.01:_Propiedades_básicas), /content/body/div[39]/div/div[1]/p[5]/span[5]/span, line 1, column 1
    
    (0,\nicefrac{1}{2}) = (-\nicefrac{1}{2},\nicefrac{1}{2})\). Lo importante a tener en cuenta es en qué espacio métrico estamos trabajando.

    Dejar\((X,d)\) ser un espacio métrico. Un conjunto\(V \subset X\) está abierto si por cada\(x \in V\), existe un\(\delta > 0\) tal que\(B(x,\delta) \subset V\). Ver. Un conjunto\(E \subset X\) se cierra si el complemento\(E^c = X \setminus E\) está abierto. Cuando el espacio ambiental no\(X\) está claro desde el contexto decimos que\(V\) está abierto\(X\) y\(E\) está cerrado en\(X\).

    Si\(x \in V\) y\(V\) está abierto, entonces decimos que\(V\) es un barrio abierto de\(x\) (o a veces solo barrio).

    Intuitivamente, un set abierto es un conjunto que no incluye su “límite”, dondequiera que estemos en el set, se nos permite “menearnos” un poco y quedarnos en el set. Tenga en cuenta que no todos los conjuntos son abiertos o cerrados, de hecho generalmente la mayoría de los subconjuntos tampoco lo son.

    El conjunto no\([0,1) \subset {\mathbb{R}}\) está abierto ni cerrado. En primer lugar, cada bola\({\mathbb{R}}\) alrededor\(0\),\((-\delta,\delta)\), contiene números negativos y por lo tanto no está contenida en\([0,1)\) y por lo tanto no\([0,1)\) está abierta. Segundo, cada bola\({\mathbb{R}}\) alrededor\(1\),\((1-\delta,1+\delta)\) contiene números estrictamente menores que 1 y mayores que 0 (por ejemplo,\(1-\nicefrac{\delta}{2}\) siempre y cuando\(\delta < 2\)). Así no\({\mathbb{R}}\setminus [0,1)\) está abierto, y por lo tanto no\([0,1)\) está cerrado.

    [prop:topology:open] Dejar\((X,d)\) ser un espacio métrico.

    1. [topología:openi]\(\emptyset\) y\(X\) están abiertos en\(X\).
    2. [topología:openii] Si\(V_1, V_2, \ldots, V_k\) están abiertos entonces también\[\bigcap_{j=1}^k V_j\] está abierto. Es decir, la intersección finita de conjuntos abiertos es abierta.
    3. [topology:openiii] Si\(\{ V_\lambda \}_{\lambda \in I}\) es una colección arbitraria de conjuntos abiertos, entonces también\[\bigcup_{\lambda \in I} V_\lambda\] está abierta. Es decir, la unión de sets abiertos es abierta.

    Tenga en cuenta que el índice establecido en [topology:openiii] es arbitrariamente grande. Por simplemente\(\bigcup_{\lambda \in I} V_\lambda\) nos referimos al conjunto de todos los\(x\) tales que\(x \in V_\lambda\) para al menos uno\(\lambda \in I\).

    Los sets\(X\) y obviamente\(\emptyset\) están abiertos en\(X\).

    Demostremos [topología:openii]. Si\(x \in \bigcap_{j=1}^k V_j\), entonces\(x \in V_j\) para todos\(j\). Como todos\(V_j\) están abiertos, para cada\(j\) existe un\(\delta_j > 0\) tal que\(B(x,\delta_j) \subset V_j\). Tomar\(\delta := \min \{ \delta_1,\delta_2,\ldots,\delta_k \}\) y darse cuenta\(\delta > 0\). Tenemos\(B(x,\delta) \subset B(x,\delta_j) \subset V_j\) para todos\(j\) y así\(B(x,\delta) \subset \bigcap_{j=1}^k V_j\). En consecuencia, la intersección está abierta.

    Demostremos [topología:openiii]. Si\(x \in \bigcup_{\lambda \in I} V_\lambda\), entonces\(x \in V_\lambda\) para algunos\(\lambda \in I\). Como\(V_\lambda\) está abierto, existe\(\delta > 0\) tal que\(B(x,\delta) \subset V_\lambda\). Pero entonces\(B(x,\delta) \subset \bigcup_{\lambda \in I} V_\lambda\) y así el sindicato está abierto.

    Lo principal a notar es la diferencia entre los ítems [topología:openii] y [topología:openiii]. El ítem [topology:openii] no es cierto para una intersección arbitraria, por ejemplo\(\bigcap_{n=1}^\infty (-\nicefrac{1}{n},\nicefrac{1}{n}) = \{ 0 \}\), que no está abierta.

    Se deja como ejercicio la prueba de la siguiente proposición análoga para conjuntos cerrados.

    [prop:topology:closed] Dejar\((X,d)\) ser un espacio métrico.

    1. [topología:closedi]\(\emptyset\) y\(X\) están cerrados en\(X\).
    2. [topología:closedii] Si\(\{ E_\lambda \}_{\lambda \in I}\) es una colección arbitraria de conjuntos cerrados, entonces también\[\bigcap_{\lambda \in I} E_\lambda\] se cierra. Es decir, la intersección de conjuntos cerrados es cerrada.
    3. [topología:closediii] Si\(E_1, E_2, \ldots, E_k\) están cerrados entonces también\[\bigcup_{j=1}^k E_j\] se cierra. Es decir, la unión finita de conjuntos cerrados es cerrada.

    Aún no hemos demostrado que la bola abierta esté abierta y la bola cerrada esté cerrada. Demostremos ahora este hecho para justificar la terminología.

    [prop:topology:bolsopenclosed] Dejar\((X,d)\) ser un espacio métrico,\(x \in X\), y\(\delta > 0\). Después\(B(x,\delta)\) se abre y\(C(x,\delta)\) se cierra.

    Vamos\(y \in B(x,\delta)\). Vamos\(\alpha := \delta-d(x,y)\). Por supuesto\(\alpha > 0\). Ahora vamos\(z \in B(y,\alpha)\). Entonces\[d(x,z) \leq d(x,y) + d(y,z) < d(x,y) + \alpha = d(x,y) + \delta-d(x,y) = \delta .\] Por lo tanto\(z \in B(x,\delta)\) para cada\(z \in B(y,\alpha)\). Entonces\(B(y,\alpha) \subset B(x,\delta)\) y\(B(x,\delta)\) está abierto.

    El comprobante que\(C(x,\delta)\) está cerrado se deja como ejercicio.

    Nuevamente tenga cuidado con lo que es el espacio métrico ambiental. Al igual\([0,\nicefrac{1}{2})\) que una bola abierta en\([0,1]\), esto significa que\([0,\nicefrac{1}{2})\) es un set abierto en\([0,1]\). Por otro lado no\([0,\nicefrac{1}{2})\) es ni abierto ni cerrado en\({\mathbb{R}}\).

    Una manera útil de pensar en un set abierto es como una unión de bolas abiertas. Si\(U\) está abierto, entonces para cada uno\(x \in U\), hay un\(\delta_x > 0\) (dependiendo de\(x\)) tal que\(B(x,\delta_x) \subset U\). Entonces\(U = \bigcup_{x\in U} B(x,\delta_x)\).

    Se deja como ejercicio la prueba de la siguiente proposición. Tenga en cuenta que hay muchos otros conjuntos abiertos y cerrados en\({\mathbb{R}}\).

    [prop:topología:intervalos:abiertocerrado] Dejan\(a < b\) ser dos números reales. Entonces\((a,b)\),\((a,\infty)\), y\((-\infty,b)\) están abiertos en\({\mathbb{R}}\). También\([a,b]\),\([a,\infty)\), y\((-\infty,b]\) están cerrados en\({\mathbb{R}}\).

    Conjuntos conectados

    Un espacio métrico no vacío\((X,d)\) está conectado si los únicos subconjuntos de los\(X\) que están abiertos y cerrados son\(\emptyset\) y\(X\) sí mismo. Si no\((X,d)\) está conectado decimos que está desconectado.

    Cuando aplicamos el término conectado a un subconjunto no vacío\(A \subset X\), simplemente queremos decir que\(A\) con el subespacio la topología está conectada.

    En otras palabras, un no vacío\(X\) está conectado si cada vez que escribimos\(X = X_1 \cup X_2\) dónde\(X_1 \cap X_2 = \emptyset\)\(X_1\) y y\(X_2\) estamos abiertos, entonces cualquiera\(X_1 = \emptyset\) o\(X_2 = \emptyset\). Entonces para mostrar\(X\) está desconectado, necesitamos encontrar conjuntos abiertos disjuntos no vacíos\(X_1\) y\(X_2\) cuya unión es\(X\). Para los subconjuntos, declaramos esta idea como una proposición.

    Dejar\((X,d)\) ser un espacio métrico. Un conjunto no vacío no\(S \subset X\) está conectado si y solo si existen conjuntos abiertos\(U_1\) y\(U_2\) en\(X\), tal que\(U_1 \cap U_2 \cap S = \emptyset\),\(U_1 \cap S \not= \emptyset\),\(U_2 \cap S \not= \emptyset\), y\[S = \bigl( U_1 \cap S \bigr) \cup \bigl( U_2 \cap S \bigr) .\]

    Si\(U_j\) está abierto en\(X\), entonces\(U_j \cap S\) está abierto en\(S\) en la topología subespacial (con métrica subespacial). Para ver esto, tenga en cuenta que si\(B_X(x,\delta) \subset U_j\), entonces como\(B_S(x,\delta) = S \cap B_X(x,\delta)\), tenemos\(B_S(x,\delta) \subset U_j \cap S\). Entonces si\(U_1\) y\(U_2\) como antes existen, entonces\(X\) se desconecta con base en la discusión anterior.

    La prueba de la otra dirección sigue utilizando para encontrar\(U_1\) y a\(U_2\) partir de dos subconjuntos abiertos disjuntos de\(S\).

    \(S \subset {\mathbb{R}}\)Sea tal que\(x < z < y\) con\(x,y \in S\) y\(z \notin S\). Reclamación: no\(S\) está conectada. Comprobante: Aviso\[\bigl( (-\infty,z) \cap S \bigr) \cup \bigl( (z,\infty) \cap S \bigr) = S .\]

    Un conjunto no vacío\(S \subset {\mathbb{R}}\) está conectado si y sólo si es un intervalo o un solo punto.

    Supongamos que\(S\) está conectado. Si\(S\) es un solo punto entonces ya terminamos. Entonces supongamos\(x < y\) y\(x,y \in S\). Si\(z\) es tal que\(x < z < y\), entonces no\((-\infty,z) \cap S\) está vacío y no\((z,\infty) \cap S\) está vacío. Los dos conjuntos son disgregados. Como\(S\) está conectado, debemos tenerlos su unión no lo es\(S\), así\(z \in S\).

    Supongamos que\(S\) está acotado, conectado, pero no un solo punto. Que\(\alpha := \inf \, S\)\(\beta := \sup \, S\) y anote eso\(\alpha < \beta\). Supongamos\(\alpha < z < \beta\). Como\(\alpha\) es el infimum, entonces hay\(x \in S\) tal que\(\alpha \leq x < z\). De igual manera hay\(y \in S\) tal que\(\beta \geq y > z\). Nosotros hemos demostrado arriba eso\(z \in S\), entonces\((\alpha,\beta) \subset S\). Si\(w < \alpha\), entonces\(w \notin S\) como\(\alpha\) era el infimum, de manera similar si\(w > \beta\) entonces\(w \notin S\). Por lo tanto las únicas posibilidades para\(S\) son\((\alpha,\beta)\),\([\alpha,\beta)\),\((\alpha,\beta]\),\([\alpha,\beta]\).

    La prueba de que un conectado sin límites\(S\) es un intervalo se deja como ejercicio.

    Por otro lado supongamos que\(S\) es un intervalo. Supongamos\(U_1\) y\(U_2\) son subconjuntos abiertos de\({\mathbb{R}}\),\(U_1 \cap S\) y no\(U_2 \cap S\) están vacíos, y\(S = \bigl( U_1 \cap S \bigr) \cup \bigl( U_2 \cap S \bigr)\). Eso lo demostraremos\(U_1 \cap S\) y\(U_2 \cap S\) contendremos un punto común, por lo que no son disjuntos, y por lo tanto\(S\) deben estar conectados. Supongamos que hay\(x \in U_1 \cap S\) y\(y \in U_2 \cap S\). Sin pérdida de generalidad, asuma\(x < y\). Como\(S\) es un intervalo\([x,y] \subset S\). Vamos\(z := \inf (U_2 \cap [x,y])\). Si\(z = x\), entonces\(z \in U_1\). Si\(z > x\), entonces para cualquiera\(\delta > 0\) el balón\(B(z,\delta) = (z-\delta,z+\delta)\) contiene puntos de\(S\) que no están en\(U_2\), y así\(z \notin U_2\) como\(U_2\) está abierto. Por lo tanto,\(z \in U_1\). Como\(U_1\) está abierto,\(B(z,\delta) \subset U_1\) por un pequeño lo suficientemente\(\delta > 0\). Como\(z\) es el infimum de\(U_2 \cap [x,y]\), debe existir alguna\(w \in U_2 \cap [x,y]\) tal que\(w \in [z,z+\delta) \subset B(z,\delta) \subset U_1\). Por lo tanto\(w \in U_1 \cap U_2 \cap [x,y]\). Entonces\(U_1 \cap S\) y no\(U_2 \cap S\) son disjuntos y por lo tanto\(S\) está conectado.

    En muchos casos\(B(x,\delta)\) se conecta una pelota. Pero esto no es necesariamente cierto en todos los espacios métricos. Para un ejemplo más sencillo, toma un espacio de dos puntos\(\{ a, b\}\) con la métrica discreta. Entonces\(B(a,2) = \{ a , b \}\), que no se conecta como\(B(a,1) = \{ a \}\) y\(B(b,1) = \{ b \}\) son abiertas y disjuntas.

    Cierre y límite

    A veces deseamos tomar un set y tirar todo lo que podamos acercarnos desde el set. A este concepto se le llama el cierre.

    Dejar\((X,d)\) ser un espacio métrico y\(A \subset X\). Entonces el cierre de\(A\) es el conjunto Es\[\overline{A} := \bigcap \{ E \subset X : \text{$E$ is closed and $A \subset E$} \} .\] decir,\(\overline{A}\) es la intersección de todos los conjuntos cerrados que contienen\(A\).

    Dejar\((X,d)\) ser un espacio métrico y\(A \subset X\). El cierre\(\overline{A}\) está cerrado. Además si\(A\) se cierra entonces\(\overline{A} = A\).

    Primero, el cierre es la intersección de conjuntos cerrados, por lo que está cerrado. Segundo, si\(A\) está cerrado, entonces toma\(E = A\), de ahí que la intersección de todos los conjuntos cerrados\(E\) que contengan\(A\) debe ser igual a\(A\).

    El cierre de\((0,1)\) in\({\mathbb{R}}\) es\([0,1]\). Prueba: Simplemente observe que si\(E\) está cerrado y contiene\((0,1)\), entonces\(E\) debe contener\(0\) y\(1\) (¿por qué?). Así\([0,1] \subset E\). Pero también\([0,1]\) está cerrado. Por lo tanto, el cierre\(\overline{(0,1)} = [0,1]\).

    Tenga cuidado de notar con qué espacio métrico ambiental está trabajando. Si\(X = (0,\infty)\), entonces el cierre de\((0,1)\) in\((0,\infty)\) es\((0,1]\). Prueba: De manera similar a lo anterior\((0,1]\) se cierra en\((0,\infty)\) (¿por qué?). Cualquier conjunto cerrado\(E\) que contenga\((0,1)\) debe contener 1 (¿por qué?). Por lo tanto\((0,1] \subset E\), y de ahí\(\overline{(0,1)} = (0,1]\) cuando se trabaja en\((0,\infty)\).

    Justificemos la afirmación de que el cierre es todo lo que podemos “acercarnos” desde el set.

    [prop:msclosureappr] Dejar\((X,d)\) ser un espacio métrico y\(A \subset X\). Entonces\(x \in \overline{A}\) si y sólo si por cada\(\delta > 0\),\(B(x,\delta) \cap A \not=\emptyset\).

    Demostremos los dos contrapositivos. Demostremos eso\(x \notin \overline{A}\) si y sólo si existe\(\delta > 0\) tal eso\(B(x,\delta) \cap A = \emptyset\).

    Primero supongamos\(x \notin \overline{A}\). Sabemos que\(\overline{A}\) está cerrado. Así hay\(\delta > 0\) tal que\(B(x,\delta) \subset \overline{A}^c\). Como\(A \subset \overline{A}\) vemos eso\(B(x,\delta) \subset A^c\) y por lo tanto\(B(x,\delta) \cap A = \emptyset\).

    Por otro lado supongamos que existe\(\delta > 0\) tal que\(B(x,\delta) \cap A = \emptyset\). Entonces\({B(x,\delta)}^c\) es un set cerrado y tenemos eso\(A \subset {B(x,\delta)}^c\), pero\(x \notin {B(x,\delta)}^c\). Así como\(\overline{A}\) es la intersección de conjuntos cerrados que contienen\(A\), tenemos\(x \notin \overline{A}\).

    También podemos hablar sobre lo que hay en el interior de un conjunto y lo que está en el límite.

    Dejar\((X,d)\) ser un espacio métrico y\(A \subset X\), entonces el interior de\(A\) es el conjunto\[A^\circ := \{ x \in A : \text{there exists a $\delta > 0$ such that $B(x,\delta) \subset A$} \} .\] El límite de\(A\) es el conjunto\[\partial A := \overline{A}\setminus A^\circ.\]

    Supongamos\(A=(0,1]\) y\(X = {\mathbb{R}}\). Entonces no es difícil de ver eso\(\overline{A}=[0,1]\),\(A^\circ = (0,1)\), y\(\partial A = \{ 0, 1 \}\).

    Supongamos\(X = \{ a, b \}\) con la métrica discreta. Vamos\(A = \{ a \}\), entonces\(\overline{A} = A^\circ = A\) y\(\partial A = \emptyset\).

    Dejar\((X,d)\) ser un espacio métrico y\(A \subset X\). Después\(A^\circ\) se abre y\(\partial A\) se cierra.

    Dado\(x \in A^\circ\) que tenemos\(\delta > 0\) tal que\(B(x,\delta) \subset A\). Si\(z \in B(x,\delta)\), entonces como bolas abiertas están abiertas, hay\(\epsilon > 0\) tal que\(B(z,\epsilon) \subset B(x,\delta) \subset A\), así\(z\) está adentro\(A^\circ\). Por lo tanto\(B(x,\delta) \subset A^\circ\) y así\(A^\circ\) está abierto.

    Como\(A^\circ\) está abierto, luego\(\partial A = \overline{A} \setminus A^\circ = \overline{A} \cap {(A^\circ)}^c\) se cierra.

    El límite es el conjunto de puntos que están cerca tanto del conjunto como de su complemento.

    Dejar\((X,d)\) ser un espacio métrico y\(A \subset X\). Entonces\(x \in \partial A\) si y sólo si por cada\(\delta > 0\),\(B(x,\delta) \cap A\) y ambos\(B(x,\delta) \cap A^c\) están no vacíos.

    Supongamos\(x \in \partial A = \overline{A} \setminus A^\circ\) y dejemos\(\delta > 0\) ser arbitrarios. Por,\(B(x,\delta)\) contiene un punto de\(A\). Si no\(B(x,\delta)\) contenía puntos de\(A^c\), entonces\(x\) estaría en\(A^\circ\). De ahí que\(B(x,\delta)\) contenga un punto de\(A^c\) también.

    Demostremos el otro rumbo por contrapositivo. Si\(x \notin \overline{A}\), entonces hay alguna\(\delta > 0\) tal que\(B(x,\delta) \subset \overline{A}^c\) como\(\overline{A}\) está cerrada. Entonces no\(B(x,\delta)\) contiene puntos de\(A\), porque\(\overline{A}^c \subset A^c\).

    Ahora supongamos\(x \in A^\circ\), entonces existe\(\delta > 0\) tal que\(B(x,\delta) \subset A\), pero eso significa que no\(B(x,\delta)\) contiene puntos de\(A^c\).

    Obtenemos el siguiente corolario inmediato sobre cierres de\(A\) y\(A^c\). Simplemente aplicamos.

    Dejar\((X,d)\) ser un espacio métrico y\(A \subset X\). Entonces\(\partial A = \overline{A} \cap \overline{A^c}\).

    Ejercicios

    Demostrar. Consejo: considera los complementos de los conjuntos y aplica.

    Terminar el comprobante de demostrando que\(C(x,\delta)\) está cerrado.

    Demostrar.

    Supongamos que\((X,d)\) es un espacio métrico no vacío con la topología discreta. Mostrar que\(X\) está conectado si y sólo si contiene exactamente un elemento.

    Mostrar que si\(S \subset {\mathbb{R}}\) es un conjunto no acotado conectado, entonces es un intervalo (no acotado).

    Demuestre que cada conjunto abierto puede escribirse como una unión de conjuntos cerrados.

    a) Mostrar que\(E\) está cerrado si y solo\(\partial E \subset E\) si. b) Mostrar que\(U\) está abierto si y solo si\(\partial U \cap U = \emptyset\).

    a) Mostrar que\(A\) está abierto si y solo\(A^\circ = A\) si. b) Supongamos que\(U\) es un conjunto abierto y\(U \subset A\). \(U \subset A^\circ\)Demuéstralo.

    Dejar\(X\) ser un conjunto y\(d\),\(d'\) ser dos métricas sobre\(X\). Supongamos que existe una\(\alpha > 0\) y\(\beta > 0\) tal que\(\alpha d(x,y) \leq d'(x,y) \leq \beta d(x,y)\) para todos\(x,y \in X\). Mostrar que\(U\) está abierto en\((X,d)\) si y solo si\(U\) está abierto en\((X,d')\). Es decir, las topologías de\((X,d)\) y\((X,d')\) son las mismas.

    Supongamos\(\{ S_i \}\),\(i \in {\mathbb{N}}\) es una colección de subconjuntos conectados de un espacio métrico\((X,d)\). Supongamos que existe\(x \in X\) tal que\(x \in S_i\) para todos\(i \in {\mathbb{N}}\). Demostrar que\(\bigcup_{i=1}^\infty S_i\) está conectado.

    Dejar\(A\) ser un conjunto conectado. a) ¿Está\(\overline{A}\) conectado? Probar o encontrar un contraejemplo. b) ¿Está\(A^\circ\) conectado? Probar o encontrar un contraejemplo. Pista: Piense en sets in\({\mathbb{R}}^2\).

    La definición de conjuntos abiertos en el siguiente ejercicio suele denominarse topología subespacial. Se le pide que demuestre que obtenemos la misma topología considerando la métrica subespacial.

    [ejercicio:mssubspace] Supongamos que\((X,d)\) es un espacio métrico y\(Y \subset X\). Mostrar que con la métrica subespacial\(Y\) encendida, un conjunto\(U \subset Y\) está abierto (in\(Y\)) siempre que exista un conjunto abierto\(V \subset X\) tal que\(U = V \cap Y\).

    Dejar\((X,d)\) ser un espacio métrico. a) Para cualquier\(x \in X\) y\(\delta > 0\), espectáculo\(\overline{B(x,\delta)} \subset C(x,\delta)\). b) ¿Siempre es cierto eso\(\overline{B(x,\delta)} = C(x,\delta)\)? Probar o encontrar un contraejemplo.

    Dejar\((X,d)\) ser un espacio métrico y\(A \subset X\). \(A^\circ = \bigcup \{ V : V \subset A \text{ is open} \}\)Demuéstralo.

    Secuencias y convergencia

    Nota: 1 conferencia

    Secuencias

    La noción de una secuencia en un espacio métrico es muy similar a una secuencia de números reales. Las definiciones son esencialmente las mismas que las de los números reales en el sentido de donde\({\mathbb{R}}\) con la métrica estándar\(d(x,y)=\left\lvert {x-y} \right\rvert\) se sustituye por un espacio métrico arbitrario\((X,d)\).

    Una secuencia en un espacio métrico\((X,d)\) es una función\(x \colon {\mathbb{N}}\to X\). Como antes escribimos\(x_n\) para el elemento\(n\) th en la secuencia y usamos la notación\(\{ x_n \}\), o más precisamente\[\{ x_n \}_{n=1}^\infty .\]

    Una secuencia\(\{ x_n \}\) está delimitada si existe un punto\(p \in X\) y\(B \in {\mathbb{R}}\) tal que\[d(p,x_n) \leq B \qquad \text{for all $n \in {\mathbb{N}}$.}\] En otras palabras, la secuencia\(\{x_n\}\) se limita siempre que el conjunto\(\{ x_n : n \in {\mathbb{N}}\}\) esté acotado.

    Si\(\{ n_j \}_{j=1}^\infty\) es una secuencia de números naturales tal que\(n_{j+1} > n_j\) para todos\(j\), entonces\(\{ x_{n_j} \}_{j=1}^\infty\) se dice que la secuencia es una subsecuencia de\(\{x_n \}\).

    De igual manera también definimos convergencia. Nuevamente, estaremos haciendo trampa un poco y usaremos el artículo definido frente a la palabra límite antes de demostrar que el límite es único.

    \((X,d)\)Se dice que una secuencia\(\{ x_n \}\) en un espacio métrico converge a un punto\(p \in X\), si por cada\(\epsilon > 0\), existe\(M \in {\mathbb{N}}\) tal que\(d(x_n,p) < \epsilon\) para todos\(n \geq M\). Se dice que el punto\(p\) es el límite de\(\{ x_n \}\). Escribimos\[\lim_{n\to \infty} x_n := p .\]

    Se dice que una secuencia que converge es convergente. De lo contrario, se dice que la secuencia es divergente.

    Demostremos que el límite es único. Obsérvese que la prueba es casi idéntica a la prueba del mismo hecho para secuencias de números reales. Muchos resultados que conocemos para secuencias de números reales se pueden probar en los ajustes más generales de los espacios métricos. Debemos sustituir\(\left\lvert {x-y} \right\rvert\) con\(d(x,y)\) en las pruebas y aplicar correctamente la desigualdad triangular.

    [prop:mslimisunique] Una secuencia convergente en un espacio métrico tiene un límite único.

    Supongamos que la secuencia\(\{ x_n \}\) tiene el límite\(x\) y el límite\(y\). Toma una arbitraria\(\epsilon > 0\). De la definición encontrar un\(M_1\) tal que para todos\(n \geq M_1\),\(d(x_n,x) < \nicefrac{\epsilon}{2}\). De igual manera encontramos un\(M_2\) tal que por todo\(n \geq M_2\) lo que tenemos\(d(x_n,y) < \nicefrac{\epsilon}{2}\). Ahora toma un\(n\) tal que\(n \geq M_1\) y también\(n \geq M_2\)\[\begin{split} d(y,x) & \leq d(y,x_n) + d(x_n,x) \\ & < \frac{\epsilon}{2} + \frac{\epsilon}{2} = \epsilon . \end{split}\] En\(d(y,x) < \epsilon\) cuanto a todos\(\epsilon > 0\), entonces\(d(x,y) = 0\) y\(y=x\). De ahí que el límite (si existe) sea único.

    Las pruebas de las siguientes proposiciones se dejan como ejercicios.

    [prop:msconvbound] Una secuencia convergente en un espacio métrico está delimitada.

    [prop:msconvifa] Una secuencia\(\{ x_n \}\) en un espacio métrico\((X,d)\) converge a\(p \in X\) si y solo si existe una secuencia\(\{ a_n \}\) de números reales tal que\[d(x_n,p) \leq a_n \quad \text{for all $n \in {\mathbb{N}}$},\] y\[\lim_{n\to\infty} a_n = 0.\]

    [prop:mssubseq] Dejar\(\{ x_n \}\) ser una secuencia en un espacio métrico\((X,d)\).

    1. Si\(\{ x_n \}\) converge a\(p \in X\), entonces cada subsecuencia\(\{ x_{n_k} \}\) converge a\(p\).
    2. Si para algunos\(K \in {\mathbb{N}}\) la\(K\) -cola\(\{ x_n \}_{n=K+1}^\infty\) converge a\(p \in X\), entonces\(\{ x_n \}\) converge a\(p\).

    Convergencia en el espacio euclidiano

    Es útil señalar lo que significa convergencia en el espacio euclidiano\({\mathbb{R}}^n\).

    [prop:msconveuc] Dejar\(\{ x_j \}_{j=1}^\infty\) ser una secuencia en\({\mathbb{R}}^n\), donde escribimos\(x_j = \bigl(x_{j,1},x_{j,2},\ldots,x_{j,n}\bigr) \in {\mathbb{R}}^n\). Entonces\(\{ x_j \}_{j=1}^\infty\) converge si y solo si\(\{ x_{j,k} \}_{j=1}^\infty\) converge para cada\(k\), en cuyo caso\[\lim_{j\to\infty} x_j = \Bigl( \lim_{j\to\infty} x_{j,1}, \lim_{j\to\infty} x_{j,2}, \ldots, \lim_{j\to\infty} x_{j,n} \Bigr) .\]

    Dejemos\(\{ x_j \}_{j=1}^\infty\) ser una secuencia convergente en\({\mathbb{R}}^n\), donde escribimos\(x_j = \bigl(x_{j,1},x_{j,2},\ldots,x_{j,n}\bigr) \in {\mathbb{R}}^n\). \(y = (y_1,y_2,\ldots,y_n) \in {\mathbb{R}}^n\)Sea el límite. Dado\(\epsilon > 0\), existe\(M\) tal que por todos\(j \geq M\) tenemos\[d(y,x_j) < \epsilon.\] Fijar algunos\(k=1,2,\ldots,n\). Porque\(j \geq M\) tenemos\[\bigl\lvert y_k - x_{j,k} \bigr\rvert = \sqrt{{\bigl(y_k - x_{j,k} \bigr)}^2} \leq \sqrt{\sum_{\ell=1}^n {\bigl(y_\ell-x_{j,\ell}\bigr)}^2} = d(y,x_j) < \epsilon .\] De ahí que la secuencia\(\{ x_{j,k} \}_{j=1}^\infty\) converja a\(y_k\).

    Para la otra dirección supongamos que\(\{ x_{j,k} \}_{j=1}^\infty\) converge\(y_k\) para cada uno\(k=1,2,\ldots,n\). De ahí que\(\epsilon > 0\), dado, escoja una\(M\), tal que si\(j \geq M\) entonces\(\bigl\lvert y_k-x_{j,k} \bigr\rvert < \nicefrac{\epsilon}{\sqrt{n}}\) para todos\(k=1,2,\ldots,n\). Entonces\[d(y,x_j) = \sqrt{\sum_{k=1}^n {\bigl(y_k-x_{j,k}\bigr)}^2} < \sqrt{\sum_{k=1}^n {\left(\frac{\epsilon}{\sqrt{n}}\right)}^2} = \sqrt{\sum_{k=1}^n \frac

    ParseError: invalid DekiScript (click for details)
    Callstack:
        at (Matematicas/Analisis/Introducción_al_Análisis_Real_(Lebl)/02:_Números_reales/2.01:_Propiedades_básicas), /content/body/div[40]/div/div[2]/p[4]/span[9]/span, line 1, column 1
    
    {n}} = \epsilon .\] La secuencia\(\{ x_j \}\) converge a\(y \in {\mathbb{R}}^n\) y ya terminamos.

    Convergencia y topología

    La topología, es decir, el conjunto de conjuntos abiertos de un espacio codifica qué secuencias convergen.

    [prop:msconvtopo] Dejar\((X,d)\) ser un espacio métrico y\(\{x_n\}\) una secuencia en\(X\). Entonces\(\{ x_n \}\) converge a\(x \in X\) si y sólo si por cada barrio abierto\(U\) de\(x\), existe\(M \in {\mathbb{N}}\) tal que por todo\(n \geq M\) lo que tenemos\(x_n \in U\).

    Primero supongamos que\(\{ x_n \}\) converge. Que\(U\) sea un barrio abierto de\(x\), entonces existe\(\epsilon > 0\) tal que\(B(x,\epsilon) \subset U\). A medida que la secuencia converge, encontramos un\(M \in {\mathbb{N}}\) tal que por todo\(n \geq M\) lo que tenemos\(d(x,x_n) < \epsilon\) o en otras palabras\(x_n \in B(x,\epsilon) \subset U\).

    Demostremos la otra dirección. Dado\(\epsilon > 0\) que\(U := B(x,\epsilon)\) sea el barrio de\(x\). Entonces hay\(M \in {\mathbb{N}}\) tal que para\(n \geq M\) nosotros tenemos\(x_n \in U = B(x,\epsilon)\) o en otras palabras,\(d(x,x_n) < \epsilon\).

    Un conjunto se cierra cuando contiene los límites de sus secuencias convergentes.

    [prop:msclosedlim] Dejar\((X,d)\) ser un espacio métrico,\(E \subset X\) un conjunto cerrado y\(\{ x_n \}\) una secuencia en\(E\) que converge a algunos\(x \in X\). Entonces\(x \in E\).

    Demostremos lo contrapositivo. Supongamos que\(\{ x_n \}\) es una secuencia en la\(X\) que converge a\(x \in E^c\). Como\(E^c\) está abierto, dice que hay\(M\) tal que para todos\(n \geq M\),\(x_n \in E^c\). Entonces no\(\{ x_n \}\) es una secuencia en\(E\).

    Cuando tomamos un cierre de un set\(A\), realmente arrojamos precisamente esos puntos que son límites de secuencias en\(A\).

    [prop:msclosureapprseq] Dejar\((X,d)\) ser un espacio métrico y\(A \subset X\). Entonces\(x \in \overline{A}\) si y sólo si existe una secuencia\(\{ x_n \}\) de elementos en\(A\) tal que\(\lim\, x_n = x\).

    Vamos\(x \in \overline{A}\). Sabemos por eso dado\(\nicefrac{1}{n}\), existe un punto\(x_n \in B(x,\nicefrac{1}{n}) \cap A\). Como\(d(x,x_n) < \nicefrac{1}{n}\), tenemos\(\lim\, x_n = x\).

    Para la otra dirección, ver.

    Ejercicios

    [ejercicio:reverseclosedseq] Dejar\((X,d)\) ser un espacio métrico y dejar\(A \subset X\). \(E\)Sea el conjunto de todos\(x \in X\) tales que exista una secuencia\(\{ x_n \}\) en la\(A\) que converja a\(x\). Espectáculo\(E = \overline{A}\).

    a) Mostrar que\(d(x,y) := \min \{ 1, \left\lvert {x-y} \right\rvert \}\) define una métrica on\({\mathbb{R}}\). b) Mostrar que una secuencia converge en\(({\mathbb{R}},d)\) si y sólo si converge en la métrica estándar. c) Encuentra una secuencia acotada en\(({\mathbb{R}},d)\) que no contenga subsecuencia convergente.

    Demostrar.

    Demostrar.

    Supongamos que\(\{x_n\}_{n=1}^\infty\) converge a\(x\). Supongamos que\(f \colon {\mathbb{N}} \to {\mathbb{N}}\) es una función uno a uno. Espectáculo que\(\{ x_{f(n)} \}_{n=1}^\infty\) converge a\(x\).

    Si\((X,d)\) es un espacio métrico donde\(d\) está la métrica discreta. Supongamos que\(\{ x_n \}\) es una secuencia convergente en\(X\). Demostrar que existe\(K \in {\mathbb{N}}\) tal que por todo\(n \geq K\) lo que tenemos\(x_n = x_K\).

    Se dice que un conjunto\(S \subset X\) es denso en\(X\) si por cada\(x \in X\), existe una secuencia\(\{ x_n \}\) en la\(S\) que converge a\(x\). Demostrar que\({\mathbb{R}}^n\) contiene un subconjunto denso contable.

    Supongamos\(\{ U_n \}_{n=1}^\infty\) ser una secuencia decreciente (\(U_{n+1} \subset U_n\)para todos\(n\)) de conjuntos abiertos en un espacio métrico\((X,d)\) tal que\(\bigcap_{n=1}^\infty U_n = \{ p \}\) para algunos\(p \in X\). Supongamos que\(\{ x_n \}\) es una secuencia de puntos en\(X\) tal que\(x_n \in U_n\). ¿Converge\(\{ x_n \}\) necesariamente a\(p\)? Probar o construir un contraejemplo.

    Dejar\(E \subset X\) ser cerrado y dejar\(\{ x_n \}\) ser una secuencia en\(X\) converger a\(p \in X\). Supongamos\(x_n \in E\) para infinitamente muchos\(n \in {\mathbb{N}}\). Espectáculo\(p \in E\).

    \({\mathbb{R}}^* = \{ -\infty \} \cup {\mathbb{R}}\cup \{ \infty \}\)Tomen ser los reales extendidos. Definir\(d(x,y) := \bigl\lvert \frac{x}{1+\left\lvert {x} \right\rvert} - \frac{y}{1+\left\lvert {y} \right\rvert} \bigr\rvert\) si\(x, y \in {\mathbb{R}}\), definir\(d(\infty,x) := \bigl\lvert 1 - \frac{x}{1+\left\lvert {x} \right\rvert} \bigr\rvert\),\(d(-\infty,x) := \bigl\lvert 1 + \frac{x}{1+\left\lvert {x} \right\rvert} \bigr\rvert\) para todos\(x \in {\mathbb{R}}\), y dejar\(d(\infty,-\infty) := 2\). a) Mostrar que\(({\mathbb{R}}^*,d)\) es un espacio métrico. b) Supongamos que\(\{ x_n \}\) es una secuencia de números reales tal que para cada\(M \in {\mathbb{R}}\), existe un\(N\) tal que\(x_n \geq M\) para todos\(n \geq N\). Mostrar eso\(\lim\, x_n = \infty\) en\(({\mathbb{R}}^*,d)\). c) Mostrar que una secuencia de números reales converge a un número real en\(({\mathbb{R}}^*,d)\) si y sólo si converge\({\mathbb{R}}\) con la métrica estándar.

    Supongamos que\(\{ V_n \}_{n=1}^\infty\) es una colección de conjuntos abiertos en\((X,d)\) tal que\(V_{n+1} \supset V_n\). Dejar\(\{ x_n \}\) ser una secuencia tal que\(x_n \in V_{n+1} \setminus V_n\) y supongamos\(\{ x_n \}\) converge a\(p \in X\). Demuestre que\(p \in \partial V\) dónde\(V = \bigcup_{n=1}^\infty V_n\).

    Demostrar.

    Completitud y compacidad

    Nota: 2 conferencias

    Secuencias de Cauchy e integridad

    Al igual que con las secuencias de números reales definimos secuencias de Cauchy.

    Dejar\((X,d)\) ser un espacio métrico. Una secuencia\(\{ x_n \}\) en\(X\) es una secuencia Cauchy si por cada\(\epsilon > 0\) existe una\(M \in {\mathbb{N}}\) tal que para todos\(n \geq M\) y para todos\(k \geq M\) tenemos\[d(x_n, x_k) < \epsilon .\]

    La definición es otra vez simplemente una traducción del concepto de los números reales a los espacios métricos. Entonces una secuencia de números reales es Cauchy en el sentido de si y solo si es Cauchy en el sentido anterior, siempre que equipemos los números reales con la métrica estándar\(d(x,y) = \left\lvert {x-y} \right\rvert\).

    Una secuencia convergente en un espacio métrico es Cauchy.

    Supongamos que\(\{ x_n \}\) converge a\(x\). Dado que\(\epsilon > 0\) hay\(M\) tal que para\(n \geq M\) nosotros tenemos\(d(x,x_n) < \nicefrac{\epsilon}{2}\). De ahí por todo\(n,k \geq M\) lo que tenemos\(d(x_n,x_k) \leq d(x_n,x) + d(x,x_k) < \nicefrac{\epsilon}{2} + \nicefrac{\epsilon}{2} = \epsilon\).

    Dejar\((X,d)\) ser un espacio métrico. Decimos que\(X\) es completo o Cauchy-completo si cada secuencia de Cauchy\(\{ x_n \}\) en\(X\) converge en un\(x \in X\).

    El espacio\({\mathbb{R}}^n\) con la métrica estándar es un espacio métrico completo.

    Para la\({\mathbb{R}}= {\mathbb{R}}^1\) completitud se comprobó en. La prueba de la propuesta anterior es una reducción al caso unidimensional.

    Dejar\(\{ x_j \}_{j=1}^\infty\) ser una secuencia de Cauchy en\({\mathbb{R}}^n\), donde escribimos\(x_j = \bigl(x_{j,1},x_{j,2},\ldots,x_{j,n}\bigr) \in {\mathbb{R}}^n\). Como la secuencia es Cauchy\(\epsilon > 0\), dada, existe\(M\) tal que por todos\(i,j \geq M\) tenemos\[d(x_i,x_j) < \epsilon.\]

    Arreglar algunos\(k=1,2,\ldots,n\), porque\(i,j \geq M\) tenemos\[\bigl\lvert x_{i,k} - x_{j,k} \bigr\rvert = \sqrt{{\bigl(x_{i,k} - x_{j,k}\bigr)}^2} \leq \sqrt{\sum_{\ell=1}^n {\bigl(x_{i,\ell}-x_{j,\ell}\bigr)}^2} = d(x_i,x_j) < \epsilon .\] De ahí que la secuencia\(\{ x_{j,k} \}_{j=1}^\infty\) sea Cauchy. Como\({\mathbb{R}}\) es completa la secuencia converge; existe\(y_k \in {\mathbb{R}}\) tal que existe\(y_k = \lim_{j\to\infty} x_{j,k}\).

    Escribir\(y = (y_1,y_2,\ldots,y_n) \in {\mathbb{R}}^n\). Por tenemos que\(\{ x_j \}\) converge hasta\(y \in {\mathbb{R}}^n\) y por lo tanto\({\mathbb{R}}^n\) está completo.

    Tenga en cuenta que un subconjunto de\({\mathbb{R}}^n\) con la métrica subespacial no necesita estar completo. Por ejemplo,\((0,1]\) con la métrica subespacial no está completa como lo\(\{ \nicefrac{1}{n} \}\) es una secuencia de Cauchy en\((0,1]\) sin límite en\((0,1]\). Pero vea también.

    Compacidad

    Dejar\((X,d)\) ser un espacio métrico y\(K \subset X\). \(K\)Se dice que el conjunto es compacto si para cualquier colección de conjuntos abiertos\(\{ U_{\lambda} \}_{\lambda \in I}\) tal que\[K \subset \bigcup_{\lambda \in I} U_\lambda ,\] existe un subconjunto finito\(\{ \lambda_1, \lambda_2,\ldots,\lambda_k \} \subset I\) tal que\[K \subset \bigcup_{j=1}^k U_{\lambda_j} .\]

    Se dice que una colección de conjuntos abiertos\(\{ U_{\lambda} \}_{\lambda \in I}\) como el anterior es una cubierta abierta de\(K\). Entonces una manera de decir que\(K\) es compacto es decir que cada cubierta abierta de\(K\) tiene una subcubierta finita.

    Dejar\((X,d)\) ser un espacio métrico. Un conjunto compacto\(K \subset X\) está cerrado y acotado.

    Primero, demostramos que un conjunto compacto está acotado. Arreglar\(p \in X\). Tenemos la tapa abierta\[K \subset \bigcup_{n=1}^\infty B(p,n) = X .\] Si\(K\) es compacta, entonces existe algún conjunto de índices\(n_1 < n_2 < \ldots < n_k\) tal que\[K \subset \bigcup_{j=1}^k B(p,n_j) = B(p,n_k) .\] As\(K\) está contenido en una bola,\(K\) está acotado.

    A continuación, mostramos un conjunto que no está cerrado no es compacto. Supongamos\(\overline{K} \not= K\), es decir, hay un punto\(x \in \overline{K} \setminus K\). Si\(y \not= x\), entonces para\(n\) con\(\nicefrac{1}{n} < d(x,y)\) nosotros tenemos\(y \notin C(x,\nicefrac{1}{n})\). Además\(x \notin K\), así\[K \subset \bigcup_{n=1}^\infty {C(x,\nicefrac{1}{n})}^c .\] como una bola cerrada está cerrada,\({C(x,\nicefrac{1}{n})}^c\) está abierta, y así tenemos una cubierta abierta. Si tomamos alguna colección finita de índices\(n_1 < n_2 < \ldots < n_k\), entonces\[\bigcup_{j=1}^k {C(x,\nicefrac{1}{n_j})}^c = {C(x,\nicefrac{1}{n_k})}^c\] As\(x\) está en el cierre,\(C(x,\nicefrac{1}{n_k}) \cap K \not= \emptyset\). Por lo que no hay una subcubierta finita y no\(K\) es compacta.

    Demostramos a continuación que en el espacio euclidiano dimensional finito cada conjunto delimitado cerrado es compacto. Por lo tanto, los conjuntos delimitados cerrados de\({\mathbb{R}}^n\) son ejemplos de conjuntos compactos. No es cierto que en cada espacio métrico, cerrado y acotado sea equivalente a compacto. Un ejemplo sencillo sería un espacio métrico incompleto como es el\((0,1)\) caso de la métrica subespacial. Pero hay muchos espacios métricos completos y muy útiles donde cerrado y acotado no es suficiente para dar compacidad, ver:\(C([a,b],{\mathbb{R}})\) es un espacio métrico completo, pero la bola unitaria cerrada no\(C(0,1)\) es compacta. No obstante, ver.

    Una propiedad útil de los conjuntos compactos en un espacio métrico es que cada secuencia tiene una subsecuencia convergente. Tales conjuntos a veces se llaman secuencialmente compactos. Demostremos que en el contexto de los espacios métricos, un conjunto es compacto si y solo si es secuencialmente compacto. Primero probamos un lema.

    [ms:lebesgue] Dejar\((X,d)\) ser un espacio métrico y\(K \subset X\). Supongamos que cada secuencia en\(K\) tiene una subsecuencia convergente en\(K\). Dada una cubierta abierta\(\{ U_\lambda \}_{\lambda \in I}\) de\(K\), existe\(\delta > 0\) tal que para cada\(x \in K\), existe un\(\lambda \in I\) con\(B(x,\delta) \subset U_\lambda\).

    Es importante reconocer lo que dice el lema. Dice que dada cualquier tapadera hay una sola\(\delta > 0\). El\(\delta\) puede depender de la cubierta, pero claro que no depende de\(x\).

    Demostremos el lema por contrapositivo. Si la conclusión no es cierta, entonces hay una cubierta abierta\(\{ U_\lambda \}_{\lambda \in I}\) de\(K\) con la siguiente propiedad. Por cada\(n \in {\mathbb{N}}\) existe un\(x_n \in K\) tal que no\(B(x_n,\nicefrac{1}{n})\) es un subconjunto de ninguno\(U_\lambda\). Dado alguno\(x \in K\), hay\(\lambda \in I\) tal que\(x \in U_\lambda\). De ahí que exista\(\epsilon > 0\) tal que\(B(x,\epsilon) \subset U_\lambda\). Toma\(M\) tal que\(\nicefrac{1}{M} < \nicefrac{\epsilon}{2}\). Si\(y \in B(x,\nicefrac{\epsilon}{2})\) y\(n \geq M\), entonces por triángulo desigualdad\[B(y,\nicefrac{1}{n}) \subset B(y,\nicefrac{1}{M}) \subset B(y,\nicefrac{\epsilon}{2}) \subset B(x,\epsilon) \subset U_\lambda .\] En otras palabras, para todos\(n \geq M\),\(x_n \notin B(x,\nicefrac{\epsilon}{2})\). De ahí que la secuencia no pueda tener una subsecuencia convergente a\(x\). Como\(x \in K\) fue arbitrario ya terminamos.

    [thm:mscompactisseqcpt] Dejar\((X,d)\) ser un espacio métrico. Entonces\(K \subset X\) es un conjunto compacto si y solo si cada secuencia en\(K\) tiene una subsecuencia que converge a un punto en\(K\).

    Dejar\(K \subset X\) ser un conjunto y\(\{ x_n \}\) una secuencia en\(K\). Supongamos que para cada uno\(x \in K\), hay una pelota\(B(x,\alpha_x)\) para algunos\(\alpha_x > 0\) tal que\(x_n \in B(x,\alpha_x)\) por sólo finitamente muchos\(n \in {\mathbb{N}}\). Entonces\[K \subset \bigcup_{x \in K} B(x,\alpha_x) .\] Cualquier colección finita de estas bolas va a contener sólo finitamente muchas\(x_n\). Así para cualquier colección finita de tales bolas hay una\(x_n \in K\) que no está en la unión. Por lo tanto, no\(K\) es compacto.

    Entonces si\(K\) es compacto, entonces existe un\(x \in K\) tal que para cualquiera\(\delta > 0\),\(B(x,\delta)\) contiene\(x_k\) para infinitamente muchos\(k \in {\mathbb{N}}\). El balón\(B(x,1)\) contiene algunos\(x_k\) así que vamos\(n_1 := k\). Si\(n_{j-1}\) se define, entonces debe existir\(k > n_{j-1}\) tal que\(x_k \in B(x,\nicefrac{1}{j})\), así definir\(n_j := k\). Observe eso\(d(x,x_{n_j}) < \nicefrac{1}{j}\). Por,\(\lim\, x_{n_j} = x\).

    Para la otra dirección, supongamos que cada secuencia en\(K\) tiene una subsecuencia que converge en\(K\). Tome una cubierta abierta\(\{ U_\lambda \}_{\lambda \in I}\) de\(K\). Usando el lema que cubre Lebesgue arriba, encontramos un\(\delta > 0\) tal que para cada\(x\), hay un\(\lambda \in I\) con\(B(x,\delta) \subset U_\lambda\).

    Escoge\(x_1 \in K\) y encuentra\(\lambda_1 \in I\) tal que\(B(x_1,\delta) \subset U_{\lambda_1}\). Si\(K \subset U_{\lambda_1}\), nos detenemos ya que hemos encontrado una subcubierta finita. De lo contrario, debe haber un punto\(x_2 \in K \setminus U_{\lambda_1}\). Tenga en cuenta que\(d(x_2,x_1) \geq \delta\). Debe existir alguna\(\lambda_2 \in I\) tal que\(B(x_2,\delta) \subset U_{\lambda_2}\). Trabajamos inductivamente. Supongamos que\(\lambda_{n-1}\) se define. O\(U_{\lambda_1} \cup U_{\lambda_2} \cup \cdots \cup U_{\lambda_{n-1}}\) es una cubierta finita de\(K\), en cuyo caso nos detenemos, o debe haber un punto\(x_n \in K \setminus \bigl( U_{\lambda_1} \cup U_{\lambda_2} \cup \cdots \cup U_{\lambda_{n-1}}\bigr)\). Tenga en cuenta que\(d(x_n,x_j) \geq \delta\) para todos\(j = 1,2,\ldots,n-1\). A continuación, debe haber alguna\(\lambda_n \in I\) tal que\(B(x_n,\delta) \subset U_{\lambda_n}\).

    O en algún momento obtenemos una subcubierta finita de\(K\) o obtenemos una secuencia infinita\(\{ x_n \}\) como la anterior. Por contradicción supongamos que no hay una subcubierta finita y tenemos la secuencia\(\{ x_n \}\). Para todos\(n\) y\(k\),\(n \not= k\), tenemos\(d(x_n,x_k) \geq \delta\), así que ninguna subsecuencia de\(\{ x_n \}\) puede ser Cauchy. De ahí que ninguna subsecuencia de\(\{ x_n \}\) pueda ser convergente, lo cual es una contradicción.

    El teorema de Bolzano-Weierstrass para secuencias de números reales () dice que cualquier secuencia acotada en\({\mathbb{R}}\) tiene una subsecuencia convergente. Por lo tanto, cualquier secuencia en un intervalo cerrado\([a,b] \subset {\mathbb{R}}\) tiene una subsecuencia convergente. El límite también debe estar en la\([a,b]\) medida en que los límites preservan las desigualdades no estrictas. Por lo tanto, un intervalo delimitado cerrado\([a,b] \subset {\mathbb{R}}\) es compacto.

    Dejar\((X,d)\) ser un espacio métrico y dejar\(K \subset X\) ser compacto. Si\(E \subset K\) es un conjunto cerrado, entonces\(E\) es compacto.

    Dejar\(\{ x_n \}\) ser una secuencia en\(E\). También es una secuencia en\(K\). Por lo tanto, tiene una subsecuencia convergente\(\{ x_{n_j} \}\) que converge a algunos\(x \in K\). Como\(E\) se cierra el límite de una secuencia en también\(E\) está adentro\(E\) y así\(x \in E\). Por lo tanto,\(E\) debe ser compacto.

    [thm:msbw] Un subconjunto delimitado cerrado\(K \subset {\mathbb{R}}^n\) es compacto.

    Por lo que los subconjuntos de\({\mathbb{R}}^n\) son compactos si y solo si están cerrados y acotados, condición que es mucho más fácil de verificar. Reiteremos que el teorema de Heine-Borel sólo se sostiene para\({\mathbb{R}}^n\) y no para los espacios métricos en general. En general, compacto implica cerrado y acotado, pero no al revés.

    Porque\({\mathbb{R}}= {\mathbb{R}}^1\) si\(K \subset {\mathbb{R}}\) está cerrado y acotado, entonces cualquier secuencia\(\{ x_k \}\) en\(K\) está delimitada, por lo que tiene una subsecuencia convergente por el teorema de Bolzano-Weierstrass (). Como\(K\) está cerrado, el límite de la subsecuencia debe ser un elemento de\(K\). Así\(K\) es compacto.

    Llevemos a cabo la prueba para\(n=2\) y dejemos arbitrario\(n\) como ejercicio. Como\(K \subset {\mathbb{R}}^2\) está acotado, existe un conjunto\(B=[a,b]\times[c,d] \subset {\mathbb{R}}^2\) tal que\(K \subset B\). Demostraremos que\(B\) es compacto. Entonces\(K\), al ser un subconjunto cerrado de un compacto\(B\), también es compacto.

    Dejar\(\{ (x_k,y_k) \}_{k=1}^\infty\) ser una secuencia en\(B\). Es decir,\(a \leq x_k \leq b\) y\(c \leq y_k \leq d\) para todos\(k\). Una secuencia acotada de números reales tiene una subsecuencia convergente por lo que hay una subsecuencia\(\{ x_{k_j} \}_{j=1}^\infty\) que es convergente. La subsecuencia también\(\{ y_{k_j} \}_{j=1}^\infty\) es una secuencia acotada por lo que existe una subsecuencia\(\{ y_{k_{j_i}} \}_{i=1}^\infty\) que es convergente. Una subsecuencia de una secuencia convergente sigue siendo convergente, así\(\{ x_{k_{j_i}} \}_{i=1}^\infty\) es convergente. Let\[x := \lim_{i\to\infty} x_{k_{j_i}} \qquad \text{and} \qquad y := \lim_{i\to\infty} y_{k_{j_i}} .\] By,\(\bigl\{ (x_{k_{j_i}},y_{k_{j_i}}) \bigr\}_{i=1}^\infty\) converge a\((x,y)\). Además, como\(a \leq x_k \leq b\) y\(c \leq y_k \leq d\) para todos\(k\), eso lo sabemos\((x,y) \in B\).

    La métrica discreta vuelve a ofrecer interesantes contraejemplos. Dejar\((X,d)\) ser un espacio métrico con la métrica discreta, es decir\(d(x,y) = 1\) si\(x \not= y\). Supongamos que\(X\) es un conjunto infinito. Entonces:

    1. \((X,d)\)es un espacio métrico completo.
    2. Cualquier subconjunto\(K \subset X\) está cerrado y acotado.
    3. Un subconjunto\(K \subset X\) es compacto si y sólo si es un conjunto finito.
    4. La conclusión del lema de cobertura de Lebesgue siempre está satisfecha con\(\delta = \nicefrac{1}{2}\), por ejemplo, incluso para no compactos\(K \subset X\).

    Las pruebas de estas afirmaciones son triviales o son relegadas a los ejercicios siguientes.

    Ejercicios

    Dejar\((X,d)\) ser un espacio métrico y\(A\) un subconjunto finito de\(X\). Demostrar que\(A\) es compacto.

    \(A = \{ \nicefrac{1}{n} : n \in {\mathbb{N}}\} \subset {\mathbb{R}}\)Dejar. a) Mostrar que no\(A\) es compacto directamente usando la definición. b) Mostrar que\(A \cup \{ 0 \}\) es compacto directamente usando la definición.

    Dejar\((X,d)\) ser un espacio métrico con el discreto métrico. a) Probar que\(X\) está completo. b) Probar que\(X\) es compacto si y solo si\(X\) es un conjunto finito.

    a) Demostrar que la unión de finitamente muchos conjuntos compactos es un conjunto compacto. b) Encuentre un ejemplo donde la unión de infinitamente muchos conjuntos compactos no sea compacta.

    Demostrar por dimensión arbitraria. Pista: El truco es usar la notación correcta.

    Mostrar que un conjunto compacto\(K\) es un espacio métrico completo (usando la métrica del subespacio).

    [ejercicio:CABRComplete] Dejar\(C([a,b],{\mathbb{R}})\) ser el espacio métrico como en. Mostrar que\(C([a,b],{\mathbb{R}})\) es un espacio métrico completo.

    [exercise:msclbounnotcompt] Dejar\(C([0,1],{\mathbb{R}})\) ser el espacio métrico de. Dejar\(0\) denotar la función cero. Después demuestre que la bola cerrada no\(C(0,1)\) es compacta (aunque esté cerrada y acotada). Consejos: Construir una secuencia de distintas funciones continuas\(\{ f_n \}\) tales que\(d(f_n,0) = 1\) y\(d(f_n,f_k) = 1\) para todos\(n \not= k\). Demuestre que el conjunto\(\{ f_n : n \in {\mathbb{N}}\} \subset C(0,1)\) está cerrado pero no compacto. Ver para inspirarse.

    Demuestre que existe una métrica en\({\mathbb{R}}\) que\({\mathbb{R}}\) se convierte en un conjunto compacto.

    Supongamos que\((X,d)\) está completo y supongamos que tenemos una colección contablemente infinita de conjuntos compactos no vacíos\(E_1 \supset E_2 \supset E_3 \supset \cdots\) luego probar\(\bigcap_{j=1}^\infty E_j \not= \emptyset\).

    Dejar\(C([0,1],{\mathbb{R}})\) ser el espacio métrico de. \(K\)Sea el conjunto de\(f \in C([0,1],{\mathbb{R}})\) tal que\(f\) sea igual a un polinomio cuadrático, es decir\(f(x) = a+bx+cx^2\), y tal que\(\left\lvert {f(x)} \right\rvert \leq 1\) para todos\(x \in [0,1]\), es decir\(f \in C(0,1)\). Demostrar que\(K\) es compacto.

    [exercise:mstotbound] Dejar\((X,d)\) ser un espacio métrico completo. Mostrar que\(K \subset X\) es compacto si y solo si\(K\) está cerrado y tal que por cada\(\epsilon > 0\) existe un conjunto finito de puntos\(x_1,x_2,\ldots,x_n\) con\(K \subset \bigcup_{j=1}^n B(x_j,\epsilon)\). Nota:\(K\) Se dice que dicho conjunto está totalmente acotado, por lo que en un espacio métrico completo un conjunto es compacto si y solo si está cerrado y totalmente delimitado.

    Tomar\({\mathbb{N}}\subset {\mathbb{R}}\) usando la métrica estándar. Encuentra una cubierta abierta de\({\mathbb{N}}\) tal que la conclusión del lema de cobertura de Lebesgue no se mantenga.

    Demostrar el teorema general de Bolzano-Weierstrass: Cualquier secuencia acotada\(\{ x_k \}\) en\({\mathbb{R}}^n\) tiene una subsecuencia convergente.

    Dejar\(X\) ser un espacio métrico y\(C \subset {\mathcal{P}}(X)\) el conjunto de subconjuntos compactos no vacíos de\(X\). Usando la métrica de Hausdorff de, mostrar que\((C,d_H)\) es un espacio métrico. Es decir, mostrar que si\(L\) y\(K\) son subconjuntos compactos no vacíos entonces\(d_H(L,K) = 0\) si y solo si\(L=K\).

    [ejercicio:cerradocompleto] Dejar\((X,d)\) ser un espacio métrico completo y\(E \subset X\) un conjunto cerrado. Mostrar que\(E\) con la métrica subespacial hay un espacio métrico completo.

    Funciones continuas

    Nota: 1 conferencia

    Continuidad

    Dejar\((X,d_X)\) y\((Y,d_Y)\) ser espacios métricos y\(c \in X\). Entonces\(f \colon X \to Y\) es continuo en\(c\) si por cada\(\epsilon > 0\) hay\(\delta > 0\) tal que cuando\(x \in X\) y\(d_X(x,c) < \delta\), entonces\(d_Y\bigl(f(x),f(c)\bigr) < \epsilon\).

    Cuando\(f \colon X \to Y\) es continuo en absoluto\(c \in X\), entonces simplemente decimos que\(f\) es una función continua.

    La definición concuerda con la definición de cuándo\(f\) es una función de valor real en la línea real, si tomamos la métrica estándar\({\mathbb{R}}\).

    [prop:contiscont] Dejar\((X,d_X)\) y\((Y,d_Y)\) ser espacios métricos. Entonces\(f \colon X \to Y\) es continuo en\(c \in X\) si y solo si por cada secuencia\(\{ x_n \}\) en\(X\) converger a\(c\), la secuencia\(\{ f(x_n) \}\) converge a\(f(c)\).

    Supongamos que\(f\) es continuo en\(c\). Dejar\(\{ x_n \}\) ser una secuencia en\(X\) converger a\(c\). Dado\(\epsilon > 0\), hay\(\delta > 0\) tal que\(d_X(x,c) < \delta\) implica\(d_Y\bigl(f(x),f(c)\bigr) < \epsilon\). Así que toma\(M\) tal que para todos\(n \geq M\), tenemos\(d_X(x_n,c) < \delta\), entonces\(d_Y\bigl(f(x_n),f(c)\bigr) < \epsilon\). De ahí que\(\{ f(x_n) \}\) converja a\(f(c)\).

    Por otro lado supongamos que no\(f\) es continuo en\(c\). Entonces existe un\(\epsilon > 0\), tal que por cada\(n \in {\mathbb{N}}\) existe un\(x_n \in X\), con\(d_X(x_n,c) < \nicefrac{1}{n}\) tal que\(d_Y\bigl(f(x_n),f(c)\bigr) \geq \epsilon\). Entonces\(\{ x_n \}\) converge a\(c\), pero\(\{ f(x_n) \}\) no converge a\(f(c)\).

    Supongamos que\(f \colon {\mathbb{R}}^2 \to {\mathbb{R}}\) es un polinomio. Es decir,\[f(x,y) = \sum_{j=0}^d \sum_{k=0}^{d-j} a_{jk}\,x^jy^k = a_{0\,0} + a_{1\,0} \, x + a_{0\,1} \, y+ a_{2\,0} \, x^2+ a_{1\,1} \, xy+ a_{0\,2} \, y^2+ \cdots + a_{0\,d} \, y^d ,\] para algunos\(d \in {\mathbb{N}}\) (el grado) y\(a_{jk} \in {\mathbb{R}}\). Entonces afirmamos que\(f\) es continuo. Dejar\(\{ (x_n,y_n) \}_{n=1}^\infty\) ser una secuencia en la\({\mathbb{R}}^2\) que converja a\((x,y) \in {\mathbb{R}}^2\). Hemos demostrado que esto significa que\(\lim\, x_n = x\) y\(\lim\, y_n = y\). Entonces por lo\[\lim_{n\to\infty} f(x_n,y_n) = \lim_{n\to\infty} \sum_{j=0}^d \sum_{k=0}^{d-j} a_{jk} \, x_n^jy_n^k = \sum_{j=0}^d \sum_{k=0}^{d-j} a_{jk} \, x^jy^k = f(x,y) .\] que tenemos Así\(f\) es continuo en\((x,y)\), y como\((x,y)\) fue arbitrario\(f\) es continuo en todas partes. De igual manera, un polinomio en\(n\) variables es continuo.

    Compacidad y continuidad

    Los mapas continuos no mapean conjuntos cerrados a conjuntos cerrados. Por ejemplo,\(f \colon (0,1) \to {\mathbb{R}}\) definido por\(f(x) := x\) toma el conjunto\((0,1)\), que está cerrado en\((0,1)\), al conjunto\((0,1)\), que no está cerrado en\({\mathbb{R}}\). Por otro lado, los mapas continuos sí conservan conjuntos compactos.

    [lemma:continuouscompact] Dejar\((X,d_X)\) y\((Y,d_Y)\) ser espacios métricos y\(f \colon X \to Y\) una función continua. Si\(K \subset X\) es un conjunto compacto, entonces\(f(K)\) es un conjunto compacto.

    Una secuencia en se\(f(K)\) puede escribir como\(\{ f(x_n) \}_{n=1}^\infty\), donde\(\{ x_n \}_{n=1}^\infty\) es una secuencia en\(K\). El conjunto\(K\) es compacto y por lo tanto hay una subsecuencia\(\{ x_{n_i} \}_{i=1}^\infty\) que converge a algunos\(x \in K\). Por continuidad,\[\lim_{i\to\infty} f(x_{n_i}) = f(x) \in f(K) .\] Así que cada secuencia en\(f(K)\) tiene una subsecuencia convergente a un punto en\(f(K)\), y\(f(K)\) es compacta por.

    Como antes,\(f \colon X \to {\mathbb{R}}\) logra un mínimo absoluto\[f(x) \geq f(c) \qquad \text{ for all $x \in X$.}\] en\(c \in X\) si Por otro lado,\(f\) logra un máximo absoluto en\(c \in X\) si\[f(x) \leq f(c) \qquad \text{ for all $x \in X$.}\]

    Dejar\((X,d)\) ser un espacio métrico compacto y\(f \colon X \to {\mathbb{R}}\) una función continua. Entonces\(f\) se acota y de hecho\(f\) logra un mínimo absoluto y un máximo absoluto encendido\(X\).

    Como\(X\) es compacto y\(f\) es continuo, tenemos que\(f(X) \subset {\mathbb{R}}\) es compacto. De ahí\(f(X)\) que esté cerrado y acotado. En particular,\(\sup f(X) \in f(X)\) y\(\inf f(X) \in f(X)\), porque tanto el sup como el inf se pueden lograr mediante secuencias en\(f(X)\) y\(f(X)\) se cierra. Por lo tanto hay algunos\(x \in X\) tales que\(f(x) = \sup f(X)\) y algunos\(y \in X\) tales que\(f(y) = \inf f(X)\).

    Continuidad y topología

    Veamos cómo definir la continuidad en términos de la topología, es decir, los conjuntos abiertos. Ya hemos visto que la topología determina qué secuencias convergen, por lo que no es de extrañar que la topología también determine la continuidad de las funciones.

    [lemma:mstopocontloc] Dejar\((X,d_X)\) y\((Y,d_Y)\) ser espacios métricos. Una función\(f \colon X \to Y\) es continua en\(c \in X\) si y solo si por cada vecindario abierto\(U\) de\(f(c)\) in\(Y\), el conjunto\(f^{-1}(U)\) contiene un vecindario abierto de\(c\) in\(X\).

    Primero supongamos que\(f\) es continuo en\(c\). Que\(U\) sea un barrio abierto de\(f(c)\) adentro\(Y\), entonces\(B_Y\bigl(f(c),\epsilon\bigr) \subset U\) para algunos\(\epsilon > 0\). Por continuidad de\(f\), existe\(\delta > 0\) tal que siempre que\(x\) sea tal que\(d_X(x,c) < \delta\), entonces\(d_Y\bigl(f(x),f(c)\bigr) < \epsilon\). En otras palabras,\[B_X(c,\delta) \subset f^{-1}\bigl(B_Y\bigl(f(c),\epsilon\bigr)\bigr) \subset f^{-1}(U) ,\] y\(B_X(c,\delta)\) es un barrio abierto de\(c\).

    Para la otra dirección,\(\epsilon > 0\) déjese dar. Si\(f^{-1}\bigl(B_Y\bigl(f(c),\epsilon\bigr)\bigr)\) contiene un barrio abierto\(W\) de\(c\), contiene una bola. Es decir, hay algunos\(\delta > 0\) tales que\[B_X(c,\delta) \subset W \subset f^{-1}\bigl(B_Y\bigl(f(c),\epsilon\bigr)\bigr) .\] Eso significa precisamente que si\(d_X(x,c) < \delta\) entonces\(d_Y\bigl(f(x),f(c)\bigr) < \epsilon\), y así\(f\) es continuo en\(c\).

    [thm:mstopocont] Dejar\((X,d_X)\) y\((Y,d_Y)\) ser espacios métricos. Una función\(f \colon X \to Y\) es continua si y solo si por cada abierto\(U \subset Y\),\(f^{-1}(U)\) está abierto en\(X\).

    La prueba se desprende y se deja como ejercicio.

    Dejar\(f \colon X \to Y\) ser una función continua. nos dice que si\(E \subset Y\) está cerrado, entonces también\(f^{-1}(E) = X \setminus f^{-1}(E^c)\) está cerrado. Por lo tanto si tenemos una función continua\(f \colon X \to {\mathbb{R}}\), entonces el conjunto cero de\(f\), es decir,\(f^{-1}(0) = \{ x \in X : f(x) = 0 \}\), se cierra. Acabamos de demostrar el resultado más básico en geometría algebraica, el estudio de conjuntos cero de polinomios.

    De igual manera el conjunto donde\(f\) es no negativo, es decir,\(f^{-1}\bigl( [0,\infty) \bigr) = \{ x \in X : f(x) \geq 0 \}\) está cerrado. Por otro lado el conjunto donde\(f\) es positivo,\(f^{-1}\bigl( (0,\infty) \bigr) = \{ x \in X : f(x) > 0 \}\) es abierto.

    Continuidad uniforme

    En cuanto a las funciones continuas en la línea real, en la definición de continuidad a veces es conveniente poder elegir una\(\delta\) para todos los puntos.

    Dejar\((X,d_X)\) y\((Y,d_Y)\) ser espacios métricos. Entonces\(f \colon X \to Y\) es uniformemente continuo si por cada\(\epsilon > 0\) hay\(\delta > 0\) tal que cuando\(x,c \in X\) y\(d_X(x,c) < \delta\), entonces\(d_Y\bigl(f(x),f(c)\bigr) < \epsilon\).

    Una función uniformemente continua es continua, pero no necesariamente viceversa como hemos visto.

    [THM:XCompactFunifCont] Dejar\((X,d_X)\) y\((Y,d_Y)\) ser espacios métricos. Supongamos que\(f \colon X \to Y\) es continuo y\(X\) compacto. Entonces\(f\) es uniformemente continuo.

    Dejemos\(\epsilon > 0\) que se den. Para cada uno\(c \in X\), escoja\(\delta_c > 0\) tal que\(d_Y\bigl(f(x),f(c)\bigr) < \nicefrac{\epsilon}{2}\) cuando sea\(d_X(x,c) < \delta_c\). Las bolas\(B(c,\delta_c)\) cubren\(X\), y el espacio\(X\) es compacto. Aplicar el para obtener un\(\delta > 0\) tal que para cada\(x \in X\), hay un\(c \in X\) para el cual\(B(x,\delta) \subset B(c,\delta_c)\).

    Si\(x_1, x_2 \in X\) dónde\(d_X(x_1,x_2) < \delta\), encuentra\(c \in X\) tal que\(B(x_1,\delta) \subset B(c,\delta_c)\). Entonces\(x_2 \in B(c,\delta_c)\). Por el triángulo de la desigualdad y la definición de\(\delta_c\) tenemos\[d_Y\bigl(f(x_1),f(x_2)\bigr) \leq d_Y\bigl(f(x_1),f(c)\bigr) + d_Y\bigl(f(c),f(x_2)\bigr) < \nicefrac{\epsilon}{2}+ \nicefrac{\epsilon}{2} = \epsilon . \qedhere\]

    Ejemplos útiles de funciones uniformemente continuas son nuevamente las llamadas funciones continuas de Lipschitz. Es decir si\((X,d_X)\) y\((Y,d_Y)\) son espacios métricos, entonces\(f \colon X \to Y\) se llama Lipschitz o\(K\) -Lipschitz si existe\(K \in {\mathbb{R}}\) tal que no es\[d_Y\bigl(f(x),f(c)\bigr) \leq K d_X(x,c) \ \ \ \ \text{for all } x,c \in X.\] difícil probar que Lipschitz implica uniformemente continuo, solo toma\(\delta = \nicefrac{\epsilon}{K}\). Y ya vimos en el caso de las funciones en la línea real, una función puede ser uniformemente continua pero no Lipschitz.

    Cabe mencionar que, si una función es Lipschitz, tiende a ser más fácil simplemente demostrar que es Lipschitz aunque sólo nos interese conocer la continuidad.

    Ejercicios

    Considera\({\mathbb{N}}\subset {\mathbb{R}}\) con la métrica estándar. Dejar\((X,d)\) ser un espacio métrico y\(f \colon X \to {\mathbb{N}}\) una función continua. a) Demostrar que si\(X\) está conectado, entonces\(f\) es constante (el rango de\(f\) es un valor único). b) Encuentra un ejemplo donde\(X\) está desconectado y no\(f\) es constante.

    Dejar\(f \colon {\mathbb{R}}^2 \to {\mathbb{R}}\) definirse por\(f(0,0) := 0\), y\(f(x,y) := \frac{xy}{x^2+y^2}\) si\((x,y) \not= (0,0)\). a) Demostrar que para cualquier fijo\(x\), la función que lleva\(y\) a\(f(x,y)\) es continua. De manera similar para cualquier fijo\(y\), la función que lleva\(x\) a\(f(x,y)\) es continua. b) Mostrar que no\(f\) es continua.

    Supongamos que\(f \colon X \to Y\) es continuo para espacios métricos\((X,d_X)\) y\((Y,d_Y)\). \(A \subset X\)Dejar. a) Demostrar\(f(\overline{A}) \subset \overline{f(A)}\) eso. b) Demostrar que el subconjunto puede ser apropiado.

    Demostrar. Pista: Uso.

    [exercise:msconnconn] Supongamos que\(f \colon X \to Y\) es continuo para espacios métricos\((X,d_X)\) y\((Y,d_Y)\). Mostrar que si\(X\) está conectado, entonces\(f(X)\) está conectado.

    Demostrar la siguiente versión del. Dejar\((X,d)\) ser un espacio métrico conectado y\(f \colon X \to {\mathbb{R}}\) una función continua. Supongamos que existen\(x_0,x_1 \in X\) y\(y \in {\mathbb{R}}\) tal que\(f(x_0) < y < f(x_1)\). Entonces demuestre que existe\(z \in X\) tal eso\(f(z) = y\). Pista: Ver.

    Una función continua\(f \colon X \to Y\) para espacios métricos\((X,d_X)\) y\((Y,d_Y)\) se dice que es adecuada si para cada conjunto compacto\(K \subset Y\), el conjunto\(f^{-1}(K)\) es compacto. Supongamos que un continuo\(f \colon (0,1) \to (0,1)\) es propio y\(\{ x_n \}\) es una secuencia en la\((0,1)\) que converge a\(0\). Demostrar que no\(\{ f(x_n) \}\) tiene subsecuencia que converja en\((0,1)\).

    Dejar\((X,d_X)\) y\((Y,d_Y)\) ser espacio métrico y\(f \colon X \to Y\) ser una función uno-a-uno y en continuo. Supongamos que\(X\) es compacto. Demostrar que la inversa\(f^{-1} \colon Y \to X\) es continua.

    Tome el espacio métrico de las funciones continuas\(C([0,1],{\mathbb{R}})\). Dejar\(k \colon [0,1] \times [0,1] \to {\mathbb{R}}\) ser una función continua. Dado\(f \in C([0,1],{\mathbb{R}})\) definir\[\varphi_f(x) := \int_0^1 k(x,y) f(y) ~dy .\] a) Mostrar que\(T(f) := \varphi_f\) define una función\(T \colon C([0,1],{\mathbb{R}}) \to C([0,1],{\mathbb{R}})\).
    b) Demostrar que\(T\) es continuo.

    Dejar\((X,d)\) ser un espacio métrico.
    a) Si\(p \in X\), mostrar lo\(f \colon X \to {\mathbb{R}}\) definido por\(f(x) := d(x,p)\) es continuo.
    b) Definir una métrica en\(X \times X\) como en la parte b, y mostrar que la\(g \colon X \times X \to {\mathbb{R}}\) definida por\(g(x,y) := d(x,y)\) es continua.
    c) Demostrar que si\(K_1\) y\(K_2\) son subconjuntos compactos de\(X\), entonces existe un\(p \in K_1\) y\(q \in K_2\) tal que\(d(p,q)\) es mínimo, es decir,\(d(p,q) = \inf \{ (x,y) \colon x \in K_1, y \in K_2 \}\).

    Teorema de punto fijo y teorema de Picard nuevamente

    Nota: 1 conferencia (opcional, no requiere)

    En esta sección probamos el teorema de punto fijo para mapeos de contracción. Como aplicación probamos el teorema de Picard, el cual probamos sin espacios métricos en. La prueba que presentamos aquí es similar, pero la prueba va mucho más suave con espacios métricos y el teorema de punto fijo.

    Teorema de punto fijo

    Dejar\((X,d)\) y\((X',d')\) ser espacios métricos. \(f \colon X \to X'\)se dice que es una contracción (o un mapa contractivo) si es un mapa\(k\) -Lipschitz para algunos\(k < 1\), es decir, si existe\(k < 1\) tal que\[d'\bigl(f(x),f(y)\bigr) \leq k d(x,y) \ \ \ \ \text{for all } x,y \in X.\]

    Si\(f \colon X \to X\) es un mapa,\(x \in X\) se llama punto fijo si\(f(x)=x\).

    [Principio de mapeo de contracción o teorema de punto fijo] [thm:contr] Let\((X,d)\) ser un espacio métrico completo no vacío y\(f \colon X \to X\) una contracción. Entonces\(f\) tiene un punto fijo único.

    Las palabras completa y contracción son necesarias. Ver.

    Escoge cualquiera\(x_0 \in X\). Definir una secuencia\(\{ x_n \}\) por\(x_{n+1} := f(x_n)\). \[d(x_{n+1},x_n) = d\bigl(f(x_n),f(x_{n-1})\bigr) \leq k d(x_n,x_{n-1}) \leq \cdots \leq k^n d(x_1,x_0) .\]Supongamos\(m > n\), entonces\[\begin{split} d(x_m,x_n) & \leq \sum_{i=n}^{m-1} d(x_{i+1},x_i) \\ & \leq \sum_{i=n}^{m-1} k^i d(x_1,x_0) \\ & = k^n d(x_1,x_0) \sum_{i=0}^{m-n-1} k^i \\ & \leq k^n d(x_1,x_0) \sum_{i=0}^{\infty} k^i = k^n d(x_1,x_0) \frac{1}{1-k} . \end{split}\] En particular la secuencia es Cauchy (¿por qué?). Ya que\(X\) está completo dejamos\(x := \lim\, x_n\), y afirmamos que\(x\) es nuestro punto fijo único.

    ¿Punto fijo? La función\(f\) es continua ya que es una contracción, por lo que Lipschitz continua. De ahí\[f(x) = f( \lim \, x_n) = \lim\, f(x_n) = \lim\, x_{n+1} = x .\]

    ¿Único? Dejemos\(x\) y\(y\) ambos sean puntos fijos. \[d(x,y) = d\bigl(f(x),f(y)\bigr) \leq k d(x,y) .\]Como\(k < 1\) esto significa eso\(d(x,y) = 0\) y por lo tanto\(x=y\). Se prueba el teorema.

    La prueba es constructiva. No sólo sabemos que existe un punto fijo único. También sabemos cómo encontrarlo. Empezamos con cualquier punto\(x_0 \in X\) y simplemente iteramos\(f(x_0)\)\(f(f(x_0))\),\(f(f(f(x_0)))\),, etc... De hecho, incluso puedes encontrar qué tan lejos del punto fijo estás, ver los ejercicios. Por lo tanto, la idea de la prueba se utiliza en aplicaciones del mundo real.

    Teorema de Picard

    Antes de llegar a Picard, mencionemos a qué espacio métrico estaremos aplicando el teorema del punto fijo. Utilizaremos el espacio métrico\(C([a,b],{\mathbb{R}})\) de. Es decir,\(C([a,b],{\mathbb{R}})\) es el espacio de funciones continuas\(f \colon [a,b] \to {\mathbb{R}}\) con la métrica\[d(f,g) = \sup_{x \in [a,b]} \left\lvert {f(x)-g(x)} \right\rvert .\] Convergencia en esta métrica es convergencia en norma uniforme, o en otras palabras, convergencia uniforme. Por lo tanto, ver,\(C([a,b],{\mathbb{R}})\) es un espacio métrico completo.

    Usemos el teorema de punto fijo para probar el teorema clásico de Picard sobre la existencia y singularidad de las ecuaciones diferenciales ordinarias. Considerar la\[\frac{dy}{dx} = F(x,y) .\] ecuación Dados algunos\(x_0, y_0\) estamos buscando una función\(y=f(x)\) tal que\(f(x_0) = y_0\) y tal que\[f'(x) = F\bigl(x,f(x)\bigr) .\] Para evitar tener que llegar a muchos nombres a menudo simplemente escribimos\(y' = F(x,y)\) y\(y(x)\) para la solución.

    El ejemplo más simple es por ejemplo la ecuación\(y' = y\),\(y(0) = 1\). La solución es la exponencial\(y(x) = e^x\). Un ejemplo algo más complicado es\(y' = -2xy\),\(y(0) = 1\), cuya solución es la gaussiana\(y(x) = e^{-x^2}\).

    Hay algunas cuestiones sutiles, por ejemplo, cuánto tiempo existe la solución. Mira la ecuación\(y' = y^2\),\(y(0)=1\). Entonces\(y(x) = \frac{1}{1-x}\) es una solución. Si bien\(F\) es una función razonablemente “agradable” y en particular existe para todos\(x\) y\(y\), la solución “estalla” en\(x=1\). Para más ejemplos relacionados con el teorema de Picard ver.

    Dejar\(I, J \subset {\mathbb{R}}\) ser intervalos compactos, dejar\(I_0\) y\(J_0\) ser sus interiores, y dejar\((x_0,y_0) \in I_0 \times J_0\). Supongamos que\(F \colon I \times J \to {\mathbb{R}}\) es continuo y Lipschitz en la segunda variable, es decir, existe\(L \in {\mathbb{R}}\) tal que\[\left\lvert {F(x,y) - F(x,z)} \right\rvert \leq L \left\lvert {y-z} \right\rvert \ \ \ \text{ for all $y,z \in J$, $x \in I$} .\] Entonces existe una\(h > 0\) y una función diferenciable única\(f \colon [x_0 - h, x_0 + h] \to J \subset {\mathbb{R}}\), tal que\[f'(x) = F\bigl(x,f(x)\bigr) \qquad \text{and} \qquad f(x_0) = y_0.\]

    Sin pérdida de generalidad asumir\(x_0 =0\). Al\(I \times J\) ser compacto y\(F(x,y)\) continuo, está acotado. Así que encuentra un\(M > 0\), tal que\(\left\lvert {F(x,y)} \right\rvert \leq M\) para todos\((x,y) \in I\times J\). Escoge\(\alpha > 0\) tal que\([-\alpha,\alpha] \subset I\) y\([y_0-\alpha, y_0 + \alpha] \subset J\). Vamos a\[h := \min \left\{ \alpha, \frac{\alpha}{M+L\alpha} \right\} .\] anotar\([-h,h] \subset I\). Definir el conjunto Es\[Y := \{ f \in C([-h,h],{\mathbb{R}}) : f([-h,h]) \subset J \} . % [y_0-\alpha,y_0+\alpha] \} .\] decir,\(Y\) es el espacio de funciones continuas sobre\([-h,h]\) con valores en\(J\), es decir, exactamente esas funciones donde tiene\(F\bigl(x,f(x)\bigr)\) sentido. La métrica utilizada es la métrica estándar dada anteriormente.

    Espectáculo que\(Y \subset C([-h,h],{\mathbb{R}})\) está cerrado. Pista:\(J\) está cerrado.

    El espacio\(C([-h,h],{\mathbb{R}})\) está completo, y un subconjunto cerrado de un espacio métrico completo es un espacio métrico completo con la métrica subespacial, ver. Entonces\(Y\) con la métrica subespacial está completa.

    Definir una asignación\(T \colon Y \to C([-h,h],{\mathbb{R}})\) por\[T(f)(x) := y_0 + \int_0^x F\bigl(t,f(t)\bigr)~dt .\]

    Demostrar que si\(f \colon [-h,h] \to J\) es continuo entonces\(F\bigl(t,f(t)\bigr)\) es continuo encendido\([-h,h]\) como una función de\(t\). Usa esto para mostrar que\(T\) está bien definido y eso\(T(f) \in C([-h,h],{\mathbb{R}})\).

    Dejar\(f \in Y\) y\(\left\lvert {x} \right\rvert \leq h\). Como\(F\) está delimitado por\(M\) tenemos\[\begin{split} \left\lvert {T(f)(x) - y_0} \right\rvert &= \left\lvert {\int_0^x F\bigl(t,f(t)\bigr)~dt} \right\rvert \\ & \leq \left\lvert {x} \right\rvert M \leq hM \leq \frac{\alpha M}{M+ L\alpha} \leq \alpha . \end{split}\] Así\(T(f)([-h,h]) \subset [y_0-\alpha,y_0+\alpha] \subset J\), y\(T(f) \in Y\). En otras palabras,\(T(Y) \subset Y\). Consideramos así\(T\) como un mapeo de\(Y\) a\(Y\).

    Afirmamos\(T \colon Y \to Y\) es una contracción. Primero, para\(x \in [-h,h]\) y\(f,g \in Y\) tenemos\[\left\lvert {F\bigl(x,f(x)\bigr) - F\bigl(x,g(x)\bigr)} \right\rvert \leq L\left\lvert {f(x)- g(x)} \right\rvert \leq L \, d(f,g) .\] Por lo tanto,\[\begin{split} \left\lvert {T(f)(x) - T(g)(x)} \right\rvert &= \left\lvert {\int_0^x F\bigl(t,f(t)\bigr) - F\bigl(t,g(t)\bigr)~dt} \right\rvert \\ & \leq \left\lvert {x} \right\rvert L \, d(f,g) \leq h L\, d(f,g) \leq \frac{L\alpha}{M+L\alpha} \, d(f,g) . \end{split}\] Elegimos\(M > 0\) y así\(\frac{L\alpha}{M+L\alpha} < 1\). El reclamo se prueba tomando supremo sobre el lado izquierdo\(x \in [-h,h]\) de arriba para obtener\(d\bigl(T(f),T(g)\bigr) \leq \frac{L\alpha}{M+L\alpha} \, d(f,g)\).

    Aplicamos el teorema del punto fijo () para encontrar un único\(f \in Y\) tal que\(T(f) = f\), es decir,\[%\label{equation:msinteqpicard} f(x) = y_0 + \int_0^x F\bigl(t,f(t)\bigr)~dt .\] por el teorema fundamental del cálculo,\(T(f)\) es la función diferenciable única cuya derivada es\(F\bigl(x,f(x)\bigr)\) y\(T(f)(0) = y_0\). Por lo tanto\(f\) es la solución única de\(f'(x) = F\bigl(x,f(x)\bigr)\) y\(f(0) = y_0\).

    Demostrar que la afirmación “Sin pérdida de generalidad asuma\(x_0 = 0\)” está justificada. Es decir, probar que si conocemos el teorema con\(x_0 = 0\), el teorema es cierto como se afirma.

    Ejercicios

    Para más ejercicios relacionados con el teorema de Picard ver.

    Dejar que\(F \colon {\mathbb{R}}\to {\mathbb{R}}\) se defina por\(F(x) := kx + b\) donde\(0 < k < 1\),\(b \in {\mathbb{R}}\).
    a) Demostrar que\(F\) es una contracción.
    b) Encontrar el punto fijo y mostrar directamente que es único.

    Dejar\(f \colon [0,\nicefrac{1}{4}] \to [0,\nicefrac{1}{4}]\) ser definido por\(f(x) := x^2\) es una contracción.
    a) Demostrar que\(f\) es una contracción, y encontrar lo mejor (el más pequeño)\(k\) a partir de la definición que funcione.
    b) Encontrar el punto fijo y mostrar directamente que es único.

    [ejercicio:nofixedpoint] a) Encuentra un ejemplo de una contracción\(f \colon X \to X\) del espacio métrico no completo\(X\) sin punto fijo. b) Encuentra un mapa 1-Lipschitz\(f \colon X \to X\) de un espacio métrico completo\(X\) sin punto fijo.

    Considerar\(y' =y^2\),\(y(0)=1\). Utilice el esquema de iteración a partir de la prueba del principio de mapeo de contracción. Empezar con\(f_0(x) = 1\). Encuentra algunas iteraciones (al menos hasta\(f_2\)). Demostrar que el límite puntual de\(f_n\) es\(\frac{1}{1-x}\), es decir para cada uno\(x\) con\(\left\lvert {x} \right\rvert < h\) para algunos\(h > 0\), demuéstralo\(\lim\limits_{n\to\infty}f_n(x) = \frac{1}{1-x}\).

    Supongamos que\(f \colon X \to X\) es una contracción para\(k < 1\). Supongamos que usa el procedimiento de iteración con\(x_{n+1} := f(x_n)\) como en la prueba del teorema del punto fijo. Supongamos que\(x\) es el punto fijo de\(f\).
    a) Demostrar eso\(d(x,x_n) \leq k^n d(x_1,x_0) \frac{1}{1-k}\) para todos\(n \in {\mathbb{N}}\).
    b) Supongamos\(d(y_1,y_2) \leq 16\) para todos\(y_1,y_2 \in X\), y\(k= \nicefrac{1}{2}\). Encuentra un\(N\) tal que empiece en cualquier punto\(x_0 \in X\),\(d(x,x_n) \leq 2^{-16}\) para todos\(n \geq N\).

    Vamos\(f(x) := x-\frac{x^2-2}{2x}\). (Puede reconocer el método de Newton para\(\sqrt{2}\))
    a) Probar\(f\bigl([1,\infty)\bigr) \subset [1,\infty)\).
    b) Demostrar que\(f \colon [1,\infty) \to [1,\infty)\) es una contracción.
    c) Aplicar el teorema de punto fijo para encontrar\(x \geq 1\) tal eso\(f(x) = x\), y demostrarlo\(x = \sqrt{2}\).

    Supongamos que\(f \colon X \to X\) es una contracción, y\((X,d)\) es un espacio métrico con la métrica discreta, es decir\(d(x,y) = 1\) siempre\(x \not= y\). Demostrar que\(f\) es constante, es decir, existe\(c \in X\) tal que\(f(x) = c\) para todos\(x \in X\).


    1. El término “moderno” se refiere a finales del siglo XIX hasta la actualidad.
    2. Para los fans del programa de televisión Futurama, hay una sala de cine en un episodio llamado\(\aleph_0\) -plex.
    3. Un algebraista diría que\({\mathbb{Z}}\) es un anillo ordenado, o quizás más precisamente un anillo ordenado conmutativo.
    4. La singularidad depende del isomorfismo, pero deseamos evitar el uso excesivo del álgebra. Para nosotros, simplemente basta con asumir que existe un conjunto de números reales. Consulte Rudin para la construcción y más detalles.
    5. El nombre del matemático suizo Jacob Bernoulli (1655 — 1705).
    6. La hipótesis de la amplitud es para la simplicidad, se puede descartar si permitimos los números reales extendidos.
    7. usa la notación\((x_n)\) para denotar una secuencia en lugar de\(\{ x_n \}\), que es lo que usa. Ambos son comunes.
    8. El nombre del físico y matemático inglés Isaac Newton (1642 — 1726/7).
    9. El nombre del matemático checo Bernhard Placidus Johann Nepomuk Bolzano (1781 — 1848), y del matemático alemán Karl Theodor Wilhelm Weierstrass (1815 — 1897).
    10. A veces se dice que\(\{ x_n \}\) converge al infinito.
    11. El nombre del matemático francés Augustin-Louis Cauchy (1789—1857).
    12. La divergencia de las series armónicas se conoció antes de que la teoría de las series se hiciera rigurosa. De hecho la prueba que damos es la prueba más temprana y fue dada por Nicole Oresme (¿1323? —1382).
    13. Demostración de este hecho es lo que hizo famoso al matemático suizo Leonhard Paul Euler (1707 — 1783).
    14. Llamado así por el matemático italiano Ernesto Ces à ro (1859 — 1906).
    15. El nombre del matemático alemán Johannes Karl Thomae (1840 — 1921).
    16. El nombre del matemático alemán Rudolf Otto Segismundo Lipschitz (1832—1903).
    17. Llamado así por el matemático alemán Gottfried Wilhelm Leibniz (1646—1716).
    18. El nombre del matemático francés Michel Rolle (1652—1719).
    19. Nombrado así por el matemático inglés Brook Taylor (1685—1731). Fue encontrado por primera vez por el matemático escocés James Gregory (1638 — 1675). La declaración que damos fue probada por Joseph-Louis Lagrange (1736 — 1813)
    20. El nombre del matemático alemán Georg Friedrich Bernhard Riemann (1826—1866).
    21. El nombre del matemático francés Jean-Gaston Darboux (1842—1917).
    22. Se dice que tal\(h\) es de variación acotada.
    23. Compare esta hipótesis con.
    24. Llamado así por el matemático suizo Leonhard Paul Euler (1707 — 1783) y el matemático italiano Lorenzo Mascheroni (1750 — 1800).
    25. Acortado del latín: sinus cardinalis
    26. La notación ni la terminología no están completamente estandarizadas. La norma también se llama la norma sup o norma del infinito, y además de\(\left\lVert {f} \right\rVert_u\) y a veces\(\left\lVert {f} \right\rVert_S\) se escribe como\(\left\lVert {f} \right\rVert_{\infty}\) o\(\left\lVert {f} \right\rVert_{\infty,S}\).
    27. Llamado así por el matemático francés Charles Émile Picard (1856—1941).

    This page titled 2.1: Propiedades básicas is shared under a CC BY-SA 4.0 license and was authored, remixed, and/or curated by Jiří Lebl via source content that was edited to the style and standards of the LibreTexts platform; a detailed edit history is available upon request.